Sie sind auf Seite 1von 300

Edicin y correccin: Dania Relova Fernndez

Diseo de interior y cubierta: Frank Herrera


Diagramacin: Israel de Jess Zaldvar Pedroso
Carlos L. Menndez Gutirrez y Jess Garca, 2007

Versin impresa
ISBN 978-959-16-07-0339-3
Flix Varela

Versin electrnica
ISBN 978-959-16-0619-8
Editorial Universitaria
III
PRESENTACIN
En 1984, los autores, en colaboracin con el Dr. Jan Chudoba, del Instituto de
Tecnologa Qumica de Praga, publicaron el libro Fundamentos Tericos de
Algunos Procesos para la Purificacin de Aguas Residuales, con el prop-
sito de que sirviera de apoyo a aquellos profesionales de habla hispana que
realizaban su trabajo de doctorado en la Repblica de Checoslovaquia. Dos
aos despus se public la segunda edicin de ese ttulo.
Para continuar supliendo las crecientes necesidades de los profesionales de
este campo de trabajo, los autores publicaron en Cuba en 1991, bajo el ttulo de
Procesos para el Tratamiento Biolgico de Aguas Residuales Industriales
un texto enfocado fundamentalmente, tal como indica su ttulo, en los procesos
biolgicos para el tratamiento de aguas residuales. Doce aos despus se pre-
senta esta segunda edicin con el mismo propsito.
Como generalmente ocurre, una vez que concluy la preparacin de esta
nueva edicin, es que los autores se consideraron en condiciones de iniciar el
trabajo. Esto es consecuencia de la inconformidad natural que se experimenta,
a partir de la conviccin de que toda obra humana es perfectible. Tambin esto
forma parte de las enseanzas que nos leg Jan Chudoba.
La justificacin que nos anima ahora, como en las ocasiones anteriores,
estimulados por los resultados ya logrados, es continuar contribuyendo a la su-
peracin de los profesionales dedicados a esta esfera de trabajo, con temas
actualizados y expuestos de manera ms completa que en la primera edicin.
Otra justificacin quizs sea el de siempre haber pensado que el camino por
recorrer era ms largo del que en realidad ha sido y que sentimos la necesidad
de expresarnos. Ambas justificaciones son igualmente vlidas.
Agradecemos a todos los que han colaborado en este nuevo empeo, en
especial a la profesora Julia Mara Hernndez por su apoyo en la preparacin de
los materiales, y a nuestras familias por la paciencia manifestada una vez ms.
Deseamos dedicar esta obra, que es a su vez un nuevo punto de partida a la
memoria de Jan Chudoba, que sigue estando entre nosotros.
a JAN
Procesos para el tratamiento de aguas residuales / Carlos Menendez Gutierrez y Jess M. Prez Olmo
Ciudad de La Habana : Editorial Flix Varera - Editorial Universitaria, 2007. ISBN 978-959-16-0619-8
IV
LOS AUTORES
Carlos L. Menndez Gutirrez
Graduado de Ingeniero Qumico en la Universidad de La Habana
y de Especialista en Saneamiento Ambiental en el Centro Nacio-
nal de Investigaciones Cientficas de Cuba. Obtuvo el ttulo de
Doctor en Ciencias Tcnicas en el Instituto de Tecnologa Qu-
mica de Praga. Ha impartido cursos en el tema de tratamiento de
aguas residuales en universidades cubanas y latinoamericanas.
Miembro del Comit Doctoral del Instituto Tecnolgico de Tolu-
ca, Mxico, y del Tribunal de Grados Cientficos para Ingeniera
Qumica de Cuba. Miembro de varias sociedades cientficas. Es
autor de artculos tcnicos.
Jess M. Prez Olmo
Graduado de Ingeniero Qumico en la Universidad de La Habana
y Mster en Ingeniera Saneamiento Ambiental. Ha impartido
cursos sobre tratamiento de aguas residuales en universidades
cubanas. Es Especialista Principal en el tratamiento de aguas
residuales de la Compaa Especializada en Soluciones Integra-
les Geogrficas y Medio Ambientales (CESIGMA S.A.) y ha
brindado asesora en plantas cubanas y de otros pases latinoa-
mericanos.
Edicin auspiciada por:
Environmental Capacity Enhancement Project in Cuba.
Proyecto internacional establecido entre la University of Guelph y el
Instituto Superior Politcnico Jos Antonio Echeverra
Financiado por:
Canadian International Development Agency (CIDA)
Procesos para el tratamiento de aguas residuales / Carlos Menendez Gutierrez y Jess M. Prez Olmo
Ciudad de La Habana : Editorial Flix Varera - Editorial Universitaria, 2007. ISBN 978-959-16-0619-8
V
CAPTULO 1. Indicadores de la contaminacin / 1
1.1. Introduccin / 1
1.2. Contenido de slidos / 1
1.3. Demanda bioqumica de oxgeno (DBO) / 3
1.4. Demanda terica de oxgeno (DTO) y demanda qumica
de oxgeno (DQO) / 11
1.5. Carbono orgnico total (COT) / 13
1.6. Nitrgeno / 17
1.7. Fsforo / 19
1.8. Caractersticas de las aguas residuales / 20
1.9. Mediciones del caudal / 29
1.10. Disminucin del volumen y fortaleza de los residuales / 35
CAPTULO 2. Pretratamiento de aguas residuales / 39
2.1. Compensacin / 39
2.2. Sedimentacin / 53
CAPTULO 3. Oxidacin biolgica nitrificacin / 77
3.1. Principios de la oxidacin biolgica / 77
3.2. Metabolismo / 78
3.3. Biodegradabilidad de las aguas residuales / 82
3.4. Cintica del crecimiento biolgico y remocin de sustrato / 87
3.5. Nitrificacin-desnitrificacin / 98
CAPTULO 4. Lodo activado / 119
4.1. Generalidades / 119
4.2. Parmetros bsicos / 123
4.3. Principales modificaciones tecnolgicas / 128
NDICE
Procesos para el tratamiento de aguas residuales / Carlos Menendez Gutierrez y Jess M. Prez Olmo
Ciudad de La Habana : Editorial Flix Varera - Editorial Universitaria, 2007. ISBN 978-959-16-0619-8
VI
4.4. Aplicacin de los modelos cinticos / 135
4.5. Factores que inciden en la eficiencia de purificacin / 138
4.6. Presencia de compuestos orgnicos en el efluente / 144
4.7. Produccin de biomasa y lodo / 146
4.8. Determinacin de constantes a escala de laboratorio / 157
4.9. Requerimientos nutricionales / 160
CAPTULO 5. Transferencia de oxgeno / 165
5.1. Introduccin / 165
5.2. Consideraciones fundamentales de la transferencia
de oxgeno / 165
5.3. Requerimiento de oxgeno / 173
5.4. Implementos de aereacin / 180
5.5. Consumo de potencia / 192
CAPTULO 6. Filtros percoladores / 199
6.1. Introduccin / 199
6.2. Partes de las que consta un filtro percolador. Mecanismo
de remocin de la DBO / 201
6.3. Parmetros bsicos del proceso. Definiciones / 206
6.4. Clasificacin de los filtros percoladores / 207
6.5. Recirculacin / 208
6.6. Factores que inciden en la eficiencia de purificacin / 209
6.7. Diseo de filtros percoladores / 214
6.8. Clculo de las constantes del modelo de primer orden / 224
CAPTULO 7. Lagunas de estabilizacin / 235
7.1. Caractersticas generales / 235
7.2. Lagunas aerobias / 235
7.3. Lagunas anaerobias / 236
7.4. Lagunas facultativas / 236
7.5. Rgimen de flujo en las lagunas / 240
7.6. Diseo de lagunas de estabilizacin / 241
7.7. Relacin rea: volumen para las lagunas / 247
7.8. Remocin de patgenos / 248
Procesos para el tratamiento de aguas residuales / Carlos Menendez Gutierrez y Jess M. Prez Olmo
Ciudad de La Habana : Editorial Flix Varera - Editorial Universitaria, 2007. ISBN 978-959-16-0619-8
VII
7.9. Balance hdrico de las lagunas / 248
7.10. Criterios para la operacin y mantenimiento de las lagunas
de estabilizacin / 251
CAPTULO 8. Digestin anaerobia / 255
8.1. Introduccin / 255
8.2. Mecanismo de la digestin anaerobia / 256
8.3. Distintas variantes de los procesos de digestin anaerobia / 257
8.4. Factores que controlan el proceso de digestin / 265
8.5. Parmetros indicadores del curso de la digestin / 269
8.6. Mtodos de diseo / 272
8.7. Puesta en marcha de los digestores anaerobios / 282
CAPTULO 9. Manejo de lodos / 285
9.1. Introduccin / 285
9.2. Contenido de humedad y volumen de los lodos / 285
9.3. Lodos primarios y secundarios. Criterios de estimacin / 287
9.4. Espesamiento por gravedad / 290
9.5. Lechos de secado / 291
9.6. Volumen de agua producida / 293
ANEXO 1. Volmenes mnimos de muestras y criterios
para su conservacin / 295
ANEXO 2. Distribucin normal acumulativa / 296
Procesos para el tratamiento de aguas residuales / Carlos Menendez Gutierrez y Jess M. Prez Olmo
Ciudad de La Habana : Editorial Flix Varera - Editorial Universitaria, 2007. ISBN 978-959-16-0619-8
2
Indicadores de la Contaminacin
CAPTULO1
INDICADORES DE LA CONTAMINACIN
1.1. INTRODUCCIN
Generalmente el nivel de contaminacin de las aguas residuales no
se mide a partir del conocimiento de la concentracin de los distintos
constituyentes de un agua residual que pueden ser considerados conta-
minantes, sino determinando parmetros globales como son la demanda
bioqumica de oxgeno (DBO) y la demanda qumica de oxgeno (DQO)
entre otros. En ocasiones, fundamentalmente cuando se trabaja con
residuales industriales, las caractersticas de estos son tales que se
requiere conocer constituyentes especficos como los metales pesados,
tensoactivos, fenoles y otros.
En algunos casos, por ejemplo, para la determinacin de trazas de con-
taminantes orgnicos en aguas superficiales y para beber, se emplean tc-
nicas analticas especiales a saber la cromatografa y la espectrometra de
masas.
1.2. CONTENIDO DE SLIDOS
El conocimiento del contenido de slidos de un agua o agua residual aporta
valiosa informacin sobre sus caractersticas, lo cual permite conocer de forma
general su naturaleza y si estas requieren de algn tratamiento en particular que
facilite su remocin o eliminacin.
Cuando se habla de slidos contenidos en un agua residual se est haciendo
referencia a aquello que permanece como residuo despus de la evaporacin y
secado de la muestra a 103
o
C.
Los slidos en las aguas residuales pueden estar en forma suspendida, coloi-
dal y disuelta. Todos ellos a su vez son de naturaleza inorgnica u orgnica. La
fraccin orgnica de los slidos se determina mediante la prdida por ignicin a
600
o
C.
3
Indicadores de la Contaminacin
1.3. DEMANDA BIOQUMICA DE OXGENO (DBO)
La DBO es uno de los parmetros ms utilizados en la caracterizacin de
los contaminantes orgnicos. Esta determinacin brinda un estimado del oxge-
no disuelto requerido por los microorganismos en la degradacin de los com-
puestos biodegradables.
El mecanismo presente durante la prueba de laboratorio de la DBO es esen-
cialmente biolgico y no depende solo del oxgeno presente y utilizado en un
momento dado, sino tambin de la velocidad a la que este se consume, y por
tanto, del tiempo. As, es necesario fijar un tiempo y temperatura de incubacin
estndar, que normalmente es de cinco das, y 20 C como temperatura de refe-
rencia.
En muchos casos se necesita conocer la demanda bioqumica de oxge-
no total de un agua residual (DBO ltima). En tal sentido se requiere deter-
minar previamente la velocidad a la cual ocurre la reaccin en toda su
extensin.
La variacin en el tiempo de la demanda bioqumica de oxgeno de un
agua residual sigue una cintica de primer orden, segn se observa en la figu-
ra 1.2.
La determinacin de la DBO implica conocer el oxgeno consumido por
las bacterias en un intervalo de tiempo, ya que existe una relacin cuantita-
tiva entre el oxgeno consumido y la concentracin del material orgnico que
se transforma en dixido de carbono y amonaco. Esta relacin puede ser
representada por la siguiente ecuacin generalizada:
3
NH
2
H
2
3
2
2
CO
2
O
4
3
2 4
N O H C c O c
a
n c
b a
n
c
b
a n
+ + + +
|
.
|

\
|
|
.
|

\
|
(1.1)
La diferencia en el contenido de oxgeno al inicio y al final del anlisis,
por litro de muestra utilizada constituye la DBO de la muestra expresada en
mg de O
2
L
1
.
Lgicamente la velocidad y extensin a la cual ocurre esta degradacin
depende de la temperatura y del tiempo, de ah que se ha aceptado para
estos dos parmetros 20
o
C y cinco das respectivamente, tal como ya fue
expresado.
4
Menndez Gutirrez, C. y J. Prez Olmo
Fig. 1.2. Variacin en el tiempo de la DBO: a) remanente, b) satisfecha.
1.3.1. Cintica de la DBO
Los estudios cinticos sobre la DBO han indicado que para propsitos
prcticos esta puede ser considerada, con un buen grado de aproximacin,
como una reaccin de primer orden, lo cual se expresa matemticamente
mediante la ecuacin:
t
t
L k
dt
dL
' = (1.2)
donde:
L
t
: DBO remanente, mg L
1
.
t: tiempo, das.
k: constante de velocidad, da
1
.
Como toda constante de velocidad de reaccin, k
'
es funcin de la tempera-
tura. Aunque el anlisis tpico o estndar se realiza a 20
o
C el valor de k
'
puede
calcularse a otra temperatura mediante la relacin:
20
20

=
T
t
0 k k
(1.3)
en la que u = 1,056
La integracin de la ecuacin 1.2 conduce a:
t k
o
t k -
o t
L e L L
'
= = 10
(1.4)
5
Indicadores de la Contaminacin
donde:
L
o
: DBO ltima o total de la muestra, mg L
1
.
k: k/2,3
Tal como se aprecia en la ecuacin anterior, el conocimiento de la DBO
remanente de la muestra en un instante dado exige conocer L
o
y k. La determi-
nacin de L
t
no es lo ms comn en el trabajo rutinario. Generalmente lo que
ms interesa conocer es aquella parte de la demanda bioqumica de oxgeno que
se ha satisfecho en un intervalo de tiempo y a una temperatura dada. As, si L
o
es la demanda total o ltima de un agua residual, y L
t
es la demanda remanente
en un instante t dado, la demanda satisfecha o ya ejercida (Y) ser:
( )
t k
o t t o
L DBO L L Y

= = = 10 1
(1.5)
Para el caso del anlisis realizado a 20
0
C y cinco das:
k
o , d o
L DBO L L Y
5
C 20 5 5
10 1

= = = (1.6)
Estas expresiones describen los lugares geomtricos representados en la
figura 1.3.
Fig. 1.3. Relacin entre L
o
y L
t
.
Como ya fue expresado, es la DBO ejercida la que se determina
normalmente en el periodo de incubacin de cinco das y a la temperatura de
referencia de 20
o
C. El conocimiento de la DBO total (L
o
), se requiere solamente
en casos especficos y conlleva un tratamiento matemtico especial de los
datos experimentales.
Para que la muestra de agua residual sea analizada exitosamente es nece-
sario que contenga una determinada poblacin microbiana o en su defecto esta
6
Menndez Gutirrez, C. y J. Prez Olmo
sea suministrada, a fin de que la materia orgnica pueda ser degradada y tenga
lugar consumo de oxgeno.
Este aspecto reviste una gran importancia porque puede ocurrir que el agua
residual objeto de anlisis contenga bacterias del gnero Nitrosomonas y
Nitrobacter, las cuales no degradan la materia orgnica, sino que oxidan dife-
rentes formas del nitrgeno con el consiguiente consumo de oxgeno (fenmeno
este denominado nitrificacin), y por lo que se falsean los resultados. En las
siguientes ecuaciones se muestra la estequiometra del proceso de nitrificacin.

+
+
+ + +
3 2 2
2 2 2 3
2NO O 2NO
2H O 2H 2NO 3O 2NH
Siendo la reaccin total:
O 2H 2H 2NO 4O 2NH
2 3 2 3
+ + +
+
(1.7)
En aquellos casos en que se analizan efluentes de plantas de tratamiento
que utilizan filtros percoladores o lodos activados estn presentes las bacterias
nitrificantes, por lo que al realizar el anlisis de la DBO se debe aadir alguna
sustancia que las inhiba. El compuesto ms comnmente empleado para inhibir
el crecimiento de organismos nitrificantes es el alilthiourea (ATU).
Cuando estos tipos de bacterias estn presentes, la curva de la DBO puede
mostrar un comportamiento similar al representado en la figura 1.4.
Fig. 1.4. Efecto de la nitrificacin sobre la DBO.
Obsrvese que de estar presente el fenmeno de la nitrificacin la DBO de-
terminada resultar mayor que la que corresponde a la degradacin de la materia
orgnica, lo cual introduce un error por exceso con respecto a lo deseado.
7
Indicadores de la Contaminacin
1.3.2. Influencia de la constante de velocidad
en la determinacin de la DBO
La constante de velocidad desempea un rol importante en la fraccin de la
DBO que puede ser estimada en un periodo de cinco das. Cuanto mayor sea el
valor de esta constante, mayor ser el valor de la DBO determinada.
Esta influencia se puede apreciar en la figura 1.5 donde a manera de ejem-
plo se han considerado cuatro valores de k.
Fig. 1.5. Influencia de k sobre la DBO.
En la figura 1.5 puede apreciarse cmo para el mismo tiempo, por ejemplo
cinco das, la DBO ser distinta en funcin del valor de la constante de velocidad k.
La variacin que puede experimentar la constante k ha sido motivo de am-
plias discusiones, pero se pueden considerar como las principales causas:
La naturaleza de la materia orgnica.
La habilidad de los microorganismos para utilizar esa materia orgnica.
As por ejemplo, se puede sealar que en general la parte soluble es degra-
dada ms rpidamente que aquella que se presenta en forma coloidal o en sus-
pensin.
Otro aspecto de importancia en la determinacin de la DBO es la concen-
tracin de microorganismos que aporte el inculo, ya que de ser muy baja, la
DBO ejercida en el tiempo tambin ser baja debido a que las bacterias tardan
un tiempo apreciable en multiplicarse como para ejercer una oxidacin notable.
Este fenmeno se destaca en la figura 1.6. En el caso mostrado por la curva A,
la concentracin de microorganismos es tres rdenes de magnitud mayor que en
B. Esta es la razn por la cual este es un aspecto que debe ser tenido en cuenta
en el momento de realizar el anlisis de la DBO. En el caso de efluentes de
plantas de tratamiento o residual humano este aporte est garantizado de por s,
pudiendo no ocurrir lo mismo cuando la muestra tiene otra procedencia.
8
Menndez Gutirrez, C. y J. Prez Olmo
Fig. 1.6. Efecto de la concentracin del inculo sobre la DBO.
Otro factor no menos importante en la determinacin de la DBO es el grado
de aclimatacin de las bacterias utilizadas en el curso del anlisis para el sustrato
en particular que presenta la muestra. Por supuesto este fenmeno se presenta
cuando la muestra no contiene microorganismos y resulta necesario inocularlos.
En la figura 1.7 se muestra el caso de un inculo no adaptado al sustrato de
inters. De ah que se presente retraso en la DBO ejercida en la etapa inicial.
Por el contrario, en el ejemplo representado en la figura 1.8, se ilustra una situa-
cin en la que el inculo est perfectamente adaptado al sustrato y no se pre-
senta la fase inicial de retraso.
Fig. 1.7. Efecto de la aclimatacin del inculo sobre la DBO.
Se puede apreciar que las curvas que aparecen en las figuras 1.6, 1.7 y 1.8
presentan una tendencia que no se corresponde exactamente con lo que predice
una cintica de primer orden (figura 1.4), ya que las primeras presentan una
meseta ms o menos acentuada y que pudiera ser falsamente interpretado como
9
Indicadores de la Contaminacin
un proceso de nitrificacin. La explicacin de la aparicin de estas mesetas
pudiera ser la siguiente: durante el primer y segundo da de incubacin, la mate-
ria orgnica soluble es rpidamente oxidada y la remanente es convertida en
clulas. Ya en esta Iase aparece una 'meseta caracteristica de una reducida
velocidad de oxidacin, lo cual lleva asociada una fase de respiracin endgena.
Posteriormente se aprecia un incremento en la velocidad de oxidacin, que es
atribuible a un incremento en la poblacin de protozoos, los cuales son predadores
de las bacterias. La presencia y duracin de esta meseta entre estas fases
depende del intervalo de tiempo entre el pico de la poblacin bacteriana y el de
la poblacin de protozoos.
Fig. 1.8. Efecto sobre la DBO de un inculo adaptado.
Se ha comprobado que mientras menor sea la constante de autooxidacin
de la biomasa, la meseta ser ms acentuada. Por otra parte, mientras sean los
valores mayores de esta constante la meseta tiende a no aparecer.
Otro aspecto importante relacionado con la DBO est en el hecho de que
por mucho tiempo se pens que despus de un largo periodo de incubacin (por
ejemplo, 20 das) la DBO determinada era la DBO ltima o total de la muestra
(L
o
), y que al mismo tiempo era igual a la DQO terica, cuando en realidad
existe discrepancia entre el valor determinado y la demanda terica de oxgeno.
El siguiente ejemplo pondr en evidencia esta discrepancia.
La oxidacin de la glucosa a dixido de carbono y agua requiere de 192 g de
oxgeno por mole de glucosa (180 g/mol) 1,065 mg de oxgeno por miligramo
de glucosa segn la estequiometra de la ecuacin 1.8:
O H 6 CO 6 0 6 O H C
2 2 2 6 12 6
+ +
(1.8)
As, una solucin de glucosa de 300 mg L
1
tiene una demanda terica de
oxgeno de 320 mg L
1
, sin embargo la determinacin de la DBO con periodos
10
Menndez Gutirrez, C. y J. Prez Olmo
de incubacin de 20 das ha reportado valores de L
o
en el intervalo de 250 a
285 mg L
1
. De aqu resulta evidente que no toda la glucosa ha sido oxidada a
dixido de carbono y agua, interpretndose en los siguientes trminos este fen-
meno: en el curso de la degradacin de la glucosa las bacterias obtienen energa
para su crecimiento y reproduccin, lo cual implica que parte de la materia
orgnica es convertida en tejido celular, por lo que no experimenta oxidacin
completa, hasta que ocurre la respiracin endgena. Cuando las bacterias mue-
ren este tejido celular se convierte en alimento de otras y prosigue la conversin
de la materia orgnica en dixido de carbono y agua. Una parte de las bacterias
vivas y muertas sirven de alimento a los protozoos y contina la degradacin de
la materia orgnica.
Fig. 1.9. Secuencia de degradacin de los compuestos orgnicos.
En cada una de las etapas por las que transcurre la oxidacin de la materia
orgnica se producen desechos del metabolismo que resultan resistentes a la
degradacin biolgica y que pueden explicar la discrepancia entre la demanda
terica y prctica. En el diagrama de la figura 1.9 se ilustra esta secuencia de
fenmenos.
1.3.3. Clculo de la constante de oxigenacin
A partir de la importancia que existe, a la hora de caracterizar un agua
residual, del conocimiento de la relacin entre la DBO cinco das, DBO ltima y
la constante de velocidad, k, han sido propuestos muchos mtodos para el
11
Indicadores de la Contaminacin
clculo de k.
1, 3
A continuacin se presenta uno que, aunque no muy exacto, es
sencillo y permite tener una idea del valor de esa constante antes de pasar a una
determinacin ms exacta, pero tambin ms laboriosa.
El mtodo de Rhame para determinar k, o mtodo de los dos puntos de
Rhame,
4
se basa en la relacin entre los valores de la DBO para dos tiempos
dados. El segundo tiempo es el doble del primero. Con el mtodo de Rhame
pueden calcularse la DBO ltima y la constante de de-oxigenacin:
( )
Z X
X
L

=
2
2
(1.9)
y
X Z
X
t T
k

= log
1
(1.10)
donde:
L: DBO ltima.
X: DBO en el tiempo t.
Z: DBO en el tiempo T = 2t.
Segn Rhame, para t = 10 das, los valores de L y k que se obtienen son
comparables a los que se estiman cuando se usan otros mtodos.
En la tabla 1.1 se comparan los valores de la DBO ltima y la constante de
de-oxigenacin para residuales teneros, calculados por diferentes mtodos.
Tabla 1.1. Valores de k y L
o
de residuales teneros calculados por diferentes
mtodos
1.4. DEMANDA TERICA DE OXGENO (DTO)
Y DEMANDA QUMICA DE OXGENO (DQO)
Cualquier compuesto orgnico tericamente puede ser oxidado hasta la ob-
tencin final de productos estables como H
2
O , CO
2
, NH
3
y
2
4
SO
. La cantidad
12
Menndez Gutirrez, C. y J. Prez Olmo
de oxgeno necesaria para la oxidacin se denomina demanda terica de oxge-
no (DTO). La DTO puede ser calculada considerando las relaciones
estequiomtricas:
O H
2
CO O
2 4
O H C
2 2 2
y
x
z y
x
z y x
+ |
.
|

\
|
+ +
(1.11)
MM
2 4
DTO
z y x + +
= (1.12)
donde:
MM: Masa molecular del compuesto.
Para compuestos que contienen N en su estructura, la ecuacin para la
demanda terica de oxgeno es:
3 2
2 2
NH O H
2
3
CO O
2 4
3
N O H C
p
p y
x
z p y
x
p z y x
+

+ |
.
|

\
|

+ +
(1.13)
( )
MM
8 2 3 4
DTO
z p y x +
= (1.14)
Si adems el azufre participa en su composicin,
4 2 3 2 2
2
SO H NH O H
2
3
CO
O
2
2
4
2 3
S N O H C
q p
p y
x
z
q
q p y
x
q p z y X
+ +

+
+ |
.
|

\
|
+

+ +
(1.15)
8
MM
2 3 6 4
DTO |
.
|

\
| + +
=
z p q y x
(1.16)
La demanda qumica de oxgeno brinda una medida ms real de la cantidad
de oxgeno requerida para la oxidacin de los compuestos orgnicos a CO y
H
2
O, de acuerdo con las ecuaciones 1.11, 1.13 y 1.15, cuando se utiliza un
oxidante fuerte.
2
13
Indicadores de la Contaminacin
El mejor mtodo para determinar la DQO es aquel que arroja valores igua-
les o cercanos a la DTO. Actualmente este mtodo es el estndar del dicromato.
Este mtodo utiliza una solucin de dicromato de potasio cuya concentracin
molar de equivalente es 0,25, en 50 % de cido sulfrico. Bajo estas condicio-
nes, con sulfato de plata como catalizador, la mayora de los compuestos son
oxidados entre 90 y 100 %.
6
No obstante, algunos compuestos no son oxidados o lo son solo parcialmen-
te cuando se utiliza el mtodo estndar con dos horas de reflujo. Esto se ha
comprobado en algunas bases fuertes nitrogenadas como las metilaminas,
etilaminas, piridinas y sus derivados (tabla 1.2).
Tabla 1.2. Algunos compuestos parcialmente oxidables por el dicromato
en medio cido
Otras bases heterocclicas nitrogenadas como la quinolina, pirol, pirolidina,
indol, inidiazol, purina y pirimidina, son oxidadas fcil y completamente. Sin em-
bargo, en los compuestos heterocclicos que contienen dos o ms tomos de
nitrgeno en la molcula, solamente una parte de ellos se desprende como amo-
naco y el resto como nitrgeno.
7
El azufre de los compuestos orgnicos es oxidado a sulfato. La excepcin
est constituida por el grupo CH
3
S que es oxidado a acido metilsulIonico.
Durante la prueba de la DQO se determinan tanto los compuestos
degradables como los no degradables biolgicamente. Tambin se incluyen aque-
llos compuestos inorgnicos reductores que pueden ser oxidados por el dicromato
en medio cido, como es el caso comn del anin cloruro.
1.5. CARBONO ORGNICO TOTAL (COT)
Hoy da el COT es utilizado como una medida del contenido de sustancias
orgnicas en el agua. Sin embargo, este parmetro no brinda informacin acer-
ca del oxgeno necesario para la oxidacin de las mismas. As, por ejemplo, la
Compuesto Oxidacin
%
Compuesto Oxidacin
%
Metilamina
Dimetilamina
Trimetilamina
Piridina
metil piridina
3 metil piridina
3,7
1,9
5,2
4,4
30,0
27,8
Etilamina
Dietilamina
Trietilamina
2,4 dimetilpiridina
2,4,6 trimetilpiridina
36,1
27,8
37,4
58,0
85,2
5
14
Menndez Gutirrez, C. y J. Prez Olmo
glucosa y el cido caproico poseen en su molcula el mismo nmero de tomos
de carbono, no obstante la demanda terica de oxgeno (DTO) de ambos com-
puestos es diferente:
1
2 2 2 6 12 6
gg 1,07 DTO O 6H 6CO 6O O H C

= + +
(1.14)
1
2 2 2 2 12 6
gg 2,21 DTO O 6H 6CO 8O O H C

= + +
(1.15)
En el primer caso se requieren 2,66 g de oxgeno para la oxidacin de 1 g de
carbono. En el segundo caso cada gramo de carbono necesita 3,55 g de oxgeno
para la oxidacin total.
En el orden terico existe una relacin entre la DTO y DQO, as como entre
la DTO y el COT.
1
Por tanto, existe tambin una relacin entre la DQO y el
COT. En el mismo plano terico cabe suponer que la relacin DTO/COT vara
entre los lmites de cero, cuando los compuestos orgnicos no son oxidables por
el dicromato, hasta 5,33, que es la relacin para el metano segn la ecuacin:
O H 2 CO O 2 CH
2 2 2 4
+ + (1.16)
En la tabla 1.3 se representa esta relacin para algunos compuestos orgni-
cos comunes.
Tabla 1.3. Relacin DTO/COT de algunos compuestos
8
Compuesto Relacin DTO / COT
cido actico
Glucosa
Sacarosa
Piridina
Benceno
Etilamina
Metano
2,67
2,67
2,67
3,33
3,33
4,00
5,33
El valor lmite de 5,33 para la relacin DTO/COT es tambin vlido para
mezclas de compuestos, aunque en la medida que aumenta en complejidad la
composicin del residual, la DTO va perdiendo su inters prctico y la relacin
DTO/COT llega a ser inoperante.
Para las aguas residuales, constituidas por mezclas de sustancias cuya com-
posicin no siempre se conoce y de concentracin variable, la DTO tiene esca-
so valor prctico. De ah la importancia de la DQO y la DBO como parmetros
indicadores de la contaminacin en unidades de oxgeno.
15
Indicadores de la Contaminacin
En la prctica la relacin DQO/DTO es muy variable debido a que, como ya
se dijo, hay muchos compuestos orgnicos que no son fcilmente oxidados por
el dicromato. La relacin puede ser tan alta como 1 (para el etanol, cido oxlico
y cido benzoico)
9
y tan baja como 0,02 para la piridina.
8
Por tanto, las relacio-
nes DTO/COT que aparecen en la tabla 1.3 para compuestos puros, no son
igualmente vlidas para DQO/COT.
Hay una correspondencia entre las relaciones anteriores y el nmero de
oxidacin promedio (NOP) del carbono (figura 1.10).
Este puede calcularse por la ecuacin:
C
O
2
2 4 NOP
n
n
=
(1.17)
donde:
2
O
n
: Nmero de tomos de oxgeno necesarios para la oxidacin completa
de un compuesto dado en CO
2
, H
2
O, NH
3
y H
2
SO
4
.
n
C
: Nmero de tomos de carbono en el compuesto dado.
Fig. 1.10. Dependencia del NOP de la relacin DTO/C
org
para algunos
compuestos.
16
Menndez Gutirrez, C. y J. Prez Olmo
Cuando se aplica la ecuacin 1.17 a las molculas de glucosa, formaldehdo
y cido actico, puede llegarse a la conclusin que en esos compuestos el car-
bono tiene el NOP igual a cero.
El carbono en todas las aminas alifticas y alcoholes tiene el NOP igual
a 2. En el acido caproico es de 1,33 y en el acido citrico de 1.
Para los compuestos puros, la ecuacin 1.17 se transforma en:
org
C
DTO
2 4 NOP =
(1.18)
Para aguas residuales la ecuacin 1.18 se modifica:
12 / C
16 / DQO
2 4 NOP
org
=
org
C
DQO
2
3
4 NOP =
(1.19)
en la que tanto la DQO como la concentracin de carbono orgnico se
expresan en mg L
1
.
En el mismo plano terico puede afirmarse que cuando el NOP del
sustrato es menor que cero se hace necesario suministrar oxgeno para la
sntesis celular:
( )
O H 18 NO H C 6 O 10 NH 6 O H C 5
2 2 7 5 2 3 2 12 6
n n n n
n
+ + +
(1.20)
Para sustratos cuyo NOP es mayor que cero, el contenido de oxgeno en su
molcula es tal que no requiere un suministro adicional del mismo para que
ocurra la sntesis celular:
( )
2 2 2 7 5 3 7 8 6
O 7,5 O H 8 NO H C 6 NH 6 O H C 5 n n n n
n
+ +
(1.21)
Como se aprecia de la ecuacin 1.21 an queda una cierta fraccin del
oxgeno que puede ser utilizado para la oxidacin.
Debe destacarse que cuando la biomasa es considerada como sustrato, su
NOP es igual a cero:
( )
3 2 2 2 2 7 5
NH O H 2 CO 5 O 5 NO H C n n n n
n
+ + + (1.22)
17
Indicadores de la Contaminacin
1.6. NITRGENO
El nitrgeno en las aguas residuales se puede presentar principalmente en
forma orgnica, formando parte de protenas, y en forma amoniacal. El nitrito
(NO
2

) y los nitratos (NO


3

) rara vez aparecen en las aguas residuales crudas,


y cuando existen, se trata fundamentalmente de aguas residuales industriales.
El conocimiento del contenido de nitrgeno en sus variadas formas re-
sulta de gran inters. As por ejemplo, cuando un agua destinada a consumo
humano presenta nitrgeno orgnico, o nitrgeno amoniacal, es indicio de
contaminacin fecal reciente, lo cual es un alerta sobre su peligrosidad. La
urea contenida en la orina del hombre y animales, se descompone aportando
nitrgeno amoniacal segn muestra la siguiente ecuacin:
3 2 2 4 2
2NH CO O H H CON + +
(1.23)
As mismo, las heces fecales de animales contienen cantidades apreciables
de protenas las cuales son convertidas en nitrgeno amoniacal por las bacterias
segn:
3
NH Bacterias orgnico Nitrgeno +
(1.24)
El nitrgeno amoniacal puede ser posteriormente convertido en iones nitrito
por las bacterias Nitrosomonas:
O 2H 2H 2NO 3O 2NH
2 2
bacterias
2
(g)
3
+ + +
+
(1.25)
La presencia de iones nitrito en un agua destinada al consumo humano
puede ser considerada tambin como indicativo de contaminacin fecal.
Esta secuencia de reacciones no termina sino en los iones nitrato, que
constituyen la forma ms oxidada del nitrgeno. Esto ocurre cuando los iones
nitrito son oxidados por las Nitrobacter segn:

+
3
bacterias
2 2
2NO O 2NO
(1.26)
La secuencia de la nitrificacin se puede mostrar grficamente, tal como
indica la figura 1.11.
La presencia de diferentes formas del nitrgeno tiene otra connotacin cuando
se trata de aguas residuales. En este caso es importante conocer el contenido
de nitrgeno por ejemplo cuando se requiere disear un sistema de tratamiento
biolgico donde se debe cumplir que el contenido de nitrgeno guarde relacin
definida con la DBO y el fsforo, para que el proceso de depuracin transcurra
de manera adecuada.
18
Menndez Gutirrez, C. y J. Prez Olmo
Fig. 1.11. Transformacin de diferentes formas del nitrgeno.
Por otra parte el contenido de nitrgeno y las diferentes formas en que se
puede presentar a lo largo del sistema de tratamiento aporta criterio sobre algu-
nos de los fenmenos que pueden ocurrir.
En la tabla 1.4 se resume el impacto de diferentes formas del nitrgeno.
Tabla 1.4. Impactos causados por diferentes especies del nitrgeno
Forma del
Nitrgeno
Impacto
N-NH
3
La oxidacin del nitrgeno amoniacal en
un curso receptor produce disminucin
del oxgeno disuelto.
El nitrgeno amoniacal no ionizado
puede causar toxicidad en varios
organismos acuticos.
N-NO
3
El nitrgeno en forma de nitrato en aguas
potables puede causar metahemoglo-
binemia en nios pequeos.
Nitrgeno total El nitrgeno como nutriente es causa de
un crecimiento excesivo de organismos
acuticos que consumen oxgeno durante
la noche y excretan sustancias que causan
olor y sabor.
19
Indicadores de la Contaminacin
1.7. FSFORO
El conocimiento del contenido de fsforo de las aguas residuales reviste
inters, pues este elemento constituye un factor imprescindible para la vida
de los organismos acuticos al entrar a formar parte de su estructura.
La presencia de compuestos de fsforo en cursos receptores induce el
crecimiento de algas. Estas afectan de forma notable la calidad de las aguas
ya que pueden ser el origen de toda una secuencia de fenmenos, dado que
este elemento es el limitante para el desarrollo de estas formas de vida. La
luz y los compuestos de nitrgeno (los cuales son los otros elementos im-
prescindibles) son generalmente abundantes.
El deterioro de la calidad del agua debido a la aparicin de color o sabor
desagradables no es el nico problema que se genera como consecuencia de
la presencia del fsforo en un cuerpo de agua, sino que se produce otro no
menos grave y es que al morir las algas, estas ocasionan una contaminacin
de carcter orgnico en el agua.
Por todo lo anterior es importante disponer de datos sobre la concentra-
cin de fsforo que se vierte a los cuerpos receptores, provenientes funda-
mentalmente de las aguas residuales industriales y domsticas.
Las aguas residuales de origen domstico son relativamente ricas en
fsforo. Antes del desarrollo de los detergentes sintticos, el contenido
de fsforo usualmente estaba en el intervalo de 2 a 5 mgL
1
y las formas
orgnicas variaban de 0,5 a 1,0 mgL
1
. Esta contribucin humana se debe a
la ruptura de las protenas con la consiguiente eliminacin del fsforo en la
orina. En la actualidad muchos detergentes sintticos contienen grandes
cantidades de poli fosfatos, pudiendo contener hasta 10 % en su composi-
cin. La sustitucin del jabn por estos productos ha incrementado notable-
mente el contenido de este elemento en las aguas residuales.
20
Menndez Gutirrez, C. y J. Prez Olmo
1.7.1. Diferentes compuestos de fsforo presentes en las aguas
residuales
En las aguas residuales se pueden encontrar compuestos de fsforo tales como:
Tabla 1.5. Diferentes compuestos de fsforo
Todos los polifosfatos experimentan reacciones qumicas produciendo
ortofosfatos (PO
4
3
), siendo la velocidad con que ocurre este proceso de-
pendiente del tiempo, la temperatura y el pH.
1.8. CARACTERSTICAS DE LAS AGUAS RESIDUALES
1.8.1. Aguas residuales municipales
Estn constituidas por materia orgnica en forma suspendida, coloidal y
disuelta. La carga contaminante de estas aguas puede ser expresada en
funcin de la contribucin per cpita.
La DBO, contenido de slidos y el gasto de agua per cpita que normalmen-
te se reporta en la literatura vara de un pas a otro, de una zona o regin a otra
e incluso con el tiempo en una misma localizacin. Esto est, por supuesto, en
funcin de los hbitos alimentarios, de las costumbres y condiciones higinico-
sanitarias de la poblacin.
Los valores tpicos de la contribucin per cpita diaria a los residuales muni-
cipales son 0,5 m
3
hab
1
, 104 g hab
1
de DBO y un contenido de slidos suspen-
didos de 90,7 g hab
1
.
10
Estudios parciales realizados en Cuba arrojan valores
de 30 g hab
1
para la DBO y de 24 g hab
1
para los slidos suspendidos.
11
Como referencia, la composicin promedio de un agua residual munici-
pal se muestra en la tabla 1.6. La incorporacin de residuales industriales
pueden modificar sustancialmente los valores de estas concentraciones.
12
Nombre Frmula
Ortofosfatos
Fosfato trisdico Na
3
PO
4
Fosfato disdico Na
2
HPO
4
Fosfato monosdico NaH
2
PO
4
Fosfato diamnico (NH
4
)HPO
4
Polifosfatos
Hexametafosfato de sodio Na
3
(PO
3
)
6
Tripolifosfato de sodio Na
5
P
3
O
10
Pirofosfato tetrasdico Na
4
P
2
O
7
21
Indicadores de la Contaminacin
Tabla 1.6. Caractersticas promedio de un residual municipal
1.8.2. Caractersticas de los residuales industriales lquidos
(RIL)
Para tener un conocimiento real de las caractersticas de un agua resi-
dual no basta con conocer la magnitud, en trminos de concentracin, de los
parmetros indicadores seleccionados. Si bien el conocimiento de las con-
centraciones puede ser til en determinadas circunstancias, una mayor in-
formacin se obtiene del conocimiento del flujo msico (kgd
1
).
As, por ejemplo, suponiendo dos residuales con las siguientes caracte-
rsticas:
Residual A: DBO
5
= 500 mgL
1
Flujo = 200 m
3
d
1
.
Residual B: DBO
5
= 250 mgL
1
Flujo = 600 m
3
d
1
Pudiera pensarse que el residual A es ms contaminante que el B debido a
que tiene mayor concentracin de DBO. Obviamente el residual A presenta una
DBO superior al B, sin embargo el valor de su flujo o gasto es menor. El produc-
to de la concentracin por el flujo da el valor del flujo msico. De esta manera,
para el ejemplo anterior se tiene que:
Flujo msico de A = 100 kgd
1
Flujo msico de B = 150 kgd
1
Este simple ejemplo pone de manifiesto que el conocimiento de la concen-
tracin de un parmetro solamente no es suficiente para tener una idea real del
poder contaminante de un agua residual.
Lo anterior destaca la importancia de conocer, con el mayor grado de certe-
za posible, el flujo de agua residual que como promedio se genera. Por lo que se
debe utilizar un procedimiento de muestreo que responda en el mayor grado
Propiedad Mximo Promedio Mnimo
pH 7,5 7,2 6,8
Slidos totales (mg L
-1
) 640 453 322
Slidos totales voltiles (mg L
-1
) 503 340 225
Slidos suspendidos (mg L
-1
) 258 145 83
Slidos suspendidos voltiles
(mg L
-1
)
208 120 62
DQO (mg L
-1
) 436 288 159
DBO (mg L
-1
)
276 158 75
Cloruros (mg L
-1
) 45 35 25
22
Menndez Gutirrez, C. y J. Prez Olmo
posible a la realidad. Es decir, las determinaciones que se realicen deben ser
representativas, para lo cual estas deben ser proporcionales al flujo.
Para llegar a disponer de muestras integradas o compuestas, se requiere
que cada una de las tomas puntuales que constituirn la compuesta, sea obteni-
da teniendo en cuenta tanto el flujo promedio de agua residual como el existente
en el instante en que se toma la muestra. El flujo promedio debe ser calculado
previamente.
Algunas de las caractersticas qumicas de las aguas residuales municipales
reportadas por Walter
13
aparecen resumidas en la tabla 1.7.
Para estimar el volumen de muestra a extraer en cada momento, puede
utilizarse la expresin:
i
T
= i Q
N Q
V
V

(1.27)
donde:
Vi: volumen de muestra a tomar en un instante (L).
V
T
: volumen total de muestra a recolectar (L).
N: nmero de muestras que se van a tomar.
Q : flujo promedio determinado previamente (Lh
1
).
Q
i
: flujo puntual en el instante de la toma de muestra (Lh
1
).
En la aplicacin de este procedimiento es recomendable que durante el pe-
riodo de muestreo se tomen volmenes de muestras, se conserven en fro y se
mida el flujo puntual en el instante de tomar cada muestra.
Tabla 1.7. Caractersticas qumicas de las aguas residuales municipales
Constituyentes Tipo Concentracin
cidos voltiles Frmico, actico,
propinico, butrico
y valrico
(8,5-20) mg L
-1
cidos solubles
no voltiles
Lctico, gliclico,
ctrico y benzoico
(0,1-1,0) mg L
-1
cidos grasos
superiores
Palmtico, esterico y
oleico
60 % del contenido
de cidos grasos
Protenas y
aminocidos
Al menos veinte
tipos
(45-50 %) del
nitrgeno total
Carbohidratos Glucosa, lactosa,
sacarosa

23
Indicadores de la Contaminacin
La caracterizacin de un agua residual reviste dos objetivos muy impor-
tantes:
1. Inmediato. El conocimiento de las caractersticas del agua residual per-
mite conocer su poder contaminante y decidir si el residual
debe ser sometido a tratamiento o no.
2. Mediato. Cuando las caractersticas del agua residual son tales que
es obligado su tratamiento, los datos que aporta la caracte-
rizacin son una parte imprescindible para el diseo de la
instalacin de tratamiento.
En el curso de un proceso de caracterizacin se debe tener la mirada
puesta en la posibilidad de estudiar separadamente las diferentes corrientes
de residuales que pueden existir a fin de valorar su posible segregacin,
pues dependiendo de sus caractersticas pueden requerir de tratamientos
diferentes, por lo que no es aconsejable su mezcla. Esta ptica est, por
supuesto, condicionada en buena medida al estudio o conocimiento previo
del problema.
Para el caso particular de los residuales industriales lquidos (RIL), un
procedimiento para conocer el balance material y de flujo de todos los pro-
cesos de la industria que utilizan agua y producen residuales, as como de la
industria en su conjunto, puede resultar imprescindible.
El resultado de la caracterizacin debe brindar informacin acerca de la
fortaleza contaminante de cada corriente, as como de las alternativas de
tratamiento y de su reuso.
El procedimiento general recomendado para obtener la informacin ne-
cesaria para la caracterizacin con un mnimo de esfuerzo, y que al mismo
tiempo sea confiable, puede resumirse en cuatro etapas:
obtener el diagrama de flujo de los residuales,
elaborar el esquema de muestreo y anlisis,
efectuar el balance de flujo y materiales,
reportar la variacin estadstica de los parmetros ms significativos
de la caracterizacin.
1.8.2.1. Diagrama de flujo de los residuales
Este diagrama se construye a travs de la inspeccin de todas las operacio-
nes del proceso y con la consulta del ingeniero de la planta. El diagrama debe
indicar los posibles puntos de muestreo y el orden de magnitud del flujo de la
corriente de aguas residuales.
24
Menndez Gutirrez, C. y J. Prez Olmo
1.8.2.2. Elaboracin del esquema de muestreo y anlisis
El muestreo ptimo es el continuo, con volmenes que son una fraccin del
flujo promedio. Sin embargo, esto pocas veces es factible.
Otra posibilidad es la de las muestras integradas o compuestas con una
frecuencia de muestreo que se establece de acuerdo con el proceso que se
necesita caracterizar. El periodo de la composicin de la muestra y la frecuen-
cia de muestreo se establece de acuerdo con la naturaleza del proceso cuya
agua residual se estudia. Algunos procesos en operacin continua pueden
muestrearse cada hora e integrarse por periodos de 8, 12 y hasta de 24 h. Aque-
llos procesos en los que hay mucha variacin deben integrarse en periodos de
una o dos horas. A los procesos a templa se les toma muestras en el periodo de
vaciado.
Los anlisis a realizar dependen de las caractersticas del parmetro que se
est midiendo. Por ejemplo, el pH debe medirse puntualmente, ya que es posible
en muchos casos que la integracin resulte en una neutralizacin de elementos
cidos y bsicos perdindose durante la integracin una informacin valiosa. La
carga de DBO puede requerir integracin de las muestras durante 8 h cuando
se pretende disear tratamientos de tiempos de retencin cortos, aunque mues-
tras integradas durante 24 h resultan suficientes para tratamientos de varios
das de retencin como el de las lagunas.
Cuando se miden constituyentes como nitrgeno y fsforo para determinar
si hay necesidad o no de adicin de nutrientes en tratamientos biolgicos, son
suficientes muestras integradas de 24 h, ya que los sistemas biolgicos tienen
alta capacidad de amortiguamiento. Si se conoce la presencia de sustancias
txicas en el agua residual, estas deben ser muestreadas continuamente.
Un aspecto importante en este punto, al que ya se hizo referencia, es el de la
medicin del flujo.
1.8.2.3. Balance de flujo y materiales
Despus de realizados el muestreo y los anlisis, debe efectuarse el balance
de flujo y materiales tomando en consideracin todas las fuentes de contamina-
cin que sean significativas. En la medida en que el balance coincida con el
efluente total, se confirmar el cuidado con el que se realiz la caracterizacin.
1.8.2.4. Variacin estadstica de los parmetros
Esta informacin se obtiene de diversas formas, una de ellas consiste en
graficar en papel de probabilidades la frecuencia de ocurrencia.
25
Indicadores de la Contaminacin
La data de la caracterizacin de los RIL generalmente es muy variable y
susceptible de anlisis estadstico. La informacin puede ser reportada en tr-
minos de frecuencia de ocurrencia de una propiedad en particular, que es el
valor que se espera que tenga esa propiedad o lo exceda 10, 50 y 90 % del
tiempo. Los valores obtenidos en la caracterizacin se ordenan en orden cre-
ciente de magnitud. Por ejemplo, si se trata de la DBO,
donde:
n: Nmero total de valores de DBO.
m: Nmero consecutivo y en orden creciente asignado a cada uno de los
valores de DBO, desde uno hasta n.
1 + n
m
: posicin de cada valor en la tabla y que es equivalente al porcentaje
de ocurrencia.
Tabla 1.8. Porcentaje de ocurrencia
Al graficar estos resultados en papel de probabilidades, tal como se hace en
la figura 1.12, la DBO promedio es la que corresponde a frecuencia de ocurren-
cia del 50 %. Por otro lado, aceptando que 68 , 27 % de los valores estn
incluidos en el intervalo:
1

n
_
s X
, se tiene que:
2
9 15 1 84
1
, ,
n
X X
s

=

(1.28)
Para el ejemplo anterior:
DBO promedio = 347,8 mg L
1
m DBO
(mg L
-1
)
1 + n
m Ocurrencia
(%)
1 175 0,1 10
2 225 0,2 20
3 285 0,3 30
4 340 0,4 40
5 350 0,5 50
6 375 0,6 60
7 405 0,7 70
8 460 0,8 80
9 515 0,9 90
26
Menndez Gutirrez, C. y J. Prez Olmo
110
2
230 450
1
=

=
n
s
s: desviacin estndar.
Fig. 1.12. Porcentaje del tiempo que la DBO ser igual o menor al valor
indicado.
Cuando el nmero de observaciones es muy grande (de cincuenta a cien) se
recomienda agrupar la data por incrementos. Estos incrementos pueden tomar-
se de 50 mg L
1
.
Ejemplo 4.1
Suponga una data que tiene noventa y seis observaciones agrupadas por
intervalos de 50 mg L
1
.
donde:
n: Nmero total de muestras
m: Nmero de muestras en el intervalo analizado y los anteriores
100
1 + n
m
: posicin al graficar..
27
Indicadores de la Contaminacin
Tabla 1.9. Intervalos y nmeros de muestras
Las mediciones del caudal de aguas residuales durante el estudio pueden
llevarse a cabo por diversos mtodos, en dependencia de la accesibilidad del
lugar. Estos pueden ser, entre otros:
1. Instalacin de vertederos, se usa en canales o conductoras abiertos.
2. Cubo o cubetas y cronmetro: aplicable cuando hay bajo caudal.
3. Medicin de la duracin del bombeo: hay que hacer uso de la curva carac-
terstica de la bomba.
4. Uso de medidores de flujo de paleta.
1.8.3. Empleo de ndices en la caracterizacin
Tomando en cuenta que los procedimientos de caracterizacin son gene-
ralmente complejos y exigen de importantes recursos de tiempo y de tipo
financiero, la utilizacin de indicadores o ndices de produccin y consumo
se presenta como un importante enfoque a considerar, dentro de los proce-
dimientos utilizados para determinar, al menos aproximadamente, los niveles
de carga contaminante que son generados por las diferentes actividades de
produccin y servicios.
Los ndices de produccin constituyen un instrumento que permite determi-
nar la efectividad de medidas y programas orientados a la disminucin de la
contaminacin, facilitando la identificacin y establecimiento de prioridades de
gestin, con arreglo a la evaluacin cuantitativa de las tendencias que se mani-
fiestan en la calidad del medio.
En este sentido, es importante indicar que esta herramienta de trabajo
no sustituye ni exime, la necesidad de realizar investigaciones ms com-
pletas.
Intervalo Muestra
en el intervalo
m Posicin
en el grfico
200 - 249 5 5 5,2
250 - 299 4 9 9,3
300 -349 6 15 15,5
. . . .
. . . .
. . . .
1150 1 199 4 96 99,0
28
Menndez Gutirrez, C. y J. Prez Olmo
Tabla 1.10. Indicadores de carga contaminante de algunos procesos
PVS: Peso vivo sacrificado.
Fuente: Evaluacin rpida de fuentes de contaminacin de aire, agua y suelo, OMS, Mxico, 1988.
El indicador de carga contaminante o ndice es la relacin entre el nivel
de produccin y la carga contaminante que se genera en esa actividad. El
ndice define una cantidad de sustancia determinada que se obtiene en la
actividad productiva en un tiempo considerado, refiriendo a l la contamina-
cin generada durante el proceso de obtencin.
Los ndices han de ser el resultado de experiencias anteriores, sean pro-
pias u obtenidas de la literatura.
En la tabla 1.10 se presentan ejemplos de los ndices de produccin refe-
ridos a la contaminacin que generan.
De igual manera puede ser de importancia el ndice de consumo de agua.
Ejemplos de este tipo de ndice se reflejan en la tabla 1.11.
Proceso Unidad
DBO
(kg/u)
DQO
(kg/u)
Slidos
suspendidos
(kg/u)
Instalaciones de
cra de reses
250 - 1716
Instalaciones de
cra de pollos
1,4 - 14,6
Granjas lecheras
cabezas
539 - -
Mataderos 6,4 - 5,2
Sin recuperar
sangre
11 - -
Sin recuperar
panza se agrega
PVS
4,7 - -
Proceso de aves
de corral
1000 aves 11,9 22.4 12,7
Enlatado de
frutas y verduras
12,5 - 4,3
Refinacin de
aceite vegetal
t de
produccin
12,9 21 16,4
Produccin de
vino
m
3
de vino 0,26 - -
Produccin de
cerveza
m
3
de
cerveza
8,6 - 14,7
29
Indicadores de la Contaminacin
Tabla 1.11. Consumo de agua de la industria azucarera y derivados
Fuente: Programa integral de desarrollo de los derivados de la caa de azcar. Subprograma: Alcohol
y levaduras, septiembre 1985, Cuba.
1.9. MEDICIONES DEL CAUDAL
El conocimiento del gasto, flujo o caudal es importante no solo en el
momento de proyectar una instalacin, su sistema de tuberas y de bombeo.
Se encuentran diversas formas comunes de conocer un caudal. Desde
trabajar por datos de proyecto, pasando por el manejo de ndices de consu-
mo por unidad de produccin o por habitante, hasta la medicin directa. De
las mediciones directas del caudal de aguas residuales trata este epgrafe.
Los medidores de caudal ms usuales que pueden ser encontrados en
las plantas de tratamiento de aguas residuales son:
Canales de aforo Parshall.
Vertedores.
Medidores de caudal en lneas de presin.
1.9.1. Canal Parshall
El canal Parshall est formado por una seccin de entrada de paredes
verticales convergentes y fondo a nivel, un estrechamiento de paredes para-
lelas y fondo descendente. El canal posee adems una seccin de salida de
paredes divergentes y fondo ascendente. En la figura 1.13 se muestra un
esquema de un canal Parshall.
Este tipo de medidor es especialmente til para aguas que contienen slidos
en suspensin, debido a que la presencia de estos no afecta las mediciones. Por
otra parte, el aumento de la velocidad del agua a su paso por la parte ms
estrecha del canal, la garganta, dificulta la sedimentacin de las partculas.
La determinacin del caudal se realiza considerando la altura alcanzada
por el agua (H), tomada en una arqueta aneja conectada por un tubo
piezomtrico.
Industria ndice de consumo
Central azucarero 0.5 m
3
/t caa molida
Destilera 1.6 m
3
/HL de alcohol
70 m
3
/t de levadura
(tecnologa francesa)
Levadura
80 m
3
/t levadura
(tecnologa austraca)
30
Menndez Gutirrez, C. y J. Prez Olmo
El caudal instantneo en cada momento se obtiene por aplicacin de la
ecuacin:
( )
0,026
1,567
3,28 0,37
W
H Q =
(1.29)
siendo:
Q: caudal instantneo (m
3
s
1
).
W: ancho del estrechamiento, (m).
H: calado del agua en el punto de observacin fijo, (m).
Fig. 1.13. Esquema de un medidor Parshall.
La medicin manual de la altura (H), arroja como resultado un valor
instantneo del caudal. Existe en el mercado un conjunto de instrumentos
para realizar la medida de forma continua y as obtener el caudal medio
diario, punta, mensual, etc., de forma precisa. En ocasiones adems es de
inters acoplar el canal a un indicador con registro grfico y totalizador.
31
Indicadores de la Contaminacin
Fig. 1.14. Canal Parshall.
1.9.2. Vertedores
Los vertedores consisten bsicamente en una obstruccin en la que se es-
tanca el lquido y vierte por encima de ella (Figura 1.15).
Fig. 1.15. Esquema de un vertedor.
32
Menndez Gutirrez, C. y J. Prez Olmo
En el uso de los vertedores se emplean trminos que requieren ser defi-
nidos, tales como 'cresta, 'altura de la cresta, 'contraccion de la superIi-
cie y 'carga del vertedero. Se denomina 'cresta al borde por el que Iluye
el liquido, 'altura de la cresta a la distancia desde el Iondo del canal a la
cresta, 'contraccion de la superIicie a la Iorma curva que describe el liqui-
do sobre la cresta y 'carga del vertedor (H) a la diferencia entre la altura
de la superficie alcanzada por el liquido y la altura de la cresta medida aguas
arriba del vertedor antes de la 'contraccion.
Una vez medido el valor de la carga del vertedor, H, se puede obtener el
caudal de agua residual que pasa en cada instante.
Hay dos elementos que deben ser tomados en consideracin cuando
se desea o necesita instalar un vertedor para la medicin de caudales.
Estos son:
partiendo del hecho de que la contraccin de la vena lquida comienza a
una distancia aproximada de 2H aguas arriba del vertedor, la medida
debe efectuarse a una distancia mayor que esta,
la altura mnima de la cresta debe ser de 2,5 H para permitir la contrac-
cin completa de la lmina.
Los vertedores ms utilizados son:
Rectangular.
Triangular.
Trapezoidal.
Vertedor rectangular
Estos pueden ser sin contraccin lateral o con ella.
En aquellos casos en los que no exista contraccin lateral, la cresta ocupa
todo el ancho del canal. El caudal viene dado por la ecuacin,
5 1
84 1
,
H L , Q =
(1.30)
siendo:
Q: caudal, m
3
s
1
.
L: longitud de la cresta, m.
H: carga sobre la cresta, m.
Las dimensiones que debe cumplir el vertedor se detallan en la fi-
gura 1.16.
33
Indicadores de la Contaminacin
Fig. 1.16. Relacin de las dimensiones de un vertedero rectangular sin
contraccin lateral.
En los vertedores rectangulares con contraccin lateral, la cresta no ocupa
toda la anchura del canal. Las dimensiones de las contracciones laterales se
especifican en la figura 1.17.
La ecuacin que permite calcular el caudal en este tipo de vertedor es,
( )
5 1
1 0 84 1
,
H H n , L , Q =
(1.31)
donde n es el nmero de contracciones laterales.
Fig. 1.17. Relacin de las dimensiones en un vertedor rectangular con
contraccin lateral.
34
Menndez Gutirrez, C. y J. Prez Olmo
Vertedor triangular
Este tipo de vertedor es uno de los ms utilizados debido a que, para un
mismo caudal, se consigue mayor altura que en los rectangulares. Este efecto
se traduce en una mayor precisin en la lectura (figura 1.18).
El ngulo del vrtice de este vertedor puede tener diferentes valores. Los
ms usuales son 60 y 90. La expresin que da el caudal en este tipo de verte-
dor es:
Para 90
5 2
38 1
,
H , Q =
(1.32)
donde:
Q: gasto, m
3
s
1
.
H: altura del lquido con relacin al vrtice, m.
Para 60
5 2
79 0
,
H , Q =
(1.33)
Fig. 1.18. Relacin de las dimensiones en un vertedor triangular.
Fig. 1.19. Relacin de las dimensiones en un vertedor trapezoidal.
35
Indicadores de la Contaminacin
Vertedor trapezoidal
Este vertedor tiene una forma similar al rectangular (figura 1.19). La expre-
sin que se emplea para calcular el caudal es:
5 1
859 1
,
H L , Q =
(1.34)
L: la longitud de la cresta del vertedero, m
Medidores de caudal en lnea
Los mtodos usualmente utilizados en la determinacin de caudales en
tuberas a presin corresponden a los llamados mtodos dinmicos, que su-
ponen una aplicacin del teorema de Bernouilli entre dos puntos de una tu-
beria. Los dispositivos mas empleados son los 'tubos Venturi.
Fig. 1.20. Tubo Venturi.
1.10. DISMINUCIN DEL VOLUMEN Y FORTALEZA
DE LOS RESIDUALES
Con frecuencia, haciendo sencillas modificaciones en la industria, se
pueden lograr reducciones apreciables en el volumen y fortaleza de las aguas
residuales, obteniendo como consecuencia de ello disminuciones en los cos-
tos de inversin y operacin de las instalaciones de tratamiento, entre estas
modificaciones puede sealarse:
Recirculacin: el agua que est relativamente poco contaminada puede ser
recirculada con un mnimo de tratamiento o sin tratamiento alguno.
Clasificacin de las diversas corrientes y segregacin de las mismas: el
agua de enfriamiento y otras poco contaminadas pueden segregarse
36
Menndez Gutirrez, C. y J. Prez Olmo
con anterioridad al tratamiento. Las aguas que contengan sustancias
txicas no deben llegar nunca a las unidades de tratamiento biolgico.
Sustitucin de aditivos qumicos: hay casos en los que se facilita el
tratamiento sustituyendo algunas materias primas cuyo residuos sean
resistentes por otros ms fcilmente tratables.
Recuperacin de subproductos: en ocasiones con instalaciones de bajo
costo se logra obtener un subproducto dando lugar a una produccin
marginal, o a la recuperacin de alguna materia prima del propio proce-
so productivo.
La disminucin en las variaciones del caudal y de la fortaleza de las
aguas residuales deben ser prioritarios, lo que puede ser logrado, entre otros
medios, mediante el empleo de tanques compensadores.
Notas bibliogrficas
1 CHUDOBA, J., C. MENNDEZ Y J. PREZ O.: Fundamentos tericos de algunos
procesos para la purificacin de aguas residuales, Ed. ISPJAE, Ciu-
dad de La Habana, 1986.
2 THOMAS, M.A.: 'Graphical determination oI BOD curve constants, Water
and Sewage Works, 1950.
3 MOORE, E.W., H.A. THOMAS, ANDW.B. SNOW.: 'SimpliIied method Ior the analysis
oI BOD data, Sewage and Ind. Wastes, vol. 22, no. 10, 1950.
4 RHAME, G. A.: 'Two-Point Method Ior estimation oI Iirst stage BOD, Sewage
and Ind. Wastes, vol. 28, no. 9, 1950.
5 Standard Method for the Examination of Water and Wastewater, Ed.
APWA-AWWA, 1995.
6 MOORE, W.A.: 'Determination oI oxygen consumed value oI organic wastes,
Anal. Chemistry, 1951.
7 ARGAMAN, Y. AND A. BRENNER: 'Experimental evaluation oI modeling alter-
natives and process parameters for the single-sludge nitrogen removal
system, IAWPRC Specialised Seminar, Copenhagen, Denmark, Au-
gust, 1985.
8 MENNDEZ, C.: Reporte Indito, ISPJAE, Ciudad de La Habana, 1989.
9 ECKENFELDER, W.W.: Water Quality Engineering for Practicing Engineers,
CBI, Pub. 1980.
10 LOEHRS, H.C.: 'Variation oI wastewater parameters, Pub. Works, vol. 99,
no. 81, 1968.
11 RODRGUEZ, C.: 'Caracterizacion de las aguas residuales de Bejucal, Inge-
niera Hidrulica, vol. 19, no. 4, 1998.
37
Indicadores de la Contaminacin
12 HUNTER, J.V. AND H. HEUKELEKIAN: 'The composition oI domestic sewage
Iractions, Jour. Water Poll. Control Fed., vol. 37, no. 8, 1965.
13 WALTER, L.: 'Composition oI sewage and sewage eIIluents, partes 1 y 2,
Water Sewage Works, vol. 109, no. 11 y 12, 1961.
14 OMS, REPORTE: Evaluacin rpida de fuentes de contaminacin de aire,
agua y suelo, Mxico, 1988.
15 Programa integral de desarrollo de los derivados de la caa de azcar.
Sub-programa alcohol y levaduras, Cuba, 1985.
39
Pretratamiento de Aguas Residuales
CAPTULO 2
PRETRATAMIENTO DE AGUAS RESIDUALES
El tratamiento biolgico de las aguas residuales, fundamentalmente las
de origen industrial, en ocasiones requiere de algn tipo de pretratamiento
cuyo objetivo puede ser la remocin de contaminantes, como ocurre du-
rante la sedimentacin primaria, o el acondicionamiento del agua residual
mediante la compensacin y (o) neutralizacin, para facilitar la depuracin
biolgica.
2.1. COMPENSACIN
Los residuales lquidos industriales se caracterizan por la diversidad
de corrientes con diferencias en su composicin y concentracin. Aun una
misma corriente puede presentar variaciones horarias en sus propiedades. Cuando
los residuales manifiestan mucha variacin en su composicin se recomienda el
empleo de tanques compensadores.
La experiencia ha demostrado que los procesos de tratamiento se realizan
mejor si las fluctuaciones extremas en la carga del sistema pueden ser evitadas
o en ltima instancia ser atenuadas mediante la compensacin.
En los tanques compensadores los residuales se retienen durante un periodo
en el que se llega a obtener un efluente relativamente estable.
La neutralizacin de corrientes cidas o bsicas, la estabilizacin de la DBO
y la separacin de metales pesados, son algunos de los objetivos de la compen-
sacin.
Aguas residuales con alta acidez o alcalinidad que no son compensadas
requieren de la neutralizacin mediante la adicin de reactivos qumicos.
La mezcla en el tanque de compensacin generalmente se promueve con
aire. Un mtodo comn es el empleo de aereadores superficiales, con un consu-
mo de potencia de 4.10
5
6.10
5
W m
3
. Cuando se utiliza aire mediante
difusores, el consumo aproximado de este puede tener un valor cercano a los
4 m
3
m
3
d
1
.
1
40
Menndez Gutirrez, C. y J. Prez Olmo
Para el diseo de los tanques de compensacin pueden emplearse criterios
empricos y calcular su volumen considerando el tiempo requerido para acumu-
lar el agua residual de un ciclo de produccin o por turnos de trabajo. Siempre
que se pueda, el criterio de diseo debe tener como base el nivel de varianza
aceptable que tiene el parmetro que se desea normalizar para las etapas subsi-
guientes del tratamiento.
Los tanques compensadores pueden disearse para operar a volumen cons-
tante o volumen variable, en funcin de que interese amortiguar variaciones de
concentracin o flujo respectivamente.
2.1.2. Dinmica de los sistemas de compensacin
La compensacin se logra vertiendo el residual en un tanque o cisterna de
forma tal que se mezcle con el que le ha precedido, con lo cual se amortiguan las
variaciones bruscas.
Fig. 2.1. Efecto de un compensador sobre variaciones de flujo y / o
composicin.
A fin de ilustrar la dinmica de un compensador considere el caso ms
sencillo de un tanque perfectamente agitado de volumen constante, al cual
llega continuamente una corriente de agua residual de flujo Q y concentra-
cin del componente de inters C
E
, siendo la salida de igual flujo pero de
concentracin C
S
.
Fig. 2.2. Compensador de volumen constante.
41
Pretratamiento de Aguas Residuales
Aplicando un balance diferencial para el componente de inters:
dt
dC
V C Q C Q
S
S E
+ = (2.1)
por lo que integrando:
) )
=
=

t
t
C
C
S E
S
dt
V
Q
C C
dC
S
0
0
(2.2)
por lo tanto:
0
t
C C
C C
E
S E
=

0
ln
donde:
Q
V
0 =
entonces:
|
.
|

\
|
=
0
t
) C (C C C
E E S
exp
0
(2.3)
En la tabla 2.1 se muestra un ejemplo numrico donde se ilustra el efecto
compensador del sistema mostrado, suponiendo por simplicidad, que el flujo de
agua residual es constante y la composicin variable.
Tabla 2.1. Efecto compensador del sistema
Horario DQO, mgL
-1
1 150
2 175
3 180
4 200
5 250
6 300
7 320
8 300
9 250
10 180
11 150
12 140
42
Menndez Gutirrez, C. y J. Prez Olmo
Se ha supuesto que el tiempo de residencia en el compensador es de 10
horas (0).
Fig. 2.3. Ilustracin del efecto de un compensador de volumen constante.
En la figura 2.3 se muestran las curvas de variacin de DQO de entrada al
compensador, las del efluente compensado y la composicin promedio. De la
misma puede observarse el efecto compensador del sistema, ya que la variacin
que experimenta la composicin de salida es menor que la de entrada.
Es de destacar que el compensador no modifica la composicin promedio
del efluente con respecto al caudal de entrada, sino su varianza.
Al incrementarse el tiempo de residencia del compensador aumenta su efecto
amortiguador, por lo que para un tiempo de residencia infinito desaparecen las
fluctuaciones de la composicin de salida igualndose a la promedio.
El mtodo que se siga para calcular las dimensiones del tanque compensador
depender de las condiciones en que este operar.
2.1.3. Compensacin para flujo de residual constante
y de composicin variable
Bajo condiciones de flujo constante la relacin entre la varianza del efluente
de un tanque completamente mezclado y la del afluente, cuando las variaciones
de composicin son aleatorias, viene expresada por la relacin:
2
0 .
AT
S
S
i
e
2
2
2
=
(2.4)
43
Pretratamiento de Aguas Residuales
donde:
S
2
e
: varianza de la concentracin del efluente.
S
2
i
: varianza de la concentracin del afluente.
AT: tiempo de recoleccin de la muestra compuesta.
u: tiempo de retencin (V / Q).
De acuerdo con lo anterior puede calcularse el tiempo de retencin si se
conocen la varianza del afluente y la permisible o deseada para el efluente.
La varianza en el afluente, S
2
i
se determina segn:
( )
_
=

=
n
i
i
X Xi
n
S
1
2
2
1
1
(2.5)
donde:
Xi: valor individual de la concentracin de cada muestra tomada.
X
: valor promedio de todas las mediciones.
n: nmero de muestras.
Se define el valor tpico normal Z de la propiedad controlada como:
e S
X Xm
Z

= (2.6)
donde:
Xm: valor mximo que puede tomar la propiedad en el efluente del tanque de
compensacin.
S
e
: desviacin tpica del efluente.
Procedimiento para el diseo
El procedimiento de diseo consiste en hallar el tiempo de retencin reque-
rido en el tanque compensador para obtener las condiciones de salida deseadas.
El tiempo de retencin y el caudal de agua a tratar, definirn el volumen de
tanque necesario.
1. Determinar el valor promedio de la concentracin del afluente
( X
.
2. Calcular su desviacin tpica.
3. Fijar el valor mximo permisible de concentracin que se desea a la salida
(Xm).
4. Determinar la desviacin tpica del efluente tomando un valor de Z con un
por ciento de confiabilidad, por ejemplo, 90 %.
5. Hallar el valor de u.
44
Menndez Gutirrez, C. y J. Prez Olmo
6. Con el valor de u y el flujo que se desea procesar se estima el volumen del
compensador:
Q
V
0 =
Ejemplo 2.1
Se estudi un agua residual cuyo caudal es de 1 500 m
3
d
1
. Se tomaron
cuatro muestras diarias integradas durante 6 horas cada una en 10 das diferen-
tes. Los resultados se muestran en la tabla 2.2:
Tabla 2.2. Resultados del ejemplo 2.1
Debido al tratamiento que se emplear se requiere disminuir la variacin que
presenta la DBO se pretende disear un tanque para compensar tales efectos.
Determine el volumen del compensador si se conoce que el valor mximo
permisible de la DBO a la salida es de 1000 mgL
1
.
8 832,
n
X
X = =
_
S
i
= 367,438
Xm = 1000
De la tabla de distribucin normal (Anexo 2), para 90 % de confianza, el
valor de Z = 1,30
e S
X Xm
Z

=
2 167
30 1
8 832 1000
,
,
,
S
e
=

=
HORA 6:00 a.m. 12:00 m 6:00 p.m. 12:00 a.m.
Da Valores de DBO (mgL
-1
)
1 850 1240 600 1130
2 1430 324 1290 741
3 1550 421 750 422
4 1390 1324 1050 560
5 410 990 684 1220
6 500 270 820 530
7 420 450 750 1205
8 960 1230 680 890
9 395 1190 300 500
10 600 1304 1020 922
45
Pretratamiento de Aguas Residuales
0 .
AT
S
S
i
e
2
2
2
=
S
e
2
= 27955,84; S
i
2
= 135810,68
El tiempo de retencin en el tanque compensador ser,
h 5 14
84 27955 2
68 135810 6
,
,
,
0 =

=
u = 0,6 d, por lo tanto, el volumen es,
V = 1500 0,6 = 900 m
3
2.1.4. Compensacin cuando hay variacin simultnea de flujo
y composicin
3
Realizando un balance de masa en el tanque compensador:
C V T C Q C V T C Q + = +
2 0 1
(2.7)
donde:
C
1
: concentracin que entra al tanque durante el intervalo de muestreo T.
T: intervalo de muestreo, por ejemplo, 1 h.
Q: flujo promedio en el intervalo de muestreo.
C: concentracin en el tanque al inicio del intervalo de muestreo.
V: volumen del tanque.
C
2
: concentracin que sale del tanque al concluir el intervalo de muestreo.
En este caso se est asumiendo que la concentracin del efluente es cons-
tante durante un intervalo de muestreo. Esto es cierto si el tiempo del intervalo
es espaciado apropiadamente.
V/Q T
V/Q C T C
C
+
+
=
0 1
2
(2.8)
Procedimiento para el diseo
1. Se asume un volumen del tanque compensador, y este define el tiempo de
retencin.
2. Tomando como primer valor de C
0
el valor promedio de las mediciones, se
calculan los valores de la concentracin compensada. Para los restantes
intervalos se toma la concentracin final del intervalo anterior.
3. Si los valores de concentracin as obtenidos no llegan a satisfacer los re-
querimientos, se toma otro valor de tiempo de retencin y repite el proceso.
46
Menndez Gutirrez, C. y J. Prez Olmo
Ejemplo 2.2
Tabla 2.3. Resultados
La tabla 2.3 fue obtenida de una planta industrial durante un ciclo de produc-
cin de 8 horas. Cada intervalo de tiempo representa un periodo de muestreo de
1 hora. Determine el factor pico del efluente de un tanque compensador cuando
se utiliza un tiempo de retencin de: a) 8 horas, b) 2 horas y comprelos con el
factor pico cuando no se utiliza compensacin.
promedio Valor
mximo Valor
Pico Factor =
Al inicio:
3 2
75 173
400
,
,
FP = =
a) Asumiendo un tiempo de retencin igual a 8 h:
V = 283,87 8 = 2270,96 m
3
Q
V
Q
V
C C
C
+
+
=
1
0 1
2
Despus del primer intervalo,
C
1
= 250 mgL
1
Q = 363 m
3
h
1
C
0
= 173,75 mgL
1
Intervalo
de
tiempo
Q
(m
3
h
-1
)
Concentracin
de DBO
(mgL
-1
)
1 363 250
2 45 70
3 230 60
4 270 170
5 363 330
6 450 50
7 290 60
8 260 400
Promedio 283,87 173,75
47
Pretratamiento de Aguas Residuales
1
2
mgL 39 184
363
96 2270
1
363
96 2270
75 173 250

=
+
+
= ,
,
,
,
C
Despus del segundo intervalo:
C
1
= 70 mgL
1
Q = 45 m
3
h
1
C
0
= 184,79 mgL
1
1
2
L mg 17 182
45
96 2270
1
45
96 2270
39 184 70

=
+
+
= ,
,
,
,
C
Al proceder de manera similar para los restantes intervalos se obtienen los
resultados que se muestran en la tabla 2.4:
Tabla 2.4. Resultados
Factor Pico para un tiempo de retencin de 8 h en el tanque compensador:
1 1
75 173
70 192
,
,
,
FP = =
b) Asumiendo un tiempo de retencin igual a 2 h:
V = 283,87 2 = 567,74 m
3
Despus del primer intervalo,
C
1
= 250 mgL
1
Q = 363 m
3
h
1
C
0
= 173,75 mgL
1
Concentracin compensada (C
2
) Intervalo C
1
Q
Para u = 8 h Para u = 2 h
Primero 250 363 173,75 203,49
Segundo 70 45 182,17 193,70
Tercero 60 230 170,93 155,15
Cuarto 170 270 170,83 160,00
Quinto 330 363 192,77 226,30
Sexto 50 450 169,16 148,35
Sptimo 60 290 156,68 109,28
Octavo 400 260 156,80 200,60
Promedio 173,75 283,37
48
Menndez Gutirrez, C. y J. Prez Olmo
1
2
mgL 49 203
363
74 567
1
363
74 567
75 173 250

=
+
+
= ,
,
,
,
C
Despus del segundo intervalo:
C
1
= 70 mgL
1
Q = 45 m
3
h
1
C
0
= 203,49 mgL
1
1
2
mgL 70 193
45
74 567
1
45
74 567
39 184 70

=
+
+
= ,
,
,
,
C
Procediendo de manera similar para los restantes intervalos se obtienen los
resultados que se muestran en la tabla anterior para: u = 2 h.
Factor Pico para un tiempo de retencin de 2 h en el tanque compensador:
3 1
75 173
30 226
,
,
,
FP = =
2.1.5. Compensacin cuando hay variacin simultnea de flujo
y composicin, pero el volumen en el compensador es
variable
El volumen requerido para la homogeneizacin o compensacin se determi-
na mediante un diagrama de los caudales a tratar, en el cual se representa el
volumen de afluente acumulado a lo largo del da. El caudal medio diario, tam-
bin representado en el mismo diagrama, es la pendiente de la lnea recta traza-
da desde el origen hasta el punto final del diagrama.
Para determinar el volumen necesario, se traza una recta paralela a la que
define el caudal medio diario, tangente a la curva de caudales acumulados. El
volumen requerido es igual a la distancia vertical existente entre el punto de
tangencia y la lnea recta que representa el caudal medio, tal como muestra la
figura 2.4.
Si una parte de la curva de caudales acumulados est situada por encima de
la lnea que representa el caudal medio, el diagrama acumulado debe limitarse
con dos lneas paralelas a la del caudal medio y tangente a las dos curvas del
diagrama. El volumen requerido en este caso es igual a la distancia vertical
existente entre las dos tangentes. Figura 2.5.
49
Pretratamiento de Aguas Residuales
Fig. 2.4. Determinacin del volumen del compensador.
La interpretacin fsica de los diagramas representados es la siguiente:
En el punto inferior de tangencia el tanque de compensacin est vaco. A
partir de este punto el tanque empieza a elevar su nivel por lo que la pendiente
del diagrama de la curva de volumen del afluente es mayor que la del caudal
medio diario y contina llenndose hasta que lo hace al final del horario. Para el
segundo modelo de flujo, el tanque est totalmente lleno en el punto de tangen-
cia superior.
Fig. 2.5. Determinacin del volumen del compensador.
Horario
Horario
V
o
l
u
m
e
n
a
c
u
m
u
l
a
d
o
50
Menndez Gutirrez, C. y J. Prez Olmo
Procedimiento para el diseo
1. Se desarrolla una curva de caudales acumulados de agua expresada en me-
tros cbicos.
T Q V =
S i o f
V V V V + =
1 1
=
o
V V
2 1 1 2
+ = V V V
o
n n n
V ... .......... V V V

+ + + =
) 1 ( 2 1 1 0
2. Se traza la lnea de caudal medio diario.
3. Se determina el volumen del tanque requerido.
4. Para estimar el efecto de homogeneizacin:
a) Se calcula el volumen de lquido existente en el tanque al final de cada
periodo de tiempo mediante la expresin:
s i o f
V V V V + =
donde:
V
f
: volumen en el tanque al final del periodo de tiempo considerado.
V
0
: volumen en el tanque al final del periodo de tiempo anterior.
V
i
: volumen aportado durante el tiempo considerado.
V
S
: volumen de caudal saliente durante el tiempo considerado.
b) Se calcula la concentracin media que sale del tanque:
0
0 0
V V
X V X V
X
i
i i
f
+
=
+
donde:
X
f
: concentracin media en el caudal que sale del tanque durante el tiempo
considerado mg / L.
V
i
: volumen aportado durante el tiempo considerado, m
3
.
V
0
: concentracin del agua residual contenida en el tanque al final del perio-
do anterior.
5. La magnitud de la carga horaria se calcula utilizando la expresin:
i i
Q X = horaria Carga
El efecto de la homogeneizacin puede mostrarse numricamente a partir
de las relaciones siguientes:
punta / media; mnima / media y punta / mnima
51
Pretratamiento de Aguas Residuales
Estas relaciones se comparan para el residual antes y despus de homoge-
neizar. Para el residual homogeneizado estas relaciones estn ms prximas al
valor de 1,0.
En la prctica el volumen del tanque de compensacin debe ser algo supe-
rior al determinado tericamente (10 al 20 %) para tener en cuenta los dos
factores que se destacan a continuacin:
La operacin continua de los equipos de aereacin y mezclado no permi-
tirn un vaciado total.
Debe contemplarse un volumen adicional para hacer frente a los im-
previstos que puedan producirse para cambios no esperados del cau-
dal diario.
Ejemplo 2.3
En las columnas 1 y 2 de la tabla 2.5 que se reporta ms abajo, estn reco-
gidos los datos de flujo y concentracin de un agua residual industrial que ser
sometida a tratamiento. Para ello se necesita que el valor del flujo se mantenga
lo ms constante posible. A tales efectos se desea conocer el volumen que ha de
tener el tanque compensador requerido.
Determine adems el volumen de agua que contendr el tanque cada hora.
La industria opera durante 12 horas al da, y los datos que se ofrecen son los
promedios de cada intervalo medido.
Para estimar el volumen del tanque compensador se traza la curva de EVvs
intervalo de tiempo. Considerando que el primer intervalo en el que se tomaron
las muestras termina a las 8 am, el valor de EV es el del intervalo de 7 a 8 a.m.:
EV = 10.
Para el intervalo siguiente: EV = 10 + 15 = 25
De la misma manera se procede con los siguientes intervalos. Los valores
de EV aparecen en la columna 3 de la tabla 2.5.
Se traza la curva correspondiente y se unen mediante una recta los dos
extremos de la curva. La pendiente de esta recta es numricamente igual al
flujo promedio, y es al mismo tiempo el flujo de salida que debe tener el
compensador.
En la figura 2.6 se aprecia que la curva presenta un valor mnimo con res-
pecto a la recta a las 10:00 a.m. (horario 3 en el grfico), y un valor mximo a
las 2:00 p.m. (horario 7 en el grfico). Estos son las horas que se corresponden,
al menos tericamente con los momentos en los que el tanque est vaco y lleno
respectivamente. Mientras tanto, el efluente se mantiene constante y con un
valor igual al flujo promedio de la entrada.
1 3
h m 83 45

= , Q
52
Menndez Gutirrez, C. y J. Prez Olmo
Tabla 2.5. Datos para el tratamiento de un agua residual industrial
El volumen del tanque compensador se calcula midiendo la diferencia de
volumen entre los valores del mximo y el mnimo de la curva, con respecto a la
recta trazada.
V = 231 m
3
Columna 1 Columna 2 Columna 3 Columna 4 Intervalo
de
muestreo
Flujo
m
3
h
1
DQO
mgL
1
EV
Volumen
contenido, m
3
7-8 a.m. 10 300 10
8-9 15 275 25
9-10 35 200 60
10-11 75 170 135 29,17
11-12 170 90 305 153,34
12-1 p.m. 140 100 445 247,51
1-2 30 210 475 231,68
2-3 25 250 500 210,85
3-4 20 250 520 185,02
4-5 15 300 535 154,19
5-6 10 340 545 118,36
6-7 5 360 550 77,53
Fig. 2.6. Clculo del volumen del tanque compensador.
53
Pretratamiento de Aguas Residuales
Para estimar el volumen contenido en el tanque en cada intervalo de tiempo,
se comienza a calcular a partir de cualquiera de los dos momentos en los que a
priori se conoce el volumen contenido, esto es 10:00 a.m. 2:00 p.m.
Tomando como valor inicial el correspondiente a las 10:00 a.m.,
A las 10:00 a.m., V = 0
En el intervalo de 10:00 a 11:00 entran al compensador 75 m , y salen
45,83 m
3
, por lo tanto, el volumen a las 11:00 a.m. ser,
V
11
= V
10
+ V
10-11
- V
promedio
V
11
= 0 + 75 - 45,83 = 29,17
El volumen contenido a las 12:00 m se estima:
V
12
= V
11
+ V
11-12
- V
promedio
V
12
= 29,17 + 170 - 45,83 = 153,34
De manera similar se procede con los intervalos siguientes. Los valores as
obtenidos se reportan en la columna 4 de la tabla 2.5.
Como se aprecia en la tabla 2.5, a las 7:00 p.m., hora en que cesa la activi-
dad laboral, el tanque contiene 77,53 m
3
. Si este volumen se mantiene en el
tanque hasta que se inicia el periodo laboral al da siguiente, se tendr que:
El volumen en el tanque de 7:00 a.m. a 8:00 a.m. ser: 41,7
8:00 a.m. a 9:00 a.m.: 10,87
9:00 a.m. a 10:00 a.m.: 0,04
Este ltimo valor corrobora la estimacin inicial que se hizo al considerar
que a las 10:00 a.m. el tanque estar vaco.
2.2. SEDIMENTACIN
La sedimentacin es un proceso fsico de uso muy difundido en el trata-
miento de las aguas r esiduales. Puede utilizar se como parte de un
tratamiento primario, o como una etapa del tratamiento secundario. El obje-
tivo de la sedimentacin primaria es reducir la concentracin de slidos sus-
pendidos y la carga orgnica de un agua residual para facilitar los tratamientos
posteriores. La secundaria constituye un proceso importante para la clarifi-
cacin del efluente de las unidades de tratamiento biolgico y para el
espesamiento de los lodos orgnicos que se obtienen.
La sedimentacin es efectiva para la remocin de slidos suspendidos en la
medida en que la fuerza de gravedad que acta sobre la partcula que se desea
3
54
Menndez Gutirrez, C. y J. Prez Olmo
remover es mayor que la resultante de las fuerzas de sustentacin y la viscosa
que actan sobre ella.
2.2.1. Tipos de sedimentacin
Atendiendo a la naturaleza de los slidos suspendidos y a su concentracin,
la sedimentacin puede clasificarse en:
Discreta,
floculenta,
retardada o por zonas y
por compresin.
2.22. Sedimentacin discreta
En este tipo de sedimentacin, caracterstica de las suspensiones de arena,
cenizas y de carbn mineral entre otros, las partculas conservan su individuali-
dad y no cambian de tamao, forma ni densidad, durante el proceso de sedimen-
tacin. La velocidad de sedimentacin solo depende de las propiedades del fluido
y de las partculas que se desean remover, y es constante durante todo el proce-
so de sedimentacin.
La fuerza resultante que acta sobre una partcula que se mueve a travs de
un lquido en reposo puede calcularse:
F
r
= F
e
F
s
F
d
(2.9)
donde:
F
r
: Fuerza resultante.
F
e
: Fuerza externa que propicia que la partcula se mueva hacia abajo.
F
s
: Fuerza de sustentacin hidrulica.
F
d
: Fuerza de arrastre.
La fuerza externa, F
e
, puede estimarse segn:
F
e
= m g (2.10)
donde:
F
e
: Fuerza externa que acta sobre la partcula, N.
m: Masa de la partcula, kg.
g: Aceleracin de la gravedad, m s
2
.
La fuerza de sustentacin hidrulica puede ser calculada segn:
g m

F
s
s
=
(2.11)
55
Pretratamiento de Aguas Residuales
Fig. 2.7. Esquema de sedimentadores: clarificadores, a) Rectangular b) y
c) Circular.
56
Menndez Gutirrez, C. y J. Prez Olmo
: Densidad del fluido, kg m
3
.

s
: Densidad de la partcula, kg m
3
.
Por otra parte, la fuerza de arrastre es funcin de la forma, tamao y rugo-
sidad de la partcula, as como de su velocidad de sedimentacin y de la densi-
dad y viscosidad del fluido:
2
2
U A C
F
D
d
= (2.12)
donde:
C
D
: Coeficiente de arrastre (adimensional). Es funcin del nmero de
Reynolds para un tipo de partcula.
A: rea proyectada sobre un plano perpendicular a la direccin del movi-
miento, m
2
.
U: Velocidad de la partcula, m s
1
.
De las ecuaciones (2.10), (2.11) y (2.12) puede obtenerse.
2
2
U A C


g m
dt
dU
m
D
s
s

=
(2.13)
En la ecuacin 2.13 se ha expresado la fuerza resultante en funcin de la
masa de la partcula y de su aceleracin,
dt
dU
La aceleracin de la partcula en su movimiento de descenso a travs del
seno del lquido puede obtenerse de la ecuacin 2.13:
m
U A C


g
dt
dU
D
s
s
2
2

=
(2.14)
Durante el descenso, se llega a una velocidad a partir de la cual la acelera-
cin de la cada se hace igual a cero, y por tanto, la velocidad de sedimentacin
es constante e igual a:
2
1
2
|
.
|

\
|

=
D s
s
C A
m


g U
(2.15)
para partculas esfricas,
4
2
d a
A= (2.16)
57
Pretratamiento de Aguas Residuales
Teniendo en cuenta adems que:
volumen
masa
=
s
(2.17)
y que el volumen de partculas esfricas viene dado por:
3
6
1
d a V = (2.18)
La ecuacin 2.15, para partculas esfricas se transforma en:
2
1
3
4
|
.
|

\
|

= d
C

g U
D
s
(2.19)
El valor U es la velocidad de sedimentacin m s
1
.
O sea, que la velocidad de sedimentacin de una partcula esfrica aislada
es funcin de su dimetro y densidad, as como de la viscosidad del lquido claro
en el que se encuentra suspendida, ya que, como fue mencionado, el coeficiente
de arrastre depende del nmero de Reynolds. Para emplear la ecuacin (2.19)
se requiere conocer el valor del coeficiente de arrastre C
D
.
Fig. 2.8. Variacin del coeficiente de arrastre con el N
Re
.
El rgimen de flujo alrededor de la partcula puede ser laminar o turbulento.
En la regin laminar prevalecen las fuerzas viscosas, mientras que en regiones
turbulentas predominan las fuerzas inerciales. Por tanto, la relacin entre el
coeficiente C
D
y el nmero de Reynolds depende del rgimen de flujo que pre-
valezca, segn la figura 2.8.
58
Menndez Gutirrez, C. y J. Prez Olmo
Para partculas esfricas:
44 0
3 24
Re Re
,
N
N
C
D
+ + =
(2.20)
Cuando N
Re
< 3 4, predomina el primer trmino de la ecuacion 2.20 y esta
queda:
Re
24
N
C
D
=
(2.21)
Para N
Re
entre 1 000 y 250 000 el valor de C
D
es relativamente constante e
independiente del nmero de Reynolds (figura 2.6).
C
D
= 0,4 (2.22)
Para N
Re
intermedios se cumple:
6 0
5 18
,
Re
D
N
,
C =
(2.23)
El procedimiento normal de diseo de sedimentaciones para partculas dis-
cretas consiste en seleccionar una velocidad de sedimentacin U de forma que
todas las partculas con velocidad superior o igual a U son removidas. La velo-
cidad de diseo puede calcularse mediante la ecuacin (2.19) o ser obtenida de
pruebas experimentales. Las pruebas de sedimentacin discontinua (a templa)
dan origen a curvas similares a la de la figura 2.9. En tales pruebas la interfase
slido-lquido crece desde el fondo hacia arriba en la medida que mejora la
clarificacin de la suspensin. En el tiempo t, puede suponerse que todas las
partculas que tengan mayor tamao que la caracterizada por una velocidad de
sedimentacin de
t t
H H
o
o

habrn sedimentado.
Fig. 2.9. Variacin de la interfase slido-lquido en la sedimentacin a templa.
59
Pretratamiento de Aguas Residuales
Si la turbidez del lquido sobrenadante o la suspensin contienen cantidades
despreciables de partculas ms pequeas que la caracterizada, el dato obtenido
en la prueba puede ser utilizado en el diseo del sedimentador.
rea del sedimentador
El rea del sedimentador puede calcularse segn:
2
m
U
Q
A= (2.24)
donde:
A: rea, m
2
.
Q: Flujo de sobrenadante, m
3
s
1
.
U: Velocidad de sedimentacin seleccionada, m s
1
.
Por otro lado, la velocidad de ascenso del lquido claro, tambin llamada
carga superficial es:
1 2 3
= d m m
A
Q
V (2.25)
De las ecuaciones 2.24 y 2.25 se concluye que en la sedimentacin de par-
tculas discretas el diseo se basa en que la carga superficial V debe ser, teri-
camente, igual a la velocidad de sedimentacin U seleccionada.
En la discusin anterior se tom en consideracin una velocidad de sedi-
mentacin lmite, y la separacin de la suspensin de aquellas partculas, que
con velocidades iguales o superiores a ellas, caen a travs del lquido iniciando
su descenso a partir de la superficie. Sin embargo, al iniciarse la sedimentacin,
considerando que las partculas estn uniformemente distribuidas en todo el vo-
lumen, hay una cierta fraccin de partculas que aunque tienen la velocidad de
sedimentacin menor que U
0
se encuentran a una altura tal, que alcanzan el
fondo al mismo tiempo o en un tiempo menor que las partculas seleccionadas
para el criterio de diseo.
Fig. 2.10. Sedimentacin ideal de partculas discretas.
60
Menndez Gutirrez, C. y J. Prez Olmo
Hazen y Camp desarrollaron el concepto de tanque ideal para definir las
relaciones aplicables al diseo de sedimentadores.
Fig. 2.11. Relaciones de sedimentacin de partculas discretas en una
columna tpica.
Suponiendo que las partculas de los distintos tamaos estn distribuidas
uniformemente por toda la profundidad a la entrada del tanque de sedimenta-
cin, mediante un anlisis de la trayectoria de la partcula en la figura 2.10,
puede verse que aquellas con velocidad de sedimentacin U
p
menor a U
o
, se
eliminarn en la proporcin:
o
p
r
U
U
X =
(2.26)
La eficiencia de la sedimentacin, considerando toda la gama de velocida-
des existentes en una suspensin, puede ser determinada mediante el uso de una
columna de sedimentacin en una prueba a templa.
Procedimiento de diseo
El procedimiento consiste en tomar muestras a distintas alturas a intervalos
regulares de tiempo, y determinar en cada caso la concentracin de slidos
suspendidos de la extraccin.
Para un caudal de clarificacin dado, Q, solo las partculas con velocidad U
mayor a U
o
sern totalmente eliminadas. Las restantes partculas sedimentan
en la proporcin
o
p
U
U
. Por tanto, la fraccin total de partculas eliminadas
61
Pretratamiento de Aguas Residuales
cuando se selecciona como criterio de diseo la velocidad de sedimentacin
U
o
ser:
( )
)
+ =
r
X
o
p
r
dX
U
U
X X
0
1
(2.27)
donde:
(1 X
r
): fraccin de partculas con velocidad > U
o
.
U
o
: velocidad seleccionada como lmite.
)
r
X
o
p
dX
U
U
0
:
Desde el punto de vista prctico, los sedimentadores estn sujetos a efectos
de turbulencia, cortocircuitos de las corrientes y gradientes de velocidades. Por
tanto, nunca debe disearse considerando que la carga superficial es igual a la
velocidad de sedimentacin seleccionada. Se recomienda que la carga superfi-
cial sea menor a la velocidad de sedimentacin en un factor que pueda variar
entre 1,25 y 1,75 y el tiempo de retencin incrementarse de 1,5 a dos veces.
Ejemplo 2.4
Al someter a prueba de sedimentacin una suspensin con baja concentra-
cin de slidos se obtuvieron los datos de la tabla 2.6.
Tabla 2.6. Datos de sedimentacin obtenidos de una suspensin con baja
concentracin de slidos
fraccin de partculas con velocidad menor que U
o
y que son
removidas.
Determine el porcentaje de slidos que se eliminarn por sedimentacin si
se utiliza una carga superficial de 17,28 m
3
m
2
d
1
en el sedimentador.
Por criterio se seleccionar una velocidad de sedimentacin 1,5 veces ma-
yor que la carga superficial:
U
o
= 0,3 10
3
m s
1
= 0,03 cms
1
De la tabla que se brinda a continuacin se puede calcular en cada caso la
velocidad de sedimentacin experimental.
Tiempo requerido para
sedimentar 1,5 m (min)
70 120 230 460 630 880
Fraccin de partculas
que no sedimentaron en
el tiempo indicado
0,46 0,40 0,28 0,12 0,06 0,02
62
Menndez Gutirrez, C. y J. Prez Olmo
V: velocidad de sedimentacin (cm s
1
).
f: porcentaje de partculas con velocidad de sedimentacin menor o igual a
la indicada.
Con los datos de la tabla anterior se grafica el porcentaje de partculas con-
tra la velocidad de sedimentacin (figura 2.12).
V
(cm s
-1
)
0,0357 0,0208 0,0109 0,0054 0,0040 0,0026
f 46 40 28 12 6 2
Fig. 2.12. Porcentaje de partculas vs velocidad de sedimentacin para
el ejemplo 2.4.
( )
)
+ =
r
X
p
o
r
dX U
U
X X
0
1
1
U
o
= 0,03 cms
1
De la figura, X
r
= 45
)
+ =
r
X
p
dX U
0
15 2 1
A A A
( )
( )
4735 0
03 0
1
45 100 ,
,
X + =
X = 70,78 %
Tabla 2.7. Porcentaje de partculas y su velocidad de sedimentacin
63
Pretratamiento de Aguas Residuales
2.2.3. Sedimentacin de partculas floculentas
Los slidos suspendidos que generalmente se encuentran presentes en las
aguas residuales tanto de origen domstico como industrial son ejemplos tpicos
de partculas floculentas. Tambin presentan caractersticas floculentas los
efluentes de los tratamientos biolgicos que poseen relativamente una baja con-
centracin de slidos suspendidos.
Cuando la floculacin ocurre, la velocidad de sedimentacin de las partcu-
las se incrementa en la medida que van sedimentando, como consecuencia de la
asociacin con otras partculas individuales o conglomerados de partculas. De
tal forma, la velocidad de sedimentacin del flculo que se forma es mayor que
la de las partculas individuales que lo constituyen, producindose una trayecto-
ria curvilnea como se ilustra en la figura 2.13.
Fig. 2.13. Sedimentacin ideal de partculas floculentas.
Cuando est presente la floculacin, en el diseo del sedimentador debe
tenerse presente no solo la carga superficial, sino adems el tiempo de reten-
cin, ya que a medida que este ltimo se incrementa, aumenta la formacin de
flculos y esto incide en la velocidad de sedimentacin.
Teniendo en cuenta la influencia del tiempo de retencin en la formacin de
los flculos, se dificulta un anlisis matemtico de la situacin, y por tanto, se
hace necesario acudir a las pruebas de laboratorio para la estimacin de los
parmetros que se requieren en el diseo de los sedimentadores.
Procedimiento de diseo
Las pruebas de sedimentacin se efectan mediante un procedimiento simi-
lar al empleado en los experimentos de sedimentacin con partculas discretas.
64
Menndez Gutirrez, C. y J. Prez Olmo
En este caso las curvas de profundidad contra tiempo a las distintas concentra-
ciones tienen la forma que se muestra en la figura 2.14.
Fig. 2.14. Relaciones de sedimentacin de partculas floculentas. Curvas
de isoconcentracin.
Las curvas representan el porcentaje de remocin a cada profundidad y
tiempo, e indican la mxima trayectoria de la senda de la sedimentacin para
cada concentracin en especfico, por ejemplo, 45 % de las partculas en sus-
pensin sedimentan a una velocidad promedio superior a 1,8 / 24 = 0,075 m /
min
1
. Adicionalmente, hay otras fracciones de partculas que aunque tienen
velocidades inferiores a esta tambin sedimentarn.
La fraccin de partculas que aun teniendo una velocidad de sedimentacin
U
p
menor a la fijada U
0
sedimenta, viene dada segn la ecuacin (2.27).
0
U
U
X
p
r
=
(2.27)
Como se trabaja a un tiempo dado, esta relacin de velocidades se convierte
en una relacin de profundidades,
0
h
h
X
p
r
=
(2.28)
Considerando la figura 2.12 y suponiendo que como promedio 87,5 % de las
partculas
|
.
|

\
| +
2
75 100
sedimentan a una velocidad igual a
0
1
h
h A
, 67,5 %
65
Pretratamiento de Aguas Residuales
|
.
|

\
| +
2
60 75
tiene una velocidad de sedimentacin igual a
0
2
h
h A
, y as sucesiva-
mente, el porcentaje de partculas que sedimenta en veinticuatro minutos en un
sedimentador de 1,8 m de profundidad ser:
( )
2 2 2
%
4 3
0
3 3 2
0
2 2 1
0
1
R R
h
h A R R
h
h A R R
h
h A
R
+
+
+
+
+
=
(2.29)
donde:
R
1
= 100 % R
3
= 60 %
R
2
= 75 % R
4
= 45 %
El procedimiento puede repetirse para distintos tiempos y obtener una rela-
cin entre la velocidad de sedimentacin y el porcentaje de remocin.
Una vez que se llega al porcentaje de remocin de SS deseado, se estima la
carga superficial teniendo en cuenta que esta debe fijarse entre 1,25 y 1,75
veces menor que la velocidad de sedimentacin. As mismo, el tiempo de diseo
debe ser entre 1,5 y 2,0 veces mayor que el trabajado. De esta manera se
calcula el rea del sedimentador y el tiempo de retencin.
Ejemplo 2.5
Una suspensin floculenta que contiene 450 mgL
1
de slidos suspendidos
llena una columna de 3,0 m con salidas laterales cada 50 cm. A distintos tiempos
se toman muestras y se determina la concentracin de slidos suspendidos. Los
resultados obtenidos se muestran en la tabla que se adjunta. Determine el por-
centaje de remocin total de slidos suspendidos que se obtendr utilizando un
sedimentador de 2,75 m de profundidad trabajando con una carga superficial de
25 m
3
m
2
d
1
.
30 60 90 120 150 180 Tiempo (min)
Profundidad (m)
Concentracin de SS (mg L
-1
)
0,5 270 112 45 27 9 9
1,0 369 207 103 54 22 9
1,5 405 315 157 126 58 18
2,0 427 360 270 139 94 58
2,5 436 387 315 202 135 94
3,0 441 405 337 256 166 112
Tabla 2.8. Porcentaje de remocin de SS a diferentes profundidades
66
Menndez Gutirrez, C. y J. Prez Olmo
A partir de los datos anteriores se calcula, para cada tiempo y profundi-
dad, el por centaje de r emocin de SS y se tr azan las cur vas de
isoconcentracin segn la figura 2.16. para mayor facilidad del trazado pue-
de graficarse previamente el porcentaje de remocin contra tiempo para
cada profundidad (figura 2.15).
Fig. 2.15. Grfico auxiliar para el trazado de las curvas de la figura 2.16.
Fig. 2.16. Relaciones de sedimentacin. Curvas de iso-remocin. Ejemplo 2.5.
La carga superficial de diseo es 25 m
3
m
2
d
1
. Aplicando el factor 1,4, la
velocidad de sedimentacin lmite seleccionada ser:
U = 1,4 25 = 23 md
1
= 0,024 mmin
1
67
Pretratamiento de Aguas Residuales
Para una profundidad de 2,75 m el tiempo correspondiente ser 111 min.
( )
% ,
,
, , , ,
% R
29 78
2
100
3
5 0
2
130
3
4 0
2
150
3
6 0
2
170
3
5 0
2
190
3
75 0
= |
.
|

\
|
+
|
.
|

\
|
+ |
.
|

\
|
+ |
.
|

\
|
+ |
.
|

\
|
=
2.2.4. Sedimentacin por zona
La sedimentacin retardada o por zona se presenta en suspensiones de con-
centracin intermedia. Son caractersticas de este tipo de suspensin los flculos
de aluminio de concentracin comprendida entre 500 y 1000 mgL
1
y los efluentes
de los tanques de aeracin de los lodos activados.
En este tipo de sedimentacin las interacciones entre partculas son lo sufi-
cientemente grandes como para que cada una mantenga una posicin fija con
respecto a las restantes durante la cada.
En estos casos se aprecia una interfase generalmente muy bien definida
entre la masa de slidos que sedimentan y el lquido claro sobrenadante, cuando
se realizan pruebas de probeta.
4,5
La velocidad de sedimentacin se define como la velocidad con que des-
ciende la interfase slido-lquido. La velocidad de sedimentacin disminuye con
el incremento de la concentracin inicial aun cuando se trate de la misma sus-
pensin (figuras 2.17 y 2.18).
Fig. 2.17. Velocidad de sedimentacin a diferentes concentraciones de
slidos en suspensin.
68
Menndez Gutirrez, C. y J. Prez Olmo
La pendiente del tramo de velocidad de sedimentacin constante en la curva
de la figura 2.17 es la velocidad de sedimentacin a la concentracin inicial.
Fig. 2.18. Influencia de la concentracin de slidos suspendidos en la
velocidad de sedimentacin.
rea del sedimentador.
Varios son los mtodos que se utilizan para determinar el rea de sedimen-
tacin necesaria para la remocin de partculas que tienen el comportamiento
antes descrito.
El mtodo que a continuacin se describe, conocido como mtodo de la
curva de flux, es aplicable, tanto para la sedimentacin por zona como para
floculenta y la de compresin. El fundamento del mtodo es el anlisis del avan-
ce del flujo de masa de los slidos en toda la profundidad del sedimentador.
En un sedimentador continuo los slidos que entran sedimentan por la ac-
cin de dos componentes del flujo:
Descenso de los slidos debido a la sedimentacin por gravedad.
Descenso de los slidos como consecuencia de la extraccin de fondo del
sedimentador.
De acuerdo con lo anterior, el flujo msico de slidos por unidad de rea,
llamado de ahora en adelante flujo msico, ser:
G
t
= G
s
+ G
b
(2.30)
donde:
G
t
: flujo msico total, kg m
2
h
1
.
G
s
: flujo msico debido a la gravedad.
G
b
: flujo msico debido a la extraccin de fondo.
69
Pretratamiento de Aguas Residuales
El flujo msico por gravedad viene dado por:
G
s
= U X (2.31)
donde:
X: concentracin de slidos suspendidos, kg m
3
.
U: velocidad de sedimentacin de los slidos de concentracin X mh
1
.
Mediante experiencias en probeta, se puede obtener G
s
a distintas concen-
traciones iniciales de la suspensin. Una curva tpica de flujo debido a la grave-
dad, se muestra en la figura 2.19.
En la curva de G
s
contra X se observa que tanto para bajos como muy altos
valores de concentracin de slidos suspendidos, G
s
es pequeo. Para concen-
traciones intermedias (3-5 kgm
3
), el flujo de slidos debido a la gravedad, al-
canza su valor mximo.
El flujo de slidos debido al caudal de fondo es:
G
b
= U
u
X, kg m
2
h
1
(2.32)
U
u
: Velocidad de extraccin de fondo, m h
1
.
Si se considera el caudal de extraccin Q
u
en lugar de la velocidad de la
extraccin, la ecuacin 2.32 se transforma en:
1 2
h m kg

= , X
A
Q
G
u
b
(2.33)
donde:
Q
u
: Caudal extrado por el fondo, m
3
h
1
.
A: rea de sedimentacin, m
2
.
Fig. 2.19. Flujo msico debido a la
gravedad.
Fig. 2.20. Flujo msico debido
a la extraccin de fondo.
70
Menndez Gutirrez, C. y J. Prez Olmo
Para una extraccin de fondo constante, el flujo de slidos debido a ella
es directamente proporcional a la concentracin de slidos suspendidos (fi-
gura 2.20).
Teniendo en cuenta la ecuacin 2.30, para unas condiciones dadas de ex-
traccin de fondo, pueden sumarse grficamente los dos componentes del flujo
msico (G
s
+ G
b
) (figura 2.21).
Para cualquier valor Q
u
que se seleccione, la curva de G
t
alcanza un
valor mnimo para cierto valor de concentracin de slidos. En la figura
2.21, la curva de flujo total, G
t
alcanza el valor mnimo G
1
para la concentra-
cin de X
1
.
Este mnimo limita la velocidad a la cual los slidos pueden alcanzar el
fondo del sedimentador, entonces, para asegurar que todo el slido alcance
el fondo, la masa de slido aplicada por unidad de rea en el tiempo unitario,
o sea, G aplicado, debe ser menor o igual a G
1
:
G
ap
G
1
(2.34)
Si por cualquier razn la cantidad de slidos que se alimentan al
sedimentador es mayor que el valor lmite G
1
, estos se acumularn en el
fondo del sedimentador, producindose un aumento paulatino de su nivel en
el fondo, que puede incluso llevar a la salida de slidos por el sobrenadante.
Fig. 2.21. Flujo msico total.
La concentracin del caudal de extraccin de fondo se obtiene trazando una
recta paralela al eje x y que pasando por el mnimo de la curva de flujo, corte a
la recta G
b
. Para la figura 2.21 esta concentracin es X
r
.
71
Pretratamiento de Aguas Residuales
Una vez determinado el valor lmite del flujo de slidos G
1
, puede obtenerse
por un balance de masa, el rea necesaria para la sedimentacin:
0
1
X
G
Q
A
t
=
(2.35)
donde:
Q
t
: Flujo total que entra al sedimentador, m
3
d
1
.
X
0
: Concentracin de SS que entra al sedimentador, gm
3
.
De la figura 2.21 puede deducirse que para una suspensin dada, el rea
de sedimentacin, y por tanto, la concentracin de los slidos de fondo se
modifica al cambiar el caudal de extraccin de fondo. Debe tenerse presen-
te que cada vez que se cambie el caudal del fondo, se obtendr un G
b
dife-
rente y se requiere, por tanto, trazar de nuevo la curva de flujo msico total.
Como se aprecia, el procedimiento anterior es tedioso porque para cada
Q
u
se necesita construir nuevas curvas de flujo. Este procedimiento grfico
se simplifica notablemente cuando se utiliza el concepto de punto de situa-
cin (figura 2.22).
La concentracin de slidos a la entrada, X
0
, se representa por una lnea
vertical en el valor especificando X
0
. La velocidad de flujo de sobrenadante,
U
0
, viene dada por una lnea denominada lnea del sobrenadante, y que par-
tiendo del origen tiene una pendiente igual a U
0
. El intercepto de la lnea de
sobrenadante con la de operacin (la de X
0
), recibe el nombre de punto de
situacin.
Fig. 2.22. Definicin del punto de situacin.
72
Menndez Gutirrez, C. y J. Prez Olmo
La velocidad de extraccin de fondo se representa por una lnea denomina-
da lnea de fondo, con pendiente igual a -U
u
y que pasa por el punto de situacin
y es tangente a la curva.
El intercepto de la lnea de fondo con el eje de las concentraciones da el
valor de la concentracin del caudal de fondo X
r
, mientras que el intercepto con
el eje de flujo slidos refleja el flujo de slidos aplicado, G
ap
.
Entre las ventajas que posee este ltimo procedimiento, est el de la posibi-
lidad de analizar en un tiempo relativamente breve distintas variantes para el
clculo del rea del sedimentador. Al mismo tiempo, brinda la facilidad de eva-
luar el comportamiento de un sedimentador en funcionamiento ante la variacin
de los parmetros de operacin.
Procedimiento para el clculo del rea del sedimentador
Como puede concluirse de lo anteriormente expuesto, el mtodo de diseo
se fundamenta en el conocimiento de las velocidades de sedimentacin a distin-
tas concentraciones de slidos suspendidos. Para esto se requiere de pruebas
relativamente sencillas, a escala de laboratorio. Las pruebas consisten en seguir
en el tiempo, la trayectoria descendente de la interfase slido-lquido.
Esta interfase es fcil de distinguir cuando las suspensiones presentan ca-
ractersticas de sedimentacin por zona, o son de carcter floculento y poseen
concentraciones altas o medias. En aquellos casos en que la interfase no se
aprecia de manera definida, puede acudirse al mtodo de diseo que se basa en
las pruebas que se conocen como de tubos largos.
Descripcin del procedimiento
1. Se realizan las pruebas de sedimentacin en probeta, a temperatura constan-
te, a distintas concentraciones de slidos suspendidos (figura 2.17). La pen-
diente en la regin recta de la curva es la velocidad de sedimentacin. Para
cada una de las pruebas anteriores se determina, por gravimetra, la concen-
tracin de slidos suspendidos.
2. Para mayor facilidad en el trabajo y poder realizar interpolaciones, se reco-
mienda trazar la curva de velocidad de sedimentacin en funcin de la con-
centracin (figura 2.18).
3. De la informacin que brinda la figura 2.18, pueden obtenerse datos de flujo
msico debido a la gravedad, para distintas concentraciones, G
s
= U X.
4. Se traza la curva de flujo de slidos en funcin de la concentracin (figura 2.19).
5. Haciendo uso de la curva obtenida en la figura 2.19, y partiendo de la con-
centracin de fondo deseada, X
r
se traza la lnea de fondo tangente a la
curva de flujo (figura 2.22). El intercepto con el eje Y da el flujo de slidos
lmite (G
ap
) .
73
Pretratamiento de Aguas Residuales
6. Haciendo uso de la ecuacin (2.35), puede calcularse el rea de sedi-
mentacin.
0
X
G
Q
A
ap
t
=
A: rea del sedimentador, m
2
.
Q
t
: Flujo total a la entrada del sedimentador, m
3
h
1
.
X
0
: Concentracin de slidos suspendidos en la corriente de entrada, kgm
3
.
G
ap
: Flujo de slidos aplicado, kg m
2
h
1
.
Velocidad de flujo del sobrenadante U
0
. En la figura 2.22 se traza la lnea de
operacin definiendo el punto de situacin. La pendiente de la recta que une el
punto de situacin con el origen (recta que define la lnea del sobrenadante), da
el valor de la velocidad de flujo del sobrenadante.
Ejemplo 2.6
Se determinaron las caractersticas de sedimentacin de una suspensin
que procede de un lodo activado. Las pruebas de laboratorio se realizaron para
un intervalo de concentracin entre 1000 y 10000 mgL
1
, obtenindose la curva
de la figura 2.23.
Fig. 2.23. Clculo del rea de un sedimentador. Ejemplo 2.6.
74
Menndez Gutirrez, C. y J. Prez Olmo
Calcule la velocidad de flujo de sobrenadante mxima permisible si se desea
obtener una concentracin de slidos suspendidos en el fondo de 11 g L
1
y la de
entrada es de 3000 mgL
1
.
En la curva de la figura 2.23 se traza la lnea de fondo a partir de X = 11 g L
1
,
y que sea tangente a la curva. El intercepto en el eje de ordenadas da el flujo de
slidos lmites, que ser el aplicado, G
ap
= 7,3 kg
2
h
1
.
El rea del sedimentador, en funcin del flujo de entrada ser:
2
m 41 0 3
3 7
t
t
Q ,
,
Q
A = =
en que Q
t
est dado en m
3
h
1
para X
0
= 3 gL
1
se traza la lnea de operacin que intercepta a la lnea de fondo
en el punto P.
La velocidad de flujo de sobrenadante es la pendiente de la lnea OP
(figura 2.23).
U
0
= 1,82 m
3
m
2
h
1
La velocidad del flujo del fondo ser:
1 2 3
h m m 68 0

= = ,
X
G
U
r
ap
u
2.2.5. Evaluacin de la operacin de sedimentadores
El punto de situacin define el estado de operacin del sedimentador, y
los cambios de las condiciones de trabajo se reflejan en la curva de flujo de
la manera que se ilustra en las figuras 2.24 a 2.26.
1. Una disminucin del caudal de extraccin del fondo hace rotar, en el
sentido contrario a las manecillas del reloj, la lnea de fondo (figura 2.24),
disminuyendo el flujo de slidos a aplicar, G
ap
, y aumentando la concen-
tracin de slidos en la extraccin del fondo, X
r
.
2. Un aumento en el flujo de extraccin del sobrenadante, hace rotar, en el
sentido contrario a las manecillas del reloj, la lnea del sobrenadante y
mueve el punto de situacin hacia arriba (figura 2.25), mientras que la
pendiente de la lnea de fondo se mantiene constante.
3. Un aumento en la concentracin de los slidos que entran al sedimentador,
X
0
, mueve la lnea de operacin hacia la derecha y el punto de situacin se
mueve a lo largo de la lnea del sobrenadante. La lnea de fondo pasa por el
nuevo punto de situacin (figura 2.26).
75
Pretratamiento de Aguas Residuales
Fig. 2.24. Variacin del flujo de salida del fondo.
Fig. 2.25. Variacin del flujo del sobrenadante.
Fig. 2.26. Aumento de la concentracin de slidos a la entrada.
76
Menndez Gutirrez, C. y J. Prez Olmo
De las situaciones anteriores es necesario destacar un aspecto impor-
tante relativo a la lnea de fondo y su posicin en la curva de flujo. Cuando la
lnea de fondo:
1. Corta a la curva de flujo en un solo punto, se obtienen condiciones seguras de
trabajo.
2. Corta a la curva en un punto y es tangente a la misma en otro, prevalecen las
condiciones lmites de operacin.
3. Corta a la curva de flujo en dos puntos, existen condiciones de sobrecarga en
la operacin.
Otra informacin que puede obtenerse sobre la operacin del sedimenta-
dor al analizar las curvas anteriores es el relativo a los criterios de clarifica-
cin y espesamiento:
1. Cuando el punto de situacin queda ubicado por debajo de la curva de flujo,
la clarificacin del lquido sobrenadante est garantizada, y son las condicio-
nes de espesamiento las que controlan el proceso.
2. Cuando el punto de situacin est en la curva de flujo prevalecen las condi-
ciones crticas para la clarificacin.
3. Cuando el punto de situacin est ubicado por encima de la curva de flujo,
tanto la clarificacin como el espesamiento son ineficientes.
Notas bibliogrficas
1 ECKENFELDER, W. W.: Principles of Water Quality Management, CBI, Pu-
blishing Co. EE.UU., 1980.
2 BENEFIELD, L. AND C. RANDALL: Biological Process Design for Wastewater
Treatment, prentice Hall Series in Environmental Sciences, EE. UU., 1980.
3 PATTERSON, J.W. AND J.P. MENEZ: 'Design oI equalization basins, Am. Inst.
Chem. Engrs. Env. Prog., vol. 3, no. 1, 1984.
4 ROBIND, W. H.: 'The Theory oI the Design and Operation oI Setting Tanks,
Transaction Institute of Chemical Engineering, vol. 42, no. 158, 1964.
5 EKAMA, G. A.; Et al: Secondary Settling Tanks. Theory, Design and Opera-
tion, Sudafrica, 1984.
6 RITTMAN, B.: 'Aerobic Biological Treatment, Environmental Science Tech-
nology, vol. 21, no. 2, 1987.
6
77
Oxidacin Biolgica. Nitrificacin
CAPTULO3
OXIDACIN BIOLGICA NITRIFICACIN
3.1. PRINCIPIOS DE LA OXIDACIN BIOLGICA
Los procesos biolgicos, cualquiera que sea su nivel, implican un intercam-
bio continuo de sustancia y de energa.
Los elementos ms importantes que intervienen en el intercambio son el
hidrgeno, oxgeno, carbono, nitrgeno, fsforo y azufre. Otros muchos elemen-
tos tambin participan, pero en menor grado.
La energa para estos procesos puede obtenerse de tres fuentes que a su
vez sirven de criterio para clasificar los organismos que participan en cada caso:
Radiacin solar.
Compuestos orgnicos.
Compuestos inorgnicos.
En la biosfera la fuente fundamental de energa es la radiacin solar.
Los organismos fotoauttrofos fijan una fraccin pequea de la energa so-
lar mediante la formacin de compuestos orgnicos de alto contenido ener-
gtico (CHO) y produciendo oxgeno. El hidrgeno lo obtienen del agua, el
carbono del CO
2
y el fsforo, nitrgeno y azufre de las sales disueltas. La
disponibilidad de P y N generalmente es limitada, ya que el primero no es
fcil de encontrar en forma disuelta y la forma en que el segundo es asimi-
lado ms fcilmente es como NH
4
+
y NO
3

. Las restricciones en la disponi-


bilidad de estos dos elementos, N y P, constituyen usualmente un factor
limitante para la vida de los organismos auttrofos.
Los compuestos de alto contenido de energa sintetizados por los orga-
nismos aut trofos const ituyen la fuent e bsica de energa de los
hetertrofos para a su vez sintetizar molculas ms complejas, constitu-
yentes de la masa celular, incluyendo las protenas. Este proceso no es
muy eficiente desde el punto de vista energtico, y solo una fraccin de la
energa disponible aparece incorporada a la biomasa producida, el resto se
pierde en forma de calor.
78
Menndez Gutirrez, C. y J. Prez Olmo
Fig. 3.1. Cadena alimentaria. Relacin presa-depredador.
Esta biomasa a su vez sirve como fuente de nutriente y energa (presa) a
otros organismos que se alimentan de ella (depredador), y esta ltima es presa
de otro depredador. Cada transformacin presa-depredador va acompaada de
intercambio y prdida de la energa, aquella que se gener en la primera accin
de los organismos fotoauttrofos. Cuando las transformaciones son tales que la
energa contenida en el compuesto orgnico va reducindose, la vida de los
organismos hetertrofos tambin se reduce y se transforma en otras formas de
energa.
En el tratamiento biolgico de aguas residuales no se permite que la cadena
de vida descrita anteriormente se desarrolle hasta sus ltimas consecuencias,
sino que llegado a un punto, la biomasa portadora de la energa es separada
diariamente por procedimientos fsicos.
Los organismos quimioauttrofos obtienen la energa de la oxidacin de los
compuestos inorgnicos. En el campo del tratamiento de las aguas residuales
cobran especial importancia dentro de este grupo las bacterias nitrificantes. Estos
organismos son estrictamente aerobios y obtienen la energa de su accin oxidante
sobre el NH
4
+
y el NO
2

.
En el tratamiento de las aguas residuales las bacterias nitrificantes cons-
tituyen un vnculo muy importante en la eliminacin del nitrgeno del agua al
convertir el N-NH
3
a N-NO
3

y posteriormente, por medios adecuados,


inducir a los organismos hetertrofos a utilizar el NO
3

en su metabolismo
de material carbnico. En esta accin el NO
3

es reducido a N
2
que escapa
como gas.
3.2. METABOLISMO
Los microorganismos requieren de una fuente de energa y carbono para la
sntesis de nuevas clulas as como de nutrientes inorgnicos para sus diferen-
tes funciones.
El metabolismo es la combinacin de un conjunto de transformaciones
bioqumicas necesarias para la sntesis celular (anabolismo) y la produccin de
energa (catabolismo) que ocurren a travs de procesos enzimticos. De hecho,
79
Oxidacin Biolgica. Nitrificacin
los diferentes microorganismos pueden ser clasificados a partir de la forma en
que obtienen el carbono y la energa.
En el tratamiento biolgico de aguas residuales la energa est presente
inicialmente en los compuestos orgnicos carbonosos y nitrosos. Esta se con-
vierte en otras formas con las consecuentes prdidas de energa. La energa
carbonosa es asimilada por los organismos hetertrofos, quienes descomponen
los compuestos proteicos en sus constituyentes carbonosos y nitrosos. El amonio
obtenido es utilizado a su vez como fuente de energa por las bacterias nitrificantes.
3.2.1. Mecanismo para la obtencin de energa
por los organismos
La reaccin global involucrada en el metabolismo de los microorganismos
hetertrofos para la obtencin de energa puede esquematizarse:
Energa
clulas
Nuevas
O H CO O
orgnicos
Compuestos
2 2
ismos microorgan
2
+ + + +
En detalle lo que ocurre es que las sustancias orgnicas son descompuestas
por la accin de los microorganismos dando lugar a la liberacin de H
+
, CO
2
y
electrones. Se dice, por tanto, que la sustancia se oxida.
Los electrones (e

) y el H
+
liberados son transferidos a otra sustancia que,
por tanto, es la aceptora de e

. Las reacciones de oxidacin-reduccin contem-


plan la transferencia de electrones de una sustancia reducida (donador de e

) a
una sustancia oxidada (consumidor de e

). El sustrato o alimento es la sustancia


donante.
El metabolismo hetertrofo utiliza los compuestos orgnicos como do-
nantes de e

. En el auttrofo son los compuestos inorgnicos los donantes.


Por otro lado, el aceptor de electrones depende de que el proceso ocurra en
un medio aerobio, anxico o anaerobio. Mientras que el oxgeno es el aceptor
ltimo o terminal de la cadena oxidacin-reduccin en un medio aerobio, el
par nitrito-nitrato es el aceptor en medio anxico y el CO
2
y SO
4
2
lo son en
el anaerobio.
La cantidad de energa liberada durante el proceso de oxidacin reduccin
depende tanto del donante como del aceptor de electrones. Cuando el donante
es carbonoso y el aceptor final es el oxgeno, la energa liberada es relativamen-
te alta y 5 % mayor que cuando el aceptor
1
es el NO
2

o NO
3

. La menor
cantidad de energa liberada corresponde al CO
2
y SO
4
2
como aceptores de
electrones.
Una fraccin de la energa liberada durante los procesos descritos anterior-
mente es utilizada para la sntesis de nuevas clulas, y el resto est destinado a
80
Menndez Gutirrez, C. y J. Prez Olmo
satisfacer los requerimientos de la supervivencia y el desarrollo de la propia
clula (figura 3.2).
Cuando la materia orgnica es completamente oxidada a CO
2
y el O
2
es el
aceptor de e

, la masa de oxgeno consumida da la medida estequiomtrica de


los moles de e

transferidos. De aqu, la capacidad de cualquier sustrato como


donante de e

puede ser medida en trminos de oxgeno requerido para oxidar la


materia orgnica a CO
2
, este es el fundamento de la DQO.
Aun cuando no es objetivo de este texto profundizar en las reacciones
metablicas, se ha considerado oportuno esquematizar en la figura 3.3 las suce-
sivas etapas de oxidacin reduccin que ocurren desde que la materia orgnica
es inicialmente oxidada hasta que los electrones son transferidos hacia el ltimo
aceptor.
Fig. 3.2. Energa liberada durante la oxidacin del sustrato y su trans-
formacin.
De acuerdo con el esquema de la figura 3.3, la energa almacenada en la
materia orgnica AH
2
se libera durante la oxidacin biolgica por dehidrogenacin
del sustrato mediante la accin del NAD (dinucletido de nicotinamida y de
adenina). El FAD (flavin adenn-dinucletido) es una flavoprotena que adems
de transportar hidrgeno, es un intermediario entre las deshidrogenasas y los
citocromos para el transporte de electrones. Por su parte, los citocromos son
protenas transportadoras de electrones que se encuentran solamente en las
clulas aerobias.
Como se aprecia en el esquema, se han representado tres puntos de produc-
cin de ATP (trifosfato de adenosina) a partir de ADP (difosfato de adenosina).
El ATP es el transportador de energa qumica ms importante de las clulas de
todas las especies vivientes. A medida que el ATP transfiere su energa a otras
81
Oxidacin Biolgica. Nitrificacin
molculas, se transforma en ADP. Este a su vez, acepta energa qumica de las
transformaciones que le preceden y se transforma nuevamente en ATP, ya sea
a expensas de organismos auttrofos o hetertrofos.
Fig. 3.3. Esquema de los procesos redox hasta la transferencia de elec-
trones al ltimo aceptor. Metabolismo aerobio, anaerobio y anxico.
De esta manera, las reacciones con el ltimo aceptor y la energa asociada
a ellas pueden resumirse como se muestra a continuacin:
(aerobio) AH
2
+ O
2
CO
2
+ H
2
O + E
1
(3.2)
(anxico) AH
2
+ NO
3

N
2
+ H
2
O + E
2
(3.3)
AH
2
+ SO
4
2
H
2
S + H
2
O + E
3
(3.4)
(anaerobio) AH
2
+ SO
4
2
H
2
S + H
2
O + E
4
(3.4)
AH
2
+ CO
2
CH
4
+ H
2
O + E
5
(3.5)
AH
2
+ B BH
2
+ A + E
6
(3.6)
82
Menndez Gutirrez, C. y J. Prez Olmo
donde:
E
1
> E
2
> E
3
> E
4
> E
5
> E
6
puede apreciarse adems que la reduccin del sulfato ocurre tanto en los
procesos anaerobios como en los anxicos, aunque en los segundos se redu-
cen preferentemente los nitratos antes que los sulfatos.
El ATP y la actividad dehidrogenasa son los mejores indicadores de la biomasa
activa en los procesos de tratamiento biolgico.
3.3. BIODEGRADABILIDAD DE LAS AGUAS RESIDUALES
La purificacin biolgica puede ser aplicada efectivamente solo en aquellos
casos en los que el agua residual contenga compuestos orgnicos biodegradables.
3.3.1. Condicin de degradacin biolgica de compuestos
orgnicos
Los compuestos orgnicos solubles presentes en aguas residuales pueden
ser clasificados en cuatro grandes grupos de acuerdo con su condicin de de-
gradacin biolgica y toxicidad:
Compuestos biolgicamente degradables y no txicos (sacridos, aminocidos,
cidos grasos).
Compuestos biolgicamente degradables, pero txicos a altas concentracio-
nes (fenol, formaldehdo).
Compuestos no degradables biolgicamente y no txicos (cido hmico, ci-
dos lignosulfnicos, colorantes azo).
Compuestos no degradables biolgicamente y txicos a bajas concentracio-
nes (pesticidas, DDT).
Un agua residual en la que todos sus constituyentes son biodegradables
no txicos o solamente txicos a altas concentraciones tiene una relacin
DBO
5
/DQO en el intervalo de 0,55-0,70. Esta relacin es un buen indicador
de la posibilidad del tratamiento biolgico de un agua residual dada. Mien-
tras ms baja es esta relacin, mayor ser la proporcin de compuestos no
degradables en el agua residual. Si la relacin es 0,2 menor, se est en
presencia de un agua residual constituida, fundamentalmente, por compues-
tos no degradables por medios biolgicos, en este caso, la purificacin biol-
gica sola no es suficiente.
Las causas por las que un compuesto puede ser no biodegradable son
diversas y no todas conocidas. Los compuestos constituidos por cadenas
2
83
Oxidacin Biolgica. Nitrificacin
alifticas son degradables, sin embargo, el crecimiento de estas es disperso.
Crecimiento disperso de las cadenas alifticas:
CH
2
.(CH
2
)
X
CH
3
SO
3
Na
CH
2
.(CH
2
)
X
CH
3
CH
3
SO
3
Na
CH
2
CH
2
-(CH
2
)-CH
3

Degradable No degradable
Otra causa de no biodegradabilidad de un compuesto puede encontrarse
en la sustitucin de un anillo aromtico. Mientras que el 2,4-dinitrofenol pue-
de ser degradable biolgicamente, la sustitucin del grupo nitro de la posi-
cin cuatro a la seis hace al compuesto no biodegradable.
Sustituciones en el anillo aromtico
Degradable No degradable
OH
NO
2
NO
2
OH
NO
2 O
2
N
En general, los compuestos de alta masa molecular y/o de estructura com-
pleja son no biodegradables o de difcil degradacin. En estos casos, es ms
aconsejable el empleo de mtodos fsico-qumicos de tratamiento.
En la figura 3.4 se muestran los resultados obtenidos al analizar por
cromatografa de gel un agua residual antes y despus de ser sometida a un
tratamiento biolgico.
3
En la parte (a) de la figura se aprecian dos picos que
caracterizan a compuestos de alta masa molecular (1) y compuestos de baja
masa molecular (2). En (b), puede observarse que despus de un tratamiento
biolgico, la curva que caracteriza a los compuestos de baja masa molecular ha
desaparecido, pero persiste la de aquellos de mayor masa molecular, y que no
son removidos mediante este tipo de tratamiento.
Los compuestos orgnicos remanentes en un agua residual que ha sido
sometida a un buen tratamiento biolgico (DBO soluble < 10 mgL
1
) son
fundamentalmente no biodegradables y fcilmente determinados con la DQO.
Estos compuestos pueden clasificarse en dos subgrupos (figura 3.5). El pri-
5
84
Menndez Gutirrez, C. y J. Prez Olmo
mer subgrupo est constituido por la fraccin no biodegradable contenida en
el residual original y que pasa a travs del tratamiento sin experimentar
ningn cambio (DQO
ND
). Al segundo subgrupo pertenecen aquellos com-
puestos que son producidos por los propios microorganismos que realizan la
accin depuradora, como consecuencia de su metabolismo. Su concentra-
cin vara entre 1 y 5 % de la DQO que es removida.
Fig. 3.4. Curva de elusin de cromatografa de gel. a) Antes del trata-
miento biolgico, b) despus del tratamiento biolgico.
85
Oxidacin Biolgica. Nitrificacin
Debe ser destacado que durante el tratamiento biolgico solo se remueven
aquellos compuestos de relativa baja masa molecular.
DQO
D
+ DQO
ND
(0,01-0,05) DQO
D
+ DQO
ND
Fig. 3.5. Clasificacin de los compuestos de acuerdo con su biodegrada-
bilidad.
3.3.2. Purificacin biolgica de las aguas residuales
Cuando un agua residual se pone en contacto con los microorganismos pre-
sentes en la unidad de tratamiento, los compuestos orgnicos pueden ser remo-
vidos a travs de diversos mecanismos.
Los slidos suspendidos y coloidales son removidos por adsorcin y
coagulacin. Si son degradables, son hidrolizados por las exoenzimas. Los
productos de esta hidrlisis son enzimticamente transportados al interior
de las clulas. Si son inertes, pasan a constituir la fraccin inerte de la
biomasa.
Por su parte, los compuestos que aunque solubles son de alta masa molecular
deben tambin ser previamente hidrolizados antes de que puedan transportarse
a travs de la membrana bacteriana.
Los compuestos de baja masa molecular son transportados directamente al
interior de las clulas por las enzimas correspondientes.
Los compuestos orgnicos removidos de las aguas residuales son par-
cialmente oxidados, y utilizados en parte en la sntesis de constituyentes de
las clulas y en nuevas clulas (figura 3.6).
Los principales productos de la oxidacin son CO
2
, H
2
O y NH
3
. El propsito
fundamental de esta oxidacin es obtener la energa necesaria para las reaccio-
nes de sntesis. En principio los productos de la sntesis se almacenan como
86
Menndez Gutirrez, C. y J. Prez Olmo
productos de reserva (polisacridos, lpidos). Con posterioridad son utilizados
para la constitucin de nuevas clulas.
Fig. 3.6. Mecanismo de la oxidacin biolgica.
No obstante que la produccin de biomasa generalmente no es el objetivo
del tratamiento biolgico de los residuales (aunque puede serlo en alguna situa-
cin particular), su formacin en cierta cantidad es inevitable. En procesos
aerobios Mc Carty
1
ha obtenido valores de rendimiento (Y) (biomasa produci-
da/DBO removida), que varan entre 0,30 y 0,51 para glucosa, anilina, lactato y
acetato. Por otra parte, Servizi y Bogan han presentado valores de rendimiento
de 0,39 para una variedad de compuestos, mientras que EckenIelder y O`Connor
muestran valores de este parmetro entre 0,37 y 0,46 para muchos compuestos
biodegradables.
Para los procesos anaerobios, como la produccin de energa es menor,
tambin es menor la cantidad de esta que se dedica a la produccin de biomasa.
En estos casos, el rendimiento apenas alcanza valores comprendidos entre 0,032
y 0,27.
6
3.3.3. Sustrato exgeno y endgeno
Teniendo en cuenta que los compuestos orgnicos contenidos en el agua
residual sirven como fuente de energa a los microorganismos, son denominados
sustratos. Los sustratos se clasifican como exgenos (fuera de la clula
bacteriana) y endgenos (en el interior de la clula bacteriana).
Cuando el sustrato exgeno se agota, los microorganismos oxidan el
sustrato endgeno. En una primera etapa estos sustratos endgenos sirven
como material de reserva y cuando se agota, comienza la oxidacin de las
protenas. Si los microorganismos constituyentes de la biomasa se mantie-
5
4
87
Oxidacin Biolgica. Nitrificacin
nen durante un cierto tiempo sin sustrato exgeno, muchos de ellos morirn.
La masa remanente puede llegar a contener menos del 10 % en peso de
organismos vivos, y ser relativamente estable a la descomposicin poste-
rior. Este fenmeno es utilizado en la prctica para la estabilizacin aerobia
de los lodos.
La oxidacin del sustrato exgeno puede ser descrita segn la ecuacin:
O H
2
1
CO O
2
1
4
1
O H C
2 2 2
Y X Z Y X
z y x
+ |
.
|

\
|
+ +
(3.7)
La oxidacin del sustrato endgeno puede representarse:
3 2 2 2 2 7 5
NH O H 2 CO 5 O 5 NO H C n n n n + + + (3.8)
La frmula emprica C
5
H
7
NO
2
representa la composicin media de
la materia voltil celular. Esta biomasa contiene 12,3 % de nitrgeno y
tericamente requiere 1,42 g de oxgeno por cada gramo de biomasa
totalmente oxidado. Aunque la frmula emprica anterior, de Hoover y
Porges, es la ms difundida, hay muchas otras que tambin se utilizan. No
obstante en la prctica el equivalente de oxgeno de biomasa voltil vara en
el intervalo comprendido entre 1,3 y 1,5 gg
1
.
3.4. CINTICA DEL CRECIMIENTO BIOLGICO
Y REMOCIN DE SUSTRATO
3.4.1. Crecimiento y multiplicacin de microorganismos
Cultivo discontinuo de poblaciones puras de microorganismos
Cuando los microorganismos se ponen en contacto con un sustrato al cual
estn bien adaptados, crecen y se multiplican.
Si se parte de presuponer que para el tiempo inicial t = 0 la concentra-
cin de microorganismos es X
0
, despus de un cierto tiempo la concentracin
inicial se duplica. El tiempo de duplicacin, tg, es llamado tiempo medio de
generacin. Despus de transcurrido el tiempo de generacin t = tg, los
microorganismos se duplican nuevamente. Esto puede demostrarse mediante
el esquema siguiente:
t = 0 X
0
tg 2X
0
2
1
X
0
2tg 4X
0
2
2
X
0
3tg 8 X
0
2
3
X
0
n tg 2
n
X
0
7
88
Menndez Gutirrez, C. y J. Prez Olmo
As para el tiempo t = ntg la concentracin de microorganismos ser:
0 0
2 2 X X X
tg
t
n
t
= =
(3.9)
Aplicando logaritmos a la ecuacin 3.9:
0
ln 2 ln ln X
t
t
X
g
t
+ =
(3.10)
Escribiendo:
tg
,
tg
n
69 0 2 ln
= =
(3.11)
Sustituyendo en la ecuacin 3.10 se obtiene,
0
ln ln X nt X
t
+ = (3.12)
La ecuacin 3.12 tambin puede escribirse:
t
0 t
e X X = (3.13)
Conociendo el valor de se puede, mediante la ecuacin 3.13, calcular el
valor de la concentracin de microorganismos a cualquier tiempo t.
Diferenciando con respecto al tiempo la ecuacin 3.13, se obtiene la ecua-
cin cintica general de crecimiento.
nX
dt
dX
=
dt
dX
X
n
1
= (3.14)
El trmino es denominado velocidad especfica de crecimiento. A partir de
la ecuacin 3.14 puede concluirse que solo depende del tiempo de generacin,
tg, de un microorganismo dado. Sin embargo, esta conclusin es vlida
solamente para altas concentraciones de sustrato.
Para bajas concentraciones de sustrato, se hace dependiente de la misma
de acuerdo con la bien conocida ecuacin de Jacques Monod:
S K
S
n n
S
mx
+
=
(3.15)
donde:
S: concentracin de sustrato limitante (mgL
1
).

mx
: Velocidad especfica de crecimiento mxima h
1
.
K
S
: Constante de saturacin (mgL
1
).
K
S
: V
S
= S para =
mx
89
Oxidacin Biolgica. Nitrificacin
Fig. 3.7. Representacin grfica de la ecuacin de Monod.
Combinando las ecuaciones 3.14 y 3.15, se obtiene que:
X
S K
S
n
dt
dX
S
mx
+
=
(3.16)
Aqu se aprecia que la velocidad de crecimiento de los microorganismos es
funcin de su concentracin y de la concentracin de sustrato.
Cuando el sustrato se encuentra en condiciones lo suficientemente altas
como para que su variacin no afecte la velocidad de crecimiento, la ecuacin
3.16 se transforma:
X n
dt
dX
mx
= (3.17)
O sea, que la velocidad de crecimiento en un instante dado es solo funcin
de la concentracin de microorganismos multiplicado por la velocidad especfi-
ca mxima de crecimiento.
Todo el desarrollo anterior fue logrado inicialmente para cultivos puros y
sustratos de un solo componente. Sin embargo, su uso se ha extendido para el
caso de cultivos mixtos y sustratos multicomponentes, que es la situacin que
prevalece durante el tratamiento biolgico de aguas residuales.
Cultivo de poblaciones mixtas de microorganismos
La dinmica poblacional de los cultivos es un proceso complejo y no
totalmente conocida an.
Al aplicar la ecuacin de Monod a cultivos mixtos, debe tenerse en cuenta
que las constantes de crecimiento
mx
y K
S
representan valores promedio de
las constantes de crecimiento de las especies individuales y de su proporcin en
el cultivo.
Los cambios en la poblacin se reflejan en cambios en las constantes de
crecimiento.
90
Menndez Gutirrez, C. y J. Prez Olmo
Fig. 3.8. Relacin entre
max
y K
S
para varios sustratos. 1. Glucosa
2. Sorbitol. 3. Sacarosa. 4. Histidina. 5. Galactosa. 6. Lactosa.
En la figura 3.8 se muestra que mientras mayor es el valor de K
S
mayor es

mx
. Sin embargo, hay compuestos para los que no se ha hallado correlacin.
Tal es el caso, por ejemplo, de los cidos actico y propinico y la fenilalanina.
Por otro lado los cidos glutmico y crico muestran relaciones inversas.
Fig. 3.9. Relacin entre y la concentracin de sustrato para dos
microorganismos.
A modo de ejemplo de lo anterior puede citarse un caso extremo de un
cultivo formado por dos especies de microorganismos denominados por A y B
(figura 3.9), cuando se utiliza la lactosa como sustrato. La especie A tiene un
valor de
mx
= 0,2 h
1
y K
S
= 1 mgL
1
. Para la especie B estos valores son
respectivamente, 0,5 h
1
y 50 mgL
1
.
91
Oxidacin Biolgica. Nitrificacin
En la figura 3.9 puede apreciarse adems que para bajas concentraciones
de sustrato la velocidad de crecimiento de la especie A es mayor que la de B.
Consecuentemente, la proporcin de A en el cultivo ser mayor que la
de B. Para concentraciones de sustratos cercanas a 35 mgL (punto de
interseccin de ambas curvas), ambas especies alcanzan la misma velocidad
de crecimiento y su proporcin en el cultivo mixto es la misma. A altas
concentraciones la especie B alcanza mayor velocidad de crecimiento, y
consecuentemente se encontrar en mayor proporcin.
El significado prctico de esta conclusin terica es que queda demos-
trada una de las formas en las que se puede inhibir o evitar el crecimiento
de algunos microorganismos indeseables en la unidad de tratamiento, por
ejemplo, los filamentosos en los procesos de lodo activado y lagunas
aereadas.
3.4.2. Cintica de la remocin de los compuestos orgnicos
de las aguas residuales mediante cultivos mixtos
Durante aos se han desarrollado diferentes modelos matemticos para
explicar el mecanismo de remocin de la DBO en los procesos de trata-
mientos biolgicos.
8, 9
Todos estos modelos coinciden en que la velocidad de remocin de DBO
por unidad de biomasa (A DBO/SSV.t) es constante cuando se tienen valo-
res altos de la DBO. Esta velocidad especfica de remocin de la DBO se
mantiene constante hasta que se alcanza un valor de concentracin de DBO,
a partir del cual la velocidad de remocin se hace dependiente de la concen-
tracin y decrece con ella.
Hay otros casos en los que ha quedado demostrado que la remocin de
sustrato (DBO DQO) sigue una cintica de orden cero, o sea, es indepen-
diente de la concentracin de DBO DQO en todo el intervalo de concen-
tracin.
10
Uno u otro comportamiento se debe a que el contaminante est
constituido por una mezcla de sustancias o por una sola sustancia.
Para el estudio cintico de la remocin de la DBO es conveniente clasi-
ficar los sustratos como:
Sustratos de un solo componente que pueden ser transportados directa-
mente al interior de la clula.
Sustratos multicomponentes, constituidos por una mezcla de sustratos sim-
ples.
1
92
Menndez Gutirrez, C. y J. Prez Olmo
Sustratos de un solo componente
Como ya ha sido expresado, la remocin de sustrato de una solucin, siem-
pre va acompaada de crecimiento de biomasa, existiendo una relacin mate-
mtica que vincula ambos fenmenos:
dt
dS
Y
dt
dX
= (3.18)
donde:
Y: Constante de rendimiento o de produccin de biomasa (gg
1
).
A partir de la ecuacin que describe el crecimiento de biomasa en funcin
de la concentracin de sustrato,
X
S K
S
n
dt
dX
S
m
+
=
y de la ecuacin 3.18, puede obtenerse:
S K
S
X
Y
n
dt
dS
S
m
+
=
(3.19)
en tratamiento de residuales es frecuente encontrar que K
S
<< S, y la ecuacin
3.19 queda:
X
Y
n
dt
dS
m
= (3.20)
o tambin:
KX
dt
dS
= (3.21)
donde:
( )
1
= d
Y
n
K
m
De acuerdo con la ecuacin 3.21 los sustratos de un solo componente son
removidos siguiendo una cintica de orden cero con respecto a la concentracin
de sustrato (figura 3.10).
93
Oxidacin Biolgica. Nitrificacin
Fig. 3.10. Variacin de la concentracin de DBO con respecto al tiempo,
para sustratos de un solo componente.
Sustratos multicomponentes
La casi totalidad de las aguas residuales estn constituidas por sustratos
multicomponentes, o sea, una mezcla de compuestos que integralmente se re-
portan como DBO DQO.
Cada uno de los compuestos que conforman el residual est presente en
concentraciones distintas, y se remueven de acuerdo con una cintica de orden
cero aunque a distintas velocidades, como se ilustra en la figura 3.11(a), donde
cada recta representa la variacin, con respecto al tiempo, de la DQO de cada
uno de los constituyentes individuales.
En la figura 3.11(b) se aprecia que para un tiempo cualquiera la velocidad de
remocin del sustrato multicomponente y la concentracin total del sustrato (ex-
presada como DQO) es la suma de las velocidades de remocin de los sustratos
simples y de la concentracin de los sustratos respectivamente. En la medida
que un sustrato simple se agota, la velocidad de remocin disminuye instant-
neamente, dando origen a un punto de ruptura en la curva de remocin.
Si el nmero de componentes es suficientemente alto, como es el caso de la
prctica del tratamiento de las aguas residuales, el punto de ruptura puede ser
sustituido por una disminucin suave. Esta curva puede ser descrita por una
ecuacin diferencial general que representa la cintica de una reaccin qumica
simple:
n
n
S X k
dt
dS
0
= (3.22)
donde:
dt
dS
: Velocidad de remocin (mgL
1
d
1
).
k
n
: Constante de velocidad de remocin (d
1
).
94
Menndez Gutirrez, C. y J. Prez Olmo
S: Concentracin de DBO DQO (mgL
1
).
X
0
: Concentracin inicial de biomasa (mgL
1
).
n: Orden de la reaccin.
Este tipo de ecuacin ha sido aplicada durante aos para los procesos
microbiolgicos. Sin embargo, el punto dbil en la aplicacin de esta ecuacin,
es que presupone un incremento en la velocidad de remocin ante cualquier
incremento en la concentracin de sustrato.
Experimentalmente ha quedado demostrado que si se incrementa la con-
centracin de un sustrato multicomponente, y se mantiene constante la propor-
cin de cada componente en el sustrato, no se apreciar incremento en la
velocidad de remocin.
De la figura 3.11 se aprecia que la velocidad instantnea de remocin de un
sustrato multicomponente est relacionada con el nmero de sustratos simples
remanentes.
Fig. 3.11. Ejemplo terico de remocin de sustratos multicomponentes.
95
Oxidacin Biolgica. Nitrificacin
Al considerar la velocidad de remocin inicial, que es constante para un
cultivo y mezclas de sustratos especficos, puede expresarse la velocidad en
funcin de nmero de componentes remanentes en relacin con la situacin
original.
As:
|
.
|

\
|
=
M
m
f r
(3.23)
donde:
M: Nmero de componentes inicialmente presentes.
m: Nmero de componentes individuales remanentes.
r: Velocidad de remocin relativa.
En general, lo que ocurre es que no se conoce el nmero de componentes
originalmente presentes, ni la ley que describe su remocin de la mezcla.
Suponiendo un nmero grande de componentes, puede sustituirse el nmero
de componentes por la concentracin.
La funcin f expresa adems, en alguna forma, el decrecimiento de la velo-
cidad de remocin causada por el nmero reducido de componentes y, por tanto,
la disminucin de la concentracin total de sustrato con el tiempo. Para reflejar
esto, un exponente similar al de orden de reaccin puede utilizarse.
La ecuacin cintica se puede expresar:
n
n n
n
n
S
S
X
k
S
S
X k
dt
dS
0
0
0
0
=
|
.
|

\
|
=
(3.24)
donde:
S
0
: Concentracin inicial de DBO DQO.
S: Concentracin final de DBO.
X
0
: Concentracin de biomasa.
k
n
: Velocidad especfica de remocin de DBO.
n: Orden de la reaccin.
La ecuacin 3.24 tambin puede expresarse,
n
S
S
k
X 0
AS
|
.
|

\
|
=

0
(3.25)
Q
V
0 =
96
Menndez Gutirrez, C. y J. Prez Olmo
El modelo de velocidad de reaccin representado por la ecuacin 3.24 fue
inicialmente descrito por Grau y col.
11
Aunque bastante general, el modelo cintico
de Grau no es ms que otro de los muy diversos modelos que pretenden explicar
la cintica de los procesos biolgicos de tratamiento de aguas residuales. Entre
muchos otros modelos descritos pueden ser citados el de Eckenfelder y el de
Lawrence y McCarty.
La ecuacin que responde al modelo cintico de Eckenfelder ha tenido un
uso muy difundido a travs de los aos. Originalmente formulada para una cintica
de primer orden con respecto a la DBO, ha sido posteriormente utilizada para
rdenes de reaccin superior al primero,
( )
X S K
dt
dS
n
= (3.26)
donde:
n: Orden de reaccin.
Fig. 3.12. Diferentes valores de la constante cintica K que se obtiene al
aplicar el modelo de primer orden de Eckenfelder a un agua residual de
DBO variable.
Otra forma de expresar la ecuacin 3.26 es,
( )
n
S K
X 0
AS
=
(3.26 a)
La mayor limitacin del modelo de Eckenfelder radica en que para aguas
residuales cuya DBO es variable, se obtiene distintos valores de K en depen-
dencia de los valores de la DBO inicial (S
0
). Figura 3.12. Esta dificultad es
salvada en el modelo cintico de Grau.
Por otra parte, el modelo de Lawrence y McCarty es aplicable para cintica
de orden intermedio entre cero y primer orden,
12
13
97
Oxidacin Biolgica. Nitrificacin
X
S K
S
k
dt
dS
S
m
+
=
(3.27)
S K
S
k
X 0
AS
S
m
+
=
(3.27 a)
Cualquiera que sea el modelo cintico que se desee emplear, debe compro-
barse en qu medida se cumple en el caso particular bajo estudio.
3.4.3. Clculo de la constante de velocidad
Para estimar la constante de velocidad de remocin de DBO deben reali-
zarse corridas experimentales a diferentes tiempos de retencin y conocer para
cada caso, la concentracin de biomasa, medida como slidos suspendidos vol-
tiles, as como la DBO inicial y final del proceso.
En el ejemplo 3.1 se ilustra el procedimiento de clculo.
Ejemplo 3.1
Para determinar la constante del modelo cintico de Grau se realizaron
corridas experimentales en un reactor de 10 L con alimentacin continua. Los
resultados obtenidos aparecen resumidos en la tabla 3.1.
Determine el valor de la constante asumiendo que la reaccin de degrada-
cin es de primer orden con respecto a la DBO.
Tabla 3.1. Datos experimentales
Q S
0
S X Corrida
Experimental
No.
Ld
-1
mgL
-1
1 13 900 63 7247
2 15 950 133 4126
3 30 1200 420 3151
4 30 1200 300 4959
5 40 1500 750 2500
6 60 1500 900 2582
De acuerdo al modelo de Grau:
0
S
S
k
X 0
AS
=
98
Menndez Gutirrez, C. y J. Prez Olmo
graficando
0
S
S
vs
X 0
AS
, puede obtenerse el valor de k de la pendiente de la
recta correspondiente.
Los datos de la tabla 3.1a se arreglan segn:
Tabla 3.1.a. Transformacin de la tabla 3.1
Corrida 1 2 3 4 5 6
X 0
AS 0,15 0,30 0,75 0,55 1,20 1,40
0
S
S 0,07 0,14 0,35 0,25 0,50 0,60
Fig. 3.13. Clculo de la constante de remocin de DBO para el modelo
de Grau.
3.5. NITRIFICACIN-DESNITRIFICACIN
Es muy til relacionar las concentraciones de nitrgeno con las de DBO. La
tabla 3.2 da algunos valores en dependencia del origen del agua residual. El
valor 0,2 puede considerarse representativo de este conjunto de datos. A falta
de disponer de anlisis ms precisos, puede ser adoptada esta relacin como
buena para el clculo de instalaciones de tratamiento. Generalmente la incorpo-
racin de aguas residuales industriales a las municipales, favorecen la disminu-
cin de esa relacin.
Se obtiene la recta de la figura 3.13 cuya pendiente es el valor de la cons-
tante k (k = 2,3 d
1
):
99
Oxidacin Biolgica. Nitrificacin
Tabla 3.2. Relacin N/DBO
5
para diferentes tipos de aguas residuales
Formas del nitrgeno y su evolucin
En las aguas residuales de origen domstico, la forma ms comn en que se
presenta el nitrgeno es como nitrgeno orgnico (N-org) y nitrgeno amoniacal
(N-NH
3
). En esta agua los nitritos y nitratos aparecen generalmente en concen-
traciones casi siempre despreciables. La presencia de nitritos y nitratos en las
aguas residuales sin tratar casi siempre es debida a contaminacin de origen
industrial. Cuando la concentracin de N-NH es elevada, se produce una ac-
cin inhibidora superior sobre un grupo de las bacterias nitrificantes, las
nitrobacterias, que sobre otro, las nitrosomonas.
El nitrogeno total Kjeldahl 'N-NTK representa el total de nitrogeno en
forma orgnica y amoniacal y no incluye nitritos ni nitratos.
El nitrgeno orgnico puede ser inerte o degradable, con un orden de mag-
nitud del 12 % (10 % en suspensin y 2 % soluble) y 88 % respectivamente. Las
dos ms importantes fuentes de contaminacin nitrogenada son los vertidos de
protenas y urea.
La fraccin orgnica degradable se transforma, por accin enzimtica, en
forma amoniacal, mediante el proceso conocido como amonificacin. La cintica
de esta reaccin es muy rpida para la urea, pero inferior para las protenas.
Aun as, se puede considerar que al final del tratamiento no subsiste ms que la
fraccin orgnica inerte.
De lo anterior puede concluirse que, con el tratamiento por lodos activa-
dos, no es necesario conocer la proporcin entre las formas orgnicas y
amoniacales. En cambio, en los sistemas de tratamiento del tipo cultivos
fijos (lechos bacterianos, discos biolgicos, filtros, etc.), en los cuales el
tiempo de retencin en el reactor es muy corto, puede no completarse la
amonificacin y si puede cobrar importancia conocer la proporcin en que
se encuentran el N-org y el N-NH .
El nitrgeno amoniacal procede de la amonificacin del nitrgeno orgnico
que puede efectuarse en la propia red de colectores y que afecta a prcticamente
la totalidad de la urea y de los dems compuestos orgnicos fcilmente
Procedencia N/DBO
5
Municipal 0,17-0,20
Lecheras 0,05-0,04
Cerveceras 0,25
Papeleras 0,1-0.02
Centrales azucareros 0,1-0,2
Tintoreras 0,1-0,05
3
3
100
Menndez Gutirrez, C. y J. Prez Olmo
degradabIes. Una parte de este nitrgeno se incorpora a la masa bacteriana en
la constitucin de las clulas a travs del proceso de asimilacin.
Cuando las condiciones ambientales son favorables, en particular el tiempo
de retencin de la biomasa y la temperatura, el nitrgeno llega a oxidarse a la
En un reactor anxico algunas bacterias hetertrofas son capaces de con-
sumir oxgeno de los nitratos, los cuales se reducen liberando nitrgeno libre.
Este proceso se denomina denitrificacin.
Todos estos procesos se representan en el diagrama de bloques de la
Figura 3.14.
Fig. 3.14. Ciclo del nitrgeno en el tratamiento.
3.5.1. Nitrificacin
En el estudio de la nitrificacin es importante diferenciar entre las funciones
de oxidacin del carbono y del nitrgeno propiamente dicho. Ambos procesos
pueden llevarse a cabo en una sola etapa o en etapas separadas, tanto en siste-
mas de biomasa en suspensin como en lecho fijo.
En casi todos los procesos biolgicos estn presentes las bacterias
nitrificantes, sin embargo para que la nitrificacin tenga lugar es necesario que
forma de nitrato en el proceso biolgico aerobio identificado como nitrificacin.
101
Oxidacin Biolgica. Nitrificacin
existan, entre otras cosas, condiciones adecuadas para el crecimiento de las
bacterias nitrificantes. En otro caso, prevalecer el crecimiento de las bacterias
hetertrofas. La nitrificacin se propicia disminuyendo la carga orgnica de los
procesos biolgicos.
El N-NH
3
es transformado en nitrito y nitrato por bacterias autotrficas
constituyendo el oxgeno requerido para esta oxidacin, una demanda adicional
a la que posee la materia orgnica que es normalmente estabilizada durante los
tratamientos biolgicos.
La primera etapa de la nitrificacin tiene su origen en la accin de las bac-
terias Nitrosomonas, siendo las Nitrobacter las responsables de la transforma-
cin posterior del N-NO
3

. Ambas reacciones de oxidacin se representan a


continuacin:

2 2 2 4
NO O H 2H O 3/2 NH + + +
+ +
as Nitrosomon
(3.28)

3 2

2
NO O 1/2 NO +
r Nitrobacte
(3.29)
A partir de la estequiometra de estas ecuaciones puede deducirse que se
requieren 4,6 mg de oxgeno para la oxidacin completa de cada mg de N-NH
3
.
Si se acepta que el efluente de un tratamiento secundario convencional po-
see aproximadamente una concentracin de 25 mgL
1
de N-NH
3
, esto tiene
como consecuencia una demanda de oxgeno al cuerpo receptor de 114 mgL
por concepto de la ulterior oxidacin a NO
3

. Si esta demanda se compara con


la que normalmente tienen estos efluentes (demanda carboncea), aproximadamente
50 mgL
1
, puede inferirse la importancia que posee el proceso de nitrificacin,
su estudio y control.
La oxidacin del nitrgeno amoniacal no tiene lugar de forma aislada, sino
que en realidad va acompaada de la sntesis de nuevas clulas adems de
consumo de oxgeno. La sntesis de nuevas bacterias nitrificantes se expresa:
15CO
2
+ 13 NH
4
+

nas) (Nitrosomo
10 NO
2

+ 3 C
5
H
7
NO
2
+ 23 H
+
+ 4 H
2
O (3.30)
5 CO
2
+ NH
4
+
+ 10 NO
2

+ 2 H
2
O

er) (Nitrobact
10 NO
3

+ C
5
H
7
NO
2
+ H
+
(3.31)
El rendimiento celular de estas reacciones est en el intervalo de 0,04 a
0,29 mg SSV (mg N-NH
3
)
1
para las bacterias Nitrosomonas y de 0,02 a
0,084 mg SSV (mg N-NO
2

)
1
para Nitrobacter.
15
Suponiendo para estas dos reacciones respectivamente rendimientos de 0,15
mg SSV(mgN-NH
3
)
1
y 0,02 mg SSV(mgN-NO
2
)
1
puede plantearse:
55 NH
4
+
+ 76 O
2
+ 109 HCO
3


C
5
H
7
NO
2
+ 54 NO
2

+ 57 H
2
O +
+ 104 H
2
CO
3
(3.32)
14
1
102
Menndez Gutirrez, C. y J. Prez Olmo
400 NO
2

+ NH
4
+
+ 4 H
2
CO
3
+ HCO
3

+ 195 CO
2

C
5
H
7
NO
2
+
+ 3 H
2
O + 400 NO
3

(3.33)
De las ecuaciones 3.32 y 3.33 puede obtenerse:
3 2 3
2 2 7 5 3 2 4
CO H 1,88 NO 0,98
O H 1,04 NO H C 0,021 HCO 1,98 O 1,83 NH
+ +
+ + + +

+
(3.34)
Dos conclusiones importantes se derivan del anlisis de la ecuacin 3.34. La
primera de ellas es que el N-NH
3
requerido para sntesis es despreciable, 2 %,
con respecto al que se utiliza por las bacterias nitrificantes para la oxidacin. La
segunda conclusin es que la nitrificacin consume alcalinidad del medio,
requirindose 7,15 mgL
1
como CaCO
3
por cada mgL
1
de N-NH
3
oxidado,
valor que se ajusta al reportado por la literatura.
Cintica de la nitrificacin
Para los sistemas de biomasa en suspensin los factores que se apuntan
como incidentes en la nitrificacin son:
Concentracin de N - NH
3
y N- NO
2
1
Relacin DBO
5
/NKT
Concentracin de oxgeno disuelto
Temperatura
pH
Se ha demostrado que existe una relacin directa entre la capacidad de
nitrificacin en un determinado proceso y la relacin DBO /NKT. Para valores
de este cociente entre 1 y 3, el porcentaje de bacterias nitrificantes en la biomasa
oscila entre 21 y 8,3 % respectivamente.
En los sistemas de lodo activado de tipo convencional, para los que la rela-
cin emprica DBO /NKT es mayor de 3, el porcentaje de bacterias nitrificantes
es mucho menor de 8,3. En los sistemas de lecho fijo, tanto en los filtros
percoladores como en los biodiscos, tambin puede presentarse la nitrificacin.
Temperatura
La temperatura ejerce una influencia importante sobre la velocidad de
nitrificacin, aumentando esta ltima con el incremento de la temperatura del
medio. Esta influencia se refleja en las constantes asociadas a la cintica de la
nitrificacin.
El intervalo de temperatura en el que ocurre preferentemente la nitrificacin
es el comprendido entre 4 y 45 C, siendo 35 C la temperatura ptima para las
bacterias Nitrosomonas y 35 42 C para Nitrobacter.
16
5
5
103
Oxidacin Biolgica. Nitrificacin
En la tabla 3.3 se reportan valores de velocidad especfica de crecimiento
de Nitrosomonas para tres temperaturas diferentes.
Tabla 3.3. Velocidad mxima especfica de crecimiento para Nitrosomonas
Para propsitos de diseo, una expresin aceptable del efecto de la tem-
peratura sobre la velocidad mxima especfica de crecimiento para las
nitrosomonas es:
) t ( ,
nm
e , n
20 098 0
65 0

=
(3.35)
Oxgeno disuelto
Generalmente la influencia de la concentracin de oxgeno en la veloci-
dad de nitrificacin ha sido objeto de controversias. No obstante, es acepta-
do, por evidencia experimental, que bajo determinadas condiciones se puede
obtener nitrificacin completa aun a concentraciones de oxgeno tan bajas
como 0,5 mgL
1
. Un estudio relativamente reciente para analizar la influen-
cia de diversos factores en la nitrificacin ha sido reportado por Stenstrom y
Song.
17
En este estudio se concluye que:
Tanto para sistemas de biomasa en suspensin como para medio fijo, la
concentracin de oxgeno que limita la nitrificacin est en el intervalo
comprendido entre 0,5 y 2,5 mgL
1
.
La concentracin lmite de oxgeno para la nitrificacin en condiciones
estacionarias es funcin del tiempo de retencin de la biomasa y de la
resistencia difusional al transporte de masa.
En la medida en que la concentracin de oxgeno es menor, se requiere
mayor tiempo de retencin de la biomasa para garantizar una completa
nitrificacin.
Bajo condiciones transcientes de choques de carga orgnica, o de au-
mento de la resistencia a la difusin o competencia entre bacterias
hetertrofas y nitrificantes, la concentracin lmite de oxgeno puede ser
significativamente mayor que bajo otras condiciones.
Temperatura, C
n,m
10 0,3
20 0,65
30 1,2
104
Menndez Gutirrez, C. y J. Prez Olmo
Efectos del pH y alcalinidad
El pH tambin puede influir notablemente sobre la velocidad de nitrificacin.
Como ya se ha expresado, la nitrificacin lleva implcita la destruccin
de una cantidad importante de alcalinidad del medio.
Puede demostrarse que en los reactores biolgicos, para el intervalo de
pH comprendido entre los valores de 5 y 8, el pH de equilibrio esta determi-
nado por la alcalinidad y el CO presente en el sistema. En aquellos casos de
reactores abiertos, en los que ocurre desprendimiento de CO , pueden man-
tenerse altos valores de pH a bajos niveles de alcalinidad en el medio.
En sistemas cerrados, donde normalmente el despojamiento de CO se
dificulta, la alcalinidad del agua residual debe ser hasta 10 veces el valor
de la concentracin del nitrgeno que se oxida para que pueda mantenerse
el pH.
Para propsitos de diseo, es suficiente tomar en consideracin que la
velocidad de nitrificacin disminuye considerablemente en la medida que el
pH cae por debajo del intervalo de neutralidad del medio. Es mejor mante-
ner el pH entre los valores de 6,5 y 8,0.
La influencia del pH sobre la velocidad mxima de nitrificacin puede cuan-
tificarse mediante la expresin,
( )
| |
pH , ,
nm
n
nm
n = 2 7 833 0 1
(3.36)
donde:

nm
: velocidad especfica de nitrificacin a pH = 7,2.
Efecto de la relacin DBO
5
/NKT
Generalmente en todos los sistemas de nitrificacin hay suficiente mate-
ria orgnica en el reactor como para propiciar el crecimiento de las bacte-
rias hetertrofas. Cuando se incrementa el tiempo de retencin medio celular
(edad del lodo) para incrementar la concentracin de biomasa (SSV), con el
objetivo de obtener mayor remocin de DBO, se corre el peligro potencial
de que la velocidad de crecimiento de las bacterias nitrificantes sea mayor
que el de las hetertrofas. En caso contrario, y se reduzca mucho el TRMC,
se lavaran del sistema. Por tanto, la condicin de trabajo ha de ser tal que el
tiempo de retencin de los slidos sea mayor que un valor limite que se
establezca.
Opuestamente a lo que sucede en los sistemas de biomasa en suspen-
sin, en aquellos en los que la biomasa esta adherida a un medio, sistemas de
biopelcula, el factor que controla es el proceso de transporte de masa.
2
2
2
105
Oxidacin Biolgica. Nitrificacin
Las condiciones que existen en el entorno cercano de los microorganismos
en la biopelcula, no son las mismas que para los sistemas de biomasa en
suspensin. En la biopelcula la concentracin de sustrato vara con la pro-
fundidad a lo largo del espesor, y es significativamente menor que en el seno
del lquido. Siendo esto vlido tanto para la concentracin de oxgeno como
para la de nitrgeno amoniacal, la velocidad de nitrificacin en biopelculas
es menor que la que podra esperarse tomando como referencia la concen-
tracin de nitrgeno en el lquido.
El transporte de masa o resistencia difusional incide sobre la nitrificacin de
la manera que se describe:
No es vlida la suposicin de que la etapa limitante de la nitrificacin
sea la oxidacin del nitrgeno amoniacal a nitrito.
Aumenta el tiempo de retencin que se requiere para los slidos.
Cambia la concentracin lmite de oxgeno para que ocurra la nitri-
ficacin.
Investigaciones realizadas por Downing, Painter y Knowles concluyen que
la cintica de la nitrificacin puede ser formulada en trminos de la relacin de
Monod.
dt
dN
Y
dt
dX
a
n
n
=
(3.37)
y
a n
a
nm n
N K
N
n n
+
= (3.38)
donde:
X
n
: Concentracin de microorganismos nitrificantes (mgL
1
).
N
a
: Concentracin de N-NH
3
(mgL
1
).
Yn: Masa de organismos nitrificantes obtenidos por masa de N-NH
3
utilizado.

n
: Velocidad especfica de crecimiento de los organismos nitrificantes a la
concentracin N
a
(mg Xn (mg Xn)
1
d
1
).
K
n
: Coeficiente de saturacin para nitrosomonas (mgL
1
N-NH
3
).
Est demostrado que la energa que liberada durante el proceso de conver-
sin de N-NH
3
a N-NO
2

es aproximadamente tres veces mayor a la corres-


pondiente a la transformacin del N-NO
2

a N-NO
3

.
19
O sea, se requiere producir
el triple de N-NO
3

que N-NO
2

para liberar una misma cantidad de energa.


Segn Adams y Eckenfelder
20
esta es la posible explicacin al hecho de que
generalmente el N-NO
2

se halla en bajas concentraciones en los efluentes de


los tratamientos biolgicos.
18
106
Menndez Gutirrez, C. y J. Prez Olmo
Teniendo en cuenta que las Nitrobacter tienen una velocidad especfica de
crecimiento mayor que el de las Nitrosomonas, la modelacin de la nitrificacin
puede considerarse como un proceso de una sola etapa, la limitante, que es la
conversin del N-NH
3
a N-NO
2

.
21
Esta es la razn por la cual los diferentes trminos definidos para las
ecuaciones 3.37 y 3.38 tienen como base el N-NH
3
.
La velocidad de crecimiento de las bacterias nitrificantes puede expresarse
como:
n n
n
X n
dt
dX
=
(3.39)
Combinando las ecuaciones 3.37, 3.38 y 3.39 para explicar la velocidad de
crecimiento en funcin del consumo de N-NH
3
se obtiene:
n
a n
a
nm
n
X
N K
N
n
Y dt
dNa
+
=
1
(3.40)
O sea, que la velocidad de nitrificacin es de primer orden con respecto a la
concentracin de Nitrosomas, y de un orden intermedio con respecto a la con-
centracin de N-NH
3
.
La ecuacin 3.40 puede escribirse de una forma equivalente si se define la
velocidad especfica mxima de consumo de sustrato, K
m
n
nm
m
Y
n
K =
(3.41)
Entonces la velocidad especfica de consumo de sustrato puede expresarse:
a n
a
m
N K
N
K K
+
=
(3.42)
y la velocidad de consumo de sustrato, o sea, de consumo de N-NH
3
para el
proceso de nitrificacin ser:
n
a n
a
m
a
X
N K
N
K
dt
dN
+
=
(3.43)
Se conoce que el valor de K
n
a 20
o
C es menor de 1 mgL
1
.
22
En la tabla 3.4 se presentan tres valores reportados en la literatura de velo-
cidades especficas de crecimiento y coeficientes de saturacin para
Nitrosomonas, a 20
o
C.
En aquellos casos en los que la concentracin de N-NH
3
tenga un valor
cercano a los 25 mgL
1
o mayor, puede considerarse, desde el punto de vista
107
Oxidacin Biolgica. Nitrificacin
prctico, que la velocidad de nitrificacin es de orden cero con respecto a la
concentracin de N-NH
3
. Para N
a
>> K
n
, la ecuacin 3.41 se transforma en:
n m
a
X K
dt
dN
=
(3.44)
o sea, la velocidad de nitrificacin solo es funcin de la concentracin de las
bacterias nitrificantes, su velocidad especfica de crecimiento mxima y del ren-
dimiento celular.
Tabla 3.4. Velocidad especfica mxima de crecimiento y coeficientes
de saturacin para Nitrosomonas a 20
o
C
Con el objetivo de llegar a conocer las condiciones a partir de las cuales
ocurre la nitrificacin puede hacerse un balance de biomasa nitrificante (fi-
gura 3.15).

nm
d
-1
K
n
mgL
-1
N-NH
3
Referencia
1,32 3,6 22
0,84 1,0 23
1,62 0,6 24
Fig. 3.15. Esquema para el balance en la etapa de nitrificacin.
w n n n n
a n
a
m n
n
Q X V X b V X
N K
N
n
At
AX
V
(

+
=
(3.4)
donde:
V: Volumen del reactor.
At
AX
n
: Velocidad de crecimiento de las bacterias nitrificantes.

nm
: Velocidad especfica de crecimiento.
108
Menndez Gutirrez, C. y J. Prez Olmo
X
n
: Concentracin de las bacterias nitrificantes.
N
a
: Concentracin de N-NH
3
.
b
n
: Constante de respiracin endgena.
Q
W
: Flujo de extraccin de lodo.
Transformando la ecuacin 3.45 se obtiene:
V
Q
X X b X
Na K
Na
n
At
AX
w
n n n n
n
nm
n

+
=
(3.46)
Considerando que:
x
n w
n
0
X Q
X V
=
(3.47)
y que en condiciones de estado estacionario se cumple que
0 =
At
X A
n
entonces, de la ecuacin 3.46 se tiene que,
|
.
|

\
|
+
|
.
|

\
|
+
=
x
n m n
x
n n
a
0
b n
0
b K
N
1
1
(3.48)
De la ecuacin 3.48 puede afirmarse que: la concentracin de nitrgeno
amoniacal en el reactor, y por tanto, en el efluente, es funcin de la edad del
lodo e independiente tanto de la concentracin de amonio en el afluente N
ai
como del coeficiente de rendimiento Y
n
.
El desarrollo anterior demuestra que la nitrificacin no siempre est pre-
sente, sino que se requiere un valor de edad de lodo mnima para que ocurra.
Suponiendo valores tpicos
21
se obtiene la figura 3.16.
En la figura 3.16 se aprecia que para altas edades de lodo el valor del
nitrgeno amoniacal es pequeo, pues ha ocurrido la trasformacin de la
forma amoniacal a N-NO
2

y N-NO
3

. Para edades de lodo, cercanas a 3,5


d, la concentracin de N
a
se incrementa bruscamente. Si u
x
< 3,5 d, el valor
de N-NH
3
aumenta hasta que se obtiene:
N
a
= N
ai
, que es la condicin lmite.
109
Oxidacin Biolgica. Nitrificacin
Fig. 3.16 Influencia de la edad del lodo en la nitrificacin.
Desde el punto de vista cuantitativo el valor de edad de lodo por debajo del
cual no existe nitrificacin se obtiene haciendo N
a
= N
ai
en la ecuacin 3.48.
Teniendo en cuenta, que N
a
rara vez es menor de 20 mgL
1
y que K
n
~ 1,0 mgL
1
,
se cumple que:
1 ((
ai
o
N
K
Suponiendo que
0 ~
ai
o
N
K
, lo que es vlido desde el punto de vista prctico, y
resolviendo la ecuacin 3.48 para
xm
0 , se tiene que:
n m n
m x
b n
0

=
,
,
1
(3.49)
De aqu puede concluirse que aunque a los efect os pr cticos,
generalmente se considera que para que ocurra nitrificacin se requiere una
edad de lodo superior a tres das, esto no es absoluto, ya que la edad de lodo
mnima depende de la velocidad especfica de crecimiento mxima de las
Nitrosomonas y de su coeficiente de respiracin endgena. Tambin a los
efectos prcticos, el valor del coeficiente b
n
puede asumirse despreciable en
la ecuacin 3.49.
Esto explica el fenmeno de nitrificacin que se ha reportado a valores de
edades de lodo de dos das en el tratamiento de residuales de pulpa y papel.
22
110
Menndez Gutirrez, C. y J. Prez Olmo
Ejemplo 3.2
Calcular la velocidad mxima especfica de crecimiento de las bacterias
nitrificantes bajo condiciones no limitantes de pH y oxgeno disuelto. Considere
una temperatura de 30
o
C.
Para propsitos de diseo puede utilizarse le ecuacin 3.35:
20 098 0
65 0

=
t ,
n,m
e , n
20 30 098 0
65 0

=
,
n,m
e , n
m
n,m
= 1,73 d
1
Ejemplo 3.3
Determine el tiempo de retencin celular de diseo (edad del lodo de diseo)
para que ocurra nitrificacin.
En estado estacionario, la cantidad de slidos que salen del sistema es nu-
mricamente igual a la que se produce. Por tanto, la relacin entre la edad de
lodo mnima para que ocurra nitrificacin y
n
ser,
n,neto n n,m
x,m
n b n
0
= =
1
Asumiendo que b
n
= 0,
n,m
x,m
n
0
=
1
Para disear puede considerarse que la edad del lodo que debe tomarse
debe ser al menos 3,5 veces la edad del lodo mnima,
x,m x,d
0 , 0 5 3 =
d 0
x,d
2 =
Ejemplo 3.4
Considerando que la alcalinidad del medio en los ejemplos anteriores es
25 mgL
1
como CaCO
3
, estime la que debe ser aadida para satisfacer la de-
manda exigida por la nitrificacin.
Cada mg de N-NH
3
al oxidarse a NO
3

, consume 7,1 mg de alcalinidad.


Para determinar el N-NH
3
que se oxida,
111
Oxidacin Biolgica. Nitrificacin
a n
a
n,m n
N K
N
n n
+
=
5 0
1
,
0
n
,d
x
n
= =
suponiendo que K
n
= 1,0
a
a
N
N
, ,
+
=
1
73 1 5 0
N
a
= 0,41 mgL
1
La alcalinidad a consumirse durante la nitrificacin es,
25 0,41 = 24,59 mgL
1
alcalinidad consumida = 24,59 7,1 = 175 mgL
1
alcalinidad a adicionar = 175 150 =25 mgL
1
3.5.2. Desnitrificacin
La denitrificacin biolgica implica la reduccin microbiolgica del ni-
trato a nitrito y finalmente del nitrito a nitrgeno gaseoso. El nitrato y nitrito
constituyen la fuente de oxgeno para este proceso. De tal forma, la
desnitrificacin normalmente ocurre en ausencia de oxgeno molecular, pero
con la presencia de nitrato. Estos procesos son denominados anxicos. De
modo que la desnitrificacin transforma el nitrgeno de una forma poten-
cialmente objetable, nitrato, a otra biolgicamente inerte, nitrgeno gaseoso.
La desnitrificacin presenta cinco campos de inters, sin considerar los
riesgos que para la salud puede implicar un efluente con alto contenido de
nitrato:
Consigue recuperar una parte de la alcalinidad que fue consumida
durante el proceso de nitrificacin. Esto es:
3,5 g como CaCO
3
por cada g N-NO
3
reducido.
Disminuye considerablemente los riesgos potenciales comnmente aso-
ciados a la desnitrificacin incontrolada que puede desarrollare en la
sedimentacin secundaria.
Recupera una parte del oxgeno consumido por la nitrificacin:
2,6 g de oxgeno por cada g de N-NO
3
reducido.
Un reactor anxico previo a la etapa de aeracin mejora la sedimentabilidad
de los fangos, disminuyendo el ndice volumtrico de lodos.
Contrariamente al pequeo nmero de especies nitrificantes, las de las
desnitrificantes es mucho mayor. De ah que resulten ms aptas para su acli-
112
Menndez Gutirrez, C. y J. Prez Olmo
matacin y, por consiguiente, tengan una mayor y mejor resistencia a las
agresiones del medio ambiente.
Muchos microorganismos presentes en los sistemas de lodos activados mu-
nicipales pueden llevar a cabo procesos de desnitrificacin, aunque estos no se
hayan diseado especficamente para ello. Su presencia se debe en parte a que
son organismos facultativos. O sea, que pueden utilizar indistintamente el oxge-
no o el nitrato como ltimo aceptor de electrones. Por otro lado las bacterias
desnitrificantes pueden tambin proliferar en sistemas aerobios, debido a su
habilidad para emplear el oxgeno molecular, y oxidar eficientemente la materia
orgnica.
La capacidad de ubicuidad de las bacterias desnitrificantes minimiza la ne-
cesidad de crear un ambiente con condiciones especiales para su subsistencia,
tal como ocurre con las nitrificantes.
3.5.2.1. Metabolismo de la desnitrificacin
En la desnitrificacin el nitrato y nitrito actan como aceptores de electro-
nes en la cadena respiratoria, tal como lo hace el oxgeno en un medio aerobio.
El transporte a travs de esta cadena es el mecanismo fundamental mediante el
cul las clulas liberan energa.
En el sistema generalmente existe un mecanismo de autocontrol, de manera
tal que si hay oxgeno presente en el medio donde est teniendo lugar el proceso,
el oxgeno se utilizar preferentemente sobre el nitrato. En ausencia de oxgeno, el
nitrato ser la especie que se favorecer como aceptor final de electrones sobre
cualquier otro compuesto que pueda ser reducido, como por ejemplo el sulfato.
El mecanismo de control de las bacterias desnitrificadoras ocurre en dos
niveles:
Sntesis de las enzimas requeridas para la desnitrificacin
Actividad enzimtica propiamente dicho
Es conocido que son necesarias entre 2 y 3 horas para la sntesis enzimtica
en cultivos puros cuando el hbitat de las bacterias es modificado de un medio
aerobio para uno anxico. Sin embargo, estudios han demostrado que las enzimas
desnitrificantes pueden estar presentes aun cuando el medio no sea anxico,
sugiriendo que el periodo de sntesis de las enzimas puede ser innecesario con
anterioridad al inicio de la desnitrificacin.
23
Por otro lado el oxgeno inhibe la actividad de las enzimas desnitrificadoras.
Para cultivos puros se ha reportado que a valores de la concentracin de oxge-
no de 0,2 mgL
1
ya se aprecia inhibicin.
24
En lodos activados los valores
inhibitorios reportados estn comprendidos entre 0,3 y 1,5 mgL
1
.
25
113
Oxidacin Biolgica. Nitrificacin
Generalmente la masa de agua que se desea desnitrificar necesita tener
suficiente materia orgnica que pueda ser utilizada como fuente de energa para
la masa biolgica encargada de transformar el nitrato en nitrgeno gaseoso.
Cuando se carece de este contenido de materia orgnica, hay que utilizar una
fuente externa tal como el metanol, etanol o cido actico, entre otros compuestos.
3.5.2.2. Cintica de la desnitrificacin
La cintica de la desnitrificacin puede ser expresada de modo similar a la
cintica de la nitrificacin o de la oxidacin de los compuestos orgnicos.
La ecuacin de Monod es un medio til para explicar los conceptos genera-
les del proceso de desnitrificacin. Esta expresin frecuentemente es empleada
en aquellos casos en los que este proceso ocurre en reactores independientes y
se emplea el metanol como fuente de materia orgnica.
Cuando se concibe un solo reactor para que tengan lugar todos los procesos,
se dificulta el empleo de Monod debido a la heterogeneidad tanto del sustrato
como de la biomasa.
Como ya se expres, existen casos en los que la velocidad de crecimiento
de las bacterias desnitrificantes puede representarse mediante la ecuacin de
Monod:
D K
D
n n
D
Dm D
+
=
(3.50)
donde:

D
: Velocidad especfica de crecimiento de las bacterias desnitrificantes h
1
.
D: Concentracin N-NO
3
(mgL
1
).

Dm
: Velocidad especfica de crecimiento mxima h
1
.
K
D
: Constante de saturacin (mgL
1
).
Si K
D
<< D, entonces la ecuacin anterior se transforma en una expresin
de orden cero con respecto a la concentracin de nitrato,
m
D
= m
Dm
(3.51)
Por el contrario, si K
D
>> D, se estar en presencia de una reaccin de
primer orden,
D
K
n
n
D
m
D
D
=
(3.52)
Como la velocidad de crecimiento de las bacterias desnitrificantes es similar
al de las hetertrofas, mucho mayor que el de las nitrificantes, el tiempo de
114
Menndez Gutirrez, C. y J. Prez Olmo
retencin medio celular para prevenir el lavado de dichas bacterias del reactor
es mucho menor que el de estas ltimas.
La velocidad de remocin de nitrato puede relacionarse con la velocidad
especfica de crecimiento,
D K
D
U
Y
n
U
D
m
D
D
D
D
+
= =
(3.53)
U
D
: Velocidad especfica de remocin g N-NO
3

(gSSVd)
1
.
Y
D
: Coeficiente de rendimiento SSV (N-NO
3
1
remov.)
1
.
U
Dm
: Velocidad especfica mxima de remocin de nitrato
g N-NO
3
1
(g SSV)
1
.
La influencia de la concentracin de materia orgnica sobre la desnitrificacin
puede expresarse mediante una relacin similar,
S K
S
U U
S
m
D D
+
=
(3.54)
donde:
S: Concentracin de sustrato orgnico mgL
1
.
Como ambos, tanto la concentracin de nitrato como de sustrato de
origen orgnico influyen sobre la velocidad de desnitrificacin, una mejor
aproximacin se obtiene cuando la velocidad de remocin de nitrato se
expresa,
D K
D
S K
S
U U
D S
m
D D
+ +
=
(3.55)
Existen evidencias experimentales de que los valores de K
D
son relativamente
bajos (0,1-0,2 mg N-NO
3
1
L
1
).
26
A partir de este hecho, y analizando la ecua-
cin (3.52), puede concluirse que concentraciones de N-NO tan bajas como
1-2mgL
1
no tienen efecto sobre la velocidad de desnitrificacin. Por lo tanto, la
concentracin de nitrato no influye en la velocidad de ese proceso, a menos que
la concentracin sea mucho ms baja que la del intervalo analizado. Esto ltimo,
aunque cierto, quizs carezca de inters prctico por el valor tan bajo de
concentraciones de que se trata.
Por otro lado, el valor de K
S
depender del tipo de compuesto orgnico que
se emplee como dador de electrones. Valores de K
S
para el metanol han sido
reportados entre 0,1 y 72 mgL
1
.
27
Cuando se emplea agua residual municipal como sustrato orgnico, expre-
sado como DBO, los valores de K
S
son mayores que los reportados para el
3
1
115
Oxidacin Biolgica. Nitrificacin
metanol, por esta razn, en estos casos, la velocidad de desnitrificacin puede
considerarse que depende de la concentracin de sustrato.
28
La influencia del oxgeno en la velocidad de desnitrificacin puede tambin
ser reflejada en la ecuacin (3.55) de la velocidad,
O O
O
D S
m
D D
S K
K
D K
D
S K
S
U U
+ + +
=
(3.56)
donde el subndice (O) representa los parmetros para el oxgeno.
Por otro lado, como la desnitrificacin lleva implcita el consumo de materia
orgnica, su cintica puede ser expresada solo en funcin de la velocidad de
consumo de DBO.
Considerando que la ecuacin que describe la cintica de remocin de DBO
es similar a la 3.19,
S K
S
q q
S
Sm S
+
=
(3.57)
donde K
S
tiene el mismo valor que la usada en la ecuacin 3.54.
Entonces la velocidad de remocin de nitrato puede relacionarse con la
velocidad de consumo de DBO mediante una expresin similar a la que se
emplea para relacionar el consumo de oxgeno a la utilizacin de sustrato
orgnico:
( )
d
S
ob D
b
,
,
,
q
Y , U
86 2
42 1
86 2
42 1 1 + =
(3.58)
Y
ob
: Rendimiento observado en funcin de la DQO removida.
El factor 1,42 es el equivalente de oxgeno de la biomasa en trminos de
DQO, mientras que el 2,86 representa la relacin g de O
2
(g N
_
NO
3

)
1
.
En la ecuacin (3.58) el primer trmino describe la fraccin de DQO y
nitrato que son empleados para la sntesis de biomasa. El segundo trmino re-
presenta la fraccin de nitrato utilizado en la respiracin endgena.
Al igual que ocurre con la nitrificacin, el proceso de desnitrificacin es
funcin de la temperatura y la concentracin de oxgeno disuelto en el medio.
As la velocidad de desnitrificacin puede expresarse,
29
U
D
= U
D
1,09
(t 20)
(1 OD) (3.59)
U
D
: Velocidad total de desnitrificacin, kg N-NO
3
(kgSSVd)
1
.
U
D
: Velocidad especfica de desnitrificacin a 20 C.
t: Temperatura del agua residual, C.
OD: Concentracin de oxgeno disuelto en el agua residual, mgL
1
.
116
Menndez Gutirrez, C. y J. Prez Olmo
Teniendo en cuenta lo que representa en cuanto a costo la utilizacin de una
fuente externa de carbono para satisfacer los requerimientos energticos de la
biomasa, en los procesos de desnitrificacin suelen llevarse a cabo de manera
conjunta ambos procesos, nitrificacin-desnitrificacin, para aprovechar la fuente
de carbono existente en el agua residual.
Segn se ha reportado, con este tipo de proceso secuencial se logran veloci-
dades de desnitrificacin de hasta 0,2 kg N-NO (kg SSVd)
1
.
Ejemplo 3.5
Se producen diariamente 5000 m
3
de un agua residual nitrificada. El efluente
del tanque de nitrificacin contiene 30 mgL
1
de NNO
3

, 2 mgL
1
de oxgeno
disuelto y 0,5 mgL
1
de NNO
2

.
Asumiendo que el efluente tiene un contenido mximo de NNO
3

de
3 mgL
1
y que la DBO
5
no excede los 10 mgL
1
, calcule el volumen del reactor
requerido para la desnitrificacin.
D K
D
S K
S
U U
D S
D D
m
+ +
=
suponga que:
K
S
= 25 mgL
1
para DBO
5
K
D
= 0,1 mgL
1
para N
Concentracin de N-NO
3

en el efluente = 3,0 - 0,5 = 2,5 mgL


1
1
d 165 0
10 25
10
5 2 1 0
5 2
6 0
-
D
,
, ,
,
, U =
+ +
=
dD
Dm
D
k
0
U + =
1
, considerando k
dD
= 0,07 d
1
El tiempo de retencin celular lmite se calcula,
07 0
1
6 0 ,
0
,
Dm
+ =
; q
Dm
= 10,52d
1
Aceptando un factor de seguridad de 2, el tiempo de retencin celular de
diseo ser,
u
D, diseo
= 21 d
1
Mediante un balance de nitrgeno para el proceso de desnitrificacin, se
obtiene una ecuacin similar a la 3.42. A partir de ella puede obtenerse,
3
117
Oxidacin Biolgica. Nitrificacin
( )
n D,diseo
D,diseo dD
Y 0
0 k
X V
D D Q

+
=
1
0
Si X = 2 750 mgL
1
y Y
n
= 0,8,
V = 340 m
3
Notas bibliogrficas
1 MC CARTY, P. L.: 'Thermodynamics oI biological synthesis and growth, Adv.
in Water Poll. Res., vol. 2, Pergamon Press, EE. UU. 1964.
2 JORGENSEN, P.E., T. ERIKSEN, AND B. K. JENSEN: 'Estimation oI viable biomass in
wastewater and activated sludge by determination of ATP, Oxygen Utili-
zation rate and FDA Hydrolysis, Water Res., vol. 26, no. 11, 1992.
3 PREZ, J. Y C. MENNDEZ: Anlisis cromatogrfico de los residuales de pulpa y
papel antes y despus de un tratamiento biolgico, indito, ISPJAE, Ciu-
dad de La Habana, 1986.
4 SERVIZI, J.A. AND R. H. BOGAN: 'Free energy as a parameter in biological
treatment, Proceeding, ASCE, vol. 89, sa, vol. 3, no. 17, 1963.
5 ECKENFELDER, W.W. AND D. J. O`CONNOR: Biological Waste Treatment, Per-
gamon Press, Oxford, 1961.
6 BENEFIELD, L. AND C. RANDALL: Biological Process Design for Wastewater
Treatment, Prentice Hall Series in Environmental Sciences, EE. UU., 1980.
7 HOOVER, S. R. AND N. PORGES: 'Assimilations oI dairy wastes by activated
sludge, Part II, Sewage and Industrial Wastes, vol. 24, no. 3, 1952.
8 MCCABE, B.J. AND W. W. ECKENFELDER: 'BOD removal and sludge growth in
the activated sludge process, Jour. Water Poll. Control Fed., vol. 33,
no. 3, 1961.
9 WILSON, I. S.: 'Concentration eIIects in the biological oxidation oI trade was-
tes, Proceding 1
st
International Conference Water Pollution Resear-
ch, Pergamon Press, London, 1962.
10 WOHRMAN, K.: Biological Treatment of Sewage and Industrial Wastes,
vol. I, Ed. Mc Cabe, EE. UU., 1956.
11 GRAU, P., M. DOHANYOS, AND J. CHUDOBA: 'Kinetics oI multicomponent subs-
trate removal by activated sludge, Water Res., vol. 9, no. 6, 1975.
12 ECKENFELDER, W.W.: Industrial Water Pollution Control, McGraw Hill,
USA 1966.
13 LAWRENCE, A.W. AND P.L. MCCARTY: 'UniIied basis Ior biological treatment
design and operation, Proc. Am. Soc. Civil Engrs., Jour. San. Eng. Div.
96 (SA3), 1970.
118
Menndez Gutirrez, C. y J. Prez Olmo
14 VAN HAANDEL, A. C. AND G. R. MARAIS: 'NitriIication and denitriIication in the
activated sludge process, Research Report no. 42 Departament of Ci-
vil Engineering University of Cape Town, 1981.
15 PODUSKA, R. AND J. ANDREWS: 'Dynamics oI nitriIication in the activated slud-
ge process, Jour. Water Poll. Control Fed., vol. 47, no. 11, 1975.
16 SHERRARD, J. H.: 'Destructions oI alkalinity in aerobic biological wastewater
treatment, Jour. Water Poll. Control Fed., vol. 48, no. 7, 1976.
17 STENSTROM Y S.S. SONG.: 'EIIect oI oxygen transport limitation on nitriIication
in the activated sludge process, Jour. Water Poll. Control Fed., vol. 63,
no. 2, 1991.
18 DOMING, A., H. PAINTER, AND G. KNOWLES: 'NitriIications in the activated slud-
ge process, Journal of the Institute of Sewage Purification, vol. 4, no.
130, 1964.
19 MC. CARTY, P. L.: Stoichiometry of Biological Reactions Progress in Water
Technology, Pergamon Press, London, 1975.
20 ADAMS, C. E. AND W. W. ECKENFELDER: 'NitriIications design approach oI
high strength ammonia wastewaters. Jour. Water Poll. Control Fed.,
vol. 49, no. 3, 1977.
21 HENZE, M., C. P. GRADY ET AL: IAWPRC Task Group on Mathematical
Modelling for Design and Operation of Biological Wastewater Treat-
ment, Final Report, 1985.
22 HALL, E. AND K. L. MURPHY: 'Estimation oI nitriIying biomass and kinetics in
wastewater, Water Res., vol. 14, no. 2, 1990.
23 ARGAMAN, Y. ANDA. BRENNE: 'Experimental evaluation oI modelling alternati-
ves and process parameters for the single-sludge nitrogen removal Sys-
tem, Presented in the IAWPRC Specialized Seminar, Copenhagen, Den-
mark, August, 1985.
24 MENNDEZ, C.: Wastewater Treatment from the Pulp and Paper Industry,
Tesis de Doctorado, Instituto de Tecnologa Qumica, Praga, 1986.
25 SIMPKIN, T.J. AND W.C. BOYLE: 'The lack oI represion by oxygen oI the deni-
triIying enzymes in activated sludge, Water Res., vol. 22, no. 2, 1988.
26 FOCHT, D.D. ANDA.C. CHANG: 'NitriIication and denitriIication processes re-
lated wastewater treatment, Adv. Appl. Microbiol, vol. 19, no. 1, 1975.
27 BURDICK, C.R., D.R. REFLING AND H.D. STENSEL: 'Advanced biological treat-
ment to achieve nutrient removal, Jour. Water Poll. Control Fed.,
vol. 54, no. 9, 1982.
28 CHRISTENSEN, M.H. AND P. HARRAMOES: 'Biological denitriIication oI sewage:
a literature review, Water Tech., vol. 8, no. 2, 1977.
29 KUGELMAN, I. J. ET AL.: 'Aerobic denitriIication in activated sludge, Proc. of
the 1991 Env. Eng. Speciality Conf. Reno, NV. 1991.
119
Lodo Activado
CAPTULO 4
LODO ACTIVADO
4.1. GENERALIDADES
Bajo la denominacin del lodo activado se incluye un conjunto de procedi-
mientos de tratamiento en los que se destaca, como elemento comn, el ntimo
contacto del agua residual con una masa biolgica preformada y en suspensin,
en un tanque aereado. Esta biomasa esta constituida por un cultivo mixto de
microorganismos que forma, junto con otras sustancias orgnicas e inorgnicas
un conglomerado floculento. Mediante este proceso los compuestos orgnicos
contenidos en las aguas residuales son utilizados como sustratos.
La primera informacin que se tiene del empleo del lodo activado para el
tratamiento de aguas residuales data de 1914 en Manchester, Inglaterra.
A diferencia de los cultivos puros en los que las bacterias se mueven libremente,
en el lodo activado estas se encuentran en forma de zooglea, de manera que forma
con facilidad flculos de buenas propiedades de sedimentacin.
Entre los grupos y especies de bacterias que con ms frecuencia se encuen-
tran presentes en los lodos activados, pueden citarse:
Pseudomonas, flavobacterias chromobacterias, azotobacterias, micrococos,
bacilos (b. subtilis, b. megaterium), alcalgenes, y nocardias. En menor propor-
cin se encuentran los hongos, levaduras y mohos.
Entre las bacterias nitrificantes pueden estar presentes las nitrosomonas y
las nitrobacterias. Es comn que se reporte en lodo activado la presencia de
protozoos tales como vorticelas, opercularias y epistlis entre otros. Otros
microorganismos superiores presentes pueden ser los rotferos.
En los lodos activados se aprecia con mucha claridad el vnculo presa-
predador en la relacin que existe entre bacterias y protozoos. El papel de las
bacterias es descomponer, por oxidacin, la materia orgnica. En el proceso del
lodo activado las bacterias son los microorganismos ms importantes debido a
que son ellas las responsables de la descomposicin de la materia orgnica con-
tenida en el agua residual a tratar. Los protozoos se alimentan de las bacterias
que se mueven libremente, y as contribuyen a la clarificacin del efluente del
120
Menndez Gutirrez, C. y J. Prez Olmo
proceso del lodo activado, que de otra forma tendra una concentracin ms
elevada de slidos voltiles en suspensin.
Por otra parte, los hongos filamentosos constituyen microorganismos inde-
seables porque su presencia puede traer como consecuencia dificultades con la
sedimentacin de la biomasa. Estos microorganismos pertenecen a los grupos:
Sphaerotilus, leptomitus, laucothrix, thiothrix, geotrchum, y otros.
La composicin del lodo activado, en lo cuantitativo, depende de la compo-
sicin del sustrato que le sirve de alimentacin y de los parmetros tecnolgicos
del proceso: tiempo de retencin hidrulico, tiempo de retencin medio celular,
(edad del lodo) y la remocin especfica de sustrato (carga de lodo).
De acuerdo con su facilidad de sedimentacin, los lodos pueden clasi-
ficarse en:
Floculentos.
No floculentos.
Abultados.
Un breve anlisis de las caractersticas fundamentales de cada uno de estos
tipos del lodo arroja los siguientes resultados:
4.1.1. Lodos floculentos
En condiciones ptimas del proceso, el lodo activado forma buenos flculos
de fcil sedimentacin. La formacin de buenos flculos no es necesariamente
una condicin para una alta eficiencia en la remocin de los compuestos orgni-
cos solubles presentes en un agua residual. Sin embargo, cuando se desea tener
un lodo denso para su recirculacin, y un efluente claro, se requiere que el lodo
activado forme buenos flculos.
El mecanismo de la biofloculacin no es totalmente conocido. La teora ms
razonable supone que el flculo se forma debido al efecto de polmeros
extracelulares de la poblacin bacteriana.
1
Desde el punto de vista prctico es importante tener presente los parmetros
tecnolgicos ya mencionados, bajo los cuales se forma un lodo con buenos flculos
y con buena sedimentacin.
4.1.2. Lodos no floculentos
En ocasiones hay condiciones de trabajo bajo las cuales los microorganismos
que constituyen el lodo no forman aglomeraciones, y por tanto, no se logran
flculos sedimentables. Las bacterias permanecen dispersas y mantienen su
individualidad.
121
Lodo Activado
El efluente de estos sistemas se presenta turbio y con relativamente alta
DQO y DBO
5
debido al contenido de sustancias que permanecen en suspensin
aun despus de la sedimentacin. No obstante, el contenido de sustancias org-
nicas en disolucin puede ser bajo.
En la prctica del tratamiento de aguas residuales, no es usual apreciar
que un lodo pueda ser no floculento en su totalidad. Normalmente, parte del
cultivo mismo forma flculos de fcil separacin por sedimentacin y parte
permanece en forma dispersa, y cuya separacin por sedimentacin simple
se hace ms difcil.
Las diferentes causas por las que un lodo puede ser no floculento pueden
encontrarse en los parmetros tecnolgicos del proceso, y en las caractersticas
de las aguas residuales que son tratadas.
Es comnmente conocido que cuando existe una baja edad del lodo (me-
nor de tres das) o alta carga, el lodo que se forma es propenso a presentar
caractersticas de un cultivo disperso no floculento.
2
Como consecuencia de
esta dispersin se produce un aumento en el ndice volumtrico de lodo, en la
medida en que la edad del lodo disminuye a partir de determinado valor.
3, 4
(Figuras 4.1 y 4.2).
Fig. 4.1. Influencia de la edad del lodo sobre la dispersin de los slidos.
Fig. 4.2. Relacin entre el ndice volumtrico del lodo (IVL) y la veloci-
dad de sedimentacin.
122
Menndez Gutirrez, C. y J. Prez Olmo
La prdida de la propiedad de formar flculos de un lodo puede deberse a
causas temporales, de forma que cuando cesa la causa el lodo recupera la capa-
cidad perdida. Este trastorno temporal puede deberse entre otros, a cambios
bruscos en el pH, alcalinidad o temperatura del agua. La de-floculacin de ca-
rcter permanente responde a otras causas, como la falta de algn nutriente o la
presencia de sustancias txicas.
4.1.3. Lodos abultados
El abultamiento de los lodos trae como consecuencia malas propiedades de
sedimentacin. Desde un punto de vista eminentemente prctico, las caracters-
ticas de sedimentacin y espesamiento de los lodos activados se expresan en
trminos del ndice volumtrico del lodo (IVL). En la medida que el valor del IVL
sea mayor, las caractersticas de sedimentabilidad de los lodos sern menores.
De acuerdo con el ndice volumtrico del lodo estos pueden clasificarse en:
Lodos normales IVL < 100 mLg
1
Lodos ligeros IVL (100 200) mLg
1
Lodos abultados IVL > 200 mLg
1
Aunque la clasificacin de los lodos de acuerdo con el ndice volumtrico se
corresponde con el comportamiento que usualmente presentan estos, debe te-
nerse tambin presente que el valor de este indicador es funcin de la concen-
tracin de los lodos.
5
En la figura 4.3 puede apreciarse que esta dependencia es ms significativa
a medida que el ndice volumtrico del lodo es mayor.
Fig. 4.3. Efecto de la concentracin del lodo sobre el IVL.
123
Lodo Activado
El abultamiento puede ser filamentoso o no filamentoso. Las causas de
este ltimo no son muy bien conocidas. Por otro lado desde el punto de vista
prctico este tipo de abultamiento no origina grandes problemas.
La causa del abultamiento debido a los organismos filamentosos puede
estar tanto en las caractersticas de las aguas residuales como en los
parmetros tecnolgicos.
La existencia de alta carga es generalmente propicia para el crecimiento
de bacterias formadoras de flculo, tal como existe en los reactores conoci-
dos como Selectores, desarrollados inicialmente por Chudoba.
6
La carga en los selectores puede ser tal alta como 12-20 kg DBO kg
SSVLM
1
d
1
.
7
No obstante, Kappeler y su mujer han reportado buenos re-
sultados del selector con cargas inferiores a esas.
8
4.1.3.1. Caractersticas de las aguas residuales
El abultamiento de los lodos se observa con ms frecuencia en el trata-
miento de aguas residuales que contiene una alta concentracin de sacridos.
El desbalance nutricional puede traer como consecuencia el crecimiento fuera
de control de algunos microorganismos filamentosos. Esto se ha apreciado,
por ejemplo, en residuales con dficit de fsforo.
9
4.1.3.2. Parmetros tecnolgicos
La presencia de microorganismos filamentosos es ms frecuente en los
sistemas de baja carga del lodo que en los de alta carga.
Por otra lodo, en reactores de flujo de pistn (poca mezcla longitudinal y
alto gradiente de concentracin de sustrato), el crecimiento de microorga-
nismos filamentosos es ms adverso que en reactores completamente mez-
clados. Este fenmeno ya fue discutido con anterioridad.
4.2. PARMETROS BSICOS
Hay una serie de parmetros del proceso del lodo activado, que por su
importancia pueden ser denominados bsicos, que deben quedar claramente
definidos antes de trabajar con las expresiones matemticas de este trata-
miento. Para la definicin de los parmetros bsicos se toma como referen-
cia el diagrama de la figura 4.4.
124
Menndez Gutirrez, C. y J. Prez Olmo
Fig. 4.4. Esquema de tratamiento.
4.2.1. Tiempo de retencin hidrulico
El tiempo de retencin hidrulico viene dado por la relacin entre el volumen
efectivo del reactor y el flujo o caudal de entrada a l.
Cuando no hay recirculacin:
0
Q
V
0 = (h d)
(4.1)
Cuando existe recirculacin Q
r
:
r
m
Q Q
V
0
+
=
0
(h d)
(4.2)
Simbologa
1. criba
2. desarenador
3. sedimentador primario
4. tanque de aereacin (TA)
5. sedimentador secundario
6. recirculacin de lodo
7. afluente al TA
8. lodo a digestin
So: DBO DQO afluente
S
2
: DBO DQO efluente
X: concentracin de lodo en el TA
X
r
: concentracin de lodo en la
recirculacin
X
2
: concentracin de lodo en el
efluente
Q
w
: flujo de exceso de lodo
Q
r
: flujo de recirculacin
125
Lodo Activado
4.2.2. Relacin de recirculacin
0
Q
Q
R
r
=
(4.3)
El tiempo de retencin hidrulico puede ser expresado en funcin de la rela-
cin de recirculacin:
( )
( )
d h
1
0
R Q
V
0
m
+
=
(4.4)
4.2.3. Carga volumtrica
Se denomina carga volumtrica a la masa de sustrato que se aplica, expre-
sado como DBO o DQO, por unidad de volumen del tanque de aereacin, en la
unidad de tiempo.
La carga volumtrica, tambin llamada en ocasiones carga orgnica, no es
ms que el flujo msico de DBO DQO por unidad de volumen efectivo del
reactor.
( )
1 3 0 0 0
d m kg

= =
0
S
V
S Q
B
v
(4.5)
(4.6)
Las ecuaciones 4.5 4.6 se utilizarn dependiendo de si hay o no recirculacin.
4.2.4. Razn de carga volumtrica
La razn de carga volumtrica es un parmetro que puede asociarse, en un
sentido fsico, con la velocidad msica de remocin del sustrato por unidad de
volumen efectivo del tanque de aereacin.
( )
( )
1 3 2 0 0
d m kg

=
V
S S Q
B A
v
(4.7)
o sea,
( )
1 3 2 0
d m kg

=
0
S S
B A
v
(4.8)
126
Menndez Gutirrez, C. y J. Prez Olmo
4.2.5. Carga de lodo
Se conoce bajo la denominacin de carga del lodo al flujo msico de sustrato,
ya sea DBO o DQO que se aplica al tanque de aeracin, por unidad de slido
orgnico en el lodo.
( )
1 1 0 0
d kg kg

=
VX
S Q
B
x
(4.9)
Combinando las ecuaciones 4.9 y 4.5, se obtiene la ecuacin 4.10, que es
otra forma de expresar la carga del lodo:
X
B
B
v
x
=
(4.10)
Cuando se utiliza la recirculacin, la carga del lodo puede calcularse segn:
( )
( )
1 1 0
d kg kg

+
=
VX
S Q Q
B
m r
x
(4.11)
4.2.6. Razn de carga de lodo
Este parmetro tambin se conoce como la relacin F/M (alimento/microor-
ganismo), y en esencia no es otra cosa que la remocin especfica de la DBO o
DQO.
( )
( )
1 1
2 0 0
d kg kg

=
VX
S S Q
B A
x
(4.12)
La ecuacin 4.12 puede expresarse de forma ms sencilla:
( )
1 1 2 0
d kg kg

=
X 0
S S
AB
x (4.13)
o tambin, combinando las ecuaciones 4.13 y 4.7:
( )
1 1
d kg kg

=
X
B A
AB
v
x
(4.14)
4.2.7. Edad del lodo
El tiempo de retencin medio celular (TRMC) o edad del lodo es hoy da el
parmetro ms importante de los utilizados para el control de los procesos del
lodo activado.
127
Lodo Activado
La edad del lodo puede calcularse a partir de la relacin que existe entre la
masa del lodo que est presente en el sistema y la masa del lodo que se extrae
diariamente.
( )
w w w
x
Q Q X Q X
G VX
0
+
+
=
0 2
(4.15)
donde:
G: Masa del lodo en el sedimentador secundario y en las tuberas. General-
mente no se comete mucho error si se supone que G = 0
Si los lodos del proceso forman buenos flculos y poseen buenas propie-
dades de sedimentacin, puede suponerse que la cantidad del lodo o biomasa
que se pierde en el sobrenadante del sedimentador secundario es desprecia-
ble cuando se compara con la que se extrae por la corriente Q
w
:
( )
w w w
Q X Q Q X ( (
0 2
(4.16)
Cuando se est en presencia de lodos no floculentos, la prdida de biomasa
en el sobrenadante del sedimentador secundario puede ser apreciable, pudiendo
llegar hasta 40 y 60 % del lodo total que se extrae, y no puede obviarse la
prdida del lodo por este concepto.
En el caso en que las condiciones de los flculos son tales que se cumple la
expresin 4.16, se tiene que:
w w
x
X Q
VX
0 =
(4.17)
Por otro lado, si la extraccin del lodo se realiza directamente del tanque de
aeracin:
w
X X =
la ecuacin 4.17 se transforma:
w
x
Q
V
0 =
(4.18)
Un aspecto importante a tener en cuenta es que en aquellos casos en los
que no exista recirculacin, se cumple que el tiempo de retencin medio celular
es igual al tiempo de retencin hidrulico.
u
x
= u cuando R = 0 (4.19)
128
Menndez Gutirrez, C. y J. Prez Olmo
4.3. PRINCIPALES MODIFICACIONES TECNOLGICAS
El proceso del lodo activado puede definirse como aquel en el que la
biomasa floculenta est circulando continuamente en ntimo contacto con la
materia orgnica que se desea eliminar o remover, en presencia de oxgeno.
Usualmente el oxgeno es suministrado a partir de las burbujas de aire que
se incorporan a la mezcla lodo-lquido (licor mezclado) en forma turbulenta,
mediante el empleo de aereadores mecnicos o de aire comprimido. El pro-
ceso implica una etapa de separacin slido-lquido. Parte del slido que se
obtiene en esta segunda etapa o sedimentacin, es recirculado y mezclado
con el agua residual que se alimenta a la etapa de aereacin. En el tanque de
aereacin ocurre:
1. La adsorcin y coagulacin rpida de los compuestos orgnicos solubles
y coloidales por la accin de la biomasa.
2. Una progresiva oxidacin de los compuestos adsorbidos y otros que son
continuamente removidos de la solucin.
3. La sntesis de nueva biomasa, en la que se emplea como sustrato la ma-
teria orgnica contenida en el agua residual.
4. La oxidacin y dispersin de las partculas del lodo cuando la aeracin se
extiende por periodos prolongados.
Los diferentes cambios que ocurren en los procesos del lodo activado se
muestran en la figura 4.5. As por ejemplo, para un albaal cualquiera en el
que la mayor fraccin de la DBO est en forma suspendida y coloidal, el
tiempo de retencin necesario para la remocin rpida de la DBO por
adsorcin de los coloides y floculacin de los slidos en suspensin se repre-
senta por t
1
. Como se observa en la figura 4.5, para el tiempo t
2
ya la mate-
ria orgnica es eliminada por oxidacin y sntesis. Esto ocurre entrada ya la
zona de respiracin endgena. Si se desea que ocurra nitrificacin, el tiempo
de retencin debe ser lo suficientemente alto, t
3
, como para que las bacte-
rias nitrificantes puedan crecer y existir en nmero lo suficientemente gran-
de como para ejercer su accin oxidante sobre el NH
3
en un grado tal que
sea detectable.
A modo de resumen puede decirse que los mecanismos que ocurren
durante el proceso del lodo activado pueden enumerarse como sigue:
1. Remocin inicial de slidos suspendidos y coloidales por aglomeracin
fsica y floculacin, as como por adsorcin sobre los flculos biolgicos.
Estos compuestos orgnicos de estructura generalmente algo compleja
son hidrolizados y descompuestos por la accin biolgica durante la
aereacin.
129
Lodo Activado
2. Una remocin, relativamente lenta, de los compuestos orgnicos solu-
bles. Esta accin, que es debida a la actividad de microorganismos, da
lugar a la formacin de productos finales oxidados, CO
2
y H
2
O y a la
sntesis de nuevos microorganismos, constituyentes de la biomasa.
3. Oxidacin del amonaco a nitrato por los organismos nitrificantes.
4. Oxidacin de la biomasa obteniendo como producto final CO
2
, NH
3
,
H
2
O y fsforo. Adicionalmente se obtiene un residuo no biodegradable,
constituido tanto por sustancias que con estas caractersticas forman
parte de la biomasa, como por una fraccin contenida en el afluente, y
que pasa a travs del tratamiento sin alteracin, segn lo discutido en el
epgrafe 3.3.1.
Fig. 4.5. a) Remocin de DBO. b) Biomasa. c) Oxgeno utilizado.
d) NH
3
. E: NO
3
.
En los ltimos sesenta aos se han desarrollado numerosas modificaciones
al proceso del lodo activado. En la tabla 4.1 se reportan algunas de las caracte-
rsticas ms importantes de estas modificaciones.
130
Menndez Gutirrez, C. y J. Prez Olmo
Tabla 4.1. Diferentes variantes del proceso del lodo activado (los parmetros
de carga estn expresados como DBO
5
y slidos suspendidos
voltiles, SSV)
4.3.1. Sistemas de baja carga
Estos sistemas son diseados para tratar pequeos volmenes de aguas
residuales de origen domstico o industrial, siempre que no sobrepasen los
7000 m
3
d
1
.
Los sistemas de baja carga requieren una edad del lodo alta, veinticinco
das o ms, para promover la oxidacin de la biomasa que se produce duran-
te la sntesis. Tericamente la pequea cantidad del lodo que se produce en
estos sistemas est constituida fundamentalmente por sustancias no
biodegradables. Debido a que los sistemas de baja carga por lo comn no se
disean con sedimentacin primaria, en el tanque de aereacin existir una
mezcla del lodo primario y biomasa.
Como el proceso de estabilizacin aerobia de los lodos es lento, el tiem-
po de retencin hidrulico debe ser tambin mucho mayor que el requerido
para la remocin de los constituyentes orgnicos solubles y coloidales del
agua residual.
En la prctica estos sistemas, conocidos comnmente como de aeracin
extendida, se disean como unidades compactas (figuras 4.6 y 4.7) o como
canales de oxidacin (figura 4.8).
Variantes B
v
(kgm
-3
d
-1
)
B
x
(kgkg
-1
d
-1
)
u
x
(d)
u
(h)
Sistemas de baja carga 0,1-0,3 0,05-0,10
> 25
24-72
Sistemas de carga
media
Estabilizacin por
contacto
0,5-1,5 0,2-0,5 3-15 4-12
Sistemas de alta carga
> 1,5 > 0,5 < 3
1-2
Lodo activado en dos
etapas:
Primera etapa
Segunda etapa
10
0,5-1,0
1-2
0,2-0,5
1
3-15
< 1
4-6
131
Lodo Activado
Fig. 4.6. Esquema de unidad compacta del lodo activado. Aereacin
extendida. 1: afluente. 2: efluente, 3: seccin de aereacin. 4: sedimen-
tacin y recirculacin interna de lodo.
Fig. 4.7. Esquema de unidad compacta. Aereacin extendida.
Fig. 4.8. Canal de oxidacin. Variante de aereacin extendida.
4.3.2. Sistemas de carga media
Estos son los sistemas del lodo activado ms frecuentemente empleados
y pueden disearse con rgimen de flujo de pistn o de mezcla completa.
132
Menndez Gutirrez, C. y J. Prez Olmo
Los sistemas de flujo a pistn tienen un diseo de tanque de aereacin lla-
mado corredor. Sus dimensiones varan dentro de los siguientes lmites:
Ancho: (4-12) m
Profundidad: (4-6) m
Longitud: (20-120) m
Los sistemas de mezcla completa tienen la ventaja de amortiguar los efec-
tos de las variaciones de las concentraciones de entrada al tanque de aereacin
y disminuir, dentro de ciertos lmites, las posibles consecuencias de la presencia
de sustancias txicas.
Entre los sistemas de carga media se destacan el convencional, la aereacin
escalonada, la dosificacin gradual y la estabilizacin por contacto.
Sistema convencional
El proceso que ocurre en un sistema convencional tiene cuatro etapas:
1. Sedimentacin primaria para la remocin de las sustancias orgnicas y sli-
dos inorgnicos sedimentables cuando estn presentes en el agua residual.
2. Aeracin de la mezcla de agua residual y biomasa activa (licor mezclado).
3. Separacin del lodo por sedimentacin.
4. Recirculacin nuevamente a la etapa de aeracin de una fraccin del lodo
separado en la tercera etapa (figura 4.9).
Fig. 4.9. Variante del esquema de un lodo activado convencional.
Aeracin escalonada
Cuando se emplean reactores de flujo de pistn en el tratamiento de aguas
residuales, la demanda de oxgeno es variable a lo largo de toda su longitud.
Si el suministro del oxgeno es uniforme, puede quedar en defecto a la entra-
da del tanque de aeracin y en exceso a la salida (figura 4.10.a).
Con el objetivo de suministrar en cada momento el oxgeno requerido para
satisfacer la demanda, surgi la variante de la aereacin escalonada (figura
4.10.b).
133
Lodo Activado
Fig. 4.10. Relacin demanda-suministro de oxgeno. a) Flujo de pistn.
b) Aereacin escalonada. c) Dosificacin gradual. d) Mezcla completa.
Fig. 4.11. Diferentes variantes del lodo activado. a) Sistema convencio-
nal. b) Dosificacin gradual. c) Aereacin escalonada.
d) Estabilizacin por contacto.
1. Sedimentador primario. 2. Tanque de aereacin.
3. Sedimentador secundario. 4. Entrada de aire.
134
Menndez Gutirrez, C. y J. Prez Olmo
Dosificacin gradual
Esta variante es conocida como sistema Gould. Para uniformar la carga
aplicada y la velocidad de consumo de oxgeno a lo largo del tanque de
aereacin, se dosifica la alimentacin, introducindola por varios puntos a lo
largo de la longitud del tanque (figura 4.11).
El tanque de aereacin se divide por medio de deflectores en tres o ms
cmaras conectadas entre s en serie.
El lodo que se recircula se introduce en la primera de las cmaras.
Este proceso se opera de acuerdo con un rgimen hidrulico de flujo a
pistn y el sistema de aereacin es a base de difusores. De esta manera se
logra una eficiencia de remocin entre 85 y 95 %.
10
Estabilizacin por contacto
Cuando el agua residual tiene una alta concentracin de contaminantes
en forma suspendida no sedimentable y en forma coloidal, puede obtenerse
una buena eficiencia en la remocin por adsorcin en el lodo bien aereado y
previamente estabilizado.
Esta variante del lodo activado se basa en el postulado de que la elimina-
cin de la DBO tiene lugar en dos etapas. La primera es la fase de adsorcin
ya mencionada, llamada fase de contacto y la segunda, fase de oxidacin,
en la que se lleva a efecto la asimilacin metablica de la materia orgnica
(figura 4.11.d).
4.3.3. Sistemas de alta carga
Estos sistemas se disean raras veces debido a que con una eficiencia
de remocin entre 60 y 75 %, producen un efluente de caractersticas
comnmente poco deseables debido a que no estn adecuadamente
estabilizados, y que generalmente, el lodo no presenta buenas propiedades
de floculacin.
Comnmente los sistemas de alta carga se utilizan en dos etapas. La
primera de las etapas para este tipo de sistema opera con elevada carga
(B
v
= 10 kg m
3
d
1
, B
x
12 kgkg
1
d
1
) y la segunda de las etapas tiene
caractersticas de cargas similares a la de los sistemas convencionales
(figura 4.12).
Los sistemas de alta carga son utilizados fundamentalmente en el tratamien-
to aerobio de residuales industriales de alta DBO.
135
Lodo Activado
Fig. 4.12. Proceso del lodo activado en dos etapas.
4.4. APLICACIN DE LOS MODELOS CINTICOS
Los modelos cinticos que describen la remocin de los compuestos orgni-
cos pueden ser utilizados con buena efectividad en la prediccin de la eficiencia
de los procesos del lodo activado.
4.4.1. Sistemas de flujo de pistn
Para sistemas discontinuos (a templa) o continuos a flujo de pistn, la ecua-
cin 3.24 puede expresarse:
) )
=
S
S
t
n
n
n
dt
S
X
k
S
dS
0
0
0
0
(4.20)
donde:
X
0
: Concentracin inicial de biomasa (mgL
1
).
S
0
: DBO inicial (mgL
1
).
S: DBO en el instante t (mgL
1
).
n: Orden de la reaccin.
k
n
: Constante de velocidad de remocin.
Si la reaccin es de orden cero (n = 0), la ecuacin 4.20 integra:
t X k S S
0 0 0
= (4.21)
Por tanto, la fraccin removida viene dada por:
t
S
X
k
S
S S
0
0
0
0
0
=

(4.22)
Para primer orden (n = 1), de la ecuacin 4.20 se obtiene:
|
|
.
|

\
|
= t
S
X
k
S
S
0
0
1
0
exp
(4.23)
136
Menndez Gutirrez, C. y J. Prez Olmo
y
|
|
.
|

\
|
=

t
S
X
k
S
S S
0
0
1
0
0
exp 1
(4.24)
Para segundo orden (n = 2):
t
S
X
k
S
S
0
0
2
0
1
1
+
=
(4.25)
y
t
S
X
k
t
S
X
k
S
S S
0
0
2
0
0
2
0
0
1+
=

(4.26)
Las unidades de la constante de velocidad de remocin en cada caso son:
k
0
: gg
1
h
1
.
k
1
: Lg
1
h
1
.
k
2
: L
2
g
2
h
1
.
4.4.2. Sistemas de mezcla completa
En los sistemas completamente mezclados, la concentracin de salida es
prcticamente igual a la concentracin del sustrato soluble remanente en el tan-
que de aereacin.
Para obtener la expresin que permita predecir la eficiencia de remocin en
sistemas completamente mezclados se requiere de un balance de sustrato alre-
dedor del sistema (figura 4.13):
Fig. 4.13. Variante de esquema de un reactor de mezcla completa.
137
Lodo Activado
V
S Q
S
S
X k
V
S Q
dt
dS
n
n
0
0
0 0

|
|
.
|

\
|
=
(4.27)
en estado estacionario
0 =
dt
dS
, y la ecuacin 4.27, queda:
0
0
0
=

n
n
n
S
S
X
k
0
S S
(4.28)
donde:
( )
d
Q
V
0
0
=
X: Concentracin de SSVLM (mgL
1
).
Resolviendo la ecuacin 4.28 para:
n = 0; n = 1; n = 2;
se obtiene:
n = 0
0
S
X
k
S
S S
0
0
0
0
=

(4.29)
n = 1
0
S
X
k
0
S
X
k
S
S S
0
1
0
1
0
0
1 +
=

(4.30)
n = 2
0
S
X
k
0
S
X
k
S
S S
0
2
0
2
0
0
2
1 4 1 +
=

(4.31)
Cuando se compara las ecuaciones desarrolladas para sistemas de flujo de
pistn y completamente mezclado, puede apreciarse que para igualdad de con-
diciones (S
0
, k
n
, X), solamente para n = 0 la prediccin de la eficiencia de remo-
cin arroja resultados similares (ecuacin 4.21 y 4.29). Para n > 0, las eficiencias
de remocin que se predicen para sistemas de flujo de pistn siempre son mayo-
res que para sistemas completamente mezclados.
138
Menndez Gutirrez, C. y J. Prez Olmo
Sin embargo, esta conclusin terica rara vez se confirma en la prctica.
La razn ms probable de que en la realidad la eficiencia de los sistemas
completamente mezclados no sea menor que en los sistemas de flujo de pistn
est en el mecanismo de remocin de sustrato multicomponente.
10
Durante toda la remocin, la DBO sigue una curva (cintica de orden
distinta a cero). Esta curva es la resultante de la suma de remociones linea-
les (orden 0) de los componentes individuales. Esta es posiblemente la razn
por la que reactores del lodo activado completamente mezclados tienen la
misma eficiencia de purificacin que los reactores de flujo de pistn en igual-
dad de condiciones.
Matemticamente esto significa que las constantes de remocin de las
ecuaciones (4.30) y (4.31) deben ser mayores que la de las ecuaciones (4.24)
y (4.26), respectivamente, lo que ha sido demostrado experimentalmente.
11
Si S
0
y S de las ecuaciones anteriores se expresan en unidades de DQO,
usualmente se sustrae aquella parte de la demanda qumica de oxgeno, S
n
,
que pertenece a los compuestos no biodegradables, (S
0
S
n
; S S
n
).
Para aguas residuales municipales los valores de S
n
varan en el intervalo
comprendido entre 2 y 40 gm
3
. En residuales industriales este resultado puede
ser mayor. En la prctica los valores de DQO y DBO se emplean indistintamen-
te obviando la mayora de las veces la fraccin no biodegradable contenida en el
agua residual.
4.5. FACTORES QUE INCIDEN EN LA EFICIENCIA
DE PURIFICACIN
Los factores que inciden en la eficiencia de purificacin pueden ser clasifi-
cados en dos grupos:
1. Factores o parmetros bsicos.
2. Factores o parmetros derivados.
Factores o parmetros bsicos
Entre los factores o parmetros bsicos ms importantes pueden dis-
tinguirse:
a) Tiempo de retencin.
b) Concentracin de biomasa.
c) Temperatura.
d) Concentracin de contaminantes.
139
Lodo Activado
Factores o parmetros derivados
a) Carga volumtrica.
b) Edad del lodo.
4.5.1. Tiempo de retencin
El tiempo de retencin es un factor determinante en la eficiencia de purifi-
cacin del proceso del lodo activado. Tal como se refleja en la tabla 4.1, de
acuerdo con la variante del lodo activado que se utilice, as ser el tiempo de
retencin que se requiera.
El tiempo de retencin es un parmetro importante por cuanto define, entre
otros factores, el volumen del tanque de aereacin.
Para todo sistema existe un tiempo de retencin denominado crtico (u
c
) por
debajo del cual la eficiencia del proceso disminuye abruptamente (figura 4.14).
Fig. 4.14. Influencia del tiempo de retencin en la eficiencia.
Este valor crtico depende en general de:
a) Carcter y concentracin de los compuestos orgnicos en el agua.
b) Concentracin de SSVLM en el tanque de aeracin.
c) Temperatura.
El tiempo de retencin crtico vara adems, de un residual a otro. Como
criterio, puede considerarse que para las aguas residuales municipales el q
c
es
cercano a las cuatro horas.
4.5.2. Concentracin y actividad del lodo activado
La concentracin de microorganismos en el tanque de aeracin tiene, al
igual que el tiempo de retencin, un efecto importante en la eficiencia de purifi-
140
Menndez Gutirrez, C. y J. Prez Olmo
cacin. Esto puede apreciarse en la figura 4.15. El grfico de esta figura fue
obtenido en un proceso a templa a partir del cual puede concluirse que existe
una relacin lineal entre la velocidad de remocin de DBO y la concentracin
inicial del lodo en el sistema.
Fig. 4.15. Dependencia de la velocidad de remocin de DBO con la
concentracin inicial de biomasa (lactosa como sustrato).
Para sistemas continuos, la relacin general entre la eficiencia y la concen-
tracin de SSVLM (X) se ilustra en la figura 4.16, en la cual puede apreciarse
que existe un valor de X X
c
( ) por encima del cual la eficiencia prcticamente no
se incrementa. Este valor crtico depende de:
a) Tiempo de retencin en el tanque de aeracin.
b) Temperatura.
En todo sistema del lodo activado existe una concentracin crtica de X.
En condiciones dadas (u, AS, T), cualquier cambio en la concentracin
de X solamente puede lograrse mediante un cambio en la edad del lodo.
La concentracin de biomasa en un reactor que trabaja en condiciones
de flujo continuo sin recirculacin est dado por la velocidad de crecimiento.
Concentraciones de biomasa por encima de este valor solo pueden lograrse
mediante la recirculacin de lodo, o sea, aumentando la edad del lodo. Sin
embargo, en la medida en que la edad del lodo se incrementa este envejece
y disminuye la fraccin de clulas activas en l. La relacin existente entre
la fraccin de clulas vivas y la edad del lodo se ilustra en la figura 4.17.
Al aumentar la edad del lodo se incrementa la concentracin de biomasa,
pero la fraccin activa de la misma disminuye.
12
Esto puede traer como conse-
cuencia que, a partir de un cierto valor de edad del lodo, no se obtengan benefi-
cios por aumentos posteriores de esta debido a que la actividad total de la biomasa
por unidad de volumen no experimentar cambios apreciables. Esta es la razn
141
Lodo Activado
por la que la eficiencia de purificacin se eleva con el aumento de la concentra-
cin del lodo, hasta un valor que se corresponda con la concentracin crtica, y
que incrementos por encima de esta concentracin no se traduzcan en aumen-
tos en la eficiencia.
Fig. 4.16. Influencia de la concentracin de SSVLM en la eficiencia de
purificacin.
Fig. 4.17. Relacin entre la edad del lodo y la fraccin de clulas activas.
Sin embargo, si el sistema del lodo activado est sobrecargado, es decir,
est siendo operado a bajos tiempos de retencin, entonces un incremento
de la concentracin del lodo dar como resultado un aumento de la eficien-
cia de purificacin.
Teniendo en cuenta el efecto que tiene sobre la eficiencia de purificacin la
relacin que existe entre el tiempo de retencin (u) y la concentracin de biomasa
(X), es que se defini la carga de lodo:
X
S
B
x
0
=
(kg kg
1
d
1
)
X
c
142
Menndez Gutirrez, C. y J. Prez Olmo
No se reconocen evidencias de una posible relacin entre la carga del lodo
y la eficiencia de purificacin. De existir esta relacin se obtendra la misma
eficiencia de purificacin para distintos pares de valores de X y u (X, u); (X/2,
2u); (2 X, u/2), etc.
4.5.3. Temperatura
La temperatura es otro de los parmetros que inciden de manera importante
en la eficiencia de purificacin.
Similarmente a lo que ocurre en las reacciones qumicas, en los procesos
biolgicos el efecto de la temperatura puede ser expresado a travs de una
constante de velocidad en la que se cumpla la ecuacin de Arrhenius:
K = A exp (-E/RT) (4.32)
donde:
K: Constante de velocidad cuyas unidades depende del orden de la reaccin.
E: Energa de activacin.
R: Constante de los gases.
T: Temperatura absoluta.
Aplicando logaritmo a la ecuacin 4.32:
A
RT
E
K ln ln + =
(4.33)
Para dos temperaturas, T
1
, T
2
y sus correspondientes constantes, K
1
, K
2
puede obtenerse:
|
|
.
|

\
|
=
2 1
2 1
2
1
ln
T T
T T
R
E
K
K
(4.34)
En la prctica del tratamiento de aguas residuales el intervalo de temperatu-
ra no es muy amplio, por tanto, el producto T
1
T
2
es aproximadamente constante
y la ecuacin 4.34 puede escribirse:
2 1
2
1 T T
v
K
K

=
(4.35)
en la que
n = exp (E/RT
1
T
2
)
143
Lodo Activado
La ecuacin 4.35 generalmente se utiliza de la forma:
20
20

=
t
C t
v k k (4.36)
El valor del coeficiente v vara para diferentes procesos en el intervalo de 1,01
a 1,10. Wuhrman
13
y Eckenfelder
14
han reportado valores para v de 1,074
y 1,085, respectivamente. Precisando ms estos valores, Wuhrman
15
ha com-
probado que para B
x
< 0,5 kgkg
1
d
1
se obtiene que el valor del coeficiente v es
aproximadamente 1,0 Por otro lado, para valores de carga del lodo superiores a
0,5 (B
x
> 0,5 kgkg
1
d
1
) el valor de v est comprendido entre 1,0-1,04
Los valores ms frecuentemente utilizados son 1,035 y 1,047.
11
El efecto de la temperatura sobre la eficiencia de purificacin depende en
cierta medida de los valores del tiempo de retencin y la concentracin de los
slidos suspendidos voltiles del licor mezclado. Hay evidencias que apuntan
que a medida que son mayores estos dos parmetros, el efecto de la temperatu-
ra tiene menos importancia.
4.5.4. Composicin y concentracin
La eficiencia de purificacin, expresada en %, depende en cierta medida de
la concentracin de los compuestos orgnicos presentes en el agua residual.
Esto puede ilustrarse mediante un ejemplo. Si la DBO disminuye desde 1000
gm
3
hasta 10 gm
3
, eso significa una eficiencia de 99 %. Para obtener esa
misma eficiencia con un agua residual que tiene una concentracin en el influente
de 100 gm
3
, se requiere que la DBO decrezca hasta 1 gm
3
, y esto es prcticamente
imposible. Por tanto, la eficiencia decrece al disminuir la contaminacin orgni-
ca. Este fenmeno recibe el nombre de efecto de la concentracin.
Por otro lado, del ejemplo anterior puede deducirse que la eficiencia de
remocin reportada en por ciento no brinda una informacin suficiente sobre el
comportamiento del proceso del lodo activado. Una visin clara de este proceso
solo puede obtenerse comparando los valores absolutos de DBO y DQO en el
afluente y efluente.
La eficiencia de purificacin puede afectarse adems como consecuencia de
un desbalance nutricional del agua residual a tratar. De estos los ms importantes
son el nitrgeno y el fsforo. Mientras las aguas residuales municipales contienen
estos elementos en exceso, muchos residuales industriales carecen de ellos. Para
propsitos de proyecto, la cantidad necesaria de estos elementos puede estimarse
aproximadamente utilizando la siguiente relacin emprica DBO
5
: N: P = 100: 5: 1.
Esta relacin de DBO
5
: N: P se cumple mejor para sistemas de carga me-
dia. Para sistemas de baja carga la demanda es menor ya que la demanda de
nutrientes aumenta en la medida que se incrementa la carga del sistema.
144
Menndez Gutirrez, C. y J. Prez Olmo
4.6. PRESENCIA DE COMPUESTOS ORGNICOS
EN EL EFLUENTE
Los compuestos orgnicos degradables en el efluente secundario pueden
dividirse en dos grupos:
a) slidos o compuestos suspendidos (SS).
b) slidos solubles o disueltos (SD).
La concentracin de compuestos solubles depende del comportamiento del
proceso del lodo activado. Por otro lado el contenido de slidos suspendidos en
el efluente est determinado en gran medida por la eficiencia del sedimentador
secundario, que rara vez alcanza valores mayores del 95 %.
Los sedimentadores secundarios operando en buenas condiciones
generalmente producen un efluente con un contenido de slidos suspendidos
entre 10 y 30 g m
3
.
Estos slidos estn constituidos por un flculo fino del lodo activado. Los
flculos contienen microorganismos vivos que consumen oxgeno y que al mis-
mo tiempo su presencia en el agua representa un aporte de DBO debido a su
naturaleza orgnica.
La DBO del efluente de un tratamiento biolgico es por tanto el resultado de
la suma de la DBO ejercida por los compuestos orgnicos disueltos y por los
slidos suspendidos:
) ) )
suspendido
5
disuelto
5
total
5
DBO DBO DBO + =
(4.37)
Es posible obtener una relacin entre la concentracin de slidos suspendi-
dos en el efluente (X
2
, ver figura 4.4) y la DBO ejercida por ellos:
)
2 5
066 0 1
86 0
X
0 ,
f ,
DBO
x
o
s
+
=
(4.38)
donde:
DBO
5
)
s
= DBO
5
suspendida (mg L
1
).
u
x
: Edad del lodo (d).
f
o
: Fraccin voltil de los slidos suspendidos.
X: slidos suspendidos totales (mg L
1
).
Como ya se ha mencionado, la fraccin voltil de los slidos suspendidos en
funcin de la edad del lodo (figura 4.18).
13
La expresin analtica de esta rela-
cin emprica es:
1 0
85 0
,
x
o
0
,
SS
SSV
f = =
(4.39)
145
Lodo Activado
Por otra parte, si:
x
o
0 ,
f ,
K
066 0 1
86 0
+
=
(4.40)
sustituyendo 4.40 y 4.38 en 4.37 se obtiene:
) )
2 5 5
X K DBO DBO
d T
+ = (4.41)
Fig. 4.18. Fraccin voltil de los slidos suspendidos y la edad del lodo.
El coeficiente K vara generalmente entre los valores 0,2 y 0,7 en dependen-
cia de la edad de lodo.
Para sistemas de baja carga, suponiendo u
x
= 30 d
f
0
= 0,60 y la ecuacin 4.41 queda:
) )
2 5 5
173 0 X , DBO DBO
d T
+ = (4.42)
Para sistemas de carga media, suponiendo u
x
= 5 d. f
0
= 0,72 y la ecuacin
4.41 queda:
) )
2 5 5
465 0 X , DBO DBO
d T
+ = (4.43)
Para sistemas de alta carga, suponiendo u
x
= 1 d, f
0
= 0,85 y de la ecuacin
4.41 se obtiene:
) )
2 5 5
686 0 X , DBO DBO
d T
+ = (4.44)
Si en lugar de trabajar con la DBO
5
el sistema se est analizando en
funcin de la DBO ltima o de la DQO, la concentracin de estas que apor-
tan los slidos suspendidos del efluente secundario puede calcularse:
14
)
2
42 1 X f , DBO
o s u
= (4.45)
146
Menndez Gutirrez, C. y J. Prez Olmo
Cuando el valor de X
2
se toma en funcin de los slidos suspendidos
voltiles en lugar de los slidos suspendidos, el valor de f
0
es igual a la uni-
dad en todos los casos.
De todo lo anterior puede concluirse que los valores totales de DBO
5
y
DQO en el efluente secundario tienen una dependencia significativa de los
flculos finos que salen en el lquido claro del sedimentador secundario.
Algunos autores plantean que en los efluentes de los sistemas de carga
baja y media 50 a 75 % de la DBO
5
)
total
es producida por los slidos
suspendidos. Su remocin o disminucin incide en una mejor calidad en las
caractersticas del efluente.
15
4.7. PRODUCCIN DE BIOMASA Y LODO
Las aguas residuales contienen compuestos orgnicos disueltos y slidos
suspendidos, tanto de origen orgnico, como inorgnico. Aunque los slidos sus-
pendidos se remueven en el tanque de sedimentacin primaria, esta nunca es
completa.
La eficiencia de la sedimentacin vara en un intervalo entre (50 y 70) %.
Esto significa que una fraccin de los slidos suspendidos es alimentada al tan-
que de aereacin.
Una parte de los slidos suspendidos que entran al tanque de aereacin es
degradable biolgicamente, y otra parte no lo es. La fraccin degradable ejerce
DBO y puede ser considerada como sustrato. Los slidos no degradables, tanto
de origen orgnico como inorgnico, son denominados slidos inertes. Estos
slidos inertes atrapados en el tanque de aeracin, forman parte del lodo activa-
do. El lodo activado es, por tanto, una mezcla del lodo biolgico (biomasa) y
slidos suspendidos inertes.
Solamente en aquellos casos en que el agua residual no contenga slidos
suspendidos inertes, el lodo activado estar constituido en su totalidad por biomasa.
Para simplificar el tratamiento matemtico de los procesos del lodo activa-
do, generalmente no se toman en consideracin los slidos suspendidos que
entran al tanque de aereacin y se supone que la biomasa que entra con el
residual es igual a cero.
4.7.1. Concentracin de biomasa y su produccin
Como una primera aproximacin al problema, que muchas veces no se
aleja de lo que para fines prcticos puede suponerse, se considerar que
todos los slidos a la entrada, disueltos y suspendidos, estn reflejados en la
DBO DQO.
147
Lodo Activado
Fig. 4.19. Esquema de un proceso del lodo activado para balance
de masa.
Tomando como referencia el esquema de la figura 4.19 y haciendo un ba-
lance de biomasa total:
Cada uno de los trminos de la ecuacin 4.46 puede plantearse:
Cambio total en el tanque:
dt
dX
b
Incremento debido al crecimiento:
0
2 0
Q
V
S S
dt
dS
Y
dt
dX
b

= =
Disminucin debido a la muerte:
b b
b
X k
dt
dX
=
donde:
Y: Rendimiento o coeficiente de produccin de biomasa (kg kg
1
).
k
b
: Constante de autooxidacin (kg kg
1
d
1
).
Teniendo en cuenta estos trminos, la ecuacin 4.46 queda:
(4.47)
Considerando que:
0
Q
V
=
0
Cambio total en
el TK =
Incremento
debido al +
crecimiento
Disminucin
debido a la +
muerte
Disminucin
debido a la +
eliminacin
del exceso
Disminucin
por lo que
sale en el
efluente
(4.46) (4.46)
148
Menndez Gutirrez, C. y J. Prez Olmo
S A S S =
2 0
( )
x b
b, w b,w w
0 X V
X Q Q X Q 1
2 0
=
+
y que en estado estacionario
0 =
dt
dX
b
, puede simplificarse la ecuacin 4.47 y
despejar el trmino X
b
que representa la concentracin de biomasa en el reactor
o tanque de aeracin.
( )
x b
x
b
0 k 0
S A 0 Y
X
+
=
1
(4.48)
La ecuacin 4.48 brinda la relacin entre la concentracin de biomasa en
estado estacionario y las variables edad del lodo, tiempo de retencin y remo-
cin de DBO una vez que son conocidos el rendimiento (Y) y la constante de
autooxidacin (k
b
).
En la tabla 4.2 se ofrecen algunos valores tpicos de estos dos parmetros.
Tabla 4.2. Valores tpicos de rendimiento y constante de autooxidacin en un
proceso aerbico
16
No obstante que en la literatura se pueden encontrar valores de Y y k
b
para
el tratamiento de distintos residuales, siempre que se pueda, estos deben ser
hallados a nivel de planta piloto o laboratorio mediante un procedimiento relati-
vamente sencillo tal como se describir en el epgrafe 4.8. basta con destacar
ahora que la ecuacin 4.47 puede transformarse para obtener:
b x
x
k B A Y
0
=
1
(4.49)
donde:
0 X
S A
AB
b
x
=
Residual Y
(base DBO)
Y
(base DQO)
kb
(d
-1
)
Municipal 0,4 0,8 0,3 0,4 0,04 0,08
Industria
Azucarera
0,33 0,04
Papel 0,36 0,07
Residual
Lcteo
0,35 0,06 0,2 0,4 0,04 0,06
149
Lodo Activado
Al graficar el inverso de la edad del lodo en funcin de AB
x
(figura 4.20), se
obtiene:
Fig. 4.20. Clculo del rendimiento Y y la constante de auto-oxidacin k
b
.
La ecuacin 4.49 que ha sido desarrollada a partir de un balance de
materiales, fue obtenida hace ms de 40 aos de forma emprica por
Heukelekian.
17
La cantidad de biomasa producida puede calcularse a partir de una modifi-
cacin de la ecuacin 4.49.
( )
1 3
d m kg

=
b b v b
X k AB Y AX
(4.50)
La produccin total de biomasa viene dada por:
( )
1
d kg

= V AX PB
b
(4.51)
donde:
V: Volumen efectivo del tanque de aereacin.
La ecuacin 4.50 puede reordenarse y obtener de ella otro parmetro im-
portante, el rendimiento verdadero o rendimiento observado, Y
ob
:
x
b
v
b
AB
k Y
AB
AX 1
=
(4.52)
El trmino
v
b
AB
AX
de la ecuacin 4.52 tiene las mismas dimensiones que Y, kg
kg
1
y es denominado rendimiento observado:
x
b
ob
AB
k
Y Y =
(4.53)
150
Menndez Gutirrez, C. y J. Prez Olmo
Otra forma til de expresar el Y
ob
, independientemente del valor de AS
cuyo conocimiento lleva implcito la ecuacin 4.53, puede obtenerse de la ecua-
cin 4.50:
x b
ob
0 k
Y
Y
+
=
1
(4.54)
Todo el desarrollo hasta aqu se ha basado en la suposicin de que toda la
biomasa es degradable, sin embargo, una discusin alrededor de la ecuacin
4.54 puede ser til para justificar la necesidad de introducir el criterio de que
toda la biomasa no es biodegradable.
De la ecuacin 4.54 puede inferirse la influencia que tiene la edad del lodo
sobre Y
ob
. Mientras menor sea la edad del lodo, la Y
ob
se aproximar ms al valor
de Y. Esa conclusin responde a la realidad ya que mientras ms pequea sea la
edad del lodo menos acentuada ser la autooxidacin o fase endgena. De he-
cho, tanto la ecuacin 4-54 como las precedentes pueden utilizarse sin mucho
error siempre que u
x
< 5 das.
El alcance o validez de la ecuacin 4.54 es ms limitado cuando la edad del
lodo es superior a los diez das.
Si se supone un caso extremo, u
x
entonces 0
ob
Y
Esta conclusin no responde a la realidad. En la prctica diaria, por muy alta
que sea la edad del lodo, la Y
ob
nunca es cero. Esto se debe a que una parte de
la biomasa orgnica producida es no degradable bajo condiciones aerbicas, y
por tanto, aparece reflejada en la Y
ob
18, 19
Consecuentemente con esto, el balan-
ce de biomasa debe contemplar tanto la fraccin degradable como la no degradable
utilizando el desarrollo propuesto por Chudoba y Tucek.
20
La biomasa total sintetizada, X
b
, est constituida por tres fracciones:
a) La fraccin orgnica degradable (X
b, D
).
b) La fraccin orgnica no degradable (X
b, N
).
c) La fraccin mineral (X
b, M
).
M , b N , b D , b b
X X X X + + =
(4.55)
De estas tres fracciones, solo X
b,D
es degradable de acuerdo con la expresin:
b,D d
b
X k
dt
dX
=
donde:
k
d
: Constante de velocidad de descomposicin de la fraccin degradable de
la biomasa (kg kg
1
d
1
).
151
Lodo Activado
Si | es la fraccin no degradable aerbicamente de biomasa orgnica y Y
0
,
el coeficiente de produccin de biomasa total, entonces |Y
0
es el coeficiente de
produccin de la parte de la biomasa orgnica que es no degradable y (1 - |) Y
0
es el coeficiente de produccin de biomasa orgnica degradable. Sea considera-
do adems f
0
m
como la fraccin mineral de la biomasa al formarse.
Por otra parte, aceptando que AS es la cantidad de sustrato removido en el
sistema, entonces Y
0
AS es la parte orgnica de la biomasa producida y
( ) | | S A Y m f / m f
0
0 0
1
es la parte inorgnica de la biomasa producida.
Un balance de masa de la biomasa degradable da origen a una ecuacin
similar a la 4.47.
Tomando en consideracin que en estado estacionario la edad del lodo de la
biomasa total u
x
es la misma para la parte degradable y no degradable, se podr
escribir:
( )
0
1
0
=

x
D , b
D , b d
0
X
X k
0
AS Y
(4.56)
despejando X
b,D
( )
( )
x d
x
D , b
0 k 0
0 AS Y
X
+

=
1
1
0
(4.57)
Mediante un balance de masa de la parte orgnica no degradable se obtiene:
0
0
=
x
N , b
0
X
0
AS Y
(4.58)
despejando X
b,N
:
0
0 AS Y
X
x
N , b
0
=
(4.59)
La concentracin de biomasa orgnica (X
b,o
), se obtiene de la suma de las
ecuaciones 4.57 y 4.58:
|
|
.
|

\
|
+
+
=
x d
x d x
b,o
0 k
0 k
0
AS0 Y
X
1
1
0
(4.60)
De acuerdo con lo anterior, la fraccin degradable de los constituyentes
orgnicos de la biomasa, f
d
, es igual a
( )
( )
x d b,o
b,D
d
0 k

X
X
f
+

= =
1
1
(4.61)
152
Menndez Gutirrez, C. y J. Prez Olmo
Al hacer el balance de la parte mineral se supone que la fraccin mineral de
las nuevas clulas es f
0
m. Entonces:
0
1
0
0
0
=

x
M . b
0
X
0
AS Y
m f
m f
(4.62)
despejando X
b,M
:
0
0 AS Y
m f
m f
X
x
M , b
0
0
0
1
=
(4.63)
La concentracin total de biomasa se obtiene sumando las ecuaciones 4.60
y 4.63:
|
|
.
|

\
|

+
+
+
=
m f
m f
0 k
0 k
0
0 AS Y
X
x d
x d x
b
0
0
0
1 1
1
(4.64)
Teniendo en cuenta que:
0
AS
0
X
Y
x
o , b
ob ,
=
0
La ecuacin 4.60 puede escribirse:
|
|
.
|

\
|
+
+
=
x d
x d
ob ,
0 k
0 k
Y Y
1
1
0 0 (4.65)
para
0
x
0
0 0
Y Y
,ob

obtenindose un resultado que es lgico.
Por otra parte, para aquellos casos en los que:

x
0
0 0
Y Y
ob ,

Este ltimo resultado de que para altas edades del lodo el valor del coefi-
ciente Y
0,ob
tiende a la fraccin no degradable de la biomasa, es coherente con la
realidad prctica. De aqu puede concluirse que para edades del lodo pequeas
y moderadas, pueden utilizarse las ecuaciones 4.48 hasta la 4.54. Para altas
edades del lodo deben tenerse en cuenta las consideraciones que le siguen hasta
la ecuacin 4.65.
153
Lodo Activado
4.7.2. Influencia de los slidos suspendidos de la alimentacin
en la concentracin del lodo activado
Los slidos suspendidos primarios, X
0
, al igual que la biomasa, estn consti-
tuidos por tres fracciones:
a) Orgnica degradable (X
O, D
)
b) Orgnica no degradable (X
O, N
)
c) Mineral (X
M
)
M O,N O,D
X X X X + + =
0
(4.66)
Si X
0
es la concentracin total de los slidos suspendidos primarios en el
agua residual, f
o
la fraccin orgnica y a la fraccin no degradable, se pueden
establecer las siguientes relaciones:
X
O, D
= (1 - o) f
o
X
0
(4.67)
X
O, N
= o f
o
X
0
(4.68)
X
M
(1 f
o
) X
0
(4.69)
Puede suponerse que la fraccin orgnica degradable est incluida en la
DBO
5
y que de acuerdo con el producto Y
0
AS es convertida en nueva
biomasa.
La suposicin de que Y
0
tiene el mismo valor para el material orgnico, sea
soluble o insoluble, es aceptable desde el punto de vista prctico.
La concentracin en estado estacionario en el tanque de aereacin de las
dos fracciones que no experimentan degradacin, X
1,N
y X
1,M,
puede obtenerse
por un balance de masa.
El balance de masa de la fraccin no degradable da origen a una ecuacin
similar a la 4,58.
0
1 0
=
x
,N o
0
X
0
X f o
(4.70)
despejando:
0
0 X f o
X
x o
,N
0
1
=
(4.71)
Del balance de la fraccin mineral:
( )
0
1
1
=

x
,M o
0
X
0
f
(4.72)
154
Menndez Gutirrez, C. y J. Prez Olmo
( )
0
0 X f
X
x o
,M
0
1
1
=
(4.73)
La concentracin total del lodo primario se obtiene sumando las ecuaciones
4.71 y 4.73:
|
|
.
|

\
|
+ =
o
o x o
f
f
o
0
0 X f
X
1
0
1 (4.74)
La concentracin total del lodo activado en el tanque de aeracin en
condiciones de estado estacionario viene dado por la suma de la concentra-
cin total de biomasa (ecuacin 4.64) y la de los slidos suspendidos prima-
rios (ecuacin 4.74):
|
|
.
|

\
|

+
+
|
|
.
|

\
|

+
+
+
= + =
o
o x o
x d
d x
b t
f
f
o
0
0 X f
m f
m f
0 k
k
0
0 AS Y
X X X
1
1 1
1
0
0
0
0
1
(4.75)
Tabla 4.3. Valores de constantes para biomasa y slidos suspendidos prima-
rios para aguas residuales domsticas
17
Ejemplo 4.1
Se desea mantener un sistema del lodo activado operando a una edad del
lodo de tres das para las siguientes condiciones de trabajo y de acuerdo con el
esquema de la figura 4.19.
Q
0
= 15000 m
3
d
1
Constante Intervalo Valor
recomendado
Asociados a
los SSP
o
fo
0,3 0,4
0,70 0,75
0,3
0,73
Asociados a
la biomasa
|
f
0
m
Y
0
k
d
0,18 0,24 0,22
0,05
0,65
0,15 (20
o
C)
155
Lodo Activado
Volumen efectivo del tanque de aereacin = 3750 m
3
S
0
= 500 mgL
1
(total) X
w
= 5,662 kg m
3
S
2
= 50 mgL
1
(soluble)
Y = 0,5 k
b
= 0,03 d
1
Determine:
a) Cantidad del lodo que debe ser extrado.
b) Flujo volumtrico de lodos a extraer.
c) Flujo de recirculacin.
a)
( )
46 0
3 03 0 1
5 0
,
,
,
Y
ob
=
+
=
PB = Y
ob
Q
0
(S
0
S
2
).
PB = 0,46 (15 000) (0,45).
PB = 3 105 kg d
1
.
b)
( ) ( )
( )
| |
3
m kg 477 2
3 03 0 1 25 0
45 0 5 0 3

=
+
= ,
, ,
, ,
X
b
Suponiendo X
2
~ 0:
( )
( )
662 5
477 2 750 3
3
, Q
,
w
=
Q
w
= 546,8 m
3
d
1
.
d) Suponiendo: X
2
~ 0
(Q
0
+ Q
r
) X = X
w
Q
w
+ X
w
Q
r
1 3
d m 57 693 10
185 3
98 095 3 155 37

=

= ,
,
,
Q
r
Ejemplo 4.2
En una planta que emplea lodo activado est siendo tratada un agua residual
con las siguientes caractersticas:
Slidos suspendidos totales = 0,275 kg m
3
Flujo de agua residual = 3 785 m
3
d
1
Volumen efectivo del tanque de aereacin = 7 570 m
3
DBO
5
de los slidos totales = 0,3 kg m
3
DBO
5
de los slidos disueltos = 0,15 kg m
3
Toda la DBO
5
es removible, por tanto AS = 0,3
156
Menndez Gutirrez, C. y J. Prez Olmo
Determine, en estado estacionario:
a) Concentracin de biomasa.
b) Concentracin de slidos primarios.
c) Concentracin total del lodo activado en el tanque de aeracin.
d) DBO
5
en el efluente de la planta si el sedimentador secundario tiene una
eficiencia de remocin de slidos suspendidos de 90 %.
Suponga las siguientes condiciones:
u
x
= 10 d f
o
= 0,73 (para los slidos primarios)
t = 26
o
C o = 0,3
1
20
15 0

= d , k
C
o
d
Y
0
= 0,65
| = 0,22
f
o
m = 0,05
a) ( )
1 20 26
26
197 0 047 1 15 0

= = d , , , k
C d
o
( ) ( )
( )
| |
3
m kg 256 0
10 197 0 1 2
10 3 0 65 0 78 0

=
+
= ,
,
, , ,
X
D b
( ) ( )
3
m kg 21 0
2
10 3 0 65 0 22 0

= = ,
, , ,
X
N b,
( )
3
m kg 0,05
2
10 3 0 65 0
95 0
05 0

= =
, ,
,
,
X
M b,
X
b
= 0,256 + 0,210 + 0,050 = 0,516 kg m
3
b)
( )
3
1
m kg 672 0
73 0
27 0
3 0
2
10 275 0 73 0

= |
.
|

\
|
+ = ,
,
,
,
, ,
X
c) X = 0,516 + 0,672 = 1,188 kg m
3
d) X
2
= 0,1 (0,256) = 0,0256 kg m
3
Para el lodo, considerando que 0
X
= 10 d: f
o
= 0,67 (de la figura 4.18)
)
1 3
5
L mg 9 10 0256 0 35 0

= = , , DBO
T
157
Lodo Activado
4.8. DETERMINACIN DE CONSTANTES A ESCALA
DE LABORATORIO
Una unidad del lodo activado a escala de banco es la que se utiliza co-
mnmente para la determinacin en el laboratorio de las constantes Y y k
b
entre otras. Las condiciones a las que se opera la unidad son las mismas
supuestas en las secciones precedentes para la obtencin de las ecuaciones
a partir de los balances de masa.
Elementos esenciales para la obtencin de buenos resultados son el man-
tener un flujo continuo de residual al reactor y un sistema completamente
mezclado en l.
Para lograr suficientes datos, la unidad debe operarse a varias edades
de lodo, generalmente en el intervalo de 3 a 20 das. La manera ms sencilla
de mantener constante la edad del lodo es la extraccin diaria de una canti-
dad constante de licor mezclado del propio reactor o tanque de aereacin.
Las muestras a cada edad del lodo no deben considerarse como representa-
tivas del proceso hasta que no haya transcurrido un tiempo que sea igual o
mayor a cinco veces la edad de lodo. Solo as podr considerarse que el
sistema est trabajando en estado estacionario.
21
La temperatura, pH y el oxgeno disuelto son parmetros que deben per-
manecer constantes o al menos ser registrados mientras duren las pruebas.
La informacin requerida para el clculo de los parmetros de opera-
cin es:
Q: Flujo de entrada (L d
1
).
Q
w
: Flujo de extraccin del lodo (L d
1
).
V: Volumen del tanque de aereacin (L).
X
b
: Concentracin de SSVLM (mg L
1
).
X
e:
Concentracin de SSV en el efluente (mg L
1
).
S
0
: DBO soluble DQO en el influente (mg L
1
).
S
e
: DBO soluble DQO en el efluente (mg L
1
).
A partir de esta informacin se calculan, para cada edad del lodo el
tiempo de retencin (ecuacin 4.1) y la razn de la carga del lodo (ecuacin
4.12). Si el exceso del lodo se extrae directamente del tanque de aeracin,
el volumen diario a extraer viene dado por:
x
w
0
V
Q =
158
Menndez Gutirrez, C. y J. Prez Olmo
Los pares de valores de
x
0
1
y
x
AB que se obtienen para cada edad del lodo
se grafican segn la figura 4.22. La pendiente de la recta que se obtiene da el
valor del rendimiento Y y el intercepto con el eje de ordenadas el valor de la
constante de autooxidacin k
b
.
Fig. 4.21. Esquema de un arreglo de equipamiento para calcular las
constantes a escala de laboratorio.
Ejemplo 4.3
Se desea determinar los parmetros Y y k
b
de un agua residual de ori-
gen industrial utilizando un tanque de aereacin con un volumen efectivo
de 10 L. El agua residual sedimentada se aliment a un flujo de 20 L d
1
para obtener un tiempo de retencin de 12 horas durante todo el experi-
mento.
La extraccin del lodo se hizo directamente del tanque de aereacin
una vez al da, excepto para la primera de las corridas, que por ser
relativamente elevado, el volumen de extraccin, se hizo en dos partes
(tabla 4.4).
159
Lodo Activado
Tabla 4.4. Datos experimentales
Como la concentracin de SSV en el efluente es baja, la edad del lodo puede
calcularse directamente a partir de la extraccin Q
w
.
A partir de los datos anteriores puede obtenerse la tabla 4.5.
Tabla 4.5. Datos obtenidos
S
0
S
e
Q
w
X X
e
(mgL
-1
) (mgL
-1
) (L d
-1
) (mgL
-1
) (mgL
-1
)
750 75 2,4 2700 10
800 96 3,0 2347 5
775 116 3,9 1757 2,5
825 148 4,8 1504 2,5
850 170 5,4 1360 2,7
Al graficar
x
0
1
contra AB
x
se obtiene la recta de la figura 4.22.
u
x
x
u
1 AB
x
(d) (d
-1
) (kg kg
-1
d
-1
)
4,16 0,24 0,50
3,33 0,30 0,60
2,56 0,39 0,75
2,08 0,48 0,90
1,85 0,54 1,00
Fig. 4.22. Ejemplo 4.3 (1/u
X
vs AB
x
).
160
Menndez Gutirrez, C. y J. Prez Olmo
Rendimiento, Y = 0,53
Constante de auto-oxidacin, k
b
= 0,007 d
1
Ejemplo 4.4
A partir de la informacin suministrada en el ejemplo anterior, determine la
constante de velocidad de reaccin:
Graficando AB
x
contra
0
S
S
de acuerdo con una modificacin de la ecuacin
4.28, se obtiene la constante k de velocidad de reaccin de la pendiente de la
recta (figura 4.23).
AB
x
kgkg
-1
d
-1
1,00 0,90 0,75 0,60 0,50
0
S
S 0,20 0,18 0,15 0,12 0,10
Fig. 4.23. Ejemplo 4.4 (AB
x
vs S/S
0
).
4.9. REQUERIMIENTOS NUTRICIONALES
La formacin de biomasa y las buenas propiedades de sedimentacin de
un lodo pueden verse afectadas si el nitrgeno y fsforo no estn presentes
en cantidades suficientes. Otros muchos elementos tambin son necesarios
para el metabolismo, pero en concentraciones tales que raras veces estn
en dficit.
De acuerdo con el modelo de Lawrence y McCarty,
22
el contenido de
nitrgeno y fsforo de la biomasa producida es de 12,3 % y 2,6 % respecti-
k = 5d
1
161
Lodo Activado
vamente. Por otra parte, a medida que disminuye la carga orgnica o au-
menta la edad del lodo, existir una acumulacin de residuos celulares no
biodegradables y cuyo contenido de nitrgeno y fsforo es de 7 % y 1 %.
Por tanto el nitrgeno y el fsforo necesarios dependen de la edad del lodo.
Bajo estas condiciones, y teniendo en cuenta adems que el requerimiento
de estos nutrientes puede ser estimado a partir de la cantidad de estos dos
elementos que estn contenidos en el lodo que se extrae del proceso, se
tiene que la necesidad de ambos elementos ser:
( )
( )
PB
,
f
, PB
,
f
, d kg N
d
78 0
0,78
07 0
78 0
123 0
d 1

+ =

(4.76)
( )
( )
PB
,
f
, PB
,
f
, d kg P
d
78 0
0,78
01 0
78 0
026 0
d 1

+ =

(4.77)
donde:
PB: Produccin de biomasa (ecuacin 4.51)
( )
( )
) - (1 0,78 ,
1
1

0 k

f
X d
d
=
+

=
f
d
: fraccin degradable de los SSVLM
|
|
.
|

\
|
=
o b
b,D
d
, X
X
f
En las ecuaciones 4.76 y 4.77 se ha considerado que 77 % de la fraccin
degradable es biolgicamente activa.
19
Notas bibliogrficas
1 PAVN, J. L., M. W. TENNEY AND W. F. ECHELBORGER: 'Bacterial exocellular
polymers and biological Iloculation, Jour. Water Poll. Control Fed., vol.
44, no. 3, 1972.
2 BISOGNI, J. J. AND W. LAWRENCE: 'Relationship between biological solids reten-
tion time and settling characteristics oI activated sludge, p. 1, Water Res.,
vol. 5, no. 9, 1971.
3 CHAO, A. C. AND T. M. KEINATH: 'InIluence oI process loading intensity on
sludge clariIications and thickening characteristics, Water Res., vol. 13,
no. 2, 1979.
4 LOVETT, D. A., B. V. KAVANAGH AND L. S. HERBERT: 'EIIect oI sludge age on
substrate composition on the settling and dewatering characteristics of
activated sludge, Water Res., vol. 17, no. 4, 1983.
162
Menndez Gutirrez, C. y J. Prez Olmo
5 CHUDOBA, J., V. OTTOVA AND V. MADERA: 'Control oI activated sludge Iila-
mentous bulking I. Effect of the hydraulic regimen of degree of mixing
in an aeration tank, Water Res., vol. 7, no. 6, 1973.
6 CHUDOBA, J., P. GRAU AND V. OTTOV: 'Control oI activated sludge Iilamen-
tous bulking II. Selection oI microorganisms by means oI a selector.
Water Res., vol. 7, no. 9, 1973.
7 CHIESA, S. C. AND S. R. LINNE: 'Discussion oI the eIIects oI sludge age and
selector configuration on the control of filamentous bulking in the acti-
vated sludge processs. Research J. Water Poll. Control Fed., vol.
62, no. 6, 1992.
8 KAPPELER, J. AND W. GUJER: 'VeriIication and application oI a mathematical
model Ior aerobic bulking, Water Res., vol. 28, no. 2, 1994.
9 GREENBERG, A. E., G. KLEIN AND W. KAUTMAN: 'EIIect oI phosphorous on
the activated sludge process, Sewage and Industrial Wastes, vol. 27,
no. 3, 1955.
10 CHUDOBA, J., C. MENNDEZ Y J. PREZ: Fundamentos Tericos de Algu-
nos Procesos para la Purificacin de Aguas Residuales, Ed. ISP-
JAE, Ciudad de La Habana, 1986.
11 TUCEK, F. AND J. CHUDOBA: 'Modeling oI wastewater treatment process,
Water Research, vol. 3, no. 2, 1969.
12 WEDDLE, C. L. AND D. JENKINS: 'The viability and activity oI activated
sludge, Water Res., vol. 5, no. 8, 1971.
13 WUHRMANN, K.: Biological Treatment of Sewage and Industrial Was-
tes, vol. I, Ed. Mac Cabe, 1956.
14 ECKENFELDER, W. AND D. O` CONNOR: Biological Waste Treatment, Per-
gamon Press, Oxford, 1961.
15 WUHRMANN, K.: 'High Rate Activated Sludge Treatment and Its Relation
to Stream Sanitation, Jour. Water Poll. Control Fed., vol. 26, no. 1,
1954.
16 MENNDEZ, C.: Influencia de la edad del lodo en la composicin de los
slidos biolgicos, indito. 1998.
17 HEUKELEKIAN, H., H. E. OXFORD AND R. MANGANELLI: 'Factors aIIecting
the quantity oI sludge production in the activated sludge process, Sewa-
ge Industrial Wastes, vol. 23, no. 8, 1951.
18 SIMONS, J. M. AND E. MC KINNEY: 'The biochemistry oI nitrogen in the
synthesis oI activated sludge, Sewage and Industrial Wastes, vol. 30,
no. 7, 1958.
19 WASHINGTON, D. R. AND L. J. HETLING: 'Volatile sludge accumulation in acti-
vated sludge plant, Jour. Water Poll. Control Fed., vol. 37, no. 4, 1965.
163
Lodo Activado
20 CHUDOBA, J. AND F. TUCEK: 'Production, degradation and composition oI
activated sludge in aeration systems without primary sedimentation,
Jour. Water Poll. Control Fed., vol. 57, no. 3, 1985.
21 VANDEVENNE, L. AND W. ECKENFELDER: 'A comparison oI models Ior com-
pletely mixed activated sludge treatment, design and operation, Water
Res., vol. 14, no. 7, 1980.
22 KOUNTZ, R.R. Y C. FORNEY: 'Metabolic energy balance in a total oxidation
activated sludge system, Sewage and Industrial Wastes, vol. 31,
no. 7, 1959.
165
Transferencia de Oxgeno
CAPTULO 5
TRANSFERENCIA DE OXGENO
5.1. INTRODUCCIN
La transferencia de oxgeno es un elemento clave en los procesos biol-
gicos aerobios.
Exceptuando casos particulares, la oxigenacin de un medio se realiza
mediante la aereacin aprovechando el hecho de que el aire contiene aproxi-
madamente 21 % en volumen de oxgeno.
La transferencia se puede realizar a travs de la introduccin de aire u
oxgeno en el agua o exponiendo el lquido a la atmsfera en forma de pe-
queas gotas.
Cualquiera que sea el procedimiento seleccionado para producir la
aereacin, existe una serie de elementos bsicos del proceso en s que son
comunes en todos ellos.
5.2. CONSIDERACIONES FUNDAMENTALES
DE LA TRANSFERENCIA DE OXGENO
En cualquier proceso de transferencia, la velocidad de esta se puede
expresar como el producto de una fuerza impulsora y un coeficiente de trans-
ferencia. En la aereacin, la fuerza impulsora es el gradiente de la concen-
tracin de oxgeno.
Lewis y Whitman,
1
desarrollaron el concepto de la doble pelcula que
considera pelculas estacionarias en las interfases del gas y del lquido
respectivamente, a travs de las cuales ocurre la transferencia de masa.
Figura 5.1.
De acuerdo con el concepto de la doble pelcula, la masa de oxgeno trans-
ferida a un medio lquido puede expresarse segn,
M = K
L
A (Cs - C) = K
g
A (P
g
- P) (5.1)
166
Menndez Gutirrez, C. y J. Prez Olmo
donde:
M: masa de oxgeno transferida en la unidad de tiempo (kg h
1
).
A: rea seccional a travs de la cual ocurre la transferencia.
K
L
, K
g
: coeficientes de transferencia en la pelcula de lquido y gas respec-
tivamente (L m
2
h
1
).
Cs: concentracin de saturacin de oxgeno en la fase lquida.
C: concentracin de oxgeno en el lquido.
P
g
, P: presin parcial de oxgeno en el aire.
Fig. 5.1. Doble pelcula segn Lewis y Whitman.
Para gases poco solubles, como el oxgeno, la pelcula controladora de la
transferencia es la de lquido. Para gases relativamente solubles, controla la
pelcula de gas.
Para el primero de los casos, al aumentar la turbulencia disminuye el
espesor de la pelcula de lquido y aumenta el coeficiente de transferencia
del gas en el lquido, K
L
.
Hay otros modelos tericos ms recientes que el de Lewis y Whitman,
tales como el de la velocidad fraccional de renovacin de superficie
(Danckwerts).
2
Sin embargo el ms aceptado sigue siendo el de Lewis y
Whitman.
Para aquellos procesos controlados por la pelcula de lquido, la ecuacin
5.1 puede transformarse:
( )
C Cs a k
dt
dC
M
V
L
= =
1
(5.2)
donde:
V
A
K a k
L L
=
, coeficiente global de transferencia de oxgeno, h
1
.
167
Transferencia de Oxgeno
Cuando los trminos vinculados a la transferencia de oxgeno se relacionan
tal como aparece en la ecuacin 5.2, vienen referidos, por problemas de como-
didad, para agua de acueducto (agua limpia), 20
0
C y una presin de 101,3 kPa.
5.2.1. Concentracin de oxgeno en condiciones de saturacin
La concentracin de saturacin de oxgeno en el agua limpia depende de la
temperatura, concentracin de slidos totales disueltos y de la presin.
En la tabla 5.1 se reporta la solubilidad del oxgeno en agua a la presin
normal y diferentes concentraciones de cloruro.
Para presiones de trabajo diferentes a la normal, y considerando la profun-
didad del tanque de aereacin, la concentracin de saturacin se calcula:
o
h
P
P , Pa
Cs Cs,m
5 0 +
=
(5.3)
Cs: concentracin de saturacin a la temperatura de trabajo y presin nor-
mal mgL
1
.
Cs,m: concentracin de saturacin promedio a la temperatura de trabajo
y a una profundidad que es igual a la mitad de la altura del tanque de
aeracin mgL
1
.
P
0
: presin normal, 101,3 kPa.
Pa: presin atmosfrica en las condiciones de trabajo kPa.
P
h
: presin hidrosttica a la altura de la descarga del aire kPa.
Tabla 5.1. Solubilidad de oxgeno en agua a 101,3 kPa
Concentracin de cloruro en agua mgL
-1
Temperatura
C
0 5 10 15 20
16 10,0 9,5 9,0 8,5 8,0
17 9,7 9,3 8,8 8,3 7,8
18 9,5 9,1 8,6 8,2 7,7
19 9,4 8,9 8,5 8,0 7,6
20 9,2 8,7 8,3 7,9 7,4
21 9,0 8,6 8,1 7,7 7,3
22 8,8 8,4 8,0 7,6 7,1
23 8,7 8,3 7,9 7,4 7,0
24 8,5 8,1 7,7 7,3 6,9
25 8,4 8,0 7,6 7,2 6,7
26 8,2 7,8 7,4 7,0 6,6
27 8,1 7,7 7,3 6,9 6,5
28 7,9 7,5 7,1 6,8 6,4
29 7,8 7,4 7,0 6,6 6,3
30 7,6 7,3 6,9 6,5 6,1
168
Menndez Gutirrez, C. y J. Prez Olmo
Para tener en consideracin adems los constituyentes del agua residual se
define el coeficiente |.
Cs
Cs,w
= =
limpia agua el en d solubilida
residual agua el en d solubilida
Los valores de | suelen variar entre 0,7 y 0,8 como consecuencia de las
variaciones de la solubilidad del oxgeno debido a la presencia de sales, slidos
suspendidos y sustancias tensoactivas.
3
5.2.2. Coeficiente global de transferencia de oxgeno
El coeficiente de transferencia de oxgeno depende del implemento de
aereacin y de las caractersticas fsicas y qumicas del sistema que se desea
oxigenar:
4
1. Temperatura. Al aumentar la temperatura aumenta el valor del coeficiente
de transferencia,
( )
( )
20
20

=
t
C
L t L
0 a k a k
o (5.4)
El valor de u es funcin de las condiciones de trabajo. Se suele situar en el
intervalo de 1,015 y 1,040. El valor tpico para todo tipo de aereador es 1,024.
2. Turbulencia. Un incremento de la turbulencia da como resultado un aumento
en el coeficiente de transferencia.
3. Altura del lquido. El efecto que tiene la altura de la columna del lquido bajo
aereacin sobre el coeficiente global de transferencia de oxgeno depende
en cierta medida del mtodo empleado en la aereacin. Para muchos siste-
mas de aereacin por difusin k
L
a vara con la altura de acuerdo con la
relacin:
n
L
L
H
H
a k
a k
|
|
.
|

\
|
=
2
1
2
1
(5.5)
donde el exponente n tiene un valor cercano a 0,7 para la mayora de los
sistemas.
4. Caractersticas del medio. El valor del coeficiente de transferencia de oxge-
no del agua pura puede diferir considerablemente del de un medio que con-
tenga compuestos orgnicos u otras sustancias en disolucin.
Con el objetivo de comparar la velocidad de transferencia de oxgeno en el
agua con la de otro medio cualquiera para un implemento de aereacin par-
ticular se define el coeficiente o.
169
Transferencia de Oxgeno
limpia agua del
medio del
u
a k
a k
L
L
=
El valor de o puede no ser constante y variar durante el proceso que tiene
lugar en el bio-reactor. En la tabla 5.2 se reportan valores tpicos de o para
diferentes implementos de aereacin.
La ecuacin 5.2 queda entonces:
( )
20
02 1

=
t
L
, C Cs a k o
dt
dC
(5.6)
y si se toma en consideracin la profundidad del tanque:
( )
20 t
L
1,02 k u
dt
dC

= C Cs,m a
(5.7)
Tabla 5.2. Valores tpicos de o
5.2.3. Clculo del coeficiente global de transferencia
de oxgeno
Para poder aplicar la ecuacin (5.7) en cualquier sistema aerobio, se requie-
re conocer o determinar previamente los coeficientes k
L
a, o, y |.
En agua limpia la velocidad de transferencia de oxgeno puede plantearse
segn la ecuacin:
( )
C Cs a k
dt
dC
L
=
mgL
1
h
1
(5.8)
Como se aprecia de la ecuacin anterior, la velocidad de la transferencia es
proporcional al dficit de oxgeno y al coeficiente global de transferencia de
oxgeno, k
L
a, tal como ya fue expresado.
Implemento de aereacin Valor tipo
Difusor de burbuja gruesa 0,85
Difusor de burbuja fina 0,50
Eyector de aereacin 0,75
Aereador superficial 0,90
Turbinas 0,85
170
Menndez Gutirrez, C. y J. Prez Olmo
El valor del coeficiente k
L
a depende, a su vez, de la turbulencia en la interfase
aire - lquido. Por tanto, es funcin del sistema de aereacin, la geometra del
tanque de aereacin, las caractersticas del lquido y de la temperatura.
El mtodo que se describe a continuacin para determinar el coeficiente de
transferencia de oxgeno se basa en estimar el tiempo necesario para que la
diferencia entre el oxgeno de saturacin, Cs y el oxgeno disuelto en un instante
dado, C, disminuya 90 %.
Integrando la ecuacin 5.8 entre t
1
y t
2
se obtiene:
( )
1 2
2
1
log 3 2 t t a k
C Cs
C Cs
,
L
=

(5.9)
Suponiendo que (Cs - C
2
) = 0,1 (Cs - C
1
), la ecuacin 5.9 queda:
90
3 2
t
,
a k
L
=
(5.10)
t
90
: tiempo durante el cual la diferencia entre la concentracin de oxgeno
en condiciones de saturacin y la del oxgeno disuelto disminuye 90 %.
5
Figura 5.2.
5.2.4. Procedimiento
Clculo de k
L
a
El procedimiento general comprende la eliminacin del oxgeno disuelto
en un volumen conocido de agua, mediante la adicin de sulfito de sodio, y la
posterior reoxigenacin hasta valores cercanos a la concentracin de satu-
racin.
5
1. El tanque de aereacin se llena con agua limpia del acueducto.
2. Debe ser reducida la concentracin de oxgeno disuelto del tanque hasta un
valor cercano a cero. Para esto puede utilizarse una solucin de sulfito de
sodio a razn de 7,88 mg por cada mg de oxgeno disuelto del agua. Cuando
se desea acelerar la etapa de de-oxigenacin puede utilizarse una solucin
de Co
2+
como catalizador a razn de 5 mgL
1
.
3. Una vez que la concentracin de oxgeno disuelto alcanza un valor cercano a
0,5 mgL
1
se pone en funcionamiento el implemento de aereacin.
4. A partir del momento en que la concentracin de oxgeno sea igual a
1 mgL
1
se hacen mediciones de la concentracin a intervalos regulares
de tiempo.
5. Se tabulan los resultados y se procede segn se ilustra en el ejemplo 5.1.
171
Transferencia de Oxgeno
Fig. 5.2. Clculo del coeficiente de transferencia de oxgeno.
5.2.5. Capacidad de oxigenacin
Con el objetivo de poder comparar las posibilidades de los distintos imple-
mentos para suministrar oxgeno, es necesario introducir el trmino capacidad
de oxigenacin (CO).
La capacidad de oxigenacin de un implemento de aereacin se define como
la cantidad de oxgeno que puede ser suministrado por ese implemento, en la
unidad de volumen y de tiempo, cuando la concentracin de oxgeno disuelto es
cero. De acuerdo con la ecuacin 5.8, cuando C = 0:
CO = 0,024 k
L
a Cs kg m
3
d
1
(5.11)
donde:
k
L
a: coeficiente de transferencia de oxgeno, h
1
.
Cs: concentracin de saturacin de oxgeno disuelto a la temperatura de
trabajo, mgL
1
.
Por tanto, para determinar la capacidad de oxigenacin solo se requiere
conocer el coeficiente de transferencia de oxgeno, y la concentracin de satu-
racin a la temperatura y presin a la que se est trabajando.
Ejemplo 5.1
Se realizan pruebas a escala de laboratorio con un aereador superficial de
paleta a tres velocidades diferentes. A partir de los datos obtenidos, calcule el
coeficiente de transferencia de oxgeno a cada velocidad (tabla 5.3).
172
Menndez Gutirrez, C. y J. Prez Olmo
t = 25
o
C Cs = 8 mgL
1
Solucin:
Se grafica en papel semilogartmico (Cs - C) en funcin del tiempo t, segn
la figura 5.3.
Tabla 5.3. Concentracin de oxgeno en funcin del tiempo de aereacin para
3 velocidades diferentes de agitacin
El tiempo t
90
se corresponde con el tiempo necesario para que la diferencia
(Cs - C) disminuya un ciclo logartmico, o sea:
Velocidad I t
90
: 6,8 min
Velocidad II t
90
: 13,0 min
Velocidad III t
90
: 21,2 min
Diferentes velocidades de rotacin del aereador Tiempo
(min)
I
O
2
mgL
-1
I
Cs-C
II
O
2
mgL
-1
II
Cs-C
III
O
2
mgL
-1
III
Cs-C
0 1.00 7.00 1,00 7,00 1,00 7,00
0,5 2,00 6,00 1,55 6,45 1,40 6,60
1,0 2,90 5,10 2,02 5,98 1,75 6,25
1,5 3,70 4,30 2,55 5,45 2,05 5,95
2,0 4,35 3,65 3,00 5,00 2,40 5,60
2,5 4,85 3,15 3,40 4,60 2,70 5,30
3,0 5,35 2,65 3,80 4,20 3,00 5,00
3,5 5,80 2,20 4,20 3,80 3,25 4,75
4,0 6,15 1,85 4,50 3,50 3,50 4,50
4,5 6,40 1,60 4,80 3,20 3,75 4,25
5,0 6,70 1,30 5,05 2,95 3,95 4,05
6,0 7,15 0,85 5,55 2,45 4,40 3,60
7,0 7,40 0,60 5,95 2,05 4,75 3,25
8,0 7,60 0,40 6,25 1,75 5,10 2,90
9,0 7,70 0,30 6,55 1,45 5,40 2,60
10,0 7,80 0,20 6,80 1,20 5,65 2,35
11,0 7,90 0,10 7,00 1,00 5,90 2,10
13,0 7,95 0,05 7,30 0,70 - -
15,0 - - 7,50 0,50 6,60 1,40
20,0 - - 7,80 0,20 7,20 0,80
25,0 - - - - 7,40 0,60
173
Transferencia de Oxgeno
Por tanto, considerando que
90
3 2
t
,
a k
L
=
, de la figura 5.3 se tiene:
Velocidad I k
L
a = 20,35 h
1
Velocidad II k
L
a = 10,60 h
1
Velocidad III k
L
a = 6,52 h
1
Fig. 5.3. Ejemplo 5.1 Clculo de k
L
a.
Ejemplo 5.2
Determine la capacidad de oxigenacin del implemento utilizado para el ejem-
plo 5.1 cuando se trabaja a la velocidad II.
Solucin:
CO = 0,024 k
L
a Cs = 0,024 x 10,60 x 8 = 2,03 kg m
3
d
1
5.3. REQUERIMIENTO DE OXGENO
La determinacin de la necesidad de oxgeno en los procesos aerobios de
biomasa en suspensin constituye una etapa importante en las actividades de
diseo y evaluacin de una instalacin de tratamiento. En la medida que se
conozca la cantidad de oxgeno que es necesario para un proceso dado, se esta-
r en mejores condiciones de seleccionar el equipamiento adecuado para sumi-
nistrarlo.
174
Menndez Gutirrez, C. y J. Prez Olmo
La velocidad de consumo de oxgeno en los sistemas de biomasa en suspen-
sin puede representarse por la siguiente ecuacin emprica:
b AB Y r
X X
+ ' = (5.12)
donde:
X
r
r
X
=
(5.13)
y
X 0
AS
AB
X
=
(5.14)
r: velocidad de consumo de oxgeno mgL
1
d
1
.
X: concentracin de slidos suspendidos voltiles (SSV) en el tanque de
aereacin mgL
1
.
Y ': kg O
2
consumido (kg DBO removido)
1
.
b: kg O
2
consumido (kg SSV autooxidado.d)
1
.
ABx: remocin especfica de DBO kg kg
1
d
1
.
AS = So - S mg L
1
.
u = tiempo de retencin hidrulico d.
La ecuacin 5.12 puede expresarse tambin tomando como base la DQO.
Valores tpicos de Y ' y b se reportan en la tabla 5.4.
Tabla 5.4. Valores tpicos de Y y b
Base Y'
(kg kg
-1
)
b
(d
-1
)
Tipo de
residual
Fuente
DBO
5
0,50 0,10 Municipal 1
DQO 0,40 0,13 Municipal 1
DQO 0,27 0,11 Pulpa y papel 4
DBO
5
0,55 - 0,80 0,03 - 0,17 Municipal 7
DQO 0,33 - 0,54 0,3 - 0,17 Municipal 7
DQO 0,35 0,08 Tenera 3
Los coeficientes Y ' y b pueden ser calculados a escala de laboratorio. (Ep-
grafe 5.3.1).
El valor de r
x
depende en gran medida de la actividad del sistema. Por
ejemplo, para un proceso convencional de lodo activado, Eckenfelder
6
ha
reportado valores de 0,84 kg kg
1
d
1
a la entrada y de 0,2 kg kg
1
d
1
a la
salida del proceso. Para residuales teneros en Cuba tratados mediante lagunas
aereadas,
7
se han obtenido valores de 1,43 kg kg
1
d
1
y 0,79 kg kg
1
d
1
para uno
175
Transferencia de Oxgeno
y tres das de tiempo de retencin, respectivamente. Desde Hoover y Porges
8
se acepta como intervalo para el valor de la velocidad especfica de consu-
mo de oxgeno para la fase de respiracin endgena el comprendido entre
0,15 y 0,50 kg kg
1
d
1
.
En estado estacionario las condiciones de trabajo son tales que el suministro
de oxgeno debe ser igual a su requerimiento:
r = o k
L
a (| Cs - C) (5.15)
El metabolismo aerobio es independiente de la concentracin de oxgeno
disuelto cuando esta es superior a un valor considerado como crtico, aproxi-
madamente entre 0,2 y 2,0 mgL
1
. Por debajo de este valor crtico la veloci-
dad de las reacciones metablicas aerobias se ven limitadas.
9,10,11
El requerimiento mnimo de aire aplicado por kilogramo de DBO alimen-
tado al tanque de aereacin es funcin de las caractersticas del sistema en
particular. Para sistemas de carga media debe ser aproximadamente de
90 kg de aire por kilogramo de DBO mientras que para aereacin extendida
puede llegar a alcanzar 120 kg por kilogramo de DBO. Estas demandas
estn calculadas sobre la base de suponer que el implemento de aereacin
es capaz de transferir al menos 1 kg de oxgeno por cada kilogramo de DBO
aplicado. En todos los casos es recomendable garantizar una concentracin
mnima de oxgeno disuelto de 2 mgL
1
.
5.3.1. Clculo de los coeficientes Y' y b
Como ya se afirm anteriormente, los coeficientes Y ' y b pueden ser calcu-
lados a partir de pruebas realizadas a escala de laboratorio. De la ecuacin 5.12,
cuando se grafica la velocidad especfica de respiracin, r
x
, en funcin de la
remocin especfica, ABx, se obtiene una recta cuya pendiente es Y ', y el inter-
cepto con el eje de ordenadas, b. Figura 5.5.
Procedimiento
Se realizan corridas experimentales a diferentes edades de lodo, 3 como
mnimo. Para cada corrida se registran los valores promedios de los siguientes
parmetros: So, S, X, u.
Para cada corrida:
1. Se toma una muestra representativa del licor mezclado del reactor y se intro-
duce en un recipiente en el que se pueda medir la disminucin en el tiempo de
la concentracin de oxgeno disuelto.
176
Menndez Gutirrez, C. y J. Prez Olmo
Fig. 5.4. Clculo de la velocidad de respiracin.
2. La muestra se aerea hasta saturacin en oxgeno.
3. Se detiene la aereacin y se mide la variacin de la concentracin de oxgeno
en el tiempo.
4. Se grafica la concentracin de oxgeno disuelto vs tiempo. Figura 5.4. La
pendiente obtenida en la seccin inicial de la curva es la velocidad de respi-
racin r.
5. Para cada corrida se divide el valor de la velocidad de consumo de oxgeno
determinado (r) por la concentracin de SSV.
6. Se prepara una tabla de r
x
vs ABx con tantos puntos experimentales como
edades de lodo se hayan prefijado.
7. Se grafica r
x
vs ABx. Figura 5.5.
Fig. 5.5. Clculo de Y y b.
177
Transferencia de Oxgeno
Ejemplo 5.3
Los datos que se dan a continuacin fueron obtenidos a escala de laborato-
rio trabajando a cuatro edades de lodo diferentes y midiendo en cada caso la
velocidad de respiracin r de la biomasa obtenida en el licor mezclado. Determi-
ne los coeficientes Y ' y b. (Tabla 5.5)
Tabla 5.5. Datos experimentales
Solucin:
A partir de los datos de velocidad de respiracin r se calcula la velocidad de
respiracin especfica r
x
.
Tabla 5.5.a. Datos de velocidad de respiracin especfica
u
x
(d)
X
(mg L
-1
)
r
(mg L
-1
h
-1
)
AB
x
(kg kg
-1
d
-1
)
3 800 26,6 1,25
5 1 200 35,0 1,00
7 1 800 30,0 0,50
10 2 500 20,8 0,25
Se grafica r
x
contra AB
x
segn la figura 5.5. De la pendiente se obtiene el
valor de Y' y del intercepto en el eje de ordenadas el valor de b.
Y' = 0,6
b = 0,075
5.3.2. Capacidad de oxigenacin necesaria
Una vez que se conoce la capacidad de oxigenacin del implemento de
aereacin, debe compararse con la que necesita el sistema, o sea, la capacidad
de oxigenacin necesaria (CO)
n
.
Cuando se disea un tanque de aereacin debe calcularse la (CO)
n
de for-
ma de conocer el oxgeno que se necesita para garantizar el suministro que
requieren las reacciones bioqumicas que tienen lugar y para el mantenimiento
de las condiciones aerobias en el medio.
r
x
(kg kg
-1
d
-1
) 0,8 0,7 0,4 0,2
AB
x
(kg kg
-1
d
-1
) 1,25 1,0 0,5 0,25
178
Menndez Gutirrez, C. y J. Prez Olmo
( ) ( )
C Cs,m
Cs,m
X b AB Y CO
V n

+ ' =
(5.16)
donde:
ABv: Razn de carga volumtrica, kg m
3
d
1
.
X: Concentracin de SSV en el tanque de aereacin, kg m
3
.
C: Concentracin lmite permisible de oxgeno disuelto, mgL
1
.
La masa de oxgeno que se disuelve en el agua en la unidad de tiempo en
condiciones normales (20
o
C y 101,3 kPa y cero oxgeno disuelto) es:
V m Cs a k G CO
C C L n 20 20
3
, 10 ) (

= = (5.17)
donde:
G: velocidad msica de disolucin (absorcin) de oxgeno, kg h
1
.
k
L
a: h
1
.
Cs,m: mgL
1
.
V: volumen del agua que se est aereando, m
3
.
Ejemplo 5.4
Determine la capacidad de oxigenacin necesaria para satisfacer las exigen-
cias de un proceso de lodo activado del que se dispone de la siguiente informacin:
t: 28
0
C
Y': 0,40
b: 0,10 d
1
ABv: 0,5 kg m
3
d
1
X = 1,5 kg m
3
OD mnimo permisible = 2 mgL
1
Altura del tanque de aereacin = 3 m
Presin atmosfrica = 101,8 kPa
Solucin:
La concentracin de saturacin de oxgeno a 28
0
C es (Tabla 5.1):
Cs = 7,9 mgL
1
De acuerdo con la ecuacin 5.3:
o
h
P
P , Pa
Cs Cs,m
5 0 +
=
Pa = 101,8 kPa
Po = 101,3 kPa
P
h
= 0,0098 x h
= 990 kg m
3
h = 3 m
P
h
= 29,1 kPa
179
Transferencia de Oxgeno
( )
1
L mg 07 9
3 101
1 29 5 0 8 101
9 7

=
+
= ,
,
, , ,
, Cs,m
(CO)
n
= 0,45 kg m
3
d
1
5.3.3. Otro procedimiento para determinar la necesidad
de oxgeno
En ausencia de clculos ms sofisticados, puede asumirse que para la remo-
cin de la DBO carbnica es necesario utilizar 1,1 kg O
2
por cada kg de DBO
5
aplicado, para el caso de procesos convencionales. Para la aereacin extendida
el requerimiento de oxgeno puede tomarse aproximadamente como 1,8 kg
de O
2
por cada kg de DBO.
Por otro lado, haciendo consideraciones tericas, la necesidad de oxgeno
puede ser determinada a partir de la DBO del agua residual y de la cantidad de
biomasa que diariamente se extrae del proceso.
Por tanto, la demanda terica de oxgeno para la remocin de las sustancias
carbonosas contenidas en el agua residual puede calcularse mediante la relacin:
kg O
2
d
1
= (kgd
1
DBO
u
total utilizado) - 1,42 (kgd
1
biomasa extrada)
En la expresin anterior ha sido considerado que 1,42 es el equivalente
de oxgeno de la biomasa, asumiendo que la frmula emprica de esta es
C
5
H
7
N O
2
.
De acuerdo con esto,
( )
P A ,
f
g / kg S S Q
d O g k 42 1
10
3
0
1
2

donde:
AP: es la biomasa producida (kgd
1
).
f: es la relacin entre DBO
5
y DBO
u
.
En el desarrollo del epgrafe 5.3, solo se consider el requerimiento de ox-
geno para satisfacer las necesidades de consumo del agua residual y de la
biomasa. Sin embargo, el suministro debe ser adecuado para:
satisfacer la DBO del agua residual,
satisfacer la respiracin endgena de la biomasa,
proporcionar un adecuado rgimen de mezcla y
mantener una concentracin de oxgeno disuelto en el reactor, que no
inhiba el proceso aerobio (generalmente superior a 0,2 mgL
1
).
180
Menndez Gutirrez, C. y J. Prez Olmo
5.4. IMPLEMENTOS DE AEREACIN
El equipamiento para producir aereacin que es ms comnmente utilizado
puede clasificarse en:
a) sistemas de aire difundido,
b) sistemas de aereacin mecnica.
5.4.1. Aire difundido
Los sistemas de aire difundido estn constituidos por difusores sumergi-
dos en el agua, las lneas o tuberas conductoras de aire, los sopladores o
compresores, y su equipamiento auxiliar.
Bsicamente se conocen dos tipos de difusores: los que producen burbu-
jas pequeas a partir de un medio poroso y aquellos que utilizan tuberas
horadadas o algn otro dispositivo para producir burbujas grandes o media-
nas. Todos estos implementos son muy diversos en dependencia de los fa-
bricantes. Figura 5.6.
Los difusores se distribuyen a ambos lados y en toda la longitud del
tanque de aereacin. Figura 5.7. Como elemento prctico puede sealarse
que para mantener una mezcla adecuada, el ancho del tanque de aereacin
debe ser aproximadamente el doble de su profundidad. Este ancho puede
duplicarse si adems se concibe una lnea central de unidades de difusin.
Tambin puede apuntarse que la ubicacin de los difusores debe ser tal que
los puntos de salida del aire deben estar unos de otros a una distancia com-
prendida entre 15 y 75 cm.
Los difusores de burbuja fina se construyen de granos de slice u xido
de aluminio. Otras unidades son tubos recubiertos de diversos compuestos
como nylon, dracn y saran. El dimetro de la burbuja suele estar compren-
dido entre 2 y 2,5 mm. Estas unidades se disean para lograr un flujo de aire
por unidad de 3 a 28 m
3
h
1
en condiciones estndar (P = 101,3 kPa, 20
0
C).
Los difusores no porosos originan burbujas de hasta 25 mm de dime-
tro. Estas burbujas grandes son la causa de que estos difusores presenten
menor rendimiento que los porosos, pero por otro lado presentan las venta-
jas de tener menor costo, y menos necesidades de mantenimiento y de
limpieza de aire.
La aereacin mediante difusores es recomendada fundamentalmente para
profundidades de lquido en el reactor entre 2,5 y 5,0 m y valores de veloci-
dades de consumo de oxgeno inferiores a 1 mgL
1
min
1
.
181
Transferencia de Oxgeno
Fig. 5.6. Implementos para la aereacin mediante aire difundido.
a) difusor poroso para burbujas finas. b) Tubos para burbujas gruesas.
Fig. 5.7. Distribucin de los difusores. Vista de planta.
5.4.2. Aereacin mecnica
Los aereadores mecnicos pueden ser superficiales o sumergidos, y ambos
a su vez, de eje vertical o de eje horizontal. En los aereadores mecnicos el
oxgeno se toma de la atmsfera. Hay modelos de aereadores mecnicos
sumergidos, las turbinas, en los que adems se introduce aire por la parte
inferior del tanque de aireacin. Figura 5.8.
182
Menndez Gutirrez, C. y J. Prez Olmo
Entre las funciones de un aereador mecnico se pueden citar:
9,12
a) Disolver oxgeno en el tanque de aereacin.
b) Mantener los SSV en suspensin.
c) Distribuir el afluente por todo el volumen del tanque de aereacin.
d) Remover parte del CO
2
que se genera.
Fig. 5.8 Aereadores mecnicos. a) Aereador superficial. b) Aereador
de turbina.
(b)
183
Transferencia de Oxgeno
5.4.2.1. Aereadores superficiales
Los aereadores de superficie, equipos para la introduccin de grandes can-
tidades de aire, y por tanto de oxgeno, consisten en turbinas de alta o baja
velocidad o en unidades flotantes de alta velocidad que giran en la superficie del
lquido, parcialmente sumergidos.
Cualquier superficie de agua tiende a absorber oxgeno del aire. Este proce-
so se acelera cuando hay turbulencia en la superficie.
Los aereadores superficiales llevan este proceso a una etapa superior al
romper mecnicamente la superficie del lquido y crear una interfase gas-lquido
en forma atomizada de pequeas gotas de agua y burbujas de aire, arrastradas
dentro del lquido. Adems el volumen total de lquido circula y se mezcla de
manera tal que el agua oxigenada se reemplaza continuamente por lquido de
otras zonas.
Los aereadores superficiales de eje vertical pueden montarse sobre una
estructura rgida o sobre flotadores. Los de eje horizontal tienen su origen
en los aereadores Kessener de cepillo y hoy se fabrican de diferentes tipos.
Figura 5.9.
Fig. 5.9. Aereadores superficial de cepillo.
La transferencia de oxgeno ocurre a travs de la accin de vrtice y por la
exposicin a la superficie de grandes volmenes de agua atomizada.
Para los implementos de eje vertical, la cantidad de oxgeno transferido al
lquido es funcin de la potencia.
184
Menndez Gutirrez, C. y J. Prez Olmo
La velocidad de transferencia depende de:
dimetro del implemento,
velocidad de rotacin y
profundidad del elemento rotatorio.
En condiciones ptimas de inmersin, la velocidad de transferencia por po-
tencia consumida es relativamente constante dentro de un amplio intervalo de
valores de dimetro del impelente.
Kormanik
13
obtuvo una correlacin entre la velocidad de transferencia de
oxgeno y la potencia por unidad de rea. As, para aereadores superficiales de
alta velocidad, puede considerarse la expresin:
N
p
= 1,973 P + 1,0045
N
p
: kg O
2
(kW-h)
1
P: kW m
2
Para mantener una concentracin uniforme de oxgeno disuelto se nece-
sita disponer de niveles de potencia de 1,2 a 2 Wm
3
. Para mantener slidos
biolgicos en suspensin es recomendable una velocidad mnima en el fondo
de 12 cm s
1
cuando la concentracin es de 5gL
1
de SSLM.
Cuando se emplea este tipo de aereador, la altura mnima de tanque reco-
mendada es de 1,8 a 2,4 m y mxima entre 3,7 a 4,9 m.
Los aereadores superficiales se utilizan cuando la velocidad de consumo de
oxgeno del sistema est en el intervalo de 1,0 a 1,4 mgL
1
min
1
. Para mayores
velocidades de consumo se recomienda el empleo de aereadores de turbina.
5.4.2.2. Aereadores sumergidos
Las unidades de aereacin a travs del empleo de turbinas dispersan el aire
comprimido mediante la accin cortante de un impelente rotatorio y promovien-
do adems la mezcla del contenido del tanque de aereacin.
Los aereadores sumergidos de eje horizontal funcionan por el mismo princi-
pio que los superficiales excepto que la agitacin del agua se lleva a efecto con
discos o paletas acoplados al eje.
Las turbinas se distribuyen en el tanque de aereacin de manera tal que
exista una cada 85 a 220 m
2
.
Las potencias de los aereadores mecnicos disponibles varan de 0,75
a 100 kW.
185
Transferencia de Oxgeno
5.4.3. Capacidad de los aereadores
Los fabricantes de equipos de aereacin generalmente ofrecen la capaci-
dad de transferencia de oxgeno de su equipamiento sobre la base de la masa de
oxgeno transferido por unidad de energa (kg de oxgeno transferido por kW h)
en condiciones normales (C.N.). Las condiciones normales o estndar estn
definidas para agua de acueducto, a 20
0
C, cero concentracin de oxgeno di-
suelto y la presin correspondiente al nivel del mar (P = 101,3 kPa). Esto no es
ms que la capacidad de oxigenacin del implemento.
Para las condiciones de campo o reales (C.R.) la capacidad de transferen-
cia o rendimiento, No, reportada en (C.N.) por el fabricante, deben ser rectifi-
cadas,
( )
20
20
0
02 1

=
t
C
, o
Cs
C Cs,m
N N
o
(5.18)
N: capacidad de transferencia de oxgeno en condiciones de trabajo o reales
kg O
2
(kW h)
1
.
N
o
: capacidad de transferencia de oxgeno en condiciones estndar.
kg O
2
(kW h)
1
.
Cs,
20

C
: concentracin de saturacin de oxgeno disuelto a la temperatura de
20
0
C mgL
1
.
Cs,m: concentracin de saturacin de oxgeno disuelto en agua limpia a la
temperatura y presin de trabajo mgL
1
.
C: Concentracin de oxgeno disuelto deseada mgL
1
.
t: temperatura de trabajo
o
C.
Cuando se evala la capacidad de transferencia de oxgeno en sistemas de
aire difundido, es necesario valorar no solo la eficiencia de los difusores, sino
tambin la capacidad de los sopladores o compresores. Esta informacin est
disponible generalmente en los manuales de operacin y mantenimiento que
ofrecen los proveedores de los equipos.
Por tanto, para determinar la transferencia de oxgeno del sistema de aereacin
por difusin en kgd
1
, adems de necesitar la capacidad de transferencia de los
difusores en condiciones reales, debe conocerse la capacidad de los sopladores
en condiciones estndar (m
3
min
1
a 20 C y la presin de 101,3 kPa). Con estos
dos elementos puede calcularse la masa de aire que el sistema es capaz de
entregar. Ver el ejemplo 5.8.
Cuando la capacidad de transferencia que se necesita calcular sea la de
un aereador superficial, debe determinarse la potencia de trabajo del motor, m,
y la capacidad o rendimiento, kg O
2
(kw-h)
1
, del aereador en particular. En
el caso de no existir otra informacin, puede asumirse que la eficiencia del
186
Menndez Gutirrez, C. y J. Prez Olmo
motor es 90 % y la de los elementos mecnicos del aereador 85 %. De
acuerdo con esto ha de considerarse que 75 % de la potencia nominal del
motor del aereador superficial es convertido en energa para la transferen-
cia de oxgeno. Ver ejemplo 5.9.
Tabla 5.6. Intervalos tpicos de valores de capacidad de transferencia de
oxgeno para algunos de los tipos de aereadores
5.4.4. Eficiencia de la transferencia de oxgeno
La eficiencia de absorcin o de la transferencia de oxgeno viene definida
para difusores y las turbinas como los kg h
1
de oxgeno transferido por kg h
1
de
oxgeno suministrado.
Los fabricantes definen la Eficiencia Estndar de Transferencia de Ox-
geno (Eo) (Standard Oxygen Transfer Eficiency, SOTE) de sus implemen-
tos de aeracin para agua de acueducto, a 20
0
C, cero oxgeno disuelto y a
nivel del mar.
Qs ,
Cs a k
E
C C
L
o o
56 16
20 20
=
(5.19)
Eo: Eficiencia estndar de transferencia %.
k
L
a: Coeficiente de transferencia de oxgeno h
1
.
Qs: Flujo de aire m
3
min
1
, a 101,3 kPa, 20
0
C.
V: Volumen del tanque de aereacin.
Los valores de Eo deben ser rectificados para las condiciones normales
de trabajo, (eficiencia real o actual de transferencia de oxgeno, E), (AOTE),
( )
20
20
0
02 1

=
t
C
, o
Cs
C Cs,m
E E
o
(5.20)
Tipo de aereador No kg O
2
(kWh)
1
Superficial (baja velocidad) 1,21 3,04
Superficial (alta velocidad) 1,21 2,19
Turbina 1,21 2,25
Rotor de cepillo y placas 0,91 2,19
Difusores de burbuja fina 1,80 3,20
Difusores de burbuja gruesa 0,90 1,50
187
Transferencia de Oxgeno
La eficiencia de absorcin tambin puede ser calculada directamente,
( )
V , o
Qs ,
C Cs,m a k
E
t C
L
o
20 20
02 1
56 16

=
(5.21)
En la tabla 5.7 se brinda informacin tpica de la eficiencia de transferencia
de oxgeno de varios tipos de difusores.
Tabla 5.7. Valores tpicos de eficiencia estndar de transferencia de diferen-
tes difusores
14
5.4.5. Correlaciones para la transferencia de oxgeno
Eckenfelder
15
ha desarrollado una correlacin general para la transferencia
de oxgeno a partir de burbujas que ascienden a travs de una columna de lqui-
do en reposo:
|
|
.
|

\
|
=
n
c d
C H
D
d K
B B
L
s L
3
1
(5.22)
H: profundidad del lquido.
d
B:
dimetro de la burbuja.
u
B:
velocidad de la burbuja.
: densidad del lquido.
: viscosidad del lquido.
C: constante.
Si se considera que para burbujas de aire en el tanque de aereacin se
cumple que:
V c d
H Qs
V
A
B B
6
=
(5.23)
Tipo de difusor Profundidad
(m)
E
0
(%)
Domos de cermica 4,2 29,0
Discos de cermica 3,7 26,0
Placas de cermica 4,5 30,0
Tubo plstico rgido poroso 4,0 27,0
Tubo plstico rgido poroso 4,5 30,0
Difusor no poroso de eje
longitudinal
4,5 12,0
188
Menndez Gutirrez, C. y J. Prez Olmo
la ecuacin 5.20 puede ser expresada en trminos de k
L
a.
Qs: flujo volumtrico de aire.
A: rea a travs de la cual ocurre la difusin.
V: volumen de lquido.
Para el intervalo de flujo de aire que normalmente se utiliza en la prctica se
cumple que,
d
B
~ Qs
n
(5.24)
Las ecuaciones 5.20, 5.21 y 5.22 pueden combinarse para obtener una ex-
presin general para la transferencia de oxgeno por difusin
( )
V
Qs H C
a k
n /
L

'
=
1 3 2
(5.25)
La ecuacin 5.25 puede ser transformada en trminos de masa de oxgeno
transferida por unidad de difusin
( )
( ) C Cs Qs H C G
n /
' =
1 3 2
(5.26)
G = masa de oxgeno transferida en la unidad de tiempo
El comportamiento de todas las unidades de aereacin por difusin pue-
de expresarse utilizando una forma modificada de la ecuacin 5.26
20
02 , 1 ) , (

| o =
t p m n
s
C m Cs W H CQ G
(5.27)
G: kg O
2
transferido. h
1
(unidad)
1
a t
o
C y 101,3 kPa.
C: constante.
W: ancho de tanque de aereacin.
n, m, p: caractersticas del implemento de aereacin.
Una correlacin similar a la anterior, pero para el caso de las turbinas es:
( )
20
02 1

=
t v n x
t
, o C Cs d Qs N C G
(5.28)
N
t:
velocidad perifrica del impelente m s
1
.
d: dimetro del impelente m.
x, n, v: caractersticas del implemento de aereacin.
Es prctica comn utilizar una relacin entre el dimetro del impelente y el
dimetro equivalente del tanque de aereacin entre 0,1 - 0,2 y una velocidad
perifrica del impelente comprendida entre 3,1 - 5,5 m s
1
.
189
Transferencia de Oxgeno
Ejemplo 5.5
Con el propsito de aerear un reactor de 5 000 m
3
de volumen efectivo se
requieren 300 kgO
2
h
1
.
Disee el sistema de aereacin mediante difusores para las siguientes con-
diciones:
temperatura 28
0
C
concentracin de oxgeno disuelto lmite = 2 mgL
1
| = 0,90 Pa = 101,8 kPa
Presin de aire requerida en la descarga = 156,5 kPa
La informacin que se dispone del proveedor se detalla:
Qs = 0,2 m
3
min
1
unidad
1
a 20
0
C y 101,3 kPa
o = 0,8
C = 0,2076
n = 1,05
m = 0,70
p = 0,32
Solucin:
Profundidad seleccionada (H) = 4,5 m
ancho = 8,0 m (s 2 H)
longitud = 139 m
Clculo de Cs,m:
a 28
0
C: Cs = 7,9 mgL
1
= 990 kg m
3
H = 4,5 m
o
P
h
P , Pa
Cs Cs,m
5 0 +
=
P
h
= 0,0098 h (kPa)
P
h
= 0,0098 990 5,0
P
h
= 43,66 kPa
1
L mg 64 9
3 101
66 43 5 0 8 101
9 7

=
+
= ,
,
, , ,
, Cs,m
Clculo de las condiciones reales de trabajo de los difusores:
G = C Qs
n
H
m
W
p
o (| Cs,m - C) 1,02
t20
G = 0,2076 0,2
1,05
4,5
0,70
10
0,32
|0,8 (0,90 9,64 - 2,0)| 1,02
8
G = 0,329 kg h
1
unidad
1
a 28
o
C
190
Menndez Gutirrez, C. y J. Prez Olmo
Nmero de unidades necesarias:
unidades 912
unidad h kg 0,329
h kg 300
u
1 1
1
= =

Espaciamiento entre unidades,


m 0,15
unidades 912
m 139
=
Flujo de aire del compresor,
Q
s
= 0,2 x 912 = 182,4 m
3
min
1
Potencia del compresor requerido:
E = eficiencia 70 %
AP = 54,7 kPa (AP = P
2
P
1
)
P
1
y P
2
= presin a la entrada y a la descarga respectivamente.
( )
E
AP s Q
= kW potencia
kW 237
min seg 60 x 0,7
54,7 182,4
T P
1
=

Capacidad de transferencia necesaria en condiciones reales.


1
2
(kWh) kgO 27 , 1
237
912 329 , 0
-
PT
G
N =

=
u
=
Capacidad de transferencia requerida en condiciones estndar.
( )
20
20
02 1
0

=
t
C
, o C Cs,m
Cs N
No
| |
8
02 1 80 0 2 64 9 90 0
2 9 27 1
, , , .
, ,
No

=
( )
1
2
h W k O k 1,86

= No
Se necesita disponer de un implemento cuya capacidad de transferencia en
condiciones estndar sea superior a 1,86 kg O
2
(k w h)
1
.
Eficiencia de la transferencia de oxgeno,
191
Transferencia de Oxgeno
100
do suministra O
absorbido O
2
2
= E
Se asume que el oxgeno absorbido es el que se necesita, 300 kg h
1
.
Oxgeno suministrado = 16,56 Q
s
= 3020 kg h
1
E = 9,9 %
La eficiencia de la transferencia en condiciones estndar:
20
C 20
02 , 1 ) (

o |
=
t
C Csm
Cs
E Eo
Eo = 15 %
El dispositivo que se utilice debe tener adems una eficiencia de transferen-
cia de 15 % en condiciones estndard.
Ejemplo 5.6
Para el mismo caso del ejemplo anterior, considere el diseo de un sistema
de aereacin con turbinas.
Informacin adicional del proveedor:
P
d
= 0,95 Eo = 20 % a = 0,85
Solucin:
Cs,m = 9,83 mgL
1
Se asume como condicin de trabajo que el oxgeno absorbido por el sistema
debe ser igual al oxgeno que se requiere = 300 kg h
1
Oxgeno suministrado = 16,56 Qs
0 0,2
do suministra oxgeno
absorbido oxgeno
= = Eo
Qs ,
,
56 16
300
2 0 =
Qs = 90,5 m
3
min
1
(20 C, 101,3 kPa)
Potencia requerida por el compresor (P
c
):
e = 70 % AP = 54,7 k Pa
W k 117,9
60 0,7
54,7 90,5
=

= Pc
192
Menndez Gutirrez, C. y J. Prez Olmo
Potencia requerida por la turbina (P
t
):
c
t
d
P
P
P =
P
t
= 0,95 117,9 = 112 k W
Capacidad de transferencia que se requiere:
( )
1
2
25 1
300

=
+
= h W k O kg ,
t
P
c
P
N
Capacidad de transferencia necesaria en condiciones ideales
( )
20
20
0
02 1
0

=
t
C
, o C Cs,m
Cs N
N
No = 1,7 kg O
2
(k W h)
1
Es necesario disponer de un dispositivo cuya capacidad de transferencia en
condiciones de referencia sea superior a 1,7 kg O
2
(k W h)
1
.
5.5. CONSUMO DE POTENCIA
La potencia terica del compresor o soplador para un sistema de aereacin
por difusin puede calcularse:
Pc = 1,1628 m Wc (5.29)
Pc: potencia terica del compresor W.
m: flujo msico de aire que se comprime kg h
1
.
Wc: trabajo en el eje del compresor kcal kg
1
.
Asumiendo que para las presiones y temperaturas consideradas en la
compresin el aire se comporta como un gas ideal (lo que es una buena
aproximacin),
(
(

|
|
.
|

\
|
|
.
|

\
|
= 1
29
1
1
2
1
A
P
P
T R A Wc
(5.30)
y
y
A

=
1
193
Transferencia de Oxgeno
T = constante adiabtica =
4 1,
C
C
v
p
=
(para aire)
Cp, Cv = calores especficos a presin y volumen constantes respectivamente.
P
2
: presin a la profundidad que se introduce el aire kPa.
R: constante de los gases = 1,987 kcal (mol K)
1
.
T
1:
temperatura de entrada del aire al compresor
0
K.
P
1:
presin del aire a la entrada del compresor kPa.
La ecuacin 5.30 puede expresarse de una forma ms sencilla,
(
(

|
|
.
|

\
|
= 1 2398 0
286 0
1
2
1
,
P
P
T , Wc
(5.31)
La potencia real del compresor est determinada por el rendimiento global
del mismo, q,
q = q
T
q
m
(5.32)
q
T
: rendimiento termodinmico (desviacin de las condiciones de reversibilidad
adiabtica)
q
m
: rendimiento considerando las prdidas en el motor y la transmisin
p
P
P
c
=
(5.33)
Para el clculo del requerimiento de potencia en el caso que se empleen
turbinas, ha de considerarse tanto el accionamiento del rotor como el funciona-
miento del compresor,
P
T
= P
c
+ P
r
(5.34)
P
T
= requerimiento de potencia total.
P
c
= potencia del compresor.
Pr = potencia del impelente o rotor.
Por otro lado, se ha demostrado que la capacidad de transferencia de oxge-
no (N) ptima de un aereador de turbina est vinculado a la relacin entre la
potencia del rotor y la del compresor:
c
r
d
P
P
P =
(5.35)
Las mejores condiciones de trabajo se presentan cuando el valor de P
d
es
cercano a 1.
194
Menndez Gutirrez, C. y J. Prez Olmo
La potencia requerida por el impelente decrece en la medida que aumenta la
cantidad de aire introducido como consecuencia de la disminucin de la densi-
dad de la mezcla bajo aereacin. Por tanto:
P
d
< 1,0 para altos caudales de aire.
P
d
> 1,0 para bajos caudales de aire
Para evitar la formacin de vrtices se suele emplear tabiques de choques.
Si el tanque de aereacin es circular, se usan 4 tabiques uniformemente distri-
buidos en el permetro del recipiente. Para tanques cuadrados se colocan 2
tabiques en paredes opuestas. En tanques rectangulares donde la relacin largo:
ancho es mayor de 1,5 no se requiere la colocacin de tabiques de choque.
Ejemplo 5.7
Para un sistema de aereacin por difusin se necesitan 720 kg h
1
de
oxgeno.
Calcular:
a) Rendimiento de la transferencia de oxgeno E.
b) Nmero de unidades de difusin.
c) Potencia real requerida.
d) Temperatura ambiente 28
0
C.
e) Presin ambiental 99,96 k Pa.
f) Profundidad de los difusores 4,0 m.
o = 0,90
| = 0,92
La concentracin de oxgeno mnima deseada en el tanque de aereacin es
2,5 mgL
1
,
Cs
20 C
= 7,9 mgL
1
(101,3 kPa)

agua
= 990 g L
1
(28
0
C)
Solucin:
a)
o
h
P
P , Pa
Cs Cs,m
5 0 +
=
P
o
= 101,3 k Pa.
P
a
= 99,96 k Pa.
P
h
= 0,0098 ,h = 0,0098 990 4,0.
g) Segn el proveedor, el flujo msico de oxgeno entregado por cada
unidad de difusin es 0,8 kgO
2
h
1
(20
0
C, 101,3 kPa, 0 mgL
1
OD).
195
Transferencia de Oxgeno
P
h
= 38,81 k Pa.
Cs,m = 9,31 mgL
1
La relacin entre la transferencia de oxgeno en las condiciones reales y las
de referencia viene dada por:
( )
20
02 1
2 9

=
t
o
, o
,
C Cs,m
E
E
( )
8
02 1 90 0
2 9
5 2 31 9 92 0
8 0 , ,
,
, , ,
, E

=
E = 0,556 kg O
2
h
1
por unidad de difusin
b) Nmero de unidades de aereacin:
unidades 1295
0,556
720
U = =
c) Pc = 1,1628 m Wc
(
(

|
|
.
|

\
|
= 1 2398 0
286 0
1
2
1
,
P
P
T , Wc
P
1
= 99,96 kPa
T
1
= 28
0
C = 301
0
K
P
2
= P
o
+ P
h
= 101,3 + 38,81 = 140,11 k Pa
Wc = 7,32 kcal kg
1
m = 720 kg h
1
Pc = 6128 W (potencia terica)
Ejemplo 5.8
Una planta tiene 3 sopladores centrfugos, cada uno de ellos con una capa-
cidad en condiciones reales de trabajo de 43 m
3
min
1
. Dos de ellos estn en
operacin y el tercero en reposo. Determine la cantidad de aire que es capaz de
entregar el sistema.
Se dispone de la siguiente informacin:
Eficiencia estndar de transferencia de oxgeno de los difusores = 12 %
(SOTE)
Profundidad de los difusores en el tanque de aereacin = 3,5 m
Presin = 101, 5 kPa; T = 28 C;
Densidad del agua = 990 kg m
3
o = 0,85; | = 0,90
196
Menndez Gutirrez, C. y J. Prez Olmo
Cs
28 C
= 7,92 mgL
1;
Cs
20 C
= 9,2 mgL
1
; C = 2,0 mgL
1
3 101
5 0
,
P , P
Cs m Cs,
h o
+
=
kPa 33,96 3,5 990 0,0098 h p 0,0098 = = =
h
P
1
L mg 26 9
3 101
96 33 5 0 5 101
92 7

=
+
= ,
,
, , ,
, m s, C
( )
( )
8
02 1 85 0
2 9
2 26 9 90 0
12 0 , ,
,
, ,
, E

=
0,082 = E
% 8,2 = E
El clculo del flujo msico de oxgeno que entregan los 2 aereadores se
realiza utilizando la eficiencia actual de los difusores y la capacidad estndar de
los sopladores.
Convertir la salida de los sopladores de las condiciones reales a las estndar
(20
o
C, 101,3 kPa):
( ) ( )
|
.
|

\
|
|
.
|

\
|
=

Pr
min min
1 3 1 3
Ps
Ts
Tr
. m s m r
R T
o
en
67 491
5
9
, C R
o
+ =
( )
R , , Ts
o
67 527 67 491 20
5
9
= + =
( )
R , , Tr
o
07 542 67 491 28
5
9
= + =
|
|
.
|

\
|
|
|
.
|

\
|
=
3 , 101
5 , 101
07 , 542
67 , 527
43
197
Transferencia de Oxgeno
s = 41,9 m
3
min
1
.
El flujo msico de oxgeno que entregan los 2 aereadores se calcula utilizan-
do la eficiencia actual de los difusores y la capacidad de los sopladoes en condi-
ciones estndar (41,9 m
3
min
1
),
kg O
2
d
1
= 2 41,9 m
3
min
1
1440 min.d
1
0,232 kgO
2
(kg aire)
1
.
1,29 kg aire m
3
0,095 = 3430
por tanto, dos sopladores, que cada uno entrega 41,9 m
3
min
1
van a transferir
3430 kg d
1
.
Ejemplo 5.9
En una instalacin existen dos aereadores superficiales mecnicos de 40
kW cada uno que son utilizados simultneamente. Segn reporta el fabri-
cante, el SOTR es de 1,8 kg O
2
(kW - h)
1
. Calcule el oxgeno transferido si
adems se brinda la siguiente informacin:
o = 0,85; | = 0,90; Csm
28
C = 7,92 mgL
1
T = 28
o
C; C = 2 mgL
1
( )
20
20
0
02 1

=
t
, o
Cs
C Cs
N N
8
02 1 85 0
2 9
2 92 7 9 0
8 1 , ,
,
, ,
, N |
.
|

\
|
=
N = 0,99 kg(kW- h)
1
Por otro lado, se asume que la potencia utilizada para transferir el gas es el
75% de la nominal del motor: (0,90 0,85)
Por lo tanto, la potencia que es utilizada para transferir el gas es:
pkW = 0,75 40 = 30 kW
O sea, el oxgeno transferido es,
kg O
2
d
1
= N pkW = 0,99 60 24 h d
1
= 1426 kW
Notas bibliogrficas
1 LEWIS, W.K. AND W. G. WHITMAN: 'Principles oI gas absorption, Ind. Eng.
Chem., vol. 16, no. 8, 1924.
2 DANCKWERTS, D. V.: 'SigniIicance oI liquid-Iilm coeIIicient in gas adsorption,
Ind. Eng. Chem, vol. 43, no. 1, 1951.
198
Menndez Gutirrez, C. y J. Prez Olmo
3 TEWARI, P. K. AND J. K. BEWTRA: 'Alpha and Beta Iactors Ior domestic was-
tewater, Jour. oI Water Poll. Control Fed., vol. 54, no. 9, 1982.
4 ECKENFELDER, W. W.: Water Quaility Engineering for Practicing Engineers,
CBI Publishing, EE.UU., 1980.
5 MENNDEZ, C. L. Y J. PREZ: 'Capacidad de oxigenacion de reactores biologi-
cos en el tratamiento de aguas residuales, Ing. Hidrulica, vol. 3, no. 2,
1982.
6 ECKENFELDER, W. W. AND J. O'CONNOR: Biological Waste Treatment, Perga-
mon Press, Oxford, 1981.
7 MENNDEZ, C. L. Y L. GUERRA: 'Laguna aereada en el tratamiento de residua-
les teneros, Ing. Hidrulica, vol. 9, no. 4, 1988.
8 HOOVER, S. R. AND S. H. PORGES: 'An interpretation oI the BOD test in terms
oI endogenous respiration oI bacteria, Sewage and Industrial Wastes,
vol. 24, no. 7, 1957.
9 CHUBOBA, J., C. MENNDEZ Y J. PREZ: Fundamentos Tericos de Algunos
Procesos para la Purificacin de Aguas Residuales, Ed. ISPJAE, Ciu-
dad de La Habana, 1986.
10 MENNDEZ, C. L.: Wastewater Treatment from the pulp and paper indus-
try, Ph.D. Thesis, Prague, 1986.
11 VASICEK, P.: 'Use oI a kinetic study to optimize the activated sludge process,
Jour. of Water Poll. Control Fed., vol. 54, no. 6, 1982.
12 CLOUGH, G. F.: 'Physical characteristics oI mechanical aerators, Jour. oI
Water Poll. Control Fed., vol. 46, no. 2, 1974.
13 KORMANIK, R. AND ET AL.: Proc. 28 th Ind. Waste Conf. Purdue University,
1973.
14 REDMON, D. T. AND ET AL.: 'TransIer eIIiciency measurements in mixed liquor
using oII-gas techniques, Jour. oI Water Poll. Control Fed., vol. 55,
no. 11, 1987.
199
Filtros Percoladores
CAPTULO 6
FILTROS PERCOLADORES
6.1. INTRODUCCIN
Los filtros percoladores o ms correctamente denominados lechos
bacterianos, son sistemas de depuracin biolgica de aguas residuales, en
los que la oxidacin de la materia orgnica se produce al hacer pasar, a
travs de un medio poroso cubierto de una pelcula biolgica, aire y agua
residual. El agua residual fluye sobre la superficie del medio poroso o empa-
que en una delgada capa que est en contacto con la pelcula biolgica por
un lado y con el aire en los espacios intersticiales del empaque por el otro. El
fundamento del proceso est basado en las acciones producidas en todo el
espesor de la pelcula biolgica.
A favor del empleo de los filtros percoladores est el hecho demostrado, que
los cultivos fijos a un soporte o medio son afectados en menor medida, que los
cultivos suspendidos ante cambios de las condiciones ambientales, que preva-
lecen en un momento dado en las condiciones de trabajo del dispositivo de
tratamiento.
1
La pelcula biolgica est constituida por un conjunto complejo de
microorganismos aislados y colonias de ellos, embebidos en una matriz de
polmeros, cuya estructura y composicin es funcin de la edad de la
biopelcula y de las condiciones ambientales.
2
Como sucede con todos los sistemas de tratamiento biolgico de aguas
r esiduales, los tipos y pr opor ciones de las diferentes especies de
microorganismos presentes en la poblacin microbiana dependen de las con-
diciones de operacin del sistema, especialmente la carga, y la composicin
del agua residual.
Las condiciones aerobias se mantienen por el flujo de aire a travs del
empaque o empaquetadura del lecho. La circulacin del aire se realiza de
forma natural o forzada, a contra corriente o en el mismo sentido del flujo
de agua. Este flujo de aire es inducido por la diferencia entre el peso espe-
cfico del aire atmosfrico dentro y fuera de la empaquetadura. En estos
200
Menndez Gutirrez, C. y J. Prez Olmo
sistemas, en muy raros casos, se emplea la ventilacin forzada. En la figu-
ra 6.1 se muestra una vista de una planta depuradora que emplea filtros
percoladores.
Fig. 6.2. Seccin de un filtro percolador circular de brazo mvil rotatorio.
Fig. 6.1. Vista de una instalacin que emplea filtros percoladores.
Peridicamente existen desprendimientos de la pelcula biolgica. Esto hace
necesario el empleo de sedimentadores secundarios para la separacin del sli-
do de la corriente de lquido tratado. La seccin transversal de un filtro se mues-
tra en la figura 6.2. Tambin es recomendable el uso de sedimentadores primarios
para evitar tupiciones en el material de relleno que dificulten el libre paso de
agua y aire a travs del mismo.
201
Filtros Percoladores
6.2. PARTES DE LAS QUE CONSTA UN FILTRO
PERCOLADOR. MECANISMO DE REMOCIN
DE LA DBO
6.2.1. Partes de que consta
Los filtros percoladores constan de 3 partes principales:
Sistema de distribucin.
Empaquetadura.
Sistema recolector.
El sistema de distribucin debe proporcionar una carga hidrulica uniforme so-
bre la superficie del filtro. Por tanto, han de evitarse atascos y paradas. Los aspersores
para la distribucin del agua residual pueden ser fijos o circulares dependiendo de
que la estructura del filtro sea rectangular o circular respectivamente.
Los aspersores fijos requieren un dispositivo ms complejo de distribucin y,
por tanto, una mayor prdida de carga (alrededor de 2 m). Los mviles consis-
ten en brazos giratorios, 2 4, que se disponen radialmente, y son movidos por
carga hidrulica. La prdida de carga prevista para estos casos es aproximada-
mente de 0,5 m. La velocidad de rotacin es de 0,3 a 5 vueltas por minuto,
dependiendo de la carga hidrulica a la que se desea someter el percolador.
Fig. 6.3. Filtro percolador rectangular de distribuidor mvil no rotatorio.
202
Menndez Gutirrez, C. y J. Prez Olmo
A travs del sistema recolector se extrae el efluente del tratamiento y se
produce adems la circulacin del aire.
Fig. 6.4. Sistema recolector de un percolador.
La recogida de agua residual tratada se efecta por medio de un dispo-
sitivo de drenaje en el fondo del lecho bacteriano. Este sistema debe tener
previsto un sistema de canales de recogida, con la caracterstica fundamen-
tal de que no debe existir sedimentacin en los mismos, ya que el agua resi-
dual contiene los flculos que sedimentarn en el decantador secundario.
Para ello la pendiente ser del 1 % 2 %, y la seccin no ir nunca llena de
agua, para que pueda realizar su funcin como canal de aereacin. Una
recomendacin de diseo indica que la zona de salida al falso fondo de agua
y aire sea 15 % 20 % de la superficie total del filtro.
La empaquetadura o relleno constituye el medio de soporte de la biomasa.
Las dos propiedades ms importantes de los filtros percoladores son la
superficie especfica y el porcentaje de huecos del empaque. La misma se
define como los m
2
de superficie de relleno por m
3
de volumen total del
empaque o relleno. Cuanto mayor sea la superficie especfica mayor ser la
cantidad de limo biolgico presente por unidad de volumen. Por otra parte
mientras mayor sea la proporcin de huecos en el empaque, se podr traba-
jar con mayores valores de carga hidrulica y con menor riesgo de que
ocurra inundacin por tupiciones.
El objetivo de los medios de empaque es el de proporcionar un soporte slido
y estable para el limo biolgico, y exponer la mxima rea superficial al flujo del
lquido que se desea tratar, y de superficie mojada al contacto del aire.
El material que se utiliza para el relleno o empaque de los filtros debe poseer:
Resistencia mecnica.
Resistencia qumica.
Alta relacin rea/volumen.
203
Filtros Percoladores
Aunque el material de relleno puede ser de granito, coke o escoria entre
otros materiales, los medios ms comnmente empleados son el estndar de
piedra y material plstico.
Medio de piedra
a) Tamao de la piedra 3-15 cm.
b) rea especfica A
v
= 40 - 80 m
2
m
3
.
c) Peso especfico = (2 - 3). 10
3
kg m
3
.
d) Espacio vaco 50 % del volumen del empaque.
Material plstico
a) Medio de diferentes formas, tamao y materiales.
b) rea especfica A
v
= 20 - 250 m
2
m
3
.
c) Peso especfico = 50 - 90 kg m
3
.
d) Espacio vaco 90-95 % del volumen del empaque.
Las principales ventajas que sobre el medio estndar de piedra tiene el me-
dio de plstico son:
a) Alta rea especfica.
b) Mayor espacio vaco.
c) Bajo peso especfico.
El limo que se desarrolla sobre los medios plsticos es, ecolgicamente,
similar al que se forma sobre los medios minerales convencionales, que con-
tienen bacterias del tipo zoogleas, protozoos y macroinvertebrados.
Teniendo en cuenta que generalmente los filtros percoladores con em-
paques plsticos operan comnmente a mucha mayor carga hidrulica que
los convencionales de piedra, la eliminacin del exceso de limo o pelcula
biolgica es el resultado de la accin de lavado por el flujo de agua ms que
por la accin de la actividad de los organismos macroinvertebrados.
La principal desventaja del medio plstico es su alto costo, aunque compensa-
do en cierta medida por el menor costo de las paredes y piso de la estructura que
lo contiene, al ser ms sencillos que si el medio soporte empleado es la piedra.
Los materiales de plstico de los que ms comnmente se dispone pueden
ser de distribucin ordenada o distribucin aleatoria, tambin llamada catica o
desordenada.
El empaque que se oferta para colocar en disposicin ordenada se presenta,
generalmente, en paquetes (estructuras modulares) de configuracin
paraleleppeda, de hojas conformadas y encoladas (por ejemplo, Flocor o Plasdek).
Tambin pueden ser de forma tubular (tubos de 8 a 10 cm de dimetro) que
llevan tabiques internos para aumentar la superficie especfica; estos tubos se
204
Menndez Gutirrez, C. y J. Prez Olmo
colocan a lo largo de toda la altura del filtro por capas (por ejemplo, el Bionet).
Figura 6.7.
Fig. 6.5. Colocacin del material de empaque en un filtro.
La manera ms econmica de contener los empaques aleatorios es dentro
de estructuras de seccin circular, y los sistemas modulares en estructuras
rectangulares. Por lo tanto, el tipo de empaque plstico prcticamente define
tambin el sistema de distribucin a emplear.
La altura del empaque dentro del filtro percolador comnmente oscila entre
1,5 y 8 metros. Cuando se emplea un medio convencional de piedra la altura
ms utilizada es la de 2 m.
Fig. 6.6. Filtro percolador tipo torre con empaque plstico.
205
Filtros Percoladores
Los tamaos de rido recomendados, como ya fue expresado, oscilan de
3 a 15 cm de dimetro. Especial atencin debe prestarse a la uniformidad
del medio, ya que mientras ms uniforme sea su tamao, mayor proporcin de
huecos o espacios vacos tendr el empaque, para un tamao dado del mismo.
Los materiales disponibles para su distribucin dentro de la estructura del
filtro de forma aleatoria estn constituidos por elementos individuales de un ta-
mao de 4 a 10 cm, dispuestos directamente en el lecho sin ninguna combina-
cin (por ejemplo, Norpac). Figura 6.7.
Fig. 6.7. Algunos tipos de empaques plsticos: a) Bionet. b) Norton.
Actifil. c) Norpac. d) Plasdek.
206
Menndez Gutirrez, C. y J. Prez Olmo
6.2.2. Mecanismo de remocin de la DBO
El mecanismo de remocin de los compuestos orgnicos en el filtro
percolador es similar al del proceso de lodo activado.
Una fraccin del volumen del lquido que se aplica al filtro pasa rpidamente
a travs del empaque y otra parte del flujo percola ms lentamente ponindose
en mayor contacto con el limo biolgico. Los contaminantes suspendidos son
rpidamente removidos por adsorcin y coagulacin. La oxidacin ocurre fun-
damentalmente en la fraccin del flujo que tiene mayor tiempo de retencin.
3
La materia orgnica contaminante del agua es degradada en la pelcula bio-
lgica. Esta pelcula no debe tener ms de 3 mm de espesor ya que no se puede
asegurar la accin del oxgeno en espesores mayores.
4,5
La pelcula biolgica se forma por adherencia de los microorganismos al
rido y a las partculas orgnicas, formando la biopelcula . Al aumentar el espe-
sor de la biopelcula, las capas ms internas de biomasa van tornndose anaerobias
al irse reduciendo paulatinamente la concentracin de oxgeno disuelto disponi-
ble. En la capa ms cercana al medio soporte, completamente anaerobia, se
produce desprendimiento de gases y rotura de la pelcula, perdiendo la capaci-
dad de adherencia al medio poroso. Se desprende la pelcula, siendo arrastrada
por el agua residual y conducida a la decantacin secundaria, donde se produci-
r la sedimentacin. Figura 6.8.
Fig. 6.8. Representacin esquemtica de la pelcula biolgica sobre un
elemento de empaquetadura.
6.3. PARMETROS BSICOS DEL PROCESO.
DEFINICIONES
Los principales parmetros que caracterizan la operacin de los filtros
percoladores son la carga hidrulica, carga orgnica, la eficiencia de purifica-
cin y la relacin de recirculacin.
207
Filtros Percoladores
Carga hidrulica (m
3
de agua residual aplicada por m
2
por da)
f
o
A
Q
c =
(m
3
m
2
d
1
) (6.1)
Q
0
: Flujo (m
3
d
1
)
A
f
: rea de la seccin transversal del filtro (m
2
)
Carga orgnica (kg DBO
5
aplicada por m
3
por da)
H
S
c
H A
S Q
B
o
f
o o
v
=

=
(kg m
3
d
1
), (6.2)
S
0
: DBO
5
del afluente
H: Profundidad o altura del empaque (m)
Eficiencia de purificacin
100
2

=
o
o
S
S S
E
(%) (6.3)
S
2
: DBO
5
del efluente
Relacin de recirculacin
o
r
Q
Q
R =
Q
r
: Flujo de recirculacin
Un diagrama de flujo tpico se representa en la figura 6.9.
Fig. 6.9. Diagrama de flujo de un sistema de filtro percolador. 1) Sedi-
mentador primario. 2) Filtro. 3) Sedimentador secundario.
6.4. CLASIFICACIN DE LOS FILTROS PERCOLADORES
Los filtros pueden clasificarse, de acuerdo con los parmetros tecnolgicos
bajo los cuales se operan, en cuatro grupos:
a) De baja velocidad.
b) De velocidad media.
208
Menndez Gutirrez, C. y J. Prez Olmo
c) De alta velocidad.
d) De velocidad super alta.
Filtros de baja velocidad:
Generalmente no emplean recirculacin
u 2,0 5,0 m
3
m
2
d
1
B
v
= 0,2 kg m
3
d
1
Filtros de velocidad media:
La dosificacin es continua, con recirculacin
u 4 10 m
3
m
2
d
1
B
v
0,2 0,5 kg m
3
d
1
Filtros de alta velocidad
Usualmente de empaque sinttico y recirculacin
u 15 30 m
3
m
2
d
1
B
v
0,7 1,0 kg m
3
d
1
Filtros de velocidad super alta
Empaque sinttico y recirculacin
Profundidades mayores a los 6 m
u 40 200 m
3
m
2
d
1
B
v
0,8 6,0 kg m
3
d
1
Los filtros con empaque de piedra no deben operarse con cargas hidru-
lica y orgnica con valores en el intervalo de 5,0 a 15 m
3
m
2
d
1
y 0,2 a 0,7
kgm
3
d
1
respectivamente. Al operar un filtro de piedra en este intervalo se
corre el riesgo de tupicin del mismo. Esto no ocurre con los filtros con
empaquetaduras plsticas. Estos ltimos pueden ser operados dentro de todo
el intervalo de valores de cargas.
6
6.5. RECIRCULACIN
Una de las razones que justifican la introduccin de la recirculacin es
para evitar la tupicin del filtro. Para flujos elevados el limo o cieno biolgi-
co se remueve ms fcilmente del filtro a travs de la autolimpieza.
La aplicacin de la recirculacin tiene como consecuencia adems una
dilucin de la concentracin de las sustancias orgnicas del agua residual.
209
Filtros Percoladores
La concentracin de DBO de la mezcla puede calcularse por un balance de
materiales a la entrada del filtro,
( )
r o m r o o
Q Q S S Q S Q + = +
2
(6.4)
R
S R S
S
o
m
+
+
=
1
2
Para el empleo del empaque de piedra en el tratamiento de aguas
residuales urbanas usualmente se recomienda que la DBO5 del agua resi-
dual a la entrada al filtro no sobrepase de 100 a 150 mg/l. Puede considerar-
se que un agua residual urbana decantada tiene una DBO5 media del orden
de 200 mg/l. Una recirculacin R del 100 al 150 % es suficiente, en general,
para respetar esta regla.
7
Para aguas residuales con slidos suspendidos debe ser utilizado un
sedimentador primario a la entrada del filtro, fundamentalmente cuando la
empaquetadura de este sea de piedra. Si el agua residual contiene bajo nivel de
slidos suspendidos y el medio filtrante es de material plstico puede obviarse
en ocasiones el uso de la sedimentacin primaria.
6.6. FACTORES QUE INCIDEN EN LA EFICIENCIA
DE PURIFICACIN
Entre los factores que inciden en la eficiencia de purificacin pueden citar-
se, como los principales, el tiempo de contacto con el limo biolgico, la tempera-
tura, la cantidad y actividad de los microorganismos en el limo y la transferencia
de oxgeno a la pelcula de limo.
6.6.1. Tiempo de contacto
El tiempo de contacto promedio a travs del filtro puede expresarse por la
relacin:
8
n
c
H
C t =
(6.5)
donde:
H: Profundidad del filtro.
u: Carga hidrulica.
C y n: Constantes que varan con el tipo de empaque.
210
Menndez Gutirrez, C. y J. Prez Olmo
La experiencia demuestra que el tiempo de contacto promedio en el filtro
vara entre 5 y 60 minutos en dependencia de la carga hidrulica.
9
La existencia
del limo tambin incrementa el tiempo de retencin. Este tiempo tiende a
incrementarse debido al almacenamiento de agua por capilaridad. No obstante,
Mehta y col.,
10
sealan que los anlisis del tiempo de retencin en los percoladores
carecen de importancia y solo dificultan distinguir los aspectos fundamentales
en discusin.
El efecto de la carga hidrulica del filtro sobre la eficiencia se ilustra en la
figura 6.10.
En la figura 6.10 se observa que para un valor de carga hidrulica constante,
la eficiencia de remocin de DBO se mantiene tambin constante, independien-
temente de la concentracin del afluente. Por otro lado, para un valor fijo de
DBO de entrada, la eficiencia de remocin disminuye cuando se incrementa la
carga hidrulica.
11
Como se ver ms adelante, esta propiedad es reflejada en el modelo de
primer orden que describe el comportamiento de los filtros percoladores.
Fig. 6.10. Influencia de la carga hidrulica sobre la eficiencia.
6.6.2. Temperatura
El efecto de la temperatura sobre el funcionamiento de los percoladores es el
resultado de dos efectos opuestos. Por un lado, la difusividad de los contaminantes
y el oxgeno aumenta al elevarse la temperatura, as como tambin se incrementa
la actividad biolgica; por otra parte, la solubilidad del oxgeno en el agua disminu-
ye. El efecto neto resultante de los dos ha sido reflejado en diferentes expresiones
empricas que establecen la relacin entre la temperatura y la eficiencia del proce-
so. Tal es el caso de Shriver y Bowers
12
entre otros.
211
Filtros Percoladores
Sin embargo, teniendo en cuenta que el efecto de la temperatura sobre los
filtros percoladores es similar al que tiene este parmetro sobre todos los procesos
biolgicos, el mismo puede expresarse mediante la ecuacin 6.6, de base tambin
emprica, pero a la que adems puede llegarse a travs de la ecuacin de Arrhenius:
k
T
= k
20

0
t 20
(6.6)
0
usualmente se toma como 1,035
13, 14
1,047.
Un criterio conservador es asumir el valor de 1,045.
15
6.6.3 Cantidad y actividad de la capa de limo
La cantidad de la pelcula biolgica en el empaque del filtro vara con la
profundidad y con la temperatura.
16,17
El espesor de la biopelcula disminuye en
la medida que se desciende a travs de la altura del empaque, ya que disminuye
la concentracin de materia orgnica biodegradable.
Se estima que para filtros de baja velocidad la cantidad de biopelcula
vara de 4,7 a 7,1 kg por cada metro cbico de empaque, mientras que para
filtros de alta carga la masa de limo que se logra est comprendida entre 3,3
y 6,5 kg m
3
.
6
La eficiencia de remocin de DBO no est tan directamente relacionada
con la cantidad de biomasa presente como con la fraccin de esta que acta
como un oxidante activo.
Por otra parte, el espesor de la biopelcula normalmente est comprendido
entre 0,1 y 2,0 mm. Existe un efecto perjudicial en la operacin del filtro percolador
si dicho espesor es superior a los 2,0 mm, pudiendo presentarse una obstruccin
del relleno, dificultando el flujo del agua residual y la transferencia de oxgeno a
los microorganismos aerobios. Jenkins
18
determin que el espesor de la biopelcula
que proporciona la mxima eficiencia est alrededor de 0,25 mm. Sin embargo,
en este sentido hay resultados muy diversos. Mientras que Bruce
19
sita la pro-
fundidad aerobia activa en un espesor entre 0,05 y 0,1 mm, Kornegay y
Andrews
20
sealan profundidades crticas entre 0,07 y 0,15 mm, La Motta
21
las
sita entre 0,012 y 0,065 mm. Por su parte, Williams,
22
reporta buenos resulta-
dos tratando aguas residuales del procesamiento de alimentos, con espesores de
biopelcula de hasta 8 mm de espesor.
La cantidad de fraccin activa vara con la carga orgnica, la carga hidru-
lica y la temperatura.
16
El espesor de pelcula activa depende adems de la
profundidad de la pelcula a travs de la cual penetra el oxgeno para mantener
las condiciones aerobias.
Adems del efecto que posee la carga hidrulica sobre el espesor de la
biopelcula, existe un control natural como consecuencia de la accin depredadora
212
Menndez Gutirrez, C. y J. Prez Olmo
ejercida por macroinvertebrados, larvas y gusanos que coexisten con las bacte-
rias, protozoos y hongos, en el limo acumulado.
6.6.4. Transferencia de oxgeno a la pelcula del limo
El oxgeno necesario para la oxidacin biolgica en el seno del filtro, es
suministrado por el aire que fluye a travs del mismo.
El fenmeno de la aereacin natural, fue estudiado por Halvorson, Savage y
Piret,
23
quienes llegaron a relacionar la diferencia de temperatura entre el aire y
agua, con el caudal y direccin del aire, formulando una ley lineal del siguiente tipo:
Qa K (Ta TL - T) (6.7)
siendo:
Qa: Caudal de aire en m
3
m
2
d.
Ta: Temperatura del agua en el interior del lecho en C.
TL: Temperatura exterior del aire en C.
T: Constante de 2 C para lechos convencionales.
K: Constante del lecho.
Para una diferencia de temperatura Ta TL = 6 C, el caudal de aire garan-
tizado es aproximadamente 18 m
3
m
2
h
1
.
El esquema de la circulacin del aire puede verse en la figura 6.11, deducido
de la siguiente expresin para filtros convencionales,
Va = 0,075 (Ta-TL) - 0,15 (6.8)
siendo Va = Velocidad del aire (en mmin
1
).
Fig. 6.11. Circulacin del aire en un lecho.
213
Filtros Percoladores
Halvorson determin que es necesaria una corriente de aire mnima
de 0,3 mmin
1
para que se realice una buena oxigenacin. Por tanto, las
diferencias ideales entre la temperatura del agua en el interior del filtro y la
del aire en el exterior, seran aquellas superiores a 6 o a 2 C. En el
intervalo definido por estos valores la aereacin es deficiente y por tanto se
pueden producir procesos anaerobios indeseables.
Tal como se afirma anteriormente, la fraccin activa de la biopelcula de-
pende en cierta medida del nivel de penetracin del oxgeno a travs del espesor
de la misma. Esta profundidad puede ser estimada por las relaciones que si-
guen.
Durante la operacin en estado estacionario de un filtro percolador, la velo-
cidad de transferencia de oxgeno del lquido a la pelcula biolgica puede
obtenerse por:
6
( )
2 1
C C A
h
D
M
L
=
kgh
1
(6.9)
donde:
C
1
y C
2
: Concentraciones de oxgeno a la profundidad 1 y 2 de la pelcula.
D
L
: Difusividad a travs del espesor de la pelcula (m
2
h
1
) h.
A: rea de la pelcula.
La velocidad de utilizacin de oxgeno por los microorganismos en la pelcu-
la puede expresarse:
h A W k M
I
= (6.10)
donde:
W: Gravedad especfica de la pelcula.
k
I
: Constante de velocidad (h
1
).
En condiciones de estado estacionario la velocidad de consumo de oxgeno
es igual a la velocidad de transferencia, por lo tanto,
( )
h A W k C C A
h
D
I
L
=
2 1
(6.11)
La penetracin mxima de oxgeno ocurrir cuando la concentracin de
oxgeno C
2
a la profundidad h sea cero, y
W
I
k
C
L
D
h

=
1
(6.12)
La mxima profundidad de la zona aerbia es de 2 a 3 mm.
214
Menndez Gutirrez, C. y J. Prez Olmo
6.7. DISEO DE FILTROS PERCOLADORES
En la actualidad son reconocidos muchos modelos para el diseo de filtros
percoladores. En este texto se abordar el estudio de dos de los ms reconoci-
dos. El modelo de Eckenfelder (primer y segundo orden),
24
que tiene su funda-
mento en los estudios de Velz
25
y Howland,
8
y el modelo emprico del National
Research Council (NRC).
26
6.7.1. Modelo de Eckenfelder
Con el objetivo de describir el comportamiento de los filtros percoladores, se
han desarrollado diferentes modelos matemticos. Muchos de estos modelos se
basan en la suposicin de que la velocidad de remocin de compuestos orgni-
cos biodegradables, siguen una cintica de primer orden, asumiendo el modelo
de flujo tubular o de pistn.
El modelo de reaccin de primer orden puede obtenerse como sigue:
S k
t d
S d
1
=
(6.13)
Asumiendo el comportamiento de flujo a pistn:
( )
t k
o
e
S
S

=
1
2
(6.14)
S
0
: DBO del afluente.
S
2
: DBO del efluente.
k
1
: constante de velocidad del proceso.
Sustituyendo para el tiempo t la ecuacin (6.5) en (6.14),
n
c
H
C k
e
S
S
o
1
2

=
(6.15)
La constante de velocidad depende de la temperatura de acuerdo con la
ecuacin 6.6:
20
20

=
t
T
k k
La constant e de velocidad tambin depende de la cantidad de
microorganismos en la pelcula. Esta cantidad es proporcional a la superficie
especfica del medio utilizado.
215
Filtros Percoladores
Sustituyendo la ecuacin (6.6) para k
1
en (6.15) y combinando todas las
constantes en una sola:
|
.
|

\
|


0
=
n
v
c H A
t
k
e
So
S
20
20
2
(6.16)
A
V
: rea especfica del empaque, m
2
m
3
.
El modelo descrito por la ecuacin (6.16) es conocido como modelo de
Eckenfelder.
24
El exponente n vara entre 0,5 y 1,0 en dependencia del medio de
empaquetadura empleado. Benjes
27
ha fijado el valor de n en 2/3 para empaque
de piedra y de 1/2 para los medios plsticos. Para empaque plstico aleatorio,
Porter y Smith
28
recomiendan el valor de 0,44 para n.
De la ecuacin 6.16 puede obtenerse una expresin para calcular el volu-
men necesario de empaque. As:
a) Cuando no hay recirculacin,
( ) H Q
H kA
S
S
V
n
V
0
2
0
ln

(
(
(
(

=
(6.17)
b) Cuando hay recirculacin,
( )H Qr Q
H kA
S
S
V
n
V
m
+
(
(
(
(

0
2
ln
(6.18)
R
S R S
S
o
m
+
+
=
1
2
(6.19),
o
r
Q
Q
R =
(6.20)
Sin embargo, la mayora de las aguas residuales contienen sustratos com-
plejos con diferentes velocidades de remocin. Esto hace necesario utilizar
una forma retardada de la ecuacin que describe la remocin global del
216
Menndez Gutirrez, C. y J. Prez Olmo
proceso. Esta ecuacin retardada puede ser obtenida del modelo de reac-
cin de segundo orden como sigue:
2
2
S k
dt
dS
=
(6.21)
Asumiendo el comportamiento de flujo a pistn:
t S k S
S
o o
+
=
2
2
1
1
(6.22)
Utilizando el mismo procedimiento que para el modelo de primer orden:
n
v
t
o
c H A o k S S
S

+
=
20
20 0
2
1
1
(6.23)
El exponente n vara en el intervalo comprendido entre 0,85 y 1,25 en depen-
dencia del medio de empaquetadura empleado.
Para el modelo de segundo orden el volumen de la empaque se calcula
segn:
a) Sin recirculacin,
( ) H
o
Q
n
H
V
kA
o
S
S
S
o
S
V

(
(
(
(
(


=
2
2
(6.24)
b) Con recirculacin,
( )
H
r
Q Q
n
H
V
kA
.
S
S
S
m
S
V +

(
(
(
(
(


=
0
0
2
2
(6.25)
Ejemplo 6.1
Se desea tratar 3217 m
3
d
1
de un agua residual industrial con una DBO de
850 mgL
1
hasta obtener un efluente de 280 mgL
1
. Se emplear una torre
de 6 m de altura cuyo empaque tiene un rea especfica de 100 m
2
m
3
.
217
Filtros Percoladores
Det ermine la car ga hidrulica, r ea de columna y r elacin de
recirculacin. La mxima DBO que debe existir a la entrada del filtro
percolador es de 570 mgL
1
.
Considere que se cumple el modelo de primer orden y,
k = 0,008 md
1
n = 0,5
t = 28 C
Clculo de R,
570
1
280 850
1
=
+
+
=
+
+
=
R
R
R
S R
o
S
m
S
R = 0,96 ~ 1,0
Clculo de la carga hidrulica, v,
( )
n
v
m
c H A k
S
S

= exp
( )
5 0
100 008 0 exp
570
280
,
c H ,

=
, n = 44 m
3
m
2
d
1
rea,
23 146
44
3217 1 3217
,
.
c
o
Q R
o
Q
A =
+
=
+
=
A = 146,23 m
2
Volumen de empaque, V = HA = 877,38 m
3
Carga orgnica,
( )
18 4
10
3
,
V
Q Q S
B
r o m
v
=
+
=

kgm
3
d
1
Los valores de carga hidrulica y orgnica chequean para el tipo de filtro
deseado.
Ejemplo 6.2
Determine el volumen de relleno necesario para tratar 1200 m
3
d
1
de un
agua residual cuya DBO
5
es de 200 gm
3
si se desea obtener 90 % de
remocin.
Considere el modelo de primer orden, empaque plstico,
218
Menndez Gutirrez, C. y J. Prez Olmo
t = 26 C H = 3,0 m A
v
= 150 m
2
m
3
n = 0,5
k
20 C
= 0,071 md
1
Asumiendo que ( )
20
20
047 1

=
t
t
, k k
Se tendr que k
26C
= 0,01
a) Sin recirculacin,
( ) H Q
n
H kA
S
S
V
V
0
0
ln

(
(
(

=
( ) 0 3 1200
0 3 150 01 0
20
200
ln
5 0
,
, ,
V
,

(
(
(

V = 940 m
3
Si H = 3 m,
entonces el dimetro de filtro a utilizar ser: 20 m
Comprobando los valores de carga hidrulica y carga orgnica:
Carga hidrulica:
8 3
313
1200
0
,
A
Q
c = = =
m
3
m
2
d
1
Carga orgnica:
940
10 1200 200
3
0 0


= =
V
Q S
B
V
B
V
= 0,25 kgm
3
d
1
Aun cuando el valor de la carga hidrulica est dentro del intervalo reco-
mendado para filtros sin recirculacin, la carga orgnica tiene un valor algo
superior al recomendado. No obstante, podra considerarse que no es necesario
el uso de la recirculacin a menos que se desee una mayor flexibilidad en la
operacin. Hoy da lo ms comn, aun en casos como este, es concebir la
recirculacin para seguridad de la operacin y obtener una mayor flexibilidad de
trabajo.
Adems de la ventaja que ofrece la recirculacin en cuanto a evitar tupiciones
en el filtro, puede citarse la de la posibilidad de utilizar mayores alturas de filtro.
Esta ventaja puede hacerse ms evidente en la medida que el material utilizado
en el empaque sea ms ligero.
219
Filtros Percoladores
b) Con recirculacin,
( )H Q Q
n
H kA
S
S
V
r
V
m
+
(
(
(

0
ln
Si R = 0,5 S
m
= 140 mgL
1
( ) 0 3 600 1200
0 3 150 01 0
20
140
ln
5 0
,
, ,
V
,
+
(
(
(

V = 1009 m
3
Si H = 3,0 m, entonces el dimetro de filtro a utilizar ser: 21 m
Comprobando los valores de carga hidrulica y carga orgnica:
Carga hidrulica:
3 5
336
1800
0
,
A
Qr Q
c = =
+
=
m
3
m
2
d
1
Carga orgnica:
( )
1009
10 1800 140
3
0


=
+
=
V
Qr Q S
B
m
v
B
V
= 0,25 kgm
3
d
1
Tal como se aprecia, los valores de carga hidrulica y carga orgnica obte-
nidos coinciden ahora con los recomendados para un filtro de velocidad media.
Si se incrementara la altura de empaquetadura, se reduce el rea y como
consecuencia aumenta la carga hidrulica sobre el filtro.
6.7.1.1. Relacin volumen - altura
La relacin entre la altura de la empaquetadura en el filtro y el volumen que
ocupa esta es funcin del valor de n. Figura 6.12. En la tabla 6.1 se ofrece el
factor por el que se afecta el volumen de la empaquetadura al aumentar la
profundidad para distintos valores de n.
220
Menndez Gutirrez, C. y J. Prez Olmo
Tabla 6.1. Factor por el que se afecta el volumen del filtro al aumentar la
altura H, para 5 valores de n
ALTURA
Fig. 6.12. Variacin del volumen de empaque con la profundidad para
distintos valores de n.
6.7.1.2. Efecto de la recirculacin
Es importante destacar el efecto de la recirculacin sobre la eficiencia de
depuracin as como sobre las cargas hidrulica y orgnica del filtro. Estos efectos
pueden apreciarse en las figuras 6.13 y 6.14.
La eficiencia del filtro, disminuye al aumentar la recirculacin, cuando esta
es medida con respecto a la DBO de entrada (S
m
) segn la figura 6.9. Sin
embargo, cuando se calcula sobre la base de S
0
, se mantiene constante, inde-
pendientemente del flujo de recirculacin. Figura 6.13.
Por otro lado, tal como se observa en la figura 6.14, el efecto de la
recirculacin sobre la carga hidrulica es ms notable que sobre la carga
orgnica. Esto se debe a que en esta ltima el incremento del flujo de entra-
da es compensado en parte por la disminucin de la concentracin que ex-
perimenta la DBO a la entrada.
n
1,5 H 2 H 3 H
0,4 0,54 V 0,35 V 0,19 V
0,5 0,67 V 0,5 V 0,33 V
0,7 0,84 V 0,75 V 0,62 V
0,8 0,91 V 0,83 V O,75 V
1 V V V
221
Filtros Percoladores
Fig. 6.13. Efecto de la recirculacin sobre la eficiencia. 1. Calculada a
partir de S
0
. 2. Calculada a partir de S
m
.
En la tabla 6.2 se destaca el efecto de la recirculacin para un caso particular.
Fig. 6.14. Efecto de la recirculacin sobre las cargas hidrulica y orgnica.
Tabla 6.2. Eficiencia, carga hidrulica y orgnica para distintos valores de R
Q
0
= 4500 m
3
d
-1
H = 2 m A
V
= 72 m
2
m
-3
S
0
= 300 mgL
-1
k = 0,02 V = 3905 m
3
R Ef/S
0
Ef/S
m
B
V
kgm
3
d
-1
u
m
3
m
-2
d
-1
0 85,0 85,0 0,34 2,30
0,5 84,7 78,0 0,37 3,46
1,0 84,9 74,0 0,40 4,61
1,5 85,3 69,8 0,42 5,76
2,0 85,6 66,5 0,44 6,92
2,5 86,0 63,8 0,47 8,07
222
Menndez Gutirrez, C. y J. Prez Olmo
6.7.2. Modelo del NRC
Ms que un modelo que pretenda explicar la respuesta de un proceso
ante la incidencia de diferentes factores, el mtodo del NRC trata de un
sistema de clculo.
Este mtodo es el resultado de un estudio realizado en la dcada de los
aos cuarenta del siglo pasado en bases militares en Estados Unidos de
Norteamrica. En el mtodo son obviados algunos parmetros como el rea
especfica de la empaquetadura y la influencia de la carga hidrulica en la
eficiencia de la depuracin.
De acuerdo con el NRC, el volumen de la empaquetadura de un filtro
percolador puede calcularse segn,
F
W
E
E ,
V
2
100
443 0
(

=
(6.26)
donde:
V: volumen m
3
.
W: carga de DBO kgd
1
.
E: eficiencia de depuracin %.
F: factor asociado a la recirculacin,
2
10
1
1
|
.
|

\
|
+
+
=
R
R
F
(6.27);
0
Q
Qr
R =
Ejemplo 6.3
Estime el volumen de empaque necesario para tratar 2500 m
3
d
1
de un
agua residual urbana cuya concentracin de DBO
5
es 250 mgL
1
si se re-
quiere un efluente con una DBO de 25 mgL
1
. Considere una altura de em-
paque de 2,0 m.
E = 80 %
Asumiendo inicialmente R = 0, F = 1
W = 250 2500 10
3
= 625 kgm
3
d
1
F
W
E
E ,
V
2
100
443 0
(

=
223
Filtros Percoladores
3
2
m 5 1962
1
625
80 100
80 443 0
,
,
V =
(

=
Si H = 2,0 m, entonces el rea ser,
A = 981,25 m
2
Verificando los valores de carga hidrulica y carga orgnica:
55 2
25 981
2500
,
,
c = =
m
3
m
2
d
1
32 0
5 1962
625
,
,
B
V
= =
kgm
3
d
1
La carga hidrulica est dentro del intervalo que corresponde a los siste-
mas sin recirculacin. Sin embargo, el valor de la carga orgnica es superior
al valor mximo de 0,2 kgm
3
d
1
recomendado para esta variante de R = 0.
Asumiendo R = 1, F = 1,65
3
2
m 1189
65 1
625
80 100
80 443 0
=
(

=
,
,
V
Si H = 2,0 m, entonces el rea ser,
A = 594,5 m
2
Verificando los valores de carga hidrulica y carga orgnica:
41 8
5 594
5000
,
,
c = =
m
3
m
2
d
1
52 0
1189
625
, B
V
= =
kgm
3
d
1
Ahora tanto la carga hidrulica como la orgnica se corresponden con los
valores del intervalo para filtros de velocidad media. Por lo tanto, el volumen a
utilizar de empaque es de 1189 m
3
y R = 1.
224
Menndez Gutirrez, C. y J. Prez Olmo
6.8. CLCULO DE LAS CONSTANTES DEL MODELO
DE PRIMER ORDEN
6.8.1. Determinacin de n y k a temperatura constante
La mayor dificultad en el uso del modelo de primer orden para el diseo de
filtros percoladores radica en la necesidad de conocer los valores de los
parmetros n y k. En el caso de n, esta informacin puede obtenerse de las
firmas que ofertan el material de empaque. El valor de k puede conocerse de
diferentes fuentes bibliogrficas.
Por otro lado, los valores de k y n pueden ser calculados en pruebas realiza-
das en plantas piloto o semipiloto.
En la figura 6.15 se presenta el diagrama de una instalacin tipo semipiloto
de filtro percolador.
Fig. 6.15. Filtro percolador semipiloto.
Esta planta puede operarse tanto con empaque de piedra como de material
plstico. Este ltimo permite mayor profundidad de empaque.
El procedimiento consiste en realizar pruebas a varios valores de carga
hidrulica y diferentes alturas de empaque. Para ello el filtro debe tener
previsto tomar muestras a diferentes profundidades. Generalmente se utili-
225
Filtros Percoladores
zan entre 3 y 5 valores diferentes de carga hidrulica, en un intervalo que
depender del tipo de empaque disponible, y 4 alturas o profundidad de em-
paque.
Una de las mayores dificultades para la realizacin de estas pruebas se
encuentra en la formacin del limo. Esta se puede lograr en un trmino de 3 a 4
semanas. Debe conocerse de antemano el rea especfica (A
V
) del material que
se utilizar como empaque.
Procedimiento
1. Para la puesta en marcha de la instalacin, con la salida del filtro cerrada, se
alimenta una vez al da el agua residual que se desea tratar, y se recircula
durante 24 horas.
2. Transcurridas 24 horas se extrae el agua residual y se alimenta una nue-
va carga. Esta operacin se repite de 2 a 3 semanas. Se sugiere que
la carga hidrulica sobre el filtro en esta etapa no debe ser mayor
de 4 m
3
m
2
d
1
. Si en ese periodo se logra la formacin de limo sobre el
empaque, se debe cerrar la recirculacin e iniciar la operacin normal del
filtro a la menor carga hidrulica de las que se seleccione para trabajar
en las siguientes etapas.
3. Determinar los valores de carga hidrulica de trabajo as como las alturas a
las que se tomarn las muestras.
4. Para cada valor de carga hidrulica se tomarn muestras a las diferentes
alturas.
5. Los anlisis de las muestras se expresan en por ciento de DBO remanente
(S/S
0
100).
Para cada valor de carga hidrulica a las que se realizan las experiencias, y
teniendo en cuenta la ecuacin:
n
Hc
V
kA
e
S
S

=
0
, (6.27)
se representa en papel semilogartmico el % de DBO remanente en funcin de
la profundidad, H:
H
n
c
V
kA
n
Hc kA
S
S
V |
.
|

\
|
=

=
|
|
.
|

\
|
0
ln
(6.28)
De esta manera puede obtenerse una familia de rectas, dependiendo de la
cantidad de cargas hidrulicas seleccionadas, y cuya pendiente en cada caso
ser, en valor absoluto, kA
V
u
n
. Figura 6.16 del ejemplo 6.4.
226
Menndez Gutirrez, C. y J. Prez Olmo
7. En un grfico doble logartmico (log-log) se representan los valores absolutos
de las pendientes de las rectas obtenidas en el paso 6, (kA
V
u
n
), en funcin
de la carga hidrulica que le dio origen:
u log log pendiente log n kA
V
= (6.29)
De la pendiente de la recta correspondiente se obtiene el valor de n.
8. El valor de k se obtiene de la ordenada, considerando como valor de origen,
el valor de abscisa correspondiente a la menor carga hidrulica. Figura 6.17.
El valor de la ordenada, en el punto tomado como origen es igual a kA
V
u
n
.
Del valor de n as como del valor de la menor carga hidrulica de las utiliza-
das, puede obtenerse la constante k.
Ejemplo 6.4
Para determinar los valores de los parmetros n y k se opera una instalacin
semipiloto con un material sinttico de empaque cuya rea especfica es
de 200 m
2
m
3
. Las pruebas se realizaron a 5 cargas hidrulicas diferentes, to-
mando muestras a 4 profundidades de filtro. Las mediciones de DBO en cada
caso arrojaron los resultados que aparecen en la tabla 6.1, reportados como
S/S
0
. Calcule n y k.
Tabla 6.3. Valores de S/S
0
100 a diferentes cargas hidrulicas y alturas
Ejemplo 6.4.
1. Para cada carga hidrulica se grafica el % DBO remanente en funcin de la
profundidad,
H
n
c
V
kA
S
S
|
.
|

\
|
=
|
|
.
|

\
|
0
ln
obteniendo la familia de rectas de la figura 6.16. Al calcular la pendiente
(-kA
V
u
n
) de cada una de las rectas,
Carga hidrulica m
3
m
-2
d
-1
Altura m 15 27 36 46 70
1,5 70 64 62 58 48
3,0 49 41 38 33 23
4,5 33 26,5 23 19 11
6,0 23 17 14 11
227
Filtros Percoladores
Fig. 6.16. % de DBO remanente en funcin de la profundidad.
2. En log-log se grafica el valor absoluto de las pendientes en funcin de la
carga hidrulica. Figura 6.17.
logu log pendiente log n kA
V
=
La pendiente de esta recta es la constante n = 0,44
3. Para obtener el valor de k: se toma la ordenada que corresponde con u = 15
m
3
m
2
d
1
, que al ser la menor de las cargas hidrulicas, representa el origen,
kA
V
u
n
= 0,49
como: u
n
= 15
0,44
= 0,30 y A
V
= 200, entonces,
k = 0,008 md
1
u m
3
m
-2
d
-1
15 27 36 46 70
Pendiente (kA
V
u
-n
) -0,490 -0,370 -0,330 -0,295 -0,247
Tabla 6.3.a. Valor de la pendiente a diferentes cargas hidrulicas
100
0

S
S
228
Menndez Gutirrez, C. y J. Prez Olmo
Fig. 6.17. Representacin grfica de kA
V
H en funcin de u.
6.8.2. Clculo de la dependencia de k con la temperatura
En la ecuacin 6.16 qued expresada la relacin que existe entre el valor de
k a una temperatura de referencia, 20 C, y su valor a cualquier otra tempera-
tura. Esta relacin permite que pueda conocerse el valor de las constantes de un
filtro percolador a cualquier temperatura, y a partir de l, determinarla a una
temperatura de inters.
En este epgrafe se describe, mediante el ejemplo 6.5 un procedimiento para
calcular, experimentalmente el valor del parmetro
0
de la ecuacin 6.16. En el
propio ejemplo se aplica una variante del procedimiento descrito en el epgrafe
6.8.1 para el clculo de k y n cuando se dispone de informacin de una sola
profundidad de empaque.
Ejemplo 6.5
Los datos de la tabla 6.4 fueron obtenidos para tres temperaturas diferentes,
20 C, 25 C, y 30 C.
Determine los parmetros n y k
t
, para el modelo de primer orden, as como
una expresin que permita conocer la influencia de la temperatura sobre la efi-
ciencia de depuracin. Profundidad del filtro 1,83 m; A
V
= 72 m
3
m
2
d
1
.
R = 0
229
Filtros Percoladores
La ecuacin (6.16) puede escribirse:
c n H A k
S
S
V t
ln ln ln ln
2
0
=
|
|
.
|

\
|
(6.30)
graficando
|
|
.
|

\
|
2
0
ln ln
S
S
en funcin de lnv ha de obtenerse una lnea recta cuya
pendiente es el parmetro n y su intercepto con el eje de ordenadas k
t
A
V
H.
En la figura 6.18 se muestra, a modo de ejemplo, la recta correspondiente a
la temperatura de 20 C. Para las restantes 2 temperaturas se procede de ma-
nera similar.
Tabla 6.4. Modelo cintico de primer orden
No. Temp C
S
0
(mgL
-1
)
S
2
(mgL
-1
)
u
(m
3
m
-2
d
-1
)
Ln u
1 44 2,176 0,777429
2 64 4,901 1,589386
3 87 6,529 1,876321
4
20 213
102 8,705 2,163933
5 26 2,176 0,777429
6 55 4,901 1,589386
7 61 6,529 1,876321
8 72 8,705 2,163933
9 96 10,881 2,387035
10 95 13,602 2,610246
11 107 16,322 2,792500
12
25 197
112 21,762 3,080182
13 22 2,176 0,777429
14 42 4,901 1,589386
15 49 6,529 1,876321
16 64 8,705 2,163933
17 88 10,881 2,387035
18 110 13,602 2,610246
19 114 16,322 2,792500
20
30 212
129 21,762 3,080182
Cuando se grafica
|
|
.
|

\
|
2
0
ln ln
S
S
vs ln u a cada temperatura, se obtienen 3
juegos de valores de n y k
t
. Tabla 6.5.
230
Menndez Gutirrez, C. y J. Prez Olmo
Tabla 6.5. Procesando los datos
Valor promedio de n = 0,6
Con el valor promedio de n y aplicando la ecuacin 6.30, se recalculan los
valores de k
t
para cada uno de los puntos experimentales.
Temperatura C
n k
t
20 0,537 0,0189
25 0,579 0,0246
30 0,701 0,0351
Fig. 6.18. Ejemplo del clculo de n para la temperatura de 20 C.
Procesando los datos de la Tabla 6.4, para el modelo cintico de primer
orden, se obtienen las tablas 6.6 a 6.8.
Tabla 6.6. Procesando datos
Temp.
20 C
n = 0,6
|
|
.
|

\
|
2
0
ln ln
S
S
Valor de k
t
obtenido
considerando n = 0,6
y aplicando la ec. 6.30
1 0,4493926 0,0189655
2 0,1843271 0,0236824
3 -0,1105025 0,0209484
4 -0,3060914 0,0204717
promedio 0,0210170
231
Filtros Percoladores
Tabla 6.7. Procesando los datos
Tabla 6.8. Procesando los datos
Temp.
25 C
n = 0,6
|
|
.
|

\
|
2
0
ln ln
S
S
Valor de k
t
obtenido
considerando n = 0,6
y aplicando la ec. 6.30
5 0,7056226 0,0245044
6 0,2436287 0,0251293
7 0,1589931 0,0274279
8 0,0065163 0,0279839
9 - 0,3300949 0,0228488
10 - 0,3156333 0,0265038
11 - 0,4936819 0,0247442
12 - 0,5714521 0,0272058
promedio 0,0257240
De esa manera se obtiene que para el sistema bajo estudio:
n = 0,6 y los valores de k
t
obtenidos son:
0,0210170 a 20 C
0,0257240 a 25 C
0,0284677 a 30 C
Para calcular la influencia de la temperatura, se hace uso de una ecuacin
similar a la 4.33,
T
p a k
t
1
ln + =
Temp.
30 C
n = 0,6
|
|
.
|

\
|
2
0
ln ln
S
S
Valor de k
t
obtenido
considerando n = 0,6
y aplicando la ec. 6.30
13 0,8178148 0,0274138
14 0,4817572 0,0318858
15 0,3816955 0,0342697
16 0,1804057 0,0332994
17 - 0,1286866 0,0279469
18 - 0,4214331 0,0238429
19 - 0,4774105 0,0251501
20 - 0,6996203 0,0239331
Promedio 0,0284677
232
Menndez Gutirrez, C. y J. Prez Olmo
Tabla 6.9. Procesamiento de datos
Aplicando estos datos a la ecuacin anterior se obtiene,
T
, , k
t
1
9 2693 4497 12 ln =
para 2 temperaturas K diferentes, por ejemplo, 298 y T,
T
T
,
k
k
T
298
298
9 2693 ln
298

=
|
|
.
|

\
|
298 T ~ (298)
2
= 88 804, entonces,
( ) 298 030 0 ln
298
=
|
|
.
|

\
|
T ,
k
k
T
20
298
030 1

=
t
T
, k k
Notas bibliogrficas
1 PEDERSEN, K.: 'BioIilm development on stainless steel and PVC surIaces,
in drinking water, Water Res., vol. 24, no. 1, 1990.
2 LAZAROBA, V. AND MANEM: 'BioIilm characterization and activity analysis in
water and wastewater treatment, Water Res., vol. 29, no. 10, 1995.
3 ECKENFELDER, W.W. AND D.J. O`CONNOR: Biological waste Treatment, Per-
gamon Press Ltd., London, 1964.
4 VASEL, J. AND P. SCHROBITGEN: 'Oxygen transIer in trickling Iilters, Water
Res., vol. 25, no. 1, 1991.
5 FRUHEN, C.: 'Spatial distribution oI species in bioIilm, Water Sci. Tech-
nol., vol. 23, no. 8, 1991.
6 CHUDOBA, J., C. MENNDEZ Y J. PREZ: Fundamentos Tericos de Algunos
procesos para la depuracin de Aguas Residuales, Edit. ISPJAE,
Cuba, 1986.
Temperatura
Kelvin
1 /T k
t
ln k
t
293 0,003412969 0,021017 - 3,8624236
298 0,003355705 0,025724 - 3,6603309
303 0,003300330 0,028467 - 3,5589852
233
Filtros Percoladores
7 IMHOFF, K. W.J. MULLER, AND THISTLETHWAYTE: Disposal of Sewage and
other Waterborne Wastes, Edit. Butterworths, 1971.
8 HOWLAND, W.E.: 'Flow over porous media as in trickling Iilter, Proc. 12
th
Ann. Ind. Waste Conf. Purdue Univ., 1958.
9 BLOODGOOD, D.E., G.H. TELETZKE, AND F.G. POHLAND: 'Fundamental hydraulic
principles oI trickling Iilters, Sewage and Ind. Wastes, vol. 31, no. 3, 1959.
10 MEHTA, D.S., H.H. DAVIES, AND R.P. KINGSBURY: Oxygen theory in biolo-
gical treatment plant design, J. San. Eng. Div. ASCE, 98 (SA 3),
1972.
11 BRUCE, A.M. AND J.C. MERKENS: 'Further studies oI partial treatment oI
sewage by high-rate biological Iiltration, Water Poll. Control, vol.
72, no. 5, 1973.
12 SHRIVER, L.E. AND D.M. BOWERS: 'Operational practices to upgrade filter
plant perIormance, Jour. Water Poll. Contro Fed., vol. 47, no. 11,
1975.
13 SHULZE, K.L.: 'Load and eIIiciency oI trickling Iilters, Jour. Water Poll.
Control Fed., vol. 32, no. 3, 1960.
14 LAMB, R. A.: 'Suggested Iormula Ior the process oI biological Iiltration,
Water Poll. Control, vol. 69, no. 2, 1970.
15 WEF AND ASCE: 'Design oI municipal wastewater treatment plants,
vol. I. Water Environment Federation, Alexandria, 1992.
16 HAWKS, H.A.: The Ecology of Waste Treatment, Ed. Pergamon Press,
London, 1963.
17 CHANG, G. AND J. HUANG: 'EIIect oI bioIilm thickness distribution on subs-
trate inhibited kinetic, Water Res., vol. 28, no. 8, 1994.
18 JENKINS, D.: 'Sewage treatment. Cap. 15, en Biochemistry of Indus-
trial Microorganisms, Eds. Rainbow, C and A.H. Rose, Academic
Press, 1963.
19 BRUCE, A.M.: 'Percolating Iilters, Process Biochem., vol. 4, no. 4, 1963.
20 KORNEGAY, B.H. AND J.F. ANDREWS: 'Kinetics oI Iixed Iilm biological reac-
tors, Jour. Water Poll. Control Fed., vol. 40, no. 2, 1968.
21 LAMOTTA, E.J.: 'Internal diIIussion and reaction in biological Iilms, Env.
Sci. Technol., vol. 10, no. 8, 1976.
22 WILLIAMS, I.L.: 'Water Poll. Control, vol. 78, no. 2, 1979.
23 HALVORSON, H.O., G.M.SAVAGE AND E.L. PIRET: 'Some Iundamental Iac-
tors concerned in the operation oI trickling Iilters, Sewage Works J.,
vol. 8, no. 6, 1936.
234
Menndez Gutirrez, C. y J. Prez Olmo
24 ECKENFELDER, W.W.: Industrial Water Pollution Control, Mc Graw Hill,
1989.
25 VELZ, C.J.: 'A basic law oI perIormance oI biological beds, Sewage
Works J., vol. 20, no. 4, 1948.
26 'National Research Council, Trickling Filters (Treatment at Military Ins-
tallations), Sewage Works J., vol. 18, no. 5, 1948.
27 BENJES, H.H.: Handbook of Biological Wastewater Treatment, Gar-
land STPM Press, New York, 1980.
28 PORTER, E.K. AND E. SMITH: 'Plastic media biological Iilters, Water Poll.
Control, vol. 78, no. 2, 1979.
235
Lagunas de Estabilizacin
CAPTULO 7
LAGUNAS DE ESTABILIZACIN
7.1. CARACTERSTICAS GENERALES
El empleo de lagunas de estabilizacin para el tratamiento de aguas residuales
que contengan compuestos orgnicos biodegradables est muy difundido.
Las lagunas son estructuras simples de fcil operacin y mantenimiento que
se basan en el proceso de autopurificacin. Generalmente estn constituidas
por embalses naturales o artificiales, en tierra, expuestos al aire y al sol, por lo
que las condiciones climticas influyen significativamente en el funcionamiento
de este dispositivo de tratamiento. Por esta razn el diseo de las lagunas es
posiblemente, de todos los procesos de tratamiento biolgico, el menos definido.
De acuerdo a la naturaleza de la actividad biolgica que tiene lugar en la
misma, y por tanto al metabolismo que prevalece durante su funcionamiento, las
lagunas de estabilizacin se clasifican en:
Aerobias.
Anaerobias.
Facultativas.
7.2. LAGUNAS AEROBIAS
Bajo esta denominacin se incluyen aquellas lagunas en las que los com-
puestos biodegradables, sean estos suspendidos o disueltos, son estabilizados
por la accin de microorganismos aerobios. El oxgeno es suministrado por la
accin fotosinttica de las algas y por el que se difunde del aire. Debido a que la
luz solar es esencial para la produccin de oxgeno mediante algas, la profundi-
dad de estas lagunas est limitada de acuerdo con la penetracin de la luz, y
normalmente es menor de 50 cm. En el caso de aguas residuales domsticas
tpicas y algunos residuales industriales, se producen aproximadamente
entre 0,5 y 0,6 kg de biomasa por kilogramo de DBO removido.
Aunque estas lagunas admiten cargas de DBO relativamente altas, su poca
profundidad hace que se requieran grandes reas para su construccin y hoy da
tienen uso limitado.
236
Menndez Gutirrez, C. y J. Prez Olmo
Las principales reacciones que pueden llevarse a cabo durante la estabiliza-
cin en este tipo de lagunas son:
degradacin aerobia
O H 28 16CO NO H C 4 4NH O 16 O H C 6
2 2 2 7 5 3 2 6 12 6
+ + + + (7.1)
a su vez, las algas sintetizan materia orgnica que incorporan a su propio
protoplasma.
fotosntesis
2 3 12 8 2 2 3
O 8,75 N O H C O H 4,5 CO 8 NH + + + (7.2)
nitrificacin


3 2 3
NO NO NH org N
(7.3)
7.3. LAGUNAS ANAEROBIAS
Las lagunas anaerobias se construyen preferentemente para reducir la car-
ga orgnica sedimentable.
En las lagunas de este tipo, la materia orgnica es estabilizada mediante un
mecanismo similar al que existe en los tanques de digestin anaerobios (forma-
cin de cidos orgnicos y de metano).
En las condiciones de climas tropicales la carga de estas lagunas puede
variar entre 60 y 300 g DBO m
2
d
1
. La profundidad est limitada por elemen-
tos prcticos: nivel fretico, tipo de suelo y facilidades para la limpieza eventual
de los lodos o fangos que se depositan en el fondo con el transcurso del tiempo.
Las ecuaciones simplificadas que representan las transformaciones que tie-
nen lugar en una laguna anaerobia son:
( )
COO HCH X O CH
3 X 2
(7.4)
2 4 3
CO CH COO HCH + (7.5)
+

4
NH org N
(7.6)
7.4. LAGUNAS FACULTATIVAS
En las lagunas facultativas se distinguen dos zonas de trabajo bien dife-
renciadas: una regin aerobia en la superficie y cercana a esta y una regin
anaerobia en el fondo. Entre ambas existe una zona, no muy bien delimitada,
facultativa. Figura 7.1.
237
Lagunas de Estabilizacin
Una laguna de estabilizacin facultativa es una estructura simple de relativa
poca profundidad (1 a 3 m) y con periodos de retencin entre 1 y 40 das.
Las lagunas facultativas se caracterizan porque:
ocurre la autodepuracin o estabilizacin natural,
la DBO es el parmetro que se utiliza para evaluar las condiciones de la
laguna,
para cargas orgnicas (CO) bajas 5-20g DBO
5
m
2
d
1
y temperaturas
entre 20 y 30 C, el estrato superior se cubre de algas microscpicas
(clorellas y euglenas), llegando el agua a sobresaturarse de oxgeno y
la penetracin de la luz es limitada (5 a 15 cm) y el estrato inferior es
anaerobio.
Fig. 7.1. a) Esquema de los mecanismos responsables de la depuracin
en lagunas facultativas. b) Simbiosis alga-bacteria en las lagunas.
7.1.b
238
Menndez Gutirrez, C. y J. Prez Olmo
No existe un lmite bien definido de carga para garantizar si una laguna es
facultativa o anaerobia. Para temperaturas entre 15 y 30 C hay una zona de
transicin entre 30 y 60 gm
2
d
1
.
En Cuba hay centrales azucareros con refinera que en pocas en que no
hay zafra la carga de las lagunas es de 45 gm
2
d
1
y son anaerobias.
1
7.4.1. Factores que afectan el proceso depurador de las lagunas
pH,
luz solar,
temperatura,
nutrientes y sustancias txicas y
La influencia de la luz solar est relacionada con la fotosntesis.
pH
Producto de la respiracin bacteriana se libera CO
2
, que tiende a bajar el
pH. Por otro lado, la accin fotosinttica provoca consumo del CO
2
con el con-
siguiente aumento del pH. Por tanto el pH esta sujeto a variaciones horarias que
se relacionan con la energa luminosa incidente.
2
Radiacin Solar
Segn Thirumurthi, para un buen funcionamiento de las lagunas facultativas
se requiere una radiacin solar mnima de 4 langleysd
1
(4 cal cm
2
d
1
) por cada
(g DBQ aplicadom
2
) (luz visible).
3
De acuerdo con Eckenfelder,
4
este valor es
de 9 langleys.d
1
. Ildeu Duarte, citado por Senz, reporta el valor de 6 langleysd

1
de radiacin visible como valor de radiacin mnima.
5
En Cuba, los meses de
menor radiacin son diciembre y enero, con un valor mnimo calculado como
promedio de 250 cal cm
2
d
1
.
6
Esta es una de las razones de las excelentes
condiciones de los pases tropicales para el funcionamiento de las lagunas facul-
tativas.
No toda la energa solar que llega a la laguna es utilizada. Despus de
un cierto valor de intensidad de luz donde las algas se saturan, el resto no
es utilizado excepto para elevar y mantener la temperatura. As, por ejem-
plo, el valor de saturacin estimado para un cierto tipo de clorella
5
es
de 60 cal cm
2
d
1
.
Debido a que la intensidad de la luz vara a lo largo del da y de acuerdo a la
poca del ao, la velocidad de crecimiento de las algas vara tambin de la
misma forma. Este fenmeno da lugar a dos efectos fundamentales: el oxgeno
239
Lagunas de Estabilizacin
disuelto y el pH del agua presentan valores mnimos en las ltimas horas de la
noche, y aumentan durante las horas de luz solar hasta alcanzar valores mxi-
mos a media tarde. A partir de ese momento, los valores decrecen al arribar
nuevamente a la noche.
2
En la figura 7.2 se refleja esta variacin cclica del
oxgeno y el pH.
Fig. 7.2. Variacin diaria de oxgeno y pH.
Para lagunas aerobias, la produccin de O
2
por algas, est relacionada a la
eficiencia de conversin de la luz y de su intensidad. As,
O
2
= 0.28 F I
L
(7.7)
O
2
: produccin de O
2
.
F: eficiencia de conversin de la luz (generalmente se toma como 4%).
I
L
: intensidad de luz (cal cm
2
d
1
) (langley).
Para estas lagunas la carga orgnica permisible del afluente en base a DBO,
(gm
2
d
1
) puede relacionarse con la intensidad de la luz. La carga hidrulica se
limita a 5 25 cm.d
1
. La carga orgnica superficial (CO) que se recomienda
debe ser menor de 25 g m
2
d
1
. La mxima produccin de algas se obtiene
cuando la profundidad no rebasa el intervalo comprendido entre 15 y 30 cm. Si
lo que se busca es produccin de O
2
,
las profundidades deseables no deben ser
mayores de 1 a 1.3 m.
La produccin de algas se estima:
Ac=0.017 F. I
L
(7.8)
Ac-produccin de algas (g m
2
d
1
)
240
Menndez Gutirrez, C. y J. Prez Olmo
Temperatura
Como ocurre en todos los procesos biolgicos, la temperatura ejerce una
influencia marcada sobre las lagunas. En general, y para los intervalos de tem-
peratura normales en las lagunas, se puede decir que la velocidad de la depura-
cin aumenta con la temperatura, en especial en lo que concierne a la actividad
de las bacterias.
Sin embargo, y en lo que respecta a las algas, se han detectado disminucio-
nes importantes en su actividad fotosinttica a temperaturas superiores a 30 C,
relacionadas con la estimulacin del crecimiento de algas menos productivas
(verdiazules) que las algas verdes (clorofceas) a las que sustituyen. Puesto que
este fenmeno coincide con una gran actividad de las bacterias, y por tanto,
grandes consumos de oxgeno, puede ampliarse la zona anaerobia en las lagunas
facultativas durante la poca de calor, y muy especialmente si el calentamiento
se produce de forma brusca. Normalmente esta situacin es solo transitoria y
las lagunas vuelven a funcionar correctamente en la medida que la temperatura
disminuye.
7.5. RGIMEN DE FLUJO EN LAS LAGUNAS
Uno de los aspectos ms controvertidos en las lagunas lo constituye el rgi-
men de flujo que prevalece en las mismas. Hay varias hiptesis:
a) mezcla completa,
b) flujo a pistn y
c) flujo disperso.
Las dos primeras hiptesis describen condiciones de flujo extremas e idea-
les. La tercera, una intermedia entre ambas, se ajusta a las condiciones de flujo
que realmente prevalecen en las lagunas. No obstante, condiciones muy cerca-
nas al comportamiento de flujo de pistn o tubular se han encontrado en lagunas
con una relacin largo ancho mayor de 3:1.
Ha sido demostrado que an cuando se sabe que el rgimen de flujo que
prevalece en las lagunas es el de flujo disperso, no se comete mucho error, a los
efectos de diseo, asumiendo un modelo de flujo de mezcla completa.
7
7.5.1. Lagunas en serie y paralelo
El mayor argumento para la construccin de una laguna grande est relacio-
nada con el beneficio que ocasionan los vientos. Sin embargo, hoy se reconocen
las ventajas de las bateras de lagunas, sean estas en serie o en paralelo.
241
Lagunas de Estabilizacin
En serie:
mejora la calidad bacteriolgica del agua,
se obtiene mayor economa del rea.
En paralelo:
no mejora la calidad del efluente pero trae ventajas en su operacin,
mejor para la puesta en operacin de las lagunas al ser estas ms peque-
as, ya que se llenan ms rpido y
mayor facilidad para la limpieza.
7.6. DISEO DE LAGUNAS DE ESTABILIZACIN
Numerosas experiencias realizadas durante ms de 40 aos han estimulado
los intentos para desarrollar modelos matemticos para el diseo de lagunas de
estabilizacin. Esto obedece al hecho ya expresado, de que el diseo de este
tipo de dispositivo de tratamiento es, de todos los procesos de tratamiento biol-
gico, el menos definido.
Entre los diferentes mtodos desarrollados para el diseo de lagunas pueden
citarse los empricos por carga superficial reportados por McGarry y Pescod,
8, 9
el
de Arceivala (WHO),
10
y el emprico volumtrico de Herman y Gloyna.
11
Otros
mtodos son el modelo racional de Marais
12
y el de flujo disperso de Thirumurthi.
2
Por su parte, Polprasert y Batharai
13
realizaron un aporte importante
correlacionando el nmero de dispersin con las formas geomtricas de la lagu-
na y propiedades del fluido, para facilitar el empleo de este ltimo modelo.
Ninguno de los mtodos de diseo de lagunas tiene una aceptacin univer-
sal. Existe mucha literatura en ese sentido, muchas veces contradictoria, que
puede inclusive llegar a confundir.
Para ilustrar la evolucin de los criterios de diseo para lagunas facultativas,
sern considerados, brevemente los temas desarrollados por Hermann y Gloyna,
emprico volumtrico, el de McGarry y Pescod, emprico superficial y por Marais
y Shaw, racional de primer orden.
7.6.1. Criterio americano
Dentro de ciertos lmites, la velocidad de las reacciones de degradacin
aumenta con el aumento de la temperatura. Si to es el tiempo de degradacin a
la temperatura To y T la temperatura para el tiempo t
T,
se tiene que:
T T
T
c
t
t

=
0
0
(7.9)
242
Menndez Gutirrez, C. y J. Prez Olmo
Aplicando la ecuacin anterior, Gloyna y col., realizando una serie de expe-
rimentos, concluyeron que, para obtener una reduccin de 80 a 90 % de la DBO
a partir de un afluente de 200 mg L
1
, se hace necesario:
a) tiempo de retencin de 7 das,
b) temperatura de 35
o
C,
c) considerar v = 1.085 y
d) aguas negras domsticas.
Teniendo en cuenta todo lo anterior, la ecuacin 7.9 se transforma:
T r
,
t

=
35
085 1
7
(7.10)
Por tanto, el tiempo de retencin, en das, necesario para una reduccin
de 80-90% de un afluente cualquiera de aguas domsticas de concentracin
y
0
ser:
T
T
,
y
t
y
= t

=
35 0 0
085 1
200
7
200
(7.11)
si
Q
V
t=
(7.12)
Sustituyendo 7.12 en 7.11:
( )
T
, y Q , V

=
35
0
2
085 1 10 5 3
(7.13)
donde:
V: Volumen de la laguna, m
3
.
Q: Flujo, m
3
d
1
.
y
o
: DBO afluente, mg L
1
.
T: Temperatura promedio del mes ms fro,
o
C.
7.6.2. McGarry y Pescod
Este mtodo destaca la importancia que tiene la temperatura sobre la carga
orgnica que es capaz de asimilar una laguna facultativa, expresada en funcin
del rea,
Carga orgnica mxima admisible:
( )
T
MAX
, , 0993 1 03 6 =
(7.14)

MAX
: gm
2
d
1
243
Lagunas de Estabilizacin
T: temperatura mnima del ao
Carga orgnica mnima recomendable:

MIN
= 2 T - 24 (7.15)

MIN
: gm
2
d
1
La carga orgnica de proyecto (), ha de cumplir la relacin,

MIN
< >
MAX
7.6.3. Mtodo racional. Marais y Shaw
Marais y Shaw basan su mtodo de diseo en la suposicin de que se logra
una mezcla instantnea entre el afluente y el volumen total de la laguna. De
hecho se supone un comportamiento de mezcla completa en las lagunas. Esta
suposicin se basa en los grandes volmenes que generalmente tienen las lagu-
nas, el relativo alto tiempo de retencin, y el efecto de mezcla proporcionado por
las brisas. De esta forma, haciendo un balance de DBO alrededor de la laguna
en estado estacionario (Figura 7.3), se obtiene que:
0 =
dt
dS
S + V Qo So - Qo
(7.16)
donde:
dt
dS
: Velocidad de degradacin de la DBO.
Q
0
: Flujo de agua a la laguna, m
3
d
1
.
S
0
, S: DBO de entrada y salida respectivamente, mgL
1
.
V: Volumen efectivo de la laguna, m
3
.
Fig. 7.3. Esquema para el balance de DBO en una laguna.
asumiendo una cintica de remocin de primer orden con respecto a la DBO,
S k
d t
dS
=
(7.17)
sustituyendo 7.17 en 7.16 y transformando se obtiene,
244
Menndez Gutirrez, C. y J. Prez Olmo
0 k S
S
+
=
1
1
0
(7.18)
donde:
S: DBO efluente (mg L
1
).
S
o
: DBO afluente (mg L
1
).
k: Constante de la reaccin de primer orden (d
1
).
u: Tiempo de retencin hidrulico (d).
Para lagunas en serie, la ecuacin de Marais puede representarse segn,
( )( ) ( ) n n
n
0 k .... 0 k 0 k
S
S
+ + +
=
1 1 1 2
2 1 1
0
(7.19)
De acuerdo a la ecuacin 7.17, se ha supuesto que la cintica de remocin
de contaminantes en la laguna es de primer orden.
La relacin de las constantes de degradacin a distintas temperaturas es
igual a la relacin de los tiempos de retencin
t
0
0
t
0
t
k
k

= =
35
35
35
(7.20)
La ecuacin 7.18 puede escribirse:
0 k
S
S
o
+ =1
(7.21)
Cuando a escala experimental se opera una laguna a distintos tiempos de
retencin (u), la constante de remocin puede calcularse como la pendiente
de la ecuacin 7.21. Figura 7.4.
Generalmente el empleo de la ecuacin de primer orden de Marais y Shaw
arroja como resultado lagunas con sobre diseo. Sin embargo esto no ha invali-
dado su empleo.
7.6.3.1. La constante k
Como en toda constante cintica, el valor de k depende, de entre otros fac-
tores, de la temperatura.
Cuando no se conoce el verdadero valor de la constante de reaccin, un
criterio conservador recomendado por Marais es el de 1,2 d
1
a 35 C. Marais
12
recomienda asumir este valor, debido a que para tiempos de retencin hasta de
245
Lagunas de Estabilizacin
40 das, una constante con ese valor corresponde a reducciones de la DBO
inferiores a las observadas experimentalmente en lagunas primarias.
35
35

=
t
t
c k k
(7.22)
v ~ 1,085
Fig. 7.4. Clculo de la constante de remocin k.
7.6.3.2. Influencia del tiempo de retencin en la eficiencia
de remocin
El tiempo de retencin influye en la eficiencia de las lagunas. Sin em-
bargo, a partir de un determinado valor, un incremento de ste influye poco
en la remocin de DBO. En la tabla 7.1 se aprecia que, independientemen-
te de la temperatura de trabajo, el % de remocin tiene un lmite, y este se
alcanza a un menor tiempo de retencin en la medida que la temperatura
es mayor.
En la prctica no deben esperarse las eficiencias de la tabla 7.1, ya que
las condiciones para las que se estimaron presuponen condiciones ideales:
mezcla instantnea y completa del afluente,
pocos slidos sedimentables,
pequeas variaciones de temperatura,
predominio de condiciones aerobias y
baja DBO en el fondo.
246
Menndez Gutirrez, C. y J. Prez Olmo
Tabla 7.1. Eficiencia de remocin terica de DBO para distintos tiempos de
retencin y valores de k (segn la temperatura)
Ejemplo 7.1
Dimensione una laguna facultativa para tratar 560 m
3
d
1
de un agua resi-
dual de un proceso y cuya constante de velocidad de remocin es de 0,15 d
1
.
La DBO de entrada a la laguna es de 350 mg L
1
y la deseada en el efluente
debe ser menor de 50 mg L
1
.
Aplicando la ecuacin 7.18
0 ,
,
15 0 1
1
14 0
+
=
u = 41 d
Volumen = 560 41 = 22 960 m
3
Asumiendo una profundidad de 1,8 m,
rea de la laguna
2
756 12
8 1
960 22
m
,
A = = =
Comprobando la carga orgnica:
1 2
15
752 12 41
960 22 350

=

= d m g O C
por tanto la carga orgnica tiene un valor que corresponde al intervalo ca-
racterstico para las lagunas facultativas.
Ejemplo 7.2
Determine el rea total requerida para las condiciones del ejemplo anterior,
pero suponiendo 2 lagunas en serie.
Considere que k
1
= k
2
= 0,15 d
1
y u
1
= u
2
Para 2 lagunas en serie:
( )
2
15 0 1
1
14 0
0 ,
,
+
=
VALORES DE k (d
-1
)
t(d) 0.24 0.35 0.53 0.80 1.2
7 62.7 71.0 78.7 84.8 89.4
10 70.6 77.8 84.1 88.9 92.3
15 78.2 84.0 88.8 92.3 94.7
20 82.7 87.5 91.4 94.1 96.0
30 87.8 91.3 94.1 96.0 97.3
40 90.6 93.3 95.5 97.0 98.0
247
Lagunas de Estabilizacin
u = 11,15 d
Tiempo total de retencin para las 2 lagunas: 22,3 d
A
T
= A
1
+ A
2
= 3 468 + 3 468 = 6 936 m
2
Comprobando la carga orgnica
para la primera laguna
1 2
28
936 6 15 11
244 6 350

=

= d m g
.
CO
para la segunda laguna
1 2
d m g 14
936 6 11,15
244 6 175
CO

=

=
La carga orgnica de la primera laguna es algo alta, pero se puede ver
compensada con la de la segunda.
En la prctica, k
1
> k
2
, y como consecuencia de ello, no siempre es tan
evidente la ventaja del empleo de 2 o ms lagunas en serie para la remocin de
DBO. La mayor justificacin en el uso de este arreglo se encuentra en la remo-
cin de patgenos.
7.7. RELACIN REA: VOLUMEN PARA LAS LAGUNAS
La carga orgnica superficial de la laguna determina su rea en la superficie,
CO
S Q
A
S
=
(7.23)
Q: flujo de agua residual, m
3
d
1
.
S: DBO en el efluente, mgL
1
.
CO: carga orgnica, gm
2
d
1
.
Una vez determinada el rea en la superficie de la laguna, el rea en el
fondo depende de la inclinacin del talud, y la profundidad que se seleccione
para la laguna. Si la inclinacin es 1:P,
( ) ( )
2
2 2 H P a L H P . A A
S F
+ + =
(7.24)
donde:
H: profundidad efectiva.
L: longitud de la laguna en la superficie.
a: ancho en la superficie.
El volumen efectivo de la laguna ser,
( )
F S F S
A A A A
H
V + + =
3
(7.25)
248
Menndez Gutirrez, C. y J. Prez Olmo
7.8. REMOCIN DE PATGENOS
Una de las ventajas de las lagunas radica en la posibilidad de eliminacin de
patgenos. Mientras que la remocin de coliformes mediante otros tratamientos
biolgicos alcanza hasta 95 %, en las lagunas se aprecian frecuentemente valo-
res de este indicador hasta (99,999,999) %.
14
Las mayores remociones de patgenos en las lagunas se logran cuando se
est en presencia de altos tiempos de retencin, baja turbiedad, alto pH y bajos
valores de DBO.
Hoy da se reconoce que la incidencia de la radiacin solar puede ser un
elemento de consideracin en la muerte de las bacterias.
11
Sin embargo,
teniendo en cuenta que la penetracin de la luz en las lagunas solo alcanza
unos (10 a 15) cm y que en la superficie raras veces se encuentran altas
concentraciones de bacterias, la posibilidad de incidencia de la radiacin
solar sobre la reduccin de patgenos es mayor en la medida que es menor
la turbiedad, de manera que se favorezca la penetracin de la luz.
Altos valores de pH aceleran la muerte de las bacterias al exponerlas a un
medio hostil. Una laguna que opere normalmente puede llegar a tener valores de
pH entre 9 y 10 durante determinadas horas del da debido a la accin fotosinttica
de las algas.
12,13
Estos valores favorecen la reduccin de patgenos.
Las bajas concentraciones de DBO en las lagunas favorecen la muerte de
patgenos al carecer del sustrato necesario para su subsistencia. Es por ello
importante mantener concentraciones de DBO< 20 mgL
1
en las lagunas para
propiciar una disminucin apreciable de patgenos. El empleo de dos o ms
lagunas en serie tambin favorece este propsito.
El empleo de altos tiempos de retencin no est asociado solamente con
la obtencin de bajos valores de concentracin de DBO en el efluente, sino
con la intencin de favorecer la eliminacin de patgenos a travs de la
sedimentacin.
7.9. BALANCE HDRICO DE LAS LAGUNAS
Un aspecto importante de las lagunas de estabilizacin, no siempre tenido en
cuenta, es el balance hdrico.
El balance hdrico permite conocer si existe o no percolacin en el dispositi-
vo de tratamiento.
El balance de flujo en la laguna puede formularse,
Qe Qa (Pr Pc) (E Pe) (7.26)
Qe: Caudal efluente.
249
Lagunas de Estabilizacin
Qa: Caudal afluente.
Pr: Precipitacin sobre la laguna.
Pc: Infiltracin de agua hacia la laguna.
E: Evaporacin.
Pe: Prdidas por percolacin.
Las variables anteriores pueden trabajarse en m
3
d
1
.
La precipitacin y evaporacin en la laguna pueden medirse experimentalmente
o tener informacin de ellas a travs de estaciones meteorolgicas.
Aun considerando el mes ms critico del ao (el de menos lluvia, el nivel
fretico ms bajo, de mayor evaporacin), el valor de Qe tiene que ser positivo.
En aquellos casos en los que Qe sea negativo, una alternativa de solu-
cin consiste en impermeabilizar la laguna. Esto ltimo puede resultar ms
econmico si se emplean lagunas anaerobias primarias. Estas ltimas pue-
den tener 60 % del rea de una equivalente facultativa, lo cual es muy im-
portante desde el punto de vista hidrulico.
La evaporacin que ocurre en una laguna puede estimarse a travs de la
ecuacin de Meyer,
( )
|
.
|

\
|
+ =
16
1 15
v
H -V Va E
r (7.27)
E: Evaporacin mensual, mm.
Va: Presin de vapor del agua a la temperatura del agua, mmHg.
V: Presin de vapor del agua a la temperatura del aire, mmHg.
H
r
: % humedad relativa.
v: Velocidad del viento km.h
1
.
Por otro lado, en aquellos casos en los que se disponga de informacin nece-
saria, las prdidas por percolacin pueden ser estimadas aplicando la ley de
Darcy,
Q = P I A (7.28)
Q: Caudal que percola m
3
s
1
.
P: Permeabilidad del suelo m s
1
.
A: rea expuesta m
2
.
L
h
I=
gradiente hidrulico (7.29)
L: Distancia del fondo de la laguna al nivel fretico.
h: Distancia de la superficie del agua en la laguna al nivel fretico.
por tanto, la profundidad de la laguna es igual a (h L).
250
Menndez Gutirrez, C. y J. Prez Olmo
Cuando se emplea como impermeabilizante algn tipo de suelo (arcilla), por
supuesto de menos permeabilidad que el suelo original, los valores de L y h son:
L = espesor de la nueva capa de arcilla
h = L + la altura del agua en la laguna.
Ejemplo 7.3
Una laguna de estabilizacin de 10 000 m
2
va a ser construida en un
suelo cuya permeabilidad es de 10
6
m.seg
1
. El nivel de agua fretica se
encuentra 3 metros por debajo del nivel del fondo de la laguna. La mxima
altura del agua en la laguna ser de 1,5 m. Determine las prdidas por
percolacin que tendr la laguna en las condiciones anteriores, y en caso
que se sustituyan 40 cm de material del fondo por una arcilla con una per-
meabilidad de 10
10
m.seg
1
.
P.I.A Q =
Para el suelo natural:
17 3
3
5 9
,
,
L
h
I = = =
1 3 2 4 6
seg m 10 17 , 3 10 17 , 3 10

= = Q
Sustituyendo 40 cm de suelo con una capa de arcilla cuya permeabilidad es
10
10
mseg
1
,
251
Lagunas de Estabilizacin
75 4
4 0
9 1
,
,
,
I = =
1 3 6 4 10
seg m 10 75 , 4 10 75 , 4 10

= = Q
7.10. CRITERIOS PARA LA OPERACIN
Y MANTENIMIENTO DE LAS LAGUNAS
DE ESTABILIZACIN
No obstante la simplicidad de estos dispositivos de tratamiento, no estn
exentos de un conjunto de requisitos a tener en cuenta para su buena opera-
cin, as como medidas a considerar para su mantenimiento. Desconocer
esto es frecuentemente la causa por la que no se obtienen las eficiencias de
depuracin que deban esperarse.
7.10.1. Lagunas facultativas
Caractersticas de una buena operacin
Dos caractersticas muy fciles de observar y que son indicadoras de que la
laguna facultativa opera adecuadamente son:
Poseer oxgeno disuelto en toda la masa lquida alcanzada por la radiacin
solar.
El efluente presenta una coloracin verde intensa, parcialmente trans-
parente y sin slidos flotantes. Una coloracin ceniza claro indica que
la carga a la que est siendo sometida la laguna es mayor a la admitida
para mantener su condicin de facultativa.
Problemas frecuentes que pueden presentarse
Aparicin de natas. Puede deberse a:
flotacin del lodo de la laguna,
floracin de algas (super produccin de algas) y
exceso de detergentes.
Olores desagradables. Puede deberse a:
sobre carga de materia orgnica, tiempo nublado o baja temperatura,
presencia de sustancias txicas,
252
Menndez Gutirrez, C. y J. Prez Olmo
corto circuito y
presencia de materia flotante.
Aparicin de moscas e insectos. Puede deberse a:
crecimiento y vegetacin de los taludes,
slidos flotantes.
7.10.2. Lagunas anaerobias
Caractersticas de una buena operacin
Elementos distintivos de una buena operacin de las lagunas anaerobias y
que se evidencian de manera inmediata lo constituyen la ausencia de oxgeno
disuelto y la coloracin oscura del agua.
Problemas frecuentes que pueden presentarse
Los problemas que suelen presentarse en la operacin de estas lagunas son
muy similares a los que se observan en las facultativas, aunque se suman otras
causales. Por ejemplo, debido a las caractersticas de los procesos que ocurren
en el metabolismo anaerobio, los malos olores pueden ser producidos adems
por deficiencia de alcalinidad del medio.
7.10.3. Operaciones de control y mantenimiento
Parmetros o elementos que deben ser controlados de manera rutinaria
Demanda Qumica y Bioqumica de Oxgeno en el afluente y efluente.
Concentracin de oxgeno disuelto.
Apariencia.
Caudal y pH de afluente y efluente.
Acciones de mantenimiento
Limpiar las obras de entrada y salida.
Mantener los taludes libres de vegetacin mediante la chapea regular.
Retirar slidos flotantes de la superficie.
Romper la nata que se forma, manualmente o mediante el empleo de
chorro de agua.
253
Lagunas de Estabilizacin
Notas bibliogrficas
1 MENNDEZ, C.: 'Principios de diseo de lagunas de oxidacion, Ing. Hidruli-
ca, vol. 10, no. 3, 1989.
2 MENNDEZ, C. Y L. GUERRA: Relaciones de alcalinidad y Ph en lagunas faculta-
tivas, Reporte de Investigacin, ISPJAE, Ciudad de La Habana, 1990.
3 THIRUMURTHI, D.: 'Design criteria Ior waste stabilization ponds, Jour. Water
Poll. Control Fed., vol.49, no. 9, 1974.
4 ECKENFELDER, W.W.: Industrial Water Pollution Control, 2d. Ed. Mc Graw
Hill, 1989.
5 SAENZ, R.: Lagunas de Estabilizacin, Manual DTIAPA, no. 14, CEPIS,
Per, 1988.
6 MARTN, M. Y C. MENNDEZ: 'Diseo de lagunas de oxidacion, Ingeniera en
Procesos Qumicos, vol. 3, no. 2, 1978.
7 MENNDEZ, C. Y L. GUERRA: 'Las Lagunas de Oxidacion y la Posibilidad del
Uso del Modelo de Mezcla Completa, Tecnol. del Agua, vol. 15, no.
136, 1995.
8 MCGARRY Y PESCOD: Stabilization Pond. Design criteria for tropical Asia.
Missouri Basin Egn. Health Council, Kansas City, 1970.
9 FINNEY, B. AND E. MIDDLEBROOKS: 'Facultative Waste Stabilization Pond De-
sign. Jour, Water Pull, Control Fed, vol. 52, no. 1, 1980.
10 WHO: 'Wastewater Stabilization Ponds. Principles oI planning and practi-
ce, WHO: EMRO Tech. Publication no. 10. World Health Organiza-
tion. Regional Office for the Eastern Mediterranean, Alejandra, 1987.
11 HERMANN, E.R. AND E.F. GLOYNA: 'Waste stabilization ponds, III Formulation
of design equations, Sewage and Ind. Wastes, vol. 30, no. 8, 1958.
12 MARAIS, G. VR.: 'New Iactors in the design, operation and perIormance oI
waste stabilization ponds, Bulletin, World Health Organization, vol. 34,
no. 5, 1966.
13 POLPRASERT, C. AND K. BHATTARAI: 'Model Ior Waste Stabilization Ponds,
Jour. Env. Eng. ASCE III, no. 1, 1985.
14 JAMES, A.: 'An alternative approach to the design oI waste stabilization ponds,
Int. Conf. On Waste Stabilization Ponds, IAWPRC, Lisbon, Portugal,
1987.
255
Digestin Anaerobia
CAPTULO 8
DIGESTIN ANAEROBIA
8.1. INTRODUCCIN
La descomposicin anaerobia conduce a la degradacin de la materia org-
nica y su transformacin, en ausencia de oxgeno libre, en compuestos ms
simples como metano y dixido de carbono.
Fig. 8.1. Secuencia de procesos en la digestin anaerobia.
Se trata de un proceso complejo que es consecuencia de muchas reaccio-
nes diferentes, tal como se esquematiza en la figura 8.1.
La principal aplicacin de este tratamiento se halla en la estabilizacin de
lodos con alto contenido de materia orgnica, as como en el tratamiento de
256
Menndez Gutirrez, C. y J. Prez Olmo
residuales de alta carga. Sin embargo, a travs de los aos se han desarrollado
nuevas tecnologas basadas en el propio metabolismo anaerobio que facilitan el
tratamiento ya no solo de los lodos, sino que pueden ser tratados exitosamente
residuales orgnicos diluidos.
8.2. MECANISMO DE LA DIGESTIN ANAEROBIA
Los microorganismos causantes de la descomposicin anaerobia generalmente
son clasificados en dos grandes grupos. El primer grupo hidroliza y transforma
los compuestos orgnicos en cidos voltiles, de los cules los ms comunes son
el actico y el propinico. El segundo grupo convierte los cidos voltiles forma-
dos en CH
4
y CO
2.
Ambos grupos de microorganismos son necesarios para la
descomposicin anaerobia. Las bacterias formadoras de cido no son capaces
de producir CH
4
y las formadoras de metano no pueden por s mismas descom-
poner la materia orgnica original en compuestos ms sencillos.
Fig. 8.2. Mecanismo de la digestin anaerobia.
257
Digestin Anaerobia
Analizando en mayor detalle la descomposicin de la materia orgnica,
puede plantearse que mediante la accin hidroltica, los microorganismos
degradan los polmeros como los polisacridos y protenas, a monmeros.
Posteriormente estos monmeros son convertidos en cidos voltiles. Finalmente
es que se produce la formacin de CH
4
y CO
2
. As en la digestin anaerobia
se distinguen cuatro niveles trficos: hidrlisis, acidognesis, acetognesis y
metanognesis. El metano se produce tanto a partir del cido actico como
del H
2
y CO
2
.
1
Figura 8.2.
No en todas las etapas de la digestin ocurre disminucin de la DBO. As, la
hidrlisis ocurre sin variacin detectable de la DBO. En general durante la aci-
dificacin la disminucin de la DBO es mnima. Por tanto es en la metanognesis
donde ocurren las mayores disminuciones de la DBO.
2
8.3. DISTINTAS VARIANTES DE LOS PROCESOS
DE DIGESTIN ANAEROBIA
Hoy en da son muchos y variados los procesos de digestin anaerobia que
se utilizan, y son diversos los factores que inciden en que se emplee una u otra
variante.
8.3.1. Reactores de primera generacin
Son aquellos en que la biomasa se encuentra en relativo reposo, con un
mnimo contacto con el sustrato, o tambin en suspensin, pero sin
recirculacin de slidos. Por esta razn la relacin (edad del lodo/tiempo de
retencin hidrulico) es igual a 1, como es el caso de los reactores comple-
tamente mezclados.
Ejemplos de reactores de primera generacin son las fosas spticas, los
tanques Imhoff, los digestores convencionales, las lagunas anaerobias y los reac-
tores completamente mezclados aplicados a la estabilizacin de los lodos de
extraccin de plantas de tratamiento de aguas residuales.
8.3.1.1. Digestores convencionales
Estos digestores trabajan a flujo discontinuo, y debido a que no son agitados,
estn estratificados: una capa superior de nata, relativamente inactiva; una capa
intermedia de la que sedimentan los slidos; y una capa inferior de slidos dige-
ridos. Figura 8.3.
258
Menndez Gutirrez, C. y J. Prez Olmo
Fig. 8.3. Esquema de un digestor de lodo convencional.
Los digestores convencionales son apropiados para el tratamiento de
lodos orgnicos y residuales lquidos de alta demanda bioqumica de oxgeno
(mayor de 20000 mgL
1
). Se requiere de altos tiempos de retencin.
Aunque en general, como ya fue mencionado, los reactores de pri-
mera generacin fueron inicialmente concebidos sin recirculacin de
lodos, existe una variante de estos que tiene incluida la recirculacin para
producir un incremento de la relacin edad del lodo / tiempo de retencin
hidrulico y hacer un proceso ms estable. Esta variante se conoce como
contacto anaerobio. Figura 8.4. En la tabla 8.1 se reflejan algunas de las
caractersticas del contacto anaerobio.
8.3.1.2. Digestores completamente mezclados
En este tipo de digestor la carga de slidos es mayor que en el convencional.
El lodo se mezcla ntimamente mediante la recirculacin del propio gas que se
genera en el proceso o a travs del empleo de agitadores mecnicos.
259
Digestin Anaerobia
Fig. 8.4. Esquema de un digestor de contacto anaerobio.
Exceptuando que trabaja con cargas mayores, y la existencia de la agita-
cin, son pocas las diferencias entre este tipo de digestor y el convencional.
Tabla 8.1. Caractersticas del reactor de contacto anaerobio
8.3.2. Reactores de segunda generacin
Se caracterizan porque los microorganismos son retenidos en el reactor por
la presencia de un soporte al que se adhieren o bien por su sedimentacin. La
primera versin de esta generacin es el filtro anaerobio (Young y McCarty).
3
Ventajas Desventajas
- Soporta aguas con slidos en
suspensin.
- Puede admitir aguas que
forman precipitados.
- Soporta picos orgnicos.
- Arranque rpido con inculo
adecuado.
- Se puede incorporar carbn
activado en polvo para tratar
aguas con compuestos
inhibidores.
- Recibe relativas bajas cargas
orgnicas (6 kg/m
3
d).
- Grandes tiempos de
retencin y volmenes de
reactor.
- Altamente dependiente de la
calidad de la sedimentacin
del lodo anaerobio.
- Costos energticos asociados
al mezclado y la
recirculacin.
260
Menndez Gutirrez, C. y J. Prez Olmo
Posteriormente se desarroll una variante ms eficiente con la introduccin de
un soporte tubular, ordenndose de esta manera el empaque (Van D Berg y
Lentz).
4
Lettinga (1980)
5
desarroll el sistema UASB (Upflow Anaerobic Sludge
Blanket) o Reactor Anaerobio de Manto de Lodos y Flujo Ascendente, con
caractersticas de pelcula fija pero sin material de soporte.
8.3.2.1. Filtro anaerobio
Este dispositivo consiste en un reactor empacado con material inerte en
el que los microorganismos crecen sobre la empaquetadura. No obstante, la
experiencia demuestra que no toda el rea disponible de empaquetadura es
cubierta por la biopelcula. En el filtro anaerobio usualmente se presenta una
relativamente alta fraccin de biomasa que no se adhiere al medio soporte,
sino que permanece atrapada en los espacios libres de la misma.
6
Los filtros
pueden operarse a flujo ascendente o descendente. Figura 8.5.
Fig. 8.5. Esquema de un filtro anaerobio de flujo ascendente.
Estos reactores son estratificados como consecuencia del poco mezclado
que existe en su interior. Por lo tanto, se aprecia una tendencia de los
microorganismos a distribuirse de acuerdo a sus requerimientos y las caracte-
rsticas del sustrato. La zona de mayor actividad es la del afuente, existiendo
mayor fraccin de microorganismos saprofitos facultativos y anaerobios que
hidrolizan y degradan la materia orgnica compleja, que en la regin ms prxi-
ma al efluente.
261
Digestin Anaerobia
La elevada concentracin de microorganismos dentro del reactor hace que
los tiempos de retencin alcancen valores comprendidos entre 3 h y 6 h
obtenindose altas eficiencias y elevada produccin de biogs.
Teniendo en cuenta el alto tiempo de retencin de los slidos, en esta varian-
te no se utiliza recirculacin de biomasa. Jennett y Dennis
7
han reportado remo-
ciones de DQO entre 80 y 98 % en el tratamiento de residuales lquidos de la
industria farmacutica con tiempos de retencin entre 12 y 48 horas operando
con cargas de 0,22 y 3,5 kg DQO m
3
d
1
respectivamente.
Algunas de las caractersticas de los filtros anaerobios se resumen en la
tabla 8.2.
Tabla 8.2. Caractersticas de los filtros anaerobios
8.3.2.2. Reactores de manto de lodo de flujo ascendente
La carga permisible en los procesos anaerobios est limitada por la edad del
lodo en los reactores. La necesidad de mantener altas edades de lodo ha sido
siempre el mayor problema prctico de la aplicacin de estos procesos, espe-
cialmente para residuales con demanda bioqumica de oxgeno relativamente
baja (~ 5000 mgL
1
). Obviamente, en general, un proceso es ms econmico en
la medida que los tiempos de retencin requeridos son menores.
La bsqueda de mayores edades de lodo en los reactores para lograr trata-
mientos ms intensivos ha conducido a las diferentes variantes de reactores
anaerobios. El digestor de manto de lodo busca este propsito. Este sistema fue
descrito inicialmente por Coulter,
8
pero debe su desarrollo actual y popularidad a
Lettinga.
5
Figura 8.6. Su caracterstica principal est dada por la capacidad que
poseen de retener biomasa sin necesidad de un soporte, como consecuencia de
la formacin de granos o pellets. Las principales caractersticas de este reactor
se resumen en la tabla 8.3.
Ventajas Desventajas
- Soporta altas cargas, hasta
(15 kg DQO/m
3
d).
- Con recirculacin es
resistente a picos orgnicos o
txicos.
- Simple construccin.
- Verstil. Aplicable a pequea
y mediana escala.
- Operacin simple.
- Re-arranque rpido.
- Arranque lento.
- Riesgo de tupicin.
- Muy sensible a la presencia
de slidos suspendidos en el
afluente.
- Presencia de slidos
suspendidos en el efluente.
262
Menndez Gutirrez, C. y J. Prez Olmo
Fig. 8.6. Esquema de reactor de manto de lodo.
En este digestor se distinguen 3 zonas bien definidas:
Zona de lecho de lodos.
Zona de los microorganismos dispersos.
Zona de separacin gas - lquido - slido.
Entre las condiciones que favorecen la buena operacin de este tipo de
reactor pueden citarse:
Elevada concentracin de biomasa, limitada solo por las condiciones del
separador gas-lquido-slido de la parte superior del reactor. La concen-
tracin de slidos totales puede alcanzar hasta 150 g.L
1
.
Elevada actividad de la biomasa.
Altos tiempos de retencin (edad del lodo) de la biomasa.
Bajos tiempos de retencin hidrulicos. Para ello el sistema de distribu-
cin del afluente debe garantizar el ntimo contacto residual-biomasa.
Produccin de biogs tal, que exista una buena distribucin del lodo sobre
el lecho y en la zona que corresponde a la biomasa dispersa.
263
Digestin Anaerobia
Tabla 8.3. Caractersticas del reactor de manto de lodo
En los reactores de manto de lodo de flujo ascendente el agua residual
se alimenta por el fondo, se distribuye uniformemente por toda el rea y
asciende a travs del manto biolgicamente activo en el que experimenta
las transformaciones tpicas de la descomposicin anaerobia. El CH
4
y
CO
2
formados ascienden a lo largo del reactor y son extrados por el domo
superior.
Su principio de funcionamiento se basa en la capacidad de sedimenta-
cin de la biomasa producida en el reactor. Esta se aglomera en forma de
granos de 1 a 3 mm de dimetro y presenta elevada actividad metanognica.
El lavado o escape de los flculos ms pequeos de la zona del manto,
generalmente se minimiza creando dentro del propio reactor una zona tran-
quila que posibilita que estas partculas vuelvan a sedimentar.
9
En parte el xito de este tipo de reactor se debe a los relativamente altos
valores de concentracin de slidos que se obtienen en el lodo, entre 100 y
150 gL
1
. Han sido reportadas eficiencias de remocin de DBO mayores del
90 % con cargas orgnicas e hidrulicas del orden de 80 kgm
3
d
1
y 5 m
3
m
2
d
1
respectivamente, con tiempos de retencin hidrulico de 8 h.
10
8.3.3. Reactores de tercera generacin
Estos reactores contienen microorganismos en forma de biopelcula ad-
herida a un soporte que se expande o fluidiza. Utilizan una corriente de
recirculacin para provocar un flujo ascendente y mantener fluidizado el
lecho de partculas de soporte. Estas partculas pueden ser arena, material
plstico o cermico. El material soporte brinda una gran rea superficial
sobre la que se adhiere la biopelcula y mantiene una buena sedimentabilidad,
garantizando la retencin celular.
11
Figura 8.7. Las caractersticas de este
reactor se reportan en la tabla 8.4.
Ventajas Desventajas
- Soporta altas cargas
(20 kg DQO/m
3
.d).
- Su construccin es
relativamente simple.
- Con inculo apropiado tiene
arranque inmediato.
- Aplicable a todas las escalas.
- Operacin sencilla.
- La granulacin es lenta y no
controlable.
- No todas las aguas favorecen
la granulacin.
- Sensible a las grasas en el
afluente.
- Sensible a las aguas que
forman precipitados.
264
Menndez Gutirrez, C. y J. Prez Olmo
En la tabla 8.5 se resumen adems datos comparativos de diferentes proce-
sos anaerobios.
12
Fig. 8.7. Esquema de un reactor de lecho fluidizado.
Tabla 8.4. Caractersticas del reactor de lecho fluidizado
Tabla 8.5. Datos de rendimientos tpicos de procesos anaerobios empleados
en el tratamiento de residuales industriales. Adaptados
12
Ventajas Desventajas
- Asimila muy altas cargas
(40 kg DQO/m
3
d).
- Pequeos tiempos de
retencin.
- Instalaciones compactas.
- Asimila slidos
suspendidos en el afluente.
- Arranque lento y difcil.
- Requiere energa para
mantener la fluidizacin por
bombeo.
- Presencia de slidos
suspendidos en el efluente.
Proceso
Afluente
DQO
mgL
-1
Tiempo de
retencin
u h
B
V
kg DQOm
-3
d
-1
%
remocin
DQO
De
contacto
1500
5000
2-10 0,5-6,0 75 - 90
UASB 5000
15000
4-12 4,0-20,0 75 - 85
Lecho fijo 10000
20000
24-48 1,0-15,0 75 - 85
Lecho
fluidizado
5000
10000
5-10 5,0-10,0 80 - 85
265
Digestin Anaerobia
8.4. FACTORES QUE CONTROLAN EL PROCESO
DE DIGESTIN
Los factores que controlan los procesos de digestin pueden clasificarse en
dos grandes grupos: Factores bsicos y factores ambientales.
8.4.1. Factores Bsicos
Dentro de los factores bsicos que intervienen en el control de los procesos
anaerobios se destacan:
Bacterias.
Sustrato.
Tiempo de retencin.
8.4.1.1. Bacterias
Las formadoras de metano son las bacterias clave en la digestin anaerobia.
Crecen ms lentamente y son ms sensibles a los cambios ambientales que las
formadoras de cido.
La fermentacin metnica es la etapa limitante de la digestin. Las
formadoras de metano son bacterias estrictamente anaerobias, y por tanto, sub-
sisten en completa ausencia de oxgeno molecular. Estas bacterias son poco
abundantes en un medio dado, como consecuencia de que la mayor fraccin de
la energa que se libera en los procesos de oxidacin anaerobia queda asociada
al CH
4
que se forma, por lo que resta menos energa para ser utilizada en el
crecimiento.
8.4.1.2. Sustrato
El sustrato para las bacterias est constituido por los compuestos orgnicos
a ser estabilizados. Los compuestos orgnicos que se someten a digestin
anaerobia comnmente son de estructura muy compleja, que como ya fue men-
cionado, son transformados a travs de sucesivas etapas hasta la obtencin de
los productos finales, constituidos fundamentalmente por CH
4
CO
2
y otros com-
puestos estables y de bajo contenido energtico.
El menor crecimiento de las bacterias anaerobias comparados con el de las
aerobias para igual cantidad de sustrato consumido causa menor requerimiento
de nutrientes. Los sistemas anaerobios producen aproximadamente 20 %
menos lodo que los aerobios para el mismo sustrato, y los rquerimientos de
266
Menndez Gutirrez, C. y J. Prez Olmo
nutrientes decrecen en la misma proporcin. Se han reportado relaciones DQO:N
tan altas como 700:5. El valor de 250:5 parece ser razonable como relacin para
procesos con altas cargas (0,8 1,2 kgDQO(kg SSV.d)
1
. Para procesos que
operan a cargas ms bajas, la relacin DQO:N puede incrementarse de forma
conservadora multiplicando la relacin 250:5 por un factor igual al valor de la
carga, dividido por 1,2.
13
8.4.1.3. Tiempo de retencin
En el anlisis del tiempo como factor que interviene en el control de los
procesos anaerobios, hay que considerar tanto la edad del lodo como el tiempo
de retencin hidrulico.
La edad del lodo tiene mucha aplicacin en el diseo y control de los reactores
completamente mezclados, pero en pelcula fija o reactores en los que la biomasa
no es homognea no tiene uso. Esto se debe a la dificultad prctica de medir la
concentracin de slidos suspendidos voltiles (SSV) dentro del reactor.
El tiempo de retencin hidrulico para la estabilizacin anaerobia depende
de la temperatura y la edad del lodo. Actualmente se conoce que los digestores
pueden operar con tiempos de retencin de hasta de un da o menos en la medi-
da que la edad del lodo sea mayor que un cierto valor crtico (u
X,cr
). Esta edad
del lodo crtica es el tiempo por debajo del cual la digestin se inhibe como
resultado del lavado de las bacterias, cuyo crecimiento es lento.
b
X,cr
k Yk
0

=
1
(8.1)
donde:
u
X,cr
: edad del lodo crtica.
Y: rendimiento.
k: velocidad especfica de remocin de sustrato, d
1
.
k
b
: constante de auto-oxidacin, d
1
.
Para edades de lodo menores de 10 das la produccin de CH
4
comienza a
disminuir y puede detenerse completamente para valores de tres o cuatro das.
14
8.4.2. Factores ambientales
Los factores ambientales para el control de la digestin anaerobia, al igual
que los bsicos, han de ser tomados en cuenta adems para el diseo. Entre los
ms importantes se destacan:
Temperatura.
267
Digestin Anaerobia
pH.
Sustancias txicas.
8.4.2.1. Temperatura
El crecimiento de las bacterias metnicas es lento en comparacin con la
mayora de los otros microorganismos. El tiempo requerido por estas bacterias
para regenerar es funcin de la temperatura. A 35 C algunas especies requie-
ren cuatro das para duplicar su nmero, mientras que otras necesitan diez das
o ms. Una disminucin de la temperatura implica un mayor tiempo de regene-
racin.
Intervalos de temperatura de inters pueden ser los correspondientes al ni-
vel mesoIilo (2045 C, con ptimo entre 28 y 33 C) y el termoIilo (4560 C,
con ptimo entre 50 y 60 C).
Considerando la produccin de gas como indicador de la degradacin de
los compuestos orgnicos, podra decirse que el intervalo termfilo es supe-
rior. De hecho, pueden obtenerse resultados similares con tiempos de reten-
cin hidrulico de 7 a 14 das en el intervalo termfilo, a los que se obtienen
en el mesfilo con retenciones de 30 das. No obstante la ventaja de trabajar
en el intervalo mesfilo en los pases tropicales radica en el hecho que no es
necesario el uso del calentamiento para mantener la temperatura dentro de
los valores del intervalo.
Harris y Dague
15
han reportado la posibilidad de incrementos notables de la
carga de los filtros anaerobios a 56 C con resultados de eficiencia de remocin
similares a los alcanzados a 32 C con la mitad de la carga.
8.4.2.2. pH
Las bacterias metnicas ejercen adecuadamente su funcin en un intervalo
de pH comprendido entre 6,6 y 7,6, con un ptimo cercano al pH 7.
16
Cuando la
velocidad de la produccin de los cidos voltiles es mayor que la de su trans-
formacin en CH
4
, el proceso puede desequilibrarse, resultando en una disminu-
cin del pH, disminucin de la produccin de gas y aumento de las fraccin de
CO
2
en el gas producido. El control del pH es por tanto esencial para garantizar
una buena operacin de los procesos anaerobios.
8.4.2.3. Sustancias txicas
Aunque se reconoce una relacin funcional entre algunos de los problemas
de operacin de la digestin anaerobia y la presencia de sustancias txicas, no
268
Menndez Gutirrez, C. y J. Prez Olmo
est clara la incidencia en las transformaciones bioqumicas que tienen estos
compuestos txicos, especialmente el nitrgeno amoniacal y los cidos voltiles
a altas concentraciones.
La toxicidad de los cidos voltiles se debe no tanto a su concentracin
como al efecto del pH, ya que se ha demostrado
17
que la toxicidad es origi-
nada por la fraccin no ionizada de ellos. Kroeker
18
ha concluido que una
concentracin de cidos voltiles no ionizados, en el intervalo comprendido
entre 30 y 60 mgL
1
(como cido actico) es txica e inhibe los procesos
anaerobios. En la figura 8.8 se muestra que aunque la concentracin de
cidos voltiles sea elevada, si el pH se mantiene relativamente alto, la con-
centracin no ionizada de cidos voltiles es pequea y la digestin puede
ocurrir sin dificultad. Por tanto, aunque altas concentraciones de por si no
son dainas. Sin embargo, su combinacin con bajos valores de pH pueden
producir situaciones indeseables.
Hay sales inorgnicas que a bajas concentraciones favorecen la digestin,
pero a valores por encima de un cierto lmite pueden inhibir el proceso. Entre
estos pueden citarse el Na
+
, K
+
, Ca
2+
, y Mg
2+
. En la tabla 8.6 se ofrecen interva-
los de concentraciones a las cuales estos cationes pueden ser inhibitorios. Otros
cationes como el zinc, cobre y nquel son txicos an a bajas concentraciones.
El amoniaco es txico a concentraciones superiores a 3000 mgL
1
e inhibi-
torio a 1500 mgL
1
. La toxicidad de amoniaco est relacionada con el pH, ya
que este compuesto es ms nocivo en su forma molecular que como catin
amonio.
14
Fig. 8.8. Relacin entre la forma molecular y disociada de los cidos
voltiles y su incidencia en la digestin.
269
Digestin Anaerobia
Tabla 8.6. Concentraciones inhibitorias de distintos cationes
20
8.5. PARMETROS INDICADORES DEL CURSO
DE LA DIGESTIN
Entre los diferentes parmetros que son de utilidad prctica para indicar si
un digestor est operando o no en buenas condiciones, y aun para predecir
futuras complicaciones en la operacin, pueden citarse:
La produccin de gases.
Contenido de cidos voltiles y pH.
La alcalinidad del medio.
8.5.1. Produccin de gases
El grado de estabilizacin de los compuestos orgnicos est directamente
relacionada con la produccin de gases y su composicin. As por ejemplo, la
produccin de gases puede disminuir abruptamente si el tiempo de retencin cae
por debajo de un determinado valor. Figura 8.9. Esto est ntimamente vinculado
con la velocidad de crecimiento de las bacterias formadoras de metano, que
como ya se ha reiterado, es menor que el de las formadoras de cido.
Concentracin mgL
-1
CATIN Moderadamente
inhibitorio
Fuertemente
inhibitorio
Na
+
1500 3000 3000
K
+
3500 5500 8000
Ca
2+
2500 4500 12000
Mg
2+
1000 - 1500 3000
Fig. 8.9. Influencia del tiempo de retencin en la produccin de biogs.
270
Menndez Gutirrez, C. y J. Prez Olmo
En condiciones de operacin normal la produccin de gases, a 101,3 kPa y 0 C,
est en el intervalo comprendido entre 0,2 y 0,7 m
3
por cada kg de DQO
removido 1 m
3
por cada kg de slido voltil estabilizado.
En la tabla 8.7 se muestran algunos rendimientos tericos de gas y su pro-
duccin por kg de slidos voltiles para sustancias que frecuentemente se en-
cuentran presentes en muchos residuales.
Por otro lado, la composicin de los gases producidos tampoco es constante.
En un digestor anaerobio que funcione en ptimas condiciones, los gases produ-
cidos estn constituidos en 75 a 85 % por CH
4
y el resto fundamentalmente por
CO
2
. Un digestor operando en condiciones desfavorables puede mantener du-
rante un tiempo constante la el volumen de gas producido y estar variando la
fraccin relativa de CH
4
s CO
2
.
Tabla 8.7. Rendimientos tericos de biogs en funcin de los constituyentes
del residual
En la figura 8.10, donde se representa la variacin de la relacin de la com-
posicin de los gases con el pH del medio, puede apreciarse cunto llega a
cambiar esa composicin ante condiciones desfavorables de operacin. En este
caso, del pH.
Compuesto % CH
4
en el gas m
3
(kg SV digerido)
-1
Carbohidratos 50 0,89
Protenas 70 1,30
Grasas 84 0,60
Fig. 8.10. Variacin de la composicin de los gases con el pH.
8.5.2. Contenido de cidos voltiles, pH y alcalinidad
Estas tres caractersticas de un digestor estn muy relacionadas entre s.
271
Digestin Anaerobia
La alcalinidad desempea un papel importante en los procesos anaerobios,
compensando no solo la produccin de los cidos voltiles, sino tambin la de
CO
2
. La alcalinidad natural de estos procesos es debida al bicarbonato, y da
lugar a un sistema tampn de pH comprendido entre 6,8 y 7,2.
En la figura 8.11 se aprecia que en los lmites de trabajo normal de un trata-
miento anaerobio se requiere disponer de una alcalinidad de aproximadamente
1500 mgL
1
cuando el gas tiene en su composicin 30 % de CO
2
. Cuando la
alcalinidad necesaria no existe en el medio, debe ser adicionada.
Fig. 8.11. Alcalinidad del medio y composicin de los gases en funcin
del pH.
Aunque la alcalinidad es esencial para una digestin balanceada, su
valor en s mismo no es un indicador adecuado que permita predecir posibles
dificultades actuales o futuras del proceso. Esto se debe a que en la medida
en que la alcalinidad al bicarbonato es consumida y destruida, se forma
alcalinidad vinculada a los cidos voltiles. Las determinaciones de alcalinidad
y cidos voltiles pueden utilizarse simultneamente como indicadores de la
condicin de la digestin.
21
La relacin entre la alcalinidad total, la debida al bicarbonato y la concentra-
cin de cidos voltiles es:
AT = AB + 0,71 AV (8.2)
donde:
AT: Alcalinidad total, mgL
1
como CaCO
3
.
AB: Alcalinidad debida al HCO
3

, mgL
1
como CaCO
3
.
AV: Concentracin de cidos voltiles, como (HCH
3
COO).
272
Menndez Gutirrez, C. y J. Prez Olmo
Es deseable que la capacidad tampn del proceso de digestin sea alta.
Esto se traduce en que la relacin cidos voltiles / alcalinidad sea baja
(entre 0 y 0,1). Valores de esta relacin cercanos a 0,5, es una seal de la
existencia de problemas en el proceso. El pH del digestor comienza a
descender rpidamente si esa relacin llega a valores tan altos como 0,8.
8.6. MTODOS DE DISEO
Esencialmente hay dos enfoques para el diseo de reactores anaerobios: los
que se basan en mtodos empricos, y los que utilizan criterios cinticos.
8.6.1. Mtodos empricos
Los mtodos empricos se utilizan fundamentalmente cuando se necesita
digerir lodos producidos en alguna etapa anterior del tratamiento.
A modo de criterio, en la tabla 8.8 se brinda la composicin, en slidos vol-
tiles y fijos, de lodos crudos y digeridos.
Tabla 8.8. Composicin de los lodos
8.6.1.1. Mtodo de las cargas
El mtodo de las cargas o tradicional para el diseo de digestores de lodo
brinda buenos resultados en la medida que los criterios que se utilicen se basen
en la experiencia previa, sin extrapolar resultados anteriores a una nueva situa-
cin. Este mtodo determina el volumen requerido de reactor en funcin de una
carga asumida.
El factor de carga ms utilizado es que se basa en los kg de slidos voltiles
aadidos por cada m
3
de reactor por da. La carga recomendada para los reac-
tores normales est entre 0,03 y 0,10 kg de slidos voltiles (SV) por m
3
por da,
con tiempos de retencin de 30 a 90 das. En el caso de reactores de alta carga
es posible utilizar cargas de hasta 1,6 a 6,4 kg de SV por cada m
3
por da, para
tiempos de retencin de 10 a 20 das.
Lodo Slidos voltiles % Slidos fijos %
Crudos 70 a 75 30 a 25
Digeridos 50 50
273
Digestin Anaerobia
8.6.1.2. Mtodo de la reduccin de volumen
A medida que la digestin tiene lugar, el lodo experimenta una reduccin de
volumen. Teniendo en cuenta esta caracterstica, el volumen del reactor puede
calcularse,
( ) t . V V V V
d f f
(

=
3
2
(8.3)
donde:
V: volumen del digestor.
V
f
: volumen diario de lodo crudo.
V
d
: volumen diario de lodo digerido.
t: tiempo de retencin.
Ejemplo 8.1
Estime el volumen de un digestor para tratar 5000 kgd
1
(base seca) de un
lodo orgnico producido en otra etapa de tratamiento. Asuma que 75 % de los
slidos es voltil. Utilice el mtodo de las cargas.
Carga: 4 kg.m
3
d
1
Slidos voltiles en los 5000 kg de lodo = 5000 0,75 = 3750 kg
3
m 5 937
1 3
4
1
3750
digestor del Volumen ,
d
-
m kg
d kg
=

=
Ejemplo 8.2
Determine el volumen de un reactor para digerir 3500 kgd
1
(base seca) de
un lodo que contiene 95 % de humedad. Asuma que 75 % de los slidos es
voltil. Considere que hay una reduccin del 60 % de los slidos voltiles, con
una humedad de 92 %. Tiempo de retencin de 25 das.
Si el lodo que contiene 95 % de humedad, su peso especfico es 1,02 kgL
1
,
por tanto el volumen de lodo a digerir es:
3
3
m 63 68
10 05 0 02 1
3500
,
. , . ,
V
f
= =
La masa de lodo despus de la digestin ser 40 % de los voltiles que
entraron ms los fijos que no se degradan (25 % de los slidos totales).
Slidos voltiles en los 3500 kg de lodo = 3500 0,75 = 2625 kg
Slidos fijos = 3500 0,25 = 875 kg
274
Menndez Gutirrez, C. y J. Prez Olmo
El lodo digerido, con 92 % de humedad, tiene un peso especfico de
1,04 kgL
1
3
3
m 14 23
10 08 0 04 1
4 0 2625 875
,
. , . ,
, .
V
d
=
+
=
( ) t V V V V
d f f

(

=
3
2
( )
25 14 23 63 68
3
2
63 68
(

= , , , V
V = 958 m
3
8.6.2. Ecuaciones de diseo basadas en criterios cinticos
El uso principal que han recibido los modelos basados en criterios cinticos,
llamados en ocasiones modelos conceptuales, es en el control y simulacin de
los procesos.
8.6.2.1. Tiempo de retencin y volumen del digestor
Para un digestor anaerobio que opere a flujo continuo y mezcla completa, el
tiempo de retencin viene dado por
S
S
k X
S A
0
0
=
(8.4)
donde:
AS = S
0
S, mgL
1
.
S: DBO DQO en el efluente.
S
0
: DBO DQO en el afluente al reactor.
X: concentracin de slidos suspendidos voltiles en el reactor.
k: constante especfica de remocin de sustrato, d
1
.
El volumen efectivo del digestor se obtiene,
V = q Q m
3
(8.5)
Q: flujo de entrada al digestor, m
3
d
1
.
275
Digestin Anaerobia
8.6.2.2. Edad del lodo y produccin de slidos
Aunque la produccin de biomasa durante la digestin anaerobia es mucho
menor que en los procesos aerobios, siempre hay que considerarla, pues ella va
a garantizar en cierta medida que el reactor no se lave con la consecuente
inhibicin del proceso.
X k AB Y X A
b V
= kgm
3
d
1
(8.6)
donde:
AX: produccin de slidos, kgm
3
d
1
.
Y: rendimiento.
AB
V
: razn de carga volumtrica, kgm
3
d
1
.
Valores tpicos de Y y k
b
.
Y: 0,1 0,2.
k
b:
0,015 0,025.
Cuando la produccin de biomasa es AX, y se desea una concentracin de
SSV en el reactor igual a X, la edad de lodo requerida es,
AX
X
0
X
=
d (8.7)
El valor de la edad de lodo siempre debe ser superior al valor crtico definido
por la ecuacin 8.1.
8.6.2.3. Volumen de gas producido
La produccin de gases en los procesos de digestin es muy variable y
depende en alguna medida de la composicin del agua residual.
22
En trminos
generales, y tomando como base la DQO, puede estimarse la produccin total
de gas a TPN como,
G
T
= 0,5 (AB
V
1,42 X) V m
3
d
1
(8.8)
0
S A
B A
v
=
donde:
Si lo que se desea es el volumen de CH
4
,
G = 0,35 (AB
V
1,42 DX)V m
3
d
1
(8.9)
276
Menndez Gutirrez, C. y J. Prez Olmo
8.6.2.4. Nutrientes
El N y P que se requiere en los procesos anaerobios es menor que los que
exigen los aerobios,
N = 0,12 AXV kgd
1
(8.10)
P = 0,025 AXV kgd
1
(8.11)
Ejemplo 8.3
Determine las dimensiones de un digestor completamente mezclado para
obtener 87 % de remocin de DQO de un agua residual cuyo flujo es de
750 m
3
d
1
.
DQO del afluente = 16000 mgL
1
Y = 0,1 k
b
= 0,02 d
1
k = 6 d
1
X = 6000 mgL
1
Edad del lodo mnima = 15 d
Clculo del tiempo de retencin a partir de la informacin cintica:
S
S
X k
AS
0
0
=
d
.
0 3
2080
16000
6000 6
13920
= =
Volumen efectivo del digestor:
V = q Q = 3 750 = 2250 m
3
Comprobacin de la edad del lodo:
AX
X
0
X
=
X k AB Y X A
b V
=
1 3 -
d m g 344 6000 02 0
3
13920
1 0

= = , , X A
Por tanto,
d , 0
X
4 17
344
6000
= =
La edad del lodo es superior a la mnima fijada por las condiciones de
diseo.
277
Digestin Anaerobia
Produccin de CH
4
a TPN:
G = 0,35 (AB
V
1,42 DX)V
G 0,35 (4640 1,42 344) 10
3
2250
G = 3269 m
3
d
1
8.6.3. Mtodo experimental para determinar los parmetros
cinticos
El procedimiento de laboratorio que se detalla a continuacin es en esen-
cia el descrito por Eckenfelder y Ford.
23
Procedimiento
Para las pruebas en laboratorio se emplea un reactor con un volumen
efectivo de 1 a 5 litros. El montaje del equipamiento necesario est
esquematizado en la figura 8.12.
1. Tomar un volumen de lodo digerido activo de una instalacin en opera-
cin. Filtrar el lodo a travs de una malla gruesa para eliminar partculas
como semillas, paja, etc. Si fuera necesario, diluir con agua corriente.
2. Introducir el lodo en el digestor. Mantener la temperatura constante a 35 C
y agitado en contenido del reactor durante todo el tiempo que dure la
experiencia.
3. No alimentar agua residual ni extraer licor mezclado del reactor hasta
que se aprecie produccin de gas. Asegurar que el dispositivo est her-
mtico para evitar fuga de gas.
4. Una vez que se aprecie produccin de gas, introducir residual aplicando
una carga a 4 kg DBO m
3
d
1
, por un periodo no menor a un tiempo
equivalente a un tiempo de retencin. Es aconsejable, durante este perio-
do inicial de aclimatacin, alimentar el reactor 2 3 veces en el da, de
manera tal que la carga total no sobrepase los 4 kg DBO m
3
d
1
.
5. El pH es un parmetro que siempre debe ser vigilado, pero ms cuidado-
samente durante la aclimatacin. En caso de disminuir por debajo de 6,
aadir alcalinidad para mantenerlo entre 6,5 y 7,5. El licor mezclado debe
tender a crear su propio sistema tampn.
6. Proceder a la alimentacin del residual con un flujo continuo, con una
extraccin diaria de un volumen igual al alimentado.
278
Menndez Gutirrez, C. y J. Prez Olmo
Fig. 8.12. Digestor anaerobio a escala de laboratorio.
7. Registrar la siguiente informacin:
Tabla 8.9. Informacin a obtener en el laboratorio
8. Registrar el volumen de gas producido diariamente y su composicin (CH
4
,
CO
2
)
9. Tabular los datos obtenidos segn se muestra en la tabla 8.10 del ejemplo 8.4.
10.Calcular las constantes k,Y y k
b
graficando la data segn la informacin de
las tablas 8.10 y 8.11. En las figuras 8.13 y 8.14 aparecen los grficos co-
rrespondientes.
Ejemplo 8.4
En pruebas de laboratorio realizadas en buenas condiciones de operacin,
se obtuvieron los datos de la tabla 8.10 para un digestor anaerobio.
Determine:
a) Velocidad especfica de remocin de DBO.
b) El rendimiento y la constante de autooxidacin.
Residual crudo (afluente) Efluente (licor mezclado)
pH
DBO DQO
PH
DBO DQO
Alcalinidad
Acidez
SSV
cidos voltiles
279
Digestin Anaerobia
Tabla 8.10. Tabla de trabajo del ejemplo 8.4
a) Considerando que se cumple que
0
S
S
X k
dt
dS
=
se prepara la tabla 8.11 1
Tabla 8.11. Tabla de trabajo del ejemplo 8.4
u
(d)
S
0
(mgL
-1
)
S
(mgL
-1
)
X
(mgL
-1
)
u
X
(d)
G
(m
3
d
-1
)
0,25 15000 13500 1200 2 14
0,50 15000 12000 1500 3 15
2,0 15400 9000 1164 4 17
3,0 14800 7250 1260 6,25 24
4,5 15150 4500 3156 20 27
5,0 14750 3750 4400 28 25
Se grafica
0
contra
S
S
B A
X segn la figura 8.13. La pendiente da el
valor de k = 6,7 d
1
b) Para el clculo de Y y k
b
, se grafica
X
0
1
a) contra AB
X
de acuerdo con la tabla 8.12 y la figura 8.14
Tabla 8.12. Tabla de trabajo del ejemplo 8.4
AB
X
(kgkg
-1
d
-1
)
5,0 4,0 2,75 2,0 0,75 0,50
0
S
S
0,9 0,8 0,58 0,49 0,30 0,25
AB
X
(kgkg
-1
d
-1
)
5,0 4,0 2,75 2,0 0,75 0,50
X
0
1
0,5 0,33 0,25 0,16 0,05 0,036
280
Menndez Gutirrez, C. y J. Prez Olmo
Fig. 8.13. Clculo de la constante de velocidad (AB
X
vs S/S
o
).
Fig. 8.14. (1/
X
vs B
X
).
8.6.4. Consideraciones de diseo para reactores UASB
El diseo de los reactores de manto de lodo de flujo ascendente en
general se basa en elementos hidrulicos para DQO menores de 1000 mgL
1
y tiempos de retencin inferiores a 8 h. La velocidad ascensional en la
zona de paso del dispositivo separador superior debe ser mantenida menor
que 3 m
3
m
2
h
1
para lodos floculentos (no granulares). Esta velocidad pue-
de llegar hasta 6 m
3
m
2
h
1
para los lodos granulares tpicos. En todos los
casos se recomienda que en la zona de sedimentacin la velocidad sea
menor que 1 mh
1
.
AB
X
281
Digestin Anaerobia
Cuando el criterio de diseo es hidrulico el volumen efectivo del reactor se
calcula,
V = u Q (8.12)
Para aguas residuales con elevada DQO, el criterio de diseo se fundamen-
ta en la carga orgnica volumtrica (B
V
). Por lo tanto, el clculo del volumen
ser,
V
B
Q S
V

=
0
(8.13)
En la figura 8.15 se refleja la variacin de la carga volumtrica recomenda-
da en el diseo de reactores UASB, en funcin de la temperatura, para reacto-
res con 25 kg SSVm
3
.
Para el tratamiento de aguas residuales diluidas, similares a las domsti-
cas, la altura recomendada para los reactores es de 3 a 5 m. Para DQO
entre 1000 y 5000 mgL
1
, los mejores resultados se obtienen con alturas de
reactor de 5 a 6 m.
Fig. 8.15. Carga orgnica volumtrica en funcin de la temperatura
para el diseo de reactores UASB.
10
En los reactores UASB el sistema de distribucin del afluente y el separador
gas-slido-lquido, revisten especial importancia.
Del sistema de distribucin depende que el manto de lodos tenga un contac-
to ms ntimo con el agua residual, evitando las zonas muertas y canalizaciones.
La distribucin se hace a travs de boquillas. Las boquillas pueden distribuirse
en el fondo del reactor con una densidad que puede variar desde 1 2 por m
2
para aguas con DQO menores de 1000 mgL
1
, y hasta 6 por cada m
2
en el caso
de B
V
4 kg DQOm
3
d
1
.
282
Menndez Gutirrez, C. y J. Prez Olmo
Entre los objetivos del separador gas-slido-lquido pueden sealarse:
Facilitar la separacin y descarga del biogs.
Prevenir, mediante decantacin, el lavado de flculos y granos.
Como ya fue destacado, en el separador se considera de importancia la
velocidad en los pasos entre colectores y en la zona de sedimentacin. Se reco-
mienda que el rea de paso entre los colectores sea aproximadamente 20 % del
rea superficial del reactor.
8.7. PUESTA EN MARCHA DE LOS DIGESTORES
ANAEROBIOS
Sin duda, uno de los aspectos ms complejos de la operacin de los digestores
es la puesta en marcha.
Aun cuando cada digestor tiene sus especificidades y por lo tanto, su propia
estrategia de puesta en marcha, se destaca un conjunto de aspectos que de
manera general ha de considerarse en todos ellos. Estos son:
Inculo.
Carga orgnica.
Control de los cidos grasos voltiles.
Como inculo de los digestores es preferible el empleo de una poblacin
de microorganismos que haya sido previamente adaptada al sustrato que va
a ser utilizado. Adems se prefiere el empleo de una mezcla de diferentes
fuentes con biomasa activa que emplear una sola.
24
El volumen inicial de
inculo nunca debe ser menor del 10 % del volumen efectivo del reactor.
Una vez cargado el reactor con el inculo y el sustrato de inters, se reco-
mienda no iniciar el flujo continuo, sino mantener el sistema a no flujo hasta
que se aprecie produccin de gas.
Teniendo en cuenta que en la etapa inicial del proceso la poblacin de
microorganismos necesita adaptarse a las condiciones de operacin que se im-
ponen, la carga inicial debe ser menor a la de la operacin normal que se preten-
de en el futuro. Puede iniciarse la puesta en marcha con un valor aproximado de
carga orgnica de 0,15 kg DBO.(kg SSV.d)
1
. Esta carga deber ser
incrementada en la medida que se observe una adecuada autoregulacin del pH
as como la alcalinidad del medio, y exista control sobre la concentracin de los
cidos voltiles. Debe propiciarse adems un determinado nivel de mezcla den-
tro del reactor para facilitar el contacto contaminante-biomasa. El funciona-
miento adecuado del digestor puede ser estimulado si junto al sustrato se adiciona
algn otro residual de fcil degradacin.
283
Digestin Anaerobia
Considerando que en un inicio el sistema buffer o tampn de pH que estos
sistemas poseen no est desarrollado, lo ms comn es que se aprecie una
tendencia a decrecer el pH debido a la formacin de los cidos grasos. El pH ha
de mantenerse en un valor cercano a 7 mediante la adicin de una base.
Notas bibliogrficas
1 HENZE, M.: 'Fundamentals oI anaerobic digestion, Water Sci. Tech., vol. 15,
no. 3, 1983.
2 ECKENFELDER, W.W.: Water Quality Engineering for Practicing Engineers,
CBI Pub. Co. Inc., EU.
3 YOUNG, J.C. AND P.L. MCCARTY: 'The anaerobic Iilter Ior waste treatment,
Jour. Water Poll, Control Fed., vol. 41, no. 1, 1969.
4 VAN DER BERG, L. AND C.P. LENTZ: 'Comparison between up and downIlow
anaerobic fixed film reactors of varying surface-to-volume ratios for the
treatment oI bean blanching waste, Proc. 34
th
Purdue Ind. Waste Conf.,
1979.
5 LETTINGA, G. ET AL.: 'Use oI UpIlow Sludge Blanket (USB) reactor concept
Ior biological wastewater treatment, Biotechnol. Bioeng, vol. 25,
no. 10, 1980.
6 YOUNG, J.C.: 'Factors aIIecting the design and perIormance oI up-Ilow anae-
robic Iilters, Water Sci. Tech., vol. 24, no. 8, 1991.
7 JENNET, J.C. AND H.D. DENNIS: 'Anaerobic Iilter treatment oI pharmaceutical
waste, Jour. Water Poll, Control fed., vol. 47, no. 1, 1975.
8 COULTER, J.E. AND M.B. ETTINGER: 'Anaerobic contact process Ior sewage
disposal, Sewage and Ind. Wastes, vol. 29, no. 4, 1957.
9 GUIOT, R.S., Y. ARCAND AND C. CHAVARIE: 'Advantages oI Iluidization on granu-
le size and activity development in upIlow anaerobic bed reactors, Water
Sci. Tech., vol. 26, no. 3-4, 1992.
10 LETTINGA, G. AND P.L. HULSHOFF: 'UASB-process design Ior various types oI
wastewaters, Water Sci. Tech., vol. 24, no. 8, 1991.
11 IZA, J.: 'Fluidized bed reactors Ior anaerobic wastewater treatment, Water
Sci. Tech., vol. 24, no. 8, 1991.
12 METCLAF AND EDDY: Ingeniera de Aguas Residuales. Tratamiento, Vertido
y Reutilizacin, 3ra. Edic., vol. 1, p. 487, Mc Graw Hill, 1996.
13 HENZE, M. AND P. HARREMOES: 'Anaerobic treatment oI wastewater in Iixed
Iilm reactors, A Literature Review. Water Sci. Tech., vol. 15, no. 8/9.
1983.
14 SPEECE, R.E.: 'Toxicity in anaerobic digestion, Proc. 4
th
Int., Symposium on
anaerobic digestion, Guanghou, China, 1985.
284
Menndez Gutirrez, C. y J. Prez Olmo
15 HARRIS, W. L. AND R. DAGUE: 'Comparative perIormance oI anaerobic Iilters
at mesophilic and thermophilic temperatures, Water Environment Res.,
vol. 65, no. 6, 1993.
16 JEWELL, W.: 'Anaerobic Sewage Treatment, Env. Sci. Tech., vol. 21,
no. 1, 1987.
17 MC CARTY, P.L.: 'Anaerobic waste treatment Iundamentals, Part 3, Toxic
Materials and their Control, Pub. Works, vol. 91, no. 11, 1964.
18 KROEKER, E.J. ET AL: 'Anaerobic treatment process stability, Jour. Water
Poll. Control, vol. 51, no. 4, 1979.
19 GUPTA, A.R., R. FLORA, M. GUPTA, G. SAYLES AND M. SUIDAN: 'Methanogene-
sis and sulphate reduction in chemostats I. Kinetic studies and experi-
ments, Water Res., vol. 28, no. 4, 1994.
20 MCCARTY, P.L. ET AL.: 'Individual volatile acid in anaerobic treatment, Jour.
Water Poll. Control Fed., vol. 35, no. 12, 1963.
21 JENKINS, S.R., J.M. MORGAN AND X. ZHANG: 'Measuring the usable carbonate
alkalinity oI operating anaerobic digesters, Res. Jour. Water Poll. Con-
trol Fed., vol. 63, no. 1, 1991.
22 ZEPENG, C. ET AL.: 'The application oI two phase anaerobic digestion on the
disposal of organic wastewater Proc. 4
th
Int. Symposium on anaerobic
digestion, Guanghou, China, 1985.
23 ECKENFELDER, W.W. AND D.L. FORD: Water Pollution Control, Jenkins Book
Pub. Co. E.U., 1970.
24 WEILAND, P. AND A. ROZZI: 'The start-up, operation and monitoring oI high-
rate anaerobic treatment systems, Water Sci. Tech., vol. 24, no. 8, 1991.
285
Manejo de Lodos
CAPTULO 9
MANEJO DE LODOS
9.1. INTRODUCCIN
Un aspecto importante en el tratamiento de los residuales industriales lqui-
dos (RIL) lo constituye la manipulacin de los lodos o fangos que se obtienen,
sean estos de naturaleza orgnica o inorgnica.
Los lodos producidos en los tratamientos biolgicos de aguas residuales pue-
den provenir de los sedimentadores primarios y secundarios as como de los
tratamientos terciarios. Estos lodos estn constituidos fundamentalmente de
materia orgnica, con una fraccin voltil entre 60 y 80 %.
En los tratamientos fsico-qumicos como puede ser la coagulacin, los lodos
que se producen son eminentemente de naturaleza inorgnica.
Uno u otro lodo posee en su composicin agua en un alto porcentaje. Esto
dificulta tanto su manipulacin como su disposicin.
La magnitud del problema se evidencia en un clculo sencillo al considerar
los residuales de una industria cuyos lodos corresponden a una poblacin equi-
valente a 500000 personas. En ese caso se producen aproximadamente 0,09 kg
de lodo por habitante. Si se asume un contenido de humedad de 97 %, esto
significa que se manejan diariamente 3000 m
3
de lodo.
La naturaleza de los lodos, orgnica o inorgnica, define el tratamiento al
que deben ser sometidos antes de su disposicin final, pero en uno y otro caso
deben ser transportados y manipulados antes y despus del tratamiento que se
emplee para los mismos.
9.2. CONTENIDO DE HUMEDAD Y VOLUMEN
DE LOS LODOS
La gravedad especfica de los slidos en los lodos puede ser determinada
mediante la expresin:
Sv
Wv
Sf
Wf
Ss
Ws
+ =
(9.1)
286
Menndez Gutirrez, C. y J. Prez Olmo
donde:
Ws: masa de slido (slido seco), kg.
Ss: gravedad especfica de los slidos secos.
Wf: masa de slidos fijos (fraccin no voltil de los slidos), kg.
Wv: masa de slidos voltiles, kg.
Sf: gravedad especfica de los slidos fijos.
Sv: gravedad especfica de los slidos voltiles.
Teniendo en cuenta que la fraccin filtrable de los lodos es relativamente
pequea comparada con la no filtrable, los slidos pueden ser tomados como el
residuo total despus de secada la muestra a 105 C.
Si en determinada circunstancia la fraccin filtrable es de tal magnitud que
no puede ser despreciada, debe trabajarse con los slidos suspendidos o no
filtrables.
La gravedad especfica de la materia orgnica puede ser tomada como
1,2 a 1,4, mientras que la de la materia inorgnica vara entre 1,5 y 2,5.
Una vez que se conoce la gravedad especfica de los slidos secos, puede
calcularse la de los lodos mediante la relacin:
Ss
Ws
Ww
Ww Ws
S
+
+
=
(9.2)
donde:
S: gravedad especfica de los lodos.
Ws: masa de slido seco.
Ww: masa de agua.
Ss: gravedad especfica de los slidos secos.
No es difcil demostrar que para los lodos orgnicos cuyo contenido de hu-
medad sea del 90 % o mayor, su gravedad especfica puede ser tomada como
1,0 sin cometer mucho error.
El volumen de lodo, calculado a partir de su contenido de slido y % de
humedad es,
S
P
Ws
V
W
|
.
|

\
|
=
100
100
(9.3)
V: volumen de lodo, m
3
.
P: contenido de humedad, %.

w:
densidad del agua, kgm
-3
.
287
Manejo de Lodos
A los efectos prcticos puede asumirse sin mucho error, que la densidad del
agua es 1000 kgm
-3
en el intervalo de temperatura entre 20 y 40 C.
La ecuacin 9.3 puede ser utilizada tambin en aquellos casos en los que se
desee calcular el flujo de lodo, Qs (m
3
d
-1
), si el trmino Ws est dado como flujo
msico (kgd
-1
).
9.3. LODOS PRIMARIOS Y SECUNDARIOS. CRITERIOS
DE ESTIMACIN
Siempre que no se disponga de toda la informacin necesaria para el clculo
preciso de los lodos en un sistema dado, se hace necesario acudir a mtodos de
clculo aproximados.
La produccin de slidos en los sedimentadores primarios y secundarios
puede estimarse de manera aproximada utilizando las siguientes ecuaciones,
Ws = Wsp + Wss (9.4)
Ws: produccin total de slido seco, kgd
-1
.
Wsp: produccin de slido seco en el sedimentador primario, kgd
-1
.
Wss: produccin de slido seco en el sedimentador secundario, kgd
-1
.
Por otro lado,
Wsp = f SS Q 10
-3
(9.5)
SS: slidos suspendidos en el agua sin sedimentar, mg L
-1
.
Q: flujo de agua residual que entra al sedimentador primario, m
3
d
-1
.
f: fraccin de los slidos suspendidos que es removida en el sedimentador
primario y
Wss f DBO Q 10
-3
(9.6)
DBO: concentracin en el agua residual que entra a la unidad de tratamiento
secundario, mg L
-1
.
f: fraccin de la DBO aplicada que aparece como exceso de lodo en la
unidad de tratamiento secundario.
El clculo estimado se basa, fundamentalmente, en la experiencia de trabajo
que se tenga con cada residual en particular, estando la mayor variacin en la
Iracciones I y I para los residuales industriales.
Para residuales domsticos: f = 0,4; I 0,6
Para residuales industriales f depende de los slidos sedimentables.
Para residuales domsticos:
Filtros percoladores de baja velocidad I 0,3
de alta velocidad I 0,5
288
Menndez Gutirrez, C. y J. Prez Olmo
Lodo activado aeracin extendida I 0,30- 0,45
convencional I 0,45 - 0,55
Para residuales industriales:
Lodo activado convencional papel I ~ 0,35
azucarero I ~ 0,65
cervecera I ~ 0,70
tenera I ~ 0,30
Ejemplo 9.1
Una industria del papel arroja diariamente 3000 m
3
de un agua residual
con una DBO de 300 mg L
-1
y una concentracin de slidos suspendidos de
270 mg L
-1
. Para el tratamiento de esos residuales est concebido una sedi-
mentacin primaria donde se elimine 65 % de los slidos suspendidos y 45 %
de la DBO.
El % de agua de los lodos primarios ser de 94 %.
La concentracin esperada de los lodos secundarios es de 15000 mg L
-1
.
Calcule el estimado de lodo a extraer en aquel caso en que se decida unir y
espesar los lodos primarios y secundarios de conjunto hasta obtener una con-
centracin de slidos de 9 %. Estime adems el flujo de lodo a extraer.
Wsp = f SS Q 10
-3
= 0,65 270 3000 10
-3
= 526,5 kgd
-1
S
P
Wsp
Qsp
W
|
.
|

\
|
=
100
100
( )
77 8
1 1000 06 0
5 526
,
,
,
Qsp =

=
m
3
d
-1
Wss f DBO Q 10
-3
= 0,55 300 0,35 3000 10
-3
(se asumio I 0,35
= 173,25 kg d
-1
S
P
Wss
ss Q
W
|
.
|

\
|
=
100
100
( )
55 11
1 1000 015 0
25 173
,
,
,
Qss =

=
m
3
d
-1
289
Manejo de Lodos
Volumen de la mezcla de lodos primarios y secundarios,
Qs = Qsp + Qss
= 8,77 + 11,55
= 20,32 m
3
d
-1
Masa de slidos en la mezcla,
Ws = Wsp + Wss
= 526,5 + 173,25
= 699,75 kg d
-1
% en peso de slidos en la mezcla antes del espesamiento:
% , x
,
,
%peso 4 3
1000
100
32 20
75 699
= =
Flujo de lodo espesado a extraer
77 7
1 1000 09 0
75 699
,
,
,
Qs =

=
m
3
d
-1
Observe que un aumento en la concentracin de slidos por espesamiento
del 3,4 % al 9 %, da lugar a una disminucin en el volumen de lodos a manipular
de 20,32 a 2,77 m
3
d
-1
.
La variacin de la concentracin de slidos que puede obtenerse en una
secuencia de tratamiento se muestra a continuacin.
Tabla 9.1. Secuencia de tratamiento
Los lodos primarios y los inorgnicos, generalmente tienen una mayor con-
centracin en cada una de las etapas de la secuencia de tratamiento que los
obtenidos de la coagulacin mediante aluminio o la de aquellos que proceden de
lodos activados.
Los procesos que normalmente se utilizan en la secuencia de deshidratacin
dependen de:
Naturaleza y caractersticas del lodo.
Mtodo que se aplicar en la disposicin final.
sedimentacin espesamiento deshidratacin
Conc.
Slidos (%)
0,5 6,0 20 12,0 18,0 50,0
290
Menndez Gutirrez, C. y J. Prez Olmo
El espesamiento por gravedad y por flotacin, la filtracin al vaco o por
presin, el tratamiento mediante calor, y el secado en lechos al aire libre, son
algunos de los tratamientos que hoy da se utilizan para disminuir el contenido de
agua de los lodos.
En los pases en desarrollo los procesos ms comnmente empleados son el
espesamiento por gravedad y los lechos de secado debido a su relativo bajo
costo de inversin, operacin y mantenimiento. No obstante en ocasiones los
volmenes de lodos son tales que se hace necesario acudir a los procesos ms
intensivos y tecnolgicamente ms complejos.
9.4. ESPESAMIENTO POR GRAVEDAD
Los principios que rigen este proceso as como los elementos fundamentales
que facilitan el diseo y operacin de los espesadores pueden ser tratados con
ms detalle. Sin embargo, aqu se dan algunos elementos de carcter prctico
para el trabajo ms cotidiano.
Debe tenerse presente que los espesadores se disean sobre la base del
rea unitaria m
2
kg
-1
d
-1
, mediante la relacin del rea superficial con los slidos
que entran y salen del espesador.
La carga msica de lodo, kgm
-2
d
-1
recomendada para el trabajo de los
espesadores vara con la facilidad que posea el lodo de perder el agua, y puede
variar entre 20 kgm
-2
d
-1
para lodos digeridos procedentes de un lodo activado,
hasta 150 kgm
-2
d
-1
para un lodo primario.
El flujo por unidad de rea que normalmente se recomienda para los
espesadores vara entre 15 y 40 m
3
m
-2
d
-1
. Cargas superficiales menores pueden
tener como consecuencia malos olores al producirse condiciones spticas. En
tales casos es recomendable el empleo de agua de dilucin para aumentar el
flujo por unidad de rea.
En los espesadores continuos es usual dejar una capa de lodo en el fondo del
mismo para propiciar el aumento de su concentracin. Normalmente el tiempo
de retencin de los slidos oscila entre 0,5 y 2 das. Este tiempo de retencin,
llamado por algunos relacin de volumen de fango (RVF), se calcula dividiendo
el volumen ocupado por el manto de lodo en el espesador, entre el flujo diario de
lodo extrado.
Ejemplo 9.2
El lodo primario que se obtiene en una planta de tratamiento de aguas
residuales es 16 m
3
d
-1
con una concentracin de slidos de 45000 mg L
-1
.
Calcule el rea requerida del espesador, asumiendo una carga msica de
lodo de 50 kgm
-2
d
-1
.
291
Manejo de Lodos
Considere adems que en el sobrenadante sale 5 % de los slidos que en-
tran al espesador y que los lodos de fondo tienen una concentracin de 8 %.
Determine el agua de dilucin que se necesita adicionar para obtener una
carga superficial de 15 m
3
m
-2
d
-1
.
Carga de lodo al espesador = 16 x 45000 x 10
-3
= 720 kg d
-1
rea de tanque requerida = A
4 14
50
720
, A = =
m
2
Dimetro = D
m 28 4
4 4 14
5 0
,
a
,
D
,
= |
.
|

\
|
=
Volumen de lodo espesado = V (ec. 9.3)
55 8
1 1000 08 0
55 8 95 0 720
,
,
, ,
V =


=
m
3
d
-1
Flujo por unidad de rea = q
11 1
4 14
16
,
,
q = =
m
3
m
-2
d
-1
El flujo de agua suplementario que se necesita para lograr una carga super-
ficial de 15 m
3
m
-2
d
-1
ser,
(15 1,11) 14,4 200 m
3
d
-1
Si el manto de lodo espesado tiene una profundidad de 1,25 m, estime el
tiempo de retencin de los lodos en el espesador,
d 1 2
55 8
4 14 25 1
,
,
, ,
0 =

=
9.5. LECHOS DE SECADO
El mtodo ms comn de secado de lodos espesados es el de los lechos de
secado, teniendo en cuenta su economa y que no requiere de personal especia-
lizado. Se utiliza fundamentalmente en plantas pequeas de tratamiento de alba-
ales y para algunos residuales industriales.
292
Menndez Gutirrez, C. y J. Prez Olmo
El secado en los lechos se produce a travs de los procesos de percolacin
y evaporacin. Por esta razn su diseo y empleo est influido por las condicio-
nes climticas prevalecientes.
La fraccin de agua que se elimina por percolacin vara de 0,2 a 0,5 en
dependencia el contenido inicial de slidos y de sus caractersticas, as como de
la radiacin solar que incide sobre el mismo.
En el tema no se insistir en el diseo emprico de los lechos de secado. No
obstante, a continuacin se resumen algunos criterios que mejor se ajustan al
diseo de estos dispositivos en pases que reciben una buena radiacin solar
durante la mayor parte del ao.
Tabla 9.2. Parmetros empricos para el diseo de lechos de secado
Cuando los lodos a secar proceden de un tratamiento de residuales indus-
triales es recomendable, siempre que se pueda, realizar pruebas de secado de
los mismos antes de proceder a disear con criterios que quizs no se ajustan a
un caso particular.
Swanwick
1,2
ha desarrollado una metodologa para el estudio de la deshidrata-
cin de los lodos en lechos de secado. Esta metodologa se describe a continuacin:
1. Utilice un tubo de vidrio de 2,50 cm de dimetro con arena en el fondo.
2. Introduzca en el tubo el lodo a secar hasta tener una altura de aproxima-
damente de 30 a 45 cm.
3. Permita el drenaje del agua durante un periodo entre 12 y 48 horas, de-
pendiendo del contenido de humedad.
4. Extraiga la torta drenada y determine su contenido de humedad.
5. Exponga la torta a la evaporacin natural en contacto directo con el aire.
Controle peridicamente el contenido de humedad hasta que se alcance
el valor deseado.
6. Determine la humedad. La diferencia entre el valor obtenido aqu y en el
paso 4 es el agua a evaporar.
7. Infrmese de la evaporacin y el rgimen de lluvia anual de la zona a
partir de datos meteorolgicos.
8. Se acepta que la evaporacin que ocurre en el lodo hmedo es 75 % de la
que ocurre en el agua as como que 43 % del agua de lluvia es drenada y
75 % restante se evapora.
Tipo de lodo digerido rea m
2
(persona)
-1
Primario 0,040
Primario y filtro percolador 0,044
Primario y lodo activado 0,112
Lodo de precipitacin qumica 0,076
293
Manejo de Lodos
9. Calcule, mes a mes, la evaporacin acumulativa multiplicada por el
factor 0,75. Grafique este resultad en funcin del tiempo. Haga lo
mismo con la lluvia cada, pero multiplicndola por el factor 0,57.
10.Estime, para cada mes, teniendo en cuenta los resultados en 9 y 6, el
tiempo requerido para el secado del lodo.
11. Tabule, para cada mes, el rea total requerida.
12.El rea de diseo ser la mxima hallada en el paso 11.
9.6. VOLUMEN DE AGUA PRODUCIDA
Como consecuencia del proceso de deshidratacin de los lodos, por cual-
quier mtodo que se utilice, se produce un volumen dado de agua que es preciso
conocer. Esta agua no siempre est libre de slidos. Si se trata de un espesador
por gravedad operando en ptimas condiciones, el lquido claro obtenido puede
contener entre 5 y 10 % de los slidos iniciales del lodo. Con otros procedimien-
tos el contenido de slidos puede ser mayor o menor.
El volumen de agua producida puede calcularse como sigue:
Se sabe que,
Ws
Cs
Ww
Cw
=
(9.7)
Cs: contenido de slidos del lodo, %.
Cw: contenido de agua en el lodo, %.
Ws: masa de slido, kg.
Ww: masa de agua, kg.
Entonces,
Ws
Cs
Ww
Cs
=
100
(9.8)
De aqu,
Ws
Cs
Ww |
.
|

\
|
= 1
100
(9.9)
Si se acepta que la densidad del agua es 1000 kg m
-3
,
Vw = Ww 10
-3
(9.10)
donde:
Vw: volumen de agua, m
3
.
294
Menndez Gutirrez, C. y J. Prez Olmo
Por tanto
3
10 1
100

|
.
|

\
|
= Ws
Cs
Vw
(9.11)
El agua que est asociada al cambio de concentracin de un lodo desde (Cs)
1
a (Cs)
2
es:
Ws
(Cs) (Cs)
Vw
|
|
.
|

\
|
=
2 1
1 1
10
1
(9.12)
Ejemplo 9.3
100 m
3
d
-1
de un lodo con un contenido de slidos del 5 % es deshidratado
por diversos tratamientos hasta alcanzar 20 %. Determine el volumen de lquido
claro producido.
(Cs)
1
= 5 %
(Cs)
2
= 20 %
Ws = 100 50000 10
-3
= 5000 kg d
-1
Aplicando la ecuacin 9.12,
5000
20
1
5
1
10
1

|
|
.
|

\
|
= Vw
Vw = 75 m
3
d
-1
Notas bibliogrficas
1 SWANWICK, J.D.: Advances in Water Pollution Research, vol. II, Pergamon
Press, N.Y., 1963.
2 _____________ : Water Poll. Research, Her Majesty`s Stacionary OIIice,
London, 1966.
ANEXOS
297
ANEXO 1
VOLMENES MNIMOS DE MUESTRAS
YCRITERIOS PARA SUCONSERVACIN
Parmetro Volumen
mnimo de
muestra, mL
Conservacin Tiempo
mximo de
conservacin
Aceites y
grasas
Acidez
Alcalinidad
DBO
DQO
Carbono
orgnico, total
Nitrato
Nitrito
Amoniaco
N-org
Fosfato
Oxgeno
disuelto
pH
1000
100
200
1000
100
100
100
100
500
500
100
300
aadir H
2
SO
4
hasta pH<
2 y refrigerar
refrigerar
refrigerar
refrigerar
aadir H
2
SO
4
hasta pH<
2 y refrigerar
refrigerar y aadir HCl
hasta pH 2
refrigerar
refrigerar
Aadir H
2
SO
4
hasta pH<
2 y refrigerar
aadir H
2
SO
4
hasta pH<
2 y refrigerar
refrigerar
Analizar inmediatamente
Analizar inmediatamente
28 d
24 h
24 h
6 h
7 d
7 d
48 h
12 h
7 d
7 d
48 h
298
ANEXO 2
DISTRIBUCIN NORMAL ACUMULATIVA
N P(X) N P(X) N P(X)
0,00 0,5000 0,55 0,7088 1,05 0,8531
0,05 0,5160 0,60 0,7257 1,10 0,8643
0,10 0,5398 0,65 0,7422 1,15 0,8749
0,15 0,5596 0,70 0,7580 1,20 0,8849
0,20 0,5793 0,75 0,7734 1,25 0,8944
0,25 0,5987 0,80 0,7881 1,30 0,9032
0,30 0,6179 0,85 0,8023 1,35 0,9115
0,35 0,6368 0,90 0,8159 1,40 0,9192
0,40 0,6554 0,95 0,8289 1,45 0,9265
0,45 0,6736 1,00 0,8413 1,50 0,9332
0,50 0,6915

Das könnte Ihnen auch gefallen